Sesap 17th Vol 2

  • Uploaded by: amr elfaresy
  • 0
  • 0
  • February 2021
  • PDF

This document was uploaded by user and they confirmed that they have the permission to share it. If you are author or own the copyright of this book, please report to us by using this DMCA report form. Report DMCA


Overview

Download & View Sesap 17th Vol 2 as PDF for free.

More details

  • Words: 93,784
  • Pages: 157
Loading documents preview...
Volume II Contents Perioperative Care Part I

001

Part II

019

Part III

030

Part IV

043

Problems in Related Specialties

057

Skin and Soft Tissue

075

Surgical Critical Care Part I

089

Part II

102

Trauma Part I

112

Part II

129

Vascular

143

Perioperative Care Part I

rhabdomyolysis. The modestly elevated sodium concentration and the urine specific gravity suggest at least a moderate degree of dehydration. Hypokalemia is evident in this case and may be even lower after rehydration.

ITEMS 1-30 For each question, select the best possible response. 1. A 19-year-old, black military recruit complains of severe pain in his legs the morning after a 5-mile march in full combat gear during mid-August at a southern basic training camp. He has no history of similar episodes and no reported previous heat-related injury. He has no history of sickle cell disease. He is awake, conversant, and oriented with normal vital signs. Other than severe discomfort in his calves and thighs, his physical examination is negative. Laboratory studies demonstrate the following: Serum

Urine

Na+ = 147 mEq/L (136-145 mEq/L) K+ = 2.8 mEq/L (3.5 5.0 mEq/L) Cl- 101 mEq/L (95-105 mEq/L) CO2 = 28 mEq/L (23 29 mEq/L) BUN = 17 mg/dL (7-20 mEq/L) Glucose = 80 mg/dL (70-100 mEq/L) Creatinine = 1.1 mg/dL (0.4-1.3 mEq/L) Serum Ca++ = 8.2 mg/dL (8.5-10.2 mEq/L) Creatine phosphokinase = 400 U/L (60-174 U/L) Mg++ = 2 mg/dL (1.5-2.5 mEq/L) Initial treatment should consist of containing A. B. C. D. E.

Numerous studies have demonstrated that serum potassium concentrations rise markedly during extreme exercise but fall to concentrations well below baseline values, even at 5minutes after cessation of the activity and remain below baseline for as much as several hours. The rise and fall of serum potassium is directly proportional to the bulk of the muscle mass used during exercise. This finding is not surprising, because muscle contains the largest body pool of this ion. The fall in potassium after exertion is thought to be due to catecholamine activation of Na+/K+-ATPase, which serves to drive potassium into cells. It could be anticipated that measured serum potassium would be even lower if intravascular volume were restored to baseline levels. Thus, rehydration should be supplemented with potassium. The clinical significance of this strategy lies in the fact that hypokalemia in presumably healthy individuals is linked to cardiac rhythm disturbances in patients with chronic conditions and possibly to sudden death after extreme exertion.

Color: dark yellow pH: 5.7 Specific gravity: 1.038 Dip Stick Blood: trace Glucose: negative Protein: trace

Microscopic: crystals present, few white blood cells, no red blood cells or casts intravenous fluids

Magnesium replacement may be necessary if serum values of potassium do not respond to potassium supplementation or if serum magnesium levels are low. Renal excretion of potassium is not reversed in the absence of adequate levels of magnesium. The addition of glucose may stimulate insulin secretion, driving potassium intracellularly and further lowering serum potassium concentration. Bicarbonate has been used, on a theoretical basis, to increase the solubility of myoglobin in rhabdomyolysis by increasing urinary pH, along with mannitol to increase urine flow. Few data exist to support this practice. There is no compelling evidence in this case that rhabdomyolysis is present or at least not to the degree that would be of concern for the development of renal failure. There is nothing to suggest that calcium supplementation is required.

magnesium. calcium. glucose. potassium. bicarbonate.

Answer:

D

Rhabdomyolysis is obviously a concern in this recruit after extreme exertion during training on a presumably hot August day. Exertional rhabdomyolysis is frequently seen in military recruits unaccustomed to vigorous exercise, particularly blacks, individuals with sickle cell disease, and those with a history of heart-related injury. However, sufficient information is available to exclude the diagnosis of rhabdomyolysis: serum creatinine kinase (creatinine phosphokinase) is not elevated to the extent seen with rhabdomyolysis, usually 10-fold the upper limit of normal. Urine myoglobin levels rise quickly with rhabdomyolysis but are rapidly cleared by renal excretion, remaining elevated for only 2 to 12 hours after injury. Myoglobinuria is suggested by the urine dipstick being positive for "trace" blood without red blood cells being identified by urine microscopy. Serum creatinine kinase levels remain elevated for several days, making it a more reliable test. Further, serum potassium levels would be expected to be markedly elevated with

2. The patient shown in figure 2.1 was recently seen by his primary physician and started on an angiotensin converting enzyme (ACE) inhibitor. Which of the following statements about this condition is true? A. It has no effect on the gastrointestinal tract. B. This reaction is unlikely to recur with repeat exposure of the inciting drug. C. Potential airway compromise prompts aggressive early intervention. D. Black patients uncommonly manifest this condition compared with patients of European descent. E. Treatment is diuretics and steroids.

Answer: 1

C

Angiotensin converting enzyme (ACE) inhibitors can interfere with pathways to dear kinins and other vasoactive peptides, leading to angioedema. Angioedema commonly affects the lips, tongue, or face, but it can also involve the bowel, leading to presentation with abdominal pain or obstruction. Rechallenge with the offending drug typically leads to recurrence of the angioedema. Black patients have a 5-fold higher incidence of this condition over white patients. Diuretics and steroids are not a part of the management of ACE inhibitor associated angioedema. The treatment is discontinuation of the drug and airway management. Early intubation with surgical backup for emergency surgical airway is indicated when this disease is encountered.

4. Which of the following statements is true regarding postoperative fluid management? A. Intravenous fluids should be administered to maintain a urine output of mL/kg/hour. B. Urine output of less than 0.5 mL/kg/hour is associated with increased hospital stay. C. Decreased urine output is a physiologic response to surgical stress. D. Fluid overload reduces postoperative morbidity. E. Enhanced recovery after surgery pathways improve outcomes by fluid loading.

Answer: 3. A 45-year-old man is involved in a motor vehicle crash and is admitted to your service for multiple lower extremity fractures. Which of the following statements regarding venous thromboembolism (VTE) is true for this patient?

What has traditionally been considered oliguria secondary to postoperative hypovolemia was recently recognized as a normal physiologic response to surgical stress. Fluids administered to achieve a urine output of 1 mL/kg/hour exceed even the classic goal of 0.5 mL/kg/hour. In a randomized trial of patients undergoing major abdominal surgery, a urine output goal of 0.2 mL/kg/hour was not associated with any injury to the kidney and did not affect length of stay. In a trial of enhanced recovery care in more than 1900 patients, fluid overloading was a strong predictor of later complications. Finally, a review of enhanced recovery pathways revealed that improved outcomes are achieved by fluid-restrictive strategies.

A. Low molecular weight heparin (LMWH) prophylaxis is preferred over unfractionated heparin. B. VTE are rare in the first few days of hospitalization. C. Chemical VTE prophylaxis should be withheld between 12 and 24 hours before open reduction and internal fixation. D. Rates of deep vein thrombosis are not influenced by surveillance bias. E. Titrating the dose of LMWH with thromboelastography decreases VTE rates.

Answer:

C

5. A 29-year-old woman is 19 weeks pregnant and presents with a changing mole on her leg. Biopsy shows a 1.5-mm Breslow depth, nonulcerated melanoma. Examination reveals a gravid uterus and no groin adenopathy. Wide local excision and sentinel node biopsy are planned. The patient should be told that

A

Venous thromboembolism (VTE) has persisted despite aggressive monitoring and prophylaxis by physicians and healthcare systems. Several studies are under way to examine the addition of aspirin to low molecular weight heparin (LMWH) or replacement of LMWH with aspirin for VTE prophylaxis. Meanwhile, a review of data from the Michigan Trauma Quality Improvement Program system suggested that LMWH was associated with reduced mortality and VTE compared with unfractionated heparin. This and another study showed a preponderance of VTE in the first few days of hospitalization.

A. surgery should be done under spinal anesthetic. B. radioactive tracers for the sentinel node biopsy are contraindicated. C. operation should be delayed until the third trimester. D. the prognosis of melanoma is not affected by pregnancy. E. preoperative CT scanning is indicated.

A randomized trial of thromboelastography-guided versus fixed-dose LMWH showed no effect on VTE rates. Chest guidelines for VTE prophylaxis in orthopedic surgery recommended starting preoperative chemical prophylaxis between 12 and 24 hours before surgery. Although VTE has been described by the US government as a "never event," VTEs do still occur despite prophylaxis. Due to surveillance bias, deep vein thrombosis rate after the implementation of routine duplex surveillance of high-risk patients is significantly higher than before surveillance—"seek and you shall find."

Answer:

D

Approximately one-third of women with melanoma are of childbearing age, and melanoma is the most common malignancy encountered during pregnancy. Traditionally, pregnancy-associated melanoma (PAM) was thought to be associated with a worse prognosis. More recent studies show no significant differences in stage at presentation; recurrence rates; or disease-free, melanoma specific, or overall survival rates between PAM and non-PAM patients. The feared event of transplacental transfer of melanoma to the fetus is rare; no

2

cases of this were found in recent reports totaling 171 PAM patients.

Acid-base disturbances among critically ill patients are some of the most common problems encountered in the intensive care unit. Understanding the nuances of add base aberrations is complicated by a history of confusing terminology, concepts that are counterintuitive, and a persistent controversy over the physio-physical-chemical mechanisms that account for the abnormalities encountered. Understanding is further complicated by differences in the buffering capacity of the 3 different bodily fluid compartments (i.e., the intravascular, extravascular, and intracellular spaces). Currently, 3 methods are used to quantify the deviations from the physiological pH at which human cells function with maximum efficiency. Careful analysis of each of these methods demonstrates that although they have a different focus, they are generally complementary.

Rather, survival in PAM patients is dependent on the same factors as non-PAM patients: Breslow thickness, tumor ulceration, and sentinel node status. Accordingly, PAM patients should be treated in the same fashion as non-PAM patients, with expeditious wide excision and sentinel lymph node biopsy. In a recent review of 15 PAM patients undergoing sentinel lymph node biopsy, all had general anesthesia, the mean gestation age at operation was 20 weeks, and all but one had injection of radioactive tracer for lymph node mapping. 99mTc sulfur colloid is safe for the fetus if the dose is kept below 50 mGy, and lymphoscintigraphy delivers less than 5 mGy to the fetus. Conversely, blue dye is often not used because of the potential teratogenic and anaphylactic risks.

The first and most commonly used approach, the "traditional approach," is based on the Henderson-Hasselbalch equation, developed in 1916 by Henderson and modified some 8 years later by Hasselbalch, who added the logarithmic expression of the ionization of carbonic add in an aqueous solution.

This patient has clinical stage T2aN0, IB melanoma; regardless of pregnancy, preoperative CT scanning is not indicated by accepted guidelines. Scans should be done postoperatively if the operation upstages the patient, by which time she may be postpartum and scanning would therefore be safer.

H+ x HCO3- ↔ H2CO3 ↔ CO2 x H2O Henderson Equation: H+ x HCO3- = pKa x CO2 x H2O Henderson-Hasselbalch Equation: pH = pKa + logl0 (HCO3/O.03 x CO2)

6. A 24-year-old man sustained multiple injuries (injury severity score = 34) as the result of a motor vehicle crash 2 weeks ago. He developed acute respiratory distress syndrome requiring mechanical ventilation. He is being treated for sepsis and appears to be improving, as indicated by normalization of vital signs with removal of pressers, improved urine output, less requirement far ventilator support, and a reduction in his leukocytosis. Current laboratory values are as follows:

Where k and pKa are the dissociation constants of carbonic add and 0.3 represents the solubility of CO2 in plasma. Use of the Henderson-Hasselbalch approach presumes that bicarbonate and carbon dioxide are independent variables with respect to pH, the dependent variable. Fortunately, this approach is simple. Results for the bicarbonate are calculated by modem blood gas analyzers readily available, accurate, and appropriate for most of the simple, direct add-base abnormalities. A change in the PaCO2 of 12 mm Hg can be expected to produce a pH gain or loss of 0.1; gain if the PaCO2 increases (respiratory acidosis) and loss if the PaCO2 decreases (respiratory alkalosis). Further, a change in pH of 0.1 equates to a change in bicarbonate of approximately 6 mEq/L (i.e., if the pH is 7.5, this represents a gain of 6 mEq/L of HCO3-, and loss of the same amount if the pH is 7.3, assuming a normal pH of 7.4). Changes in ventilation produce nearly immediate effects, whereas renal compensation for the change requires days. Compensation is never complete, returning pH to the physiological normal value of 7.4. As an example, a respiratory acidosis with a PaCO2 of 52 mm Hg and a measured pH of 7.35 represents a partial compensation by an increase in HCO3-

Serum Arterial Blood Gases Na+ = 131 mEq/L (136-145 mEq/L) PaO2 = 98 mm Hg K+ = 4.2 mEq/L (3.5-5.0 mEq/L) PaCO2 = 41 mm Hg Cl- = 86 mEq/L (95-105 mEq/L) pH = 7.32 BUN = 8 mg/dL (7-20 mEq/L) Sat = 99% Albumin = 0.8 g/dL (3.5-5.2 g/dL) Base excess = -4.8 mEq/L Creatinine = 1.0 mg/dL (0.4-1.3 mEq/L) Serum Ca++ = 3.6 mg/dL (8.5-10.2 mEq/L) Mg++ = 2.2 mg/dL (1.5-2.5 mEq/L) HCO3- = 21 mEq/L (calculated; 20-29 mEq) PO4 = 2.3 mg/dL (2.5-4.5 mg/dL)

The underlying acid-base aberration is best described as a A. B. C. D. E.

severe metabolic acidosis. mixed respiratory-metabolic acidosis. uncompensated respiratory acidosis. partially compensated respiratory acidosis. metabolic alkalosis and respiratory acidosis.

Answer:

The management of acid-base changes was further refined by the use of the concept of base excess, introduced by SieggardAndersen and named the van Slykes equation, acknowledging his contributions to the understanding of addbase physiology:

A 3

BE = 0.02786 x PaCO2 x 10(pH-6.1) + (13.77 x pH) -124.58

[Cl-] = 92 mEq/L SID = HCO3 +Alb-+ Pi-

This value is also provided by blood gas machines, obviating the need for calculation at the bedside. Some confusion, however, is introduced by the concept of a "negative" base excess, which implies an "acidosis." Simultaneous calculation of the anion gap may also shed light on an addbase relationship. Because all anions cannot be measured readily, there is always a calculated anion gap, normally 6 to 12 mEq/L. The anion gap can be expressed as follows:

Alb- = Alb g/L x (0.123 x pH - 0.631) (in g/L) Alb- = 8 x (0.123 x 7.32 - 0.631) Alb- = 6.58 g/L Note the units used in the formula. 0.8 g/dL = 8.0 g/L Pi- = Pi x (0.309 x pH - 0.469) (in mmol/L) Pi- = 2.3 x (0.309 x pH - 0.469) Pi- =4.87 therefore SID = 21+ 6.58+ 4.87 SID = 32.45 mmol/L

AG = Na++ K+- Cl—HCO3 AG-131 +4.2-86-21 AG = 28.2 mEq/L A high anion gap implies a metabolic acidosis. A low anion gap is rare and is the result of abnormal serum proteins (e.g., myeloma, monoclonal gammopathies). In reality, there can never be a true anion gap based on the principle of electrical neutrality: the sum of all anions and equal to the sum of all cations. As such, a high anion gap implies the presence of an unmeasured anion, such as in methanol ingestion, uremia, diabetic ketoacidosis, propylene glycol, isoniazid intoxication, lactate, ethylene glycol ingestion, rhabdomyolysis, or salicylates, "MUD PILERS" the mnemonic memorized by most medical students.

These values indicate that the metabolic acidosis seen in this patient is worse than suggested by the pH alone and the negative base excess: the decrease in the SID from the normal value of 40 based on plasma water excess, the increase in the corrected anion gap based on severe hypoalbuminemia, and the increase in Pi compound the acidosis. This is modified to some degree by the chloride deficit and the severe hypoalbuminemia, both of which serve to have an alkalinizing effect on the pH. There is no respiratory acidosis or alkalosis.

In the 1980s, Peter Stewart proposed a third approach to the evaluation of acid-base physiology. Stewart's approach, lauded by many and criticized by an equal number, focuses on the physical-chemical aspects of electrical neutrality. In this analysis, H4 (and consequentially pH) and HCO3- are dependent variables, and their change is the result of a change in the strong ion difference (SID) that considers the rote of nonvolatile weak acids, such as albumin and inorganic phosphate (Pi), and the impact of chloride. Stewart's method requires the simultaneous calculation of 6 different equations, a process that until the development of computer algorithms, and now smart phone apps, made this approach too cumbersome for bedside use. Perhaps the most important lessons for the clinician from this approach include the impact of (1) volume expansion/contraction as reflected in the serum sodium;(2) serum albumin [Alb] inorganic phosphate (Pi), when added to HCO3- constitute the SID (normal value 40); (3) water itself is the largest reservoir of H+ in the body,(4) strong unidentified anions cannot be measured but can be calculated; and (5) measured values of chloride, albumin, and phosphate must be mathematically corrected based on the water excess or deficit.

The implications for management of the acid-base abnormality, highlighted by this more complicated but more insightful analysis, could be inferred by the recognition of the volume excess (i.e., sodium), the profound hypoalbuminemia, and the corrected anion gap. Intravenous volume restriction in addition to measures to improve the patient’s nutritional status are indicated. As some authors have concluded, older, more familiar tools to analyze acidbase abnormalities allow the clinician to arrive at the same conclusions as the more complicated methods.

7. A 67-year-old man has a partial bowel obstruction and undergoes a colonoscopy that shows a fungating lesion in his sigmoid colon. In the recovery room; he has chest pain and has ST segment elevation on his electrocardiogram. Coronary angiogram is performed and shows severe left anterior descending stenosis; Which intervention would allow sigmoid colectomy to be performed in the next 2 weeks with minimal bleeding and thrombosis risk? A. B. C. D. E.

Values for the patient presented are as follows:

AG adjusted = AG measure + 0.25(Alb normal - Alb measured) (in mEq/L) AG adj = 28.2 + 0.25(45 - 8) AG adj = 37.5 mEq/L Cl-corr = Cl-measured x Na+normal /Na+measured (in mEq/L) Cl-corr = 86 x 140/131 = 86 x 1.068

Placement of a bare metal stent Placement of a drug-eluting stent Angioplasty of the left anterior descending lesion Coronary artery bypass grafting Continuous unfractionated heparin intravenously for 3 weeks

Answer: 4

C

In a patient with significant coronary disease who also needs urgent noncardiac surgery, the question is which revascularization strategy is best. A coronary artery bypass (CABG) will usually have significant long-term durability but is associated with a 5% in hospital mortality if performed in the setting of an acute coronary event. Angioplasty without a stent would allow the earliest intervention for the colon lesion, because there is no stent in place to thrombose if the antiplatelet drugs are stopped. Discontinuation of antiplatelet therapy after stent placement puts the patient at risk for early stent thrombosis. The earlier the discontinuation of antiplatelet therapy occurs, the greater the risk. Although initial studies showed that antiplatelet agents could be stopped sooner after bare metal stent implantation, more recent studies suggest that stent-related thrombotic complications occur for approximately 6 months, regardless of the type of stent. Administering 3 weeks of unfractionated heparin without addressing the underlying anatomical disease (left anterior descending coronary artery stenosis) will not impact the patient's subsequent cardiac risk from the noncardiac procedure.

sodium, chloride, and magnesium, may accompany potassium losses through the gastrointestinal tract. In cases of hypokalemia refractory to potassium supplementation, magnesium needs to be replaced concomitantly. Magnesium is the most common divalent cation in human cells, and magnesium deficiency impairs the Na/K-ATPase pump, leading to renal potassium wasting. Supplemental citrate and phosphate have no effect on potassium, and the administration of glucose and insulin would only drive potassium into the cell. Ammonium chloride, occasionally used to correct metabolic alkalosis, does not assist in the retention of potassium.

9. An otherwise healthy 52-year-old woman underwent an uneventful right hemicolectomy and ileocolostomy for an angiodysplastic lesion in her cecum. An ileus and her inability to take oral nourishment has kept her hospitalized. Postoperatively, she was given unfractionated heparin for deep vein thrombosis prophylaxis. On postoperative day 6, her platelet count was 30,000/mm3 (baseline 230,000/mm3). Heparin was discontinued and an anti-PF4/Heparin-ELISA was requested, which was positive (OD 1.5). After a flush of a central venous line with heparin solution an hour later, the patient developed cyanosis of the nail beds of both hands despite palpable radial artery pulses. The most appropriate management at this point is

8. A 28-year-old woman with longstanding inflammatory bowel disease underwent an urgent total abdominal colectomy for toxic megacolon after 10 days of intractable diarrhea. Postoperatively, she was found to be hypokalemic (potassium = 2.2 mEq/L; 3.5-5.0 mEq/L) and was begun on intravenous potassium chloride. Her blood glucose is 170 mg/dL (70-100 mg/dL). After 24 hours of replacement, her potassium is only 2.4 mEq/L (13.5-5.0 mEq/L). In addition to continued potassium chloride administration, successful correction of her potassium deficit will likely require the infusion of A. B. C. D. E.

A. B. C. D. E.

citrate. phosphate. magnesium. insulin and glucose. ammonium chloride.

Answer:

platelet transfusion. streptokinase. warfarin. argatroban. enoxaparin.

Answer:

D

Thrombocytopenia is common in intensive care unit patients. Approximately 50% of intensive care unit patients have platelet counts less than 150,000/mm3 (150,000400,000/mm3), and 10% will have counts less than 50,000/mm3. Humans produce approximately 150 billion platelets/day with an average lifespan of 10 days. Causes of reduced platelets include (1) hemodilution from resuscitation with intravenous fluids; (2) sequestration from hepatosplenomegaly; (3) consumption from blood loss or sepsis/systemic inflammatory response syndrome; (4) underproduction due to viral illness, drugs, and other toxins; and (5) destruction by immune complexes from heparininduced thrombocytopenia (HIT) or thrombotic thrombocytopenia purpura. More than one-half of patients exposed to heparin will develop antibodies after heparin administration, and more than 65% of patients undergoing cardiac surgery in which heparin is used will develop them. The reason for the increase in cardiac surgical patients is not clear. Despite the propensity to form antibodies, the incidence of HIT in intensive care unit patients is quite low (0.1-3%, depending on the series reported).

C

Hypokalemia is one of the most common electrolyte abnormalities encountered in medicine and in the intensive care unit in particular. Because the majority of potassium is intracellular/ serum levels do not reflect total body potassium. Although most hypokalemic patients are asymptomatic, the clinical concern is the potential for cardiac arrhythmias, especially when it is accompanied by heart failure. Muscular symptoms, when present, include weakness, fasciculations, and tetany. Gastrointestinal symptoms may include ileus, nausea, vomiting, and constipation. Potassium loss may result from diuretics, gastrointestinal loss, nephrotoxic drugs, some chemotherapeutic agents, and especially diarrhea, as in this case. Hypokalemia is also seen in some genetic syndromes (e.g., Bartter disease). Additional electrolyte loss, including

5

Platelet factor 4 (PF4), a positively charged protein released from alpha-granules in platelets, preferentially binds to the negatively charged heparin molecule even to the point that it will displace PF4 from its natural binding on vascular endothelium. The PF4-heparin complex (PF4/H) results in platelet aggregation and is a potent activator of B-cell production of an antibody against PF4/H, augmented by 10,000 times if the PF4/H is complement-coated. Most antibodies are IgG. IgM and IgA have also been identified, although their role in HIT is uncertain. The antibody-PF4/H complex bids to the FcƳlla receptor on monocytes, stimulating the release of procoagulant microparticles, tissue factor, and activated thrombin. This mechanism is believed to be the process by which spontaneous thrombosis in HIT occurs, further reducing platelet numbers.

risk for HIT by either of these systems were identified as HITpositive when laboratory tests were obtained, and many patients judged to be high risk did not demonstrate laboratory confirmation of HIT, these systems should not be used alone to make therapeutic decisions. They probably are best used to indicate which patients require further laboratory investigation. The most common laboratory test used is the immunoassay (ELISA) for the anti-PF4/H antibodies. The test does not selectively identify the IgG antibody. Because many patients who receive heparin develop antibodies without overt HIT, the false-positive rate is quite high, leading to risky overtreatment. Because the assay uses an optical density (OD) measurement, the test is prone to operator error. Further, the OD threshold for positivity is variable. The test does demonstrate an excellent negative predictive value and is generally available in most laboratories. The radiolabeledserotonin release assay (14C-SRA) is considered the standard of care, but the assay is costly, requires radioactive substances, requires specialized equipment and expertise, and is not universally available. Many other laboratory tests are more accurate but are not as widely available.

Because thrombocytopenia is a common finding in the intensive care unit, several scoring systems can aid the clinician in determining which patients are at risk for HIT. The most common is Warkentin 4T system, outlined in table 9.1. Another system, the HEP-score, incorporates the features of the 4T-score, expands some of them, and adds others in a more quantitative fashion. The HEP-score improves the accuracy of probability estimates of HIT being present, but at the cost of adding complexity and being cumbersome to use at the bedside. Because patients who were found to be a low

The approach to the patient with suspected HIT is to initiate immediate discontinuation of heparin and to perform a risk assessment using one of the tools described. Based on that score, a decision can be made regarding the need for further

Points (0,1, or 2 for each of 4 categories: maximum possible score = 8) 2 >50% platelet fall to nadir≥20

1 30-50% platelet count fall (or >50% directly resulting from surgery); or nadir 10-19

0 <30% platelet fail; or nadir<10

Timinga of platelet count fall, thrombosis, or other sequelae (1st day of putative immunizing exposure to heparin = day 0)

Days 5-10 onseta (typical/ delayed onset HIT); or ≤ 1 day (with recent heparin exposure) or within 30 days (rapid-onset HIT)

Platelet count fail <4 days (unless picture of rapid-onset HIT)

Thrombosis or other sequelae (e.g., skin lesions, anaphylactoid reactions)

Proven new thrombosis; or skin necrosis (at injection site); or postintervention heparin bolus anaphylactoid reaction No explanation for platelet count fail is evident

Consistent with days 5-10 fall, but not clear (e.g., missing platelet counts); or ≤ 1 day (heparin exposure within the past 31-100 days) (rapid-onset HIT); or platelet fall after day 10 Progressive or recurrent thrombosis; or erythematous skin lesions (at injection site); or suspected thrombosis (not proven); hemofilter thrombosis Possible other cause is evident

Thrombocytopenia

Other cause for thrombocytopenia

None

Definite other cause is present

Pretest probability score: 6-8 = high; 4-5 = intermediate; 0-3 = low HIT = heparin-induced thrombocytopenia. a First day of immunizing heparin exposure considered day 0; the day the platelet count begins to fall Is considered the day of onset of thrombocytopenia (It generally takes 1-3 more days until an arbitrary threshold that defines thrombocytopenia is passed). Usually, heparin administered at or near surgery is the most immunizing situation (i.e., day 0). * Table 9.1. The 4Ts scoring system.

6

laboratory investigation. Initially, an immunoassay is requested; if positive, a confirmatory 14C-SRA should be obtained. If the immunoassay is negative, heparin may safely be continued. A positive immunoassay then prompts the clinical decision as to whether some form of anticoagulation, either therapeutic or prophylactic, is required. When the patient has evidence of thrombosis (HITT), a direct thrombin inhibitor should be instituted. Both arterial and venous thrombosis have been reported with HITT. At the time of this writing, argatroban is the only drug approved by the US Food and Drug Administration, although there are reports of the off-label use of other direct thrombin inhibitors, as well as fondaparinux, in this clinical setting.

characteristics and the clinical situations that necessitated the need for a contrast-enhanced imaging study [e.g., coronary versus peripheral vascular angiography], urgent versus elective procedure, specific contrast material used as well as the volume administered, and many others). To complicate matters further, a multitude of published metaanalyses have directly conflicting results. It is less complicated to identify patients who may be at increased risk for CI-AKI: those with pre-existing renal dysfunction, chronic congestive heart failure, diabetes, hypertension, and advanced age. The use of multipleiodinated contrast media is also a factor. In addition, patients who are volume depleted are at increased risk for CI-AKI. This has led to the consistent recommendation that intravenous volume infusion is the mainstay of prevention. Dispute continues over which intravenous fluid should be used, sodium chloride versus a bicarbonate solution. The use of N-acetylcysteine shows conflicting results in reducing CIAKI; however, along with bicarbonate, there is no evidence of harm. Statins have been used successfully in some studies but not in others, and controversy exists over the dose and duration of treatment. Renal-dose (low-dose) dopamine has no effect on reducing the incidence of CI-AKI or the duration if it occurs.

Platelet transfusion is not recommended due to the theoretical risk of enhancing PF4/H antibody production, platelet aggregation, and perhaps sensitization of the patient to exogenous platelets. There is no direct evidence that platelet transfusion is contraindicated; it is simply not necessary unless the patient is actively bleeding. Streptokinase is not indicated due to the bleeding risk, especially for postoperative patients with thrombocytopenia. Enoxaparin, a lowmolecular weight heparin, is not approved for use in this clinical setting, although low molecular weight heparins are less associated with HIT than unfractionated heparins. Warfarin is contraindicated due to the propensity for producing acquired protein C deficiency and lower extremity venous thrombosis and necrosis during the thrombocytopenic phase of HIT. Cutaneous necrosis has also been reported.

10. What is an effective strategy to minimize risk for acute kidney injury in a polytrauma patient expected to receive multiple contrast-enhanced imaging studies?

11. A 48-year-old woman with a history of deep venous thrombosis treated with rivaroxaban presents with 12 hours of right upper quadrant pain. Physical examination, laboratory values, and ultrasound are consistent with acute cholecystitis. Laparoscopic cholecystectomy is planned. Given her normal renal function coagulation profile when will her coagulation profile normalize?

A. B. C. D. E.

A. B. C. D. E.

N-acetylcysteine Sodium bicarbonate "Renal dose" dopamine Intravenous volume infusion Statin therapy

Answer:

D

6 hours 12 hours 36 hours 72 hours 96 hours

Answer:

The prevention of contrast-induced acute kidney injury (CIAKI) is a contentious issue in the medical literature, and to date there remains significant equipoise regarding the most effective strategy. Fortunately, CI-AKI is uncommon, occurring in 1 to 5% of patients undergoing a contrastenhanced imaging study; however, the results can have a profound impact on outcome. All reported trials suffer from one or more of the following: methodological flaws, small patient numbers, heterogeneity in the definition of CI-AKI, ill-defined or possibly inappropriate endpoints (e.g., rise in serum creatinine, arbitrary reduction in glomerular filtration rate, need for dialysis or renal replacement therapy, mortality, duration of CI-AKI, potential pleiotropic effects of pharmacological strategies used, inconsistent patient

C

Rivaroxaban is a direct oral anticoagulant (DOAC), commonly used in patients diagnosed with venous thromboembolic events, which inhibits Factor Xa. The halflife of this anticoagulant is 9 to 13 hours, and its therapeutic activity wears off after 4 to 5 half-lives. Therefore, rivaroxaban should be discontinued at least 24 hours before a surgical procedure. In patients with a reduced creatinine clearance, this recommendation is changed to 3 to 5 days. Although reversal of the DOAC is possible, the risk of a thrombotic event during reversal should be weighed against the urgency of surgery. For a patient with acute cholecystitis, her coagulation profile will normalize at 24 to 36 hours after her last oral dose.

7

B. has a morbidity rate comparable with total cholecystectomy. C. is followed by postoperative endoscopic retrograde cholangiopancreatography in 40% of patients. D. is optimally performed with a fenestrating rather than a reconstituting technique. E. is not a definitive procedure.

12. A 55-year-old man with perforated diverticulitis is undergoing a laparotomy, Segmental resection of the sigmoid was performed after a difficult dissection. The patient is on norepinephrine and vasopressin infusions, and his arterial blood gases are as follows: pH = 7.05 PaCO2 = 22 mm Hg PaO2 = 89 mm Hg HCO3- = 13 mEq (20-29 mEq) Oxygen saturation = 91% Base deficit = -19 mEq/L Temperature = 35°C What is the most appropriate next step?

Answer:

In cases of acute cholecystitis with significant inflammation or a fibrosed gallbladder, subtotal cholecystectomy is advocated. Subtotal cholecystectomy can be performed laparoscopically and involves removing the front wall of the gallbladder and all intraluminal stones. Both fenestrating (leaving the infundibulum/cystic duct orifice open with placement of a drain in Morison pouch) and reconstituting (closure of a cuff of infundibulum over the top of the cystic duct orifice) techniques are described, with reported equivalent outcomes.

A. Leave the bowel in discontinuity with a temporary abdominal closure B. Mature an end colostomy with a temporary abdominal closure C. Mature an end colostomy, closing the fascia, leaving the skin open D. Perform a primary anastomosis with a primary abdominal closure E. Perform a primary anastomosis, with a diverting loop ileostomy, and primary abdominal closure

Answer:

B

This procedure is safe and feasible, particularly in cases in which conversion to an open procedure will not better delineate the inflamed triangle of Calot. It can be a definitive procedure, with less than 5% of patients needing additional surgery. The morbidity rate is comparable with total cholecystectomy, with a reported 7 to 18% rate of bile leak and need for endoscopic retrograde cholangiopancreatography in 4 to 20% of patients.

A

Damage control surgery, abbreviating a laparotomy in the physiologically exhausted patient, has been used for several decades in trauma to improve mortality. The utility of damage control techniques is increasingly recognized in the acidotic, hypothermic, coagulopathic general surgery patient. Arresting surgical bleeding and limiting enteric contamination is followed by temporary abdominal closure with resuscitation and physiologic restoration in the intensive care unit. Definitive repair of injuries and restoration of gastrointestinal continuity is delayed until second laparotomy.

Items 14-15 A 76-year-old woman with atrial fibrillation on warfarin presents with acute cholecystitis. She is started on antibiotics and is scheduled to undergo laparoscopic cholecystectomy once her international normalized ratio of 2.5 (0.8-1.2) has been reversed. Her medical history is significant for congestive heart failure, hypertension, and diabetes.

This patient is in septic shock requiring multiple vasopressors; additionally he is hypothermic and markedly acidotic. Segmental resection of the septic source should be performed. The optimal next step is to leave the stapled-off ends of the colon in discontinuity, apply a temporary abdominal closure, and transport the patient to the intensive care unit for correction of his acidosis, hypothermia, and shock. Further time in the operating room is not warranted. Once fully resuscitated, the patient may be returned to the operating room for definitive management and abdominal closure.

14. In an attempt to determine whether anticoagulant bridge therapy is needed, which of the following individual risk factors portends the highest risk of stroke if using the CHA2DS2-VASc risk assessment tool? A. B. C. D. E.

Age Congestive heart failure Hypertension Diabetes Female sex

15. She is determined to be at moderate risk for a perioperative stroke based on the CHA2DS2-VASc risk assessment tool. In this patient, anticoagulant bridging therapy should

13. Laparoscopic subtotal cholecystectomy for acute cholecystitis

A. not be instituted. B. be instituted with intravenous unfractionated heparin.

A. requires a reoperation in 20% of patients.

8

C. be instituted with subcutaneous enoxaparin. D. be instituted with intravenous dalteparin. E. be instituted with subcutaneous fondaparinux.

Answers:

variables and is a reliable means to determine the Frailty status of patients. These variables can be grouped into one of 4 categories: comorbidities, activities of daily living, attitude, and nutrition. The important comorbidities are previous history of cancer, hypertension, coronary heart disease, or dementia. The activities of daily living include whether the patient needs help with grooming, managing finances, performing housework, toileting, and walking. Assessment of attitude includes whether the patient feels less useful, is depressed, is lonely, and is sexually active. Nutrition is assessed with a serum albumin, with an albumin of less than 3 mg/dL (3.5-5.2 mg/dL) contributing to the frailty score.

A, A

Anticoagulant bridge therapy for patients with atrial fibrillation who undergo an invasive procedure reduces the time that a patient will not be therapeutically anticoagulated, thereby reducing the overall risk of a thrombotic event. Recent studies have determined that the overall risk of thrombosis is much lower than previously cited and is not necessarily less with bridging therapy. The risk of bleeding must be balanced by the risk of thrombosis. When assessing the perioperative risk of stroke, the American College of Cardiology has stratified patients based on their risk of thromboembolism. The CHA2DS2-VASc score was developed after the CHADS2 score and evaluates more risk factors to estimate the risk of stroke in patients with atrial fibrillation. Aside from a previous risk of stroke or transient ischemic attack, of the choices listed, age 75 or older portends the highest risk of stroke when using the CHA2DS2-VASc risk assessment tool.

Patients determined to be frail by this index are more likely to have postoperative complications compared with nonfrail patients. Age, sex, American Society of Anesthesiologists score, and cardiovascular status are not independently predictive of the likelihood of developing postoperative complications in this patient population. These findings suggest that physiologic reserve is the best predictor of postoperative complications in patients older than 65. 17. A previously healthy patient presents with peritonitis secondary to perforated diverticulitis. After adequate source control, she should receive 4 days of

The risk of stroke must be balanced by the risk of bleeding when determining the need for anticoagulant bridging therapy in patients with atrial fibrillation who undergo an invasive procedure. According to the American College of Cardiology's 2017 consensus guidelines, bridging recommendations are based on stroke risk (CHA 2DS2-VASc tool) and bleeding risk. For this patient who is deemed moderate risk (CHA2DS2-VASc score 5-6 or prior stroke/TIA at least 3 months previously) of stroke with increased risk of bleeding (intra-abdominal surgery; i.e., laparoscopic cholecystectomy), anticoagulant bridging therapy is not recommended. Unfractionated heparin, enoxaparin, dalteparin, and fondaparinux are suitable alternatives for anticoagulation when bridging therapy is indicated.

A. B. C. D. E.

Answer:

As a result of her perforated diverticulitis, this patient is at higher risk of developing a complicated intra-abdominal infection. Owing to its broader spectrum of antibiotic activity, piperacillin-tazobactam is the best antibiotic for the treatment of this patient.

American Society of Anesthesiology score. age. the Frailty index. sex. cardiovascular status.

Answer:

E

Recent randomized clinical trials demonstrated that in patients with intra-abdominal infections in whom adequate source control is achieved, antibiotic therapy for a fixed duration of 4 days achieves equivalent outcomes compared with longer courses of antibiotic therapies based on administering antibiotics for 2 days beyond the resolution of fever, leukocytosis, or ileus.

16. In patients over the age of 65 undergoing emergency general surgery, the likelihood of developing postoperative complications is best predicted by A. B. C. D. E.

gentamicin. tigecycline. clindamycin. fluconazole. piperacillin-tazobactam.

Aminoglycosides, such as gentamicin, should not be routinely used for empiric therapy. Instead, they should be reserved for the treatment of Gram-negative infections that are resistant to other antibiotics and are documented to be sensitive to aminoglycosides.

C

In patients over the age of 65 undergoing emergency general surgery, the likelihood of developing perioperative complications is best predicted by the Frailty index. The emergency general surgery frailty index consists of 15

Similarly, tigecycline should not be used for the empiric treatment of intraabdominal infections. It should be reserved for patients with resistant pathogens, particularly as a component of a combination regimen.

9

Clindamycin should be used only as an antianaerobic agent for the empiric treatment of intra-abdominal infection as part of a combination regimen when metronidazole cannot be used. Antifungal agents, such as fluconazole, are not recommended for empiric therapy for intra-abdominal infection. If antifungal agents are necessary, echinocandins are preferred over azoles.

be stopped immediately. If succinylcholine has been previously administered, it should not be read ministered. Dantrolene is the treatment of choice. It should be prepared and administered immediately in doses of 2.5 mg/kg. Dantrolene acts by inhibiting the ryanodine receptor (RyRl) channel which reduces the RyRl channel activity in muscle cells. Susceptibility to malignant hyperthermia is inherited as an autosomal dominant condition and is characterized by a defect in the ryanodine receptor, resulting in a dysfunctional calcium channel located in the cytoplasmic membrane.

18. A 40-year-old man is undergoing a laparoscopic cholecystectomy. At the time of attempted port placement, the patient's abdominal wall is extremely rigid. The anesthesiologist notices that the patient is hyperventilating and there has been a sudden increase in the end tidal CO2 concentration. The patient begins having recurrent 10-beat runs of ventricular tachycardia. His arterial blood gas demonstrates a pH of 7.10, a PaCO2 of 65, a base deficit of 10, and a lactate of 8 mg/dL (4.5-19.8 mg/dL). His temperature is 37.5°C. A drug that is helpful in this condition is A. B. C. D. E.

Patients in whom the onset of malignant hyperthermia is suspected should be immediately cooled using all routes available, including the administration of intravenous saline at 4°C, topical ice to all exposed areas, and peritoneal washing with cold saline. Nasogastric lavage and bladder irrigation are not recommended. Administration of calcium to a patient with malignant hyperthermia is contraindicated, because it can worsen the clinical syndrome. While the administration of sodium bicarbonate can temporize the systemic acidosis, it does nothing to prevent the progression of malignant hyperthermia. If repeated and sustained runs of ventricular dysrhythmias are present, amiodarone, not lidocaine, is the antiarrhythmic agent of choice.

succinylcholine. sodium bicarbonate. lidocaine. calcium. dantrolene.

Answer:

E

19. Which of the following is considered a serious reportable surgical event?

Malignant hyperthermia (MH) is a pharmacogenetic disorder that may occur at any time during general anesthesia or the early postoperative period. The physiologic derangement leading to the development of MH is uncontrolled hypermetabolism. This uncontrolled hypermetabolism leads to respiratory acidosis and, in most cases, metabolic acidosis due to the rapid consumption of adenosine triphosphate within cells. The etiology is an uncontrolled release of intracellular calcium from the sarcoplasmic reticulum of skeletal muscle. The enhanced intracellular calcium results in abnormal skeletal muscle metabolism, which causes skeletal muscle contraction, rigidity, and increased oxygen consumption and CO2 production.

A. Intraoperative death in an American Society of Anesthesiologists class IV patient B. Lack of a timeout before a surgical procedure C. Lack of a site marking before exploratory laparotomy D. Deep surgical site infection after colorectal surgery E. Wrong site surgery

Answer:

E

The National Quality Forum has endorsed serious reportable events (SREs) in healthcare. These were initially released in 2002 and updated in 2011. The purpose of the SREs is to have a uniform and comparable public reporting system that facilitates systematic learning and drives national improvement across healthcare organizations. The 2011 update defines 5 surgical or invasive procedure serious reportable events:

In humans, clinical malignant hyperthermia results most often from exposure to anesthetic agents. All inhalation anesthetics, except nitrous oxide, are triggers for malignant hyperthermia. The muscle relaxant succinylcholine is also a well-described trigger for the onset of malignant hyperthermia. Early recognition of MH is key to successful treatment. Patients will manifest tachycardia, a rise in the end-expired CO2 concentration (despite an increase in minute ventilation), and muscle rigidity.

1. 2. 3.

For any patient in whom the onset of malignant hyperthermia is suspected, all inhalation anesthetics should

4.

10

Surgery or other invasive procedure performed on the wrong site. Surgery or other invasive procedure performed on the wrong patient. Wrong surgical or other invasive procedure performed on a patient. Unintended retention of a foreign object in a patient after surgery or other invasive procedure.

5.

Intraoperative or immediately postoperative/postprocedure death in an American Society of Anesthesiologists class I patient.

Management of antiplatelet therapy in the setting of coronary disease and stents continues to evolve. Both the risk of stent thrombosis and the risk of bleeding must be assessed. In this case, the risk of bleeding is low (it would be higher with a mastectomy with reconstruction; table 20.1) and the risk of thrombosis is low (table 20.2). Discontinuing the clopidogrel and continuing the aspirin is reasonable.

Timeouts were widely adopted as a method to prevent SREs; however, not performing a timeout does not constitute an SRE. Site marking is highly recommended, especially when there is laterality or level. In many institutions, site marking is not required for an exploratory laparotomy. Although a deep surgical site infection is a known complication of colorectal procedures, it does not fall into any of the surgical or invasive procedure SREs.

Management of dual antiplatelet therapy in patients undergoing surgery is a balance between risk of bleeding and thrombosis (table 20.3). Stopping these agents places the patient at risk for major adverse cardiac events, while continuing one or both agents is a risk for major bleeding. Thrombotic risk appears to be time dependent. If elective surgery can be delayed to reduce the risk, it is a viable option. If surgery cannot be delayed and bleeding risk is high, continuing aspirin and holding the clopidogrel is suggested. Restarting clopidogrel should occur as soon as possible and preferably within 72 hours.

20. A 64-year-old woman has a 2-cm area of ductal carcinoma in situ in her left breast. She has a history of having had 4 bare metal stents placed 2 years ago during an acute coronary event. She has had no chest pain. She takes clopidogrel and a baby aspirin every day. In preparation for her partial mastectomy, she should A. B. C. D. E.

stop both the aspirin and the clopidogrel. keep taking both the aspirin and the clopidogrel. stop the aspirin and keep taking the clopidogrel. stop the clopidogrel and keep taking the aspirin. stop both the aspirin and clopidogrel and start enoxaparin.

Answer:

D

Low Risk

Intermediate Risk

High Risk

General, orthopedic, and urologic surgeries Hernioplasty, plastic surgery of Incisional hernias, cholecystectomy, appendectomy, colectomy, gastric resection, intestinal resection, breast surgery, hand surgery, arthroscopy, cystoscopy and ureteroscopy

Hemorrhoidectomy, splenectomy, gastrectomy, bariatric surgery, rectal resection, thyroidectomy, prosthetic shoulder, knee, foot and major spine surgery, prostate biopsy, orchiectomy

Hepatic resection, duodenocefalopancreasectomy, hip, major pelvic and proximal femur fracture surgery, nephrectomy, cystectomy, TURP, TURBT, prostatectomy

Carotid endarterectomy, bypass or endarterectomy of lower extremity, EVAR, TEVAR, limb amputations

Open abdominal aorta surgery

Open thoracic and thoracoabdominal surgery

Cardiac surgery

Mini-thoracotomy, TAVR (apical approach), OPCAB, CABG, valve replacement

Reintervention, endocarditis, CABG in PCI failure, aortic dissections

Vascular surgery

CABG = coronary artery bypass graft; EVAR = endovascular aortic aneurysm repair; OPCAB = off-pump coronary artery bypass; PCI = percutaneous coronary intervention; TAVR = transcatheter aortic valve replacement; TEVAR = thoracic endovascular aortic aneurysm repair; TURBT = transurethral resection of bladder tumor; TURP = transurethral resection of prostate. Table 20.1. Determination of hemorrhagic risk of noncardiac and cardiac surgeries.

11

low Risk >4 weeks after PCI with POBA

Intermediate Risk (1%-5%)* High Risk (>5%) * >2 weeks and ≤ weeks after PCI with POBA <2 weeks after PCI with POBA

>6 months after PCI with BMS

>1 month and months after PCI with BMS

≤1 month after PCI with BMS

>12 months after PCI with DES

>6 months and ≤12 months after PCI with DES

≤6 months after PCI with DES

>12 months after complex PCI with DES (long stents, multiple stents, overlapping, small vessels, bifurcations, left main, last remaining vessel)

≤12 months after complex PCI with DES

≤6 months after PCI for Ml Previous ST *30-day ischemic event rates of cardiovascular death and Ml. BMS = bare-metal stent(s); DES = drug-eluting stent(s); Ml = myocardial infarction; PCI = percutaneous coronary intervention; POBA - plain old balloon angioplasty ; ST « stent thrombosis.

Table 20.2. Determination of thrombotic risk.

Thrombotic Risk

Hemorrhagic Risk Low risk

Intermediate risk

High risk

Low Risk Intermediate Risk High Risk Continue ASA; discontinue Postpone elective surgery. Postpone elective surgery. P2Y12 receptor Inhibitor; resume If surgery nondeferrable: If surgery nondeferrable: continue within 24-72 h with a loading continue ASA; discontinue ASA and dose P2Y12 receptor inhibitor; resume P2Y12 receptor Inhibitor within 24-72 h with a loading perioperatively dose Continue ASA; discontinue Postpone elective surgery. P2Y12 receptor Inhibitor; r If surgery nondeferrable: Postpone elective surgery; if surgery esume within 24-72 h with a continue ASA; discontinue nondeferrable: continue ASA; loading dose P2Y1Z receptor inhibitor; resume discontinue P2Y12 receptor within 24-72 h with a loading inhibitor, resume within 24-72 h dose with a loading dose; consider bridging with shortacting IV APT Continue ASA; discontinue Postpone elective surgery. Postpone elective surgery, if surgery P2YIZ receptor Inhibitor; resume If surgery nondeferrable: nondeferrable: continue ASA; within 24-72 h with a loading continue ASA; discontinue discontinue P2Y12 receptor dose P2Y1Z receptor inhibitor, resume inhibitor; resume within 24-72 h within 24-72 h with a loading with a loading dose; consider dose bridging with short-acting IV APT

APT = antiplatelet therapy; ASA = aspirin; IV = intravenous. Table 20.3. Perioperative management of DAPT.

12

21. Which of the following is associated with an increased rate of surgical site infection in patients undergoing elective ventral hernia repair with class 1 wounds? A. B. C. D. E.

Answer:

E

Nearly half of surgical site infections (SSIs) are preventable. In this era of public reporting and decreasing reimbursement for hospital acquired infections, evidence-based strategies must be applied to surgical patients. The Centers for Disease Control and Prevention Guidelines for the Prevention of Surgical Site Infection (2017) provided new and updated evidence-based recommendations for the prevention of SSL Prehospital recommendations are shown in table 21.1. Inhospital recommendations are shown in table 21.2

Hyperthermia Hypoglycemia Facility laundering of scrubs Clipping hair around surgical site Mechanical bowel prep

Guideline 1.1. Preoperative bathing

Intervention Routine preoperative bathing with chlorhexidine (when not part of a decolonization protocol or preoperative bundle) decreases skin surface pathogen concentrations but has not been shown to reduce SSI.

1.2. Smoking cessation

Smoking cessation 4 to 6 weeks before surgery reduces SSI and Is recommended for all current smokers, especially those undergoing procedures with implanted materials. There is no literature to support cessation of marijuana and electronic cigarette use to prevent SSI, but cessation is recommended before surgery based on expert consensus. American College of Surgeons patient education materials support the use of nicotine lozenges, nicotine gum, and medication to aid in smoking cessation.

1.3. Glucose control

Optimal blood glucose control should be encouraged for all diabetic patients; however, there is no evidence that improved Hgb A1C decreases SSI risk.

1.4. MRSA screening

Decision about whether or not to Implement global Staphylococcus aureus screening and decolonization protocols should depend on baseline SSI and MRSA rates. Clinical practice guidelines from the American Society of Health-System Pharmacists recommend screening and nasal mupirocin decolonization for S aureus-colonized patients before total joint replacement and cardiac procedures. MRSA bundles (screening, decolonization, contact precautions, hand hygiene) are highly effective if adhered to, otherwise there is no benefit. No standard decolonization protocol supported by literature; consider nasal mupirocin alone vs nasal mupirocin plus chlorhexidine gluconate bathing. Decolonization protocols should be completed close to date of surgery to be effective. Vancomycin should not be administered as prophylaxis to MRSA-negative patients.

Table 21.1. Prehospital interventions. Guideline

Intervention

2.1. Glucose control

Hyperglycemia in the immediate preoperative period Is associated with an increased risk of SSI. Target perioperative blood glucose should be between 110 to 150 mg/dL in all patients, regardless of diabetic status, except in cardiac surgery patients where the target perioperative blood glucose is <180 mg/dL. Target blood glucose rates <110 mg/dL have been tied to adverse outcomes and increased episodes of hypoglycemia and do not decrease SSI risk.

22. Hair removal

Hair removal should be avoided unless hair interferes with surgery. If hair removal is necessary, clippers should be used instead of a razor.

2.3. Skin preparation

Alcohol-containing preparation should be used unless contraindication exists (eg fire hazard, surfaces involving mucosa, cornea, or ear). No dear superior agent (chlorhexidine vs iodine) when combined with alcohol. If alcohol cannot be induced in the preparation, chlorhexidine should be used instead of iodine unless contraindications exist

2.4. Surgical hand scrub

Use of a waterless chlorhexidine scrub is as effective as traditional water scrub and requires less time, but there is no superior agent If used according to manufacturer instructions.

SSI, surgical site infection. Table 21.2a. Hospital interventions.

13

Guideline

Intervention

2.5. Surgical attire

There is limited evidence to support recommendations on surgical attire. Joint Commission and Association of Perioperative Registered Nurses policies support facility scrub laundering and the use of disposable bouffant hats. American College of Surgeons guidelines support the use of a skull cap if minimal hair is exposed, removing or covering all jewelry on the head and neck, and professional attire when outside the operating room (no scrubs or clean scrubs covered with a white coat).

2.6. Antibiotic prophylaxis

Administer prophylactic antibiotics only when Indicated. Choice of prophylactic antibiotic should be dictated by the procedure and pathogens most likely to cause SSI. Prophylactic antibiotic should be administered within 1 hour before incision or within 2 hours for vancomycin or fluoroquinolones. Prophylactic antibiotic dosing should be weight-adjusted. Re-dose antibiotics to maintain adequate tissue levels based on agent half-life or for every 1,500 mL blood loss There is no evidence that prophylactic antibiotic administration after incision closure decreases SSI risk; prophylactic antibiotics should be discontinued at time of incision closure (exceptions include implant-based breast reconstruction, joint arthroplasty, and cardiac procedures where optimal duration of antibiotic therapy remains unknown).

2.7. Intraoperative normothermia

Maintain intraoperative normothermia to reduce SSI risk. Preoperative warming is recommended for all cases, and intraoperative warming methods should be employed for all but short, dean cases.

2.8. Wound protectors

Use of an impervious plastic wound protector can prevent SSI in open abdominal surgery. Evidence is strongest for elective colorectal and biliary tract procedures.

2.9. Antibiotic sutures

Triclosan antibacterial suture use is recommended for wound closure in dean and deancontaminated abdominal cases when available.

2.10. Gloves

The use of double gloves is recommended. Changing gloves before closure in colorectal cases is recommended, however, rescrubbing before closure in colorectal cases is not recommended.

2.11. Instruments 2.12. Wound closure

The use of new instruments for closure in colorectal cases is recommended. No high-quality evidence about delayed primary closure vs primary closure and SSI for contaminated and dirty incisions. Purse-string closure of stoma sites recommended over primary closure.

2.13. Topical antibiotics

Topical antibiotics can reduce SSI for specific cases, including spine surgery, total joint arthroplasty, and cataract surgery, but there is insufficient evidence to recommend routine use at this time.

2.14. Supplemental oxygen

The administration of supplemental oxygen (80%) is recommended in the immediate postoperative period after surgery performed under general anesthesia.

2.15. Wound care

There is no evidence in the literature that timing of dressing removal increases SSI risk. Early showering (12 hours postoperative) does not increase the risk of SSI. Use of wound vacuum therapy over stapled skin can reduce SSI In open colorectal (abdominal Incision) and vascular (groin incision) cases. Mupirocin topic antibiotic application can decrease SSI compared with a standard dressing. Daily wound probing can decrease SSI in contaminated wounds.

SSI, surgical site infection. Table 21.2b. Hospital interventions (continued).

14

Additional prophylactic antimicrobial agent doses are not necessary after closure, even in the presence of a drain for clean and dean-contaminated cases. Topical antimicrobial agents in the surgical incision should not be used.

protein kinases BRAF or MEK. About half of all melanoma possess such mutations. Anti-BRAF drugs include dabrafenib and vemurafenib. Anti-MEK drugs include trametinib and binimetinib. These agents are given orally and work rapidly, but they are limited by rapid drug resistance and toxidties, including skin rashes and secondary skin cancers. Giving BRAF and MEK inhibitors together seems to improve their efficacy, and the US Food and Drug Administration has approved these agents alone and in combination.

In an analysis of a large hernia registry, for those patients with a dean wound (e.g., no skin breakdown, strangulation), a preoperative bowel preparation was associated with a higher rate of SSIs. Hair removal should be avoided unless it interferes with the procedure. Hair removal, if needed, should be done with clippers rather than a razor because the latter is associated with higher rates of SSIs. Although the Joint Commission and the Association of Perioperative Registered Nurses support facility scrub laundering, data are insufficient to associate this with decreased SSI rates.

The second type of newer agents are checkpoint inhibitors, monoclonal antibodies that block the checkpoint receptors that are present on the surface of cytotoxic T cells and normally serve to downregulate these cells and induce natural apoptosis. Many tumors can ligate these receptors, "turning off" antigen-specific T cells important in tumor killing. Thus, agents that block checkpoint receptors or their ligands are immunotherapies that serve to augment antitumor T cell response. Unlike MAP kinase inhibitors/ these drugs do not require a specific tumor mutation to be effective, because all T cells express these receptors. The 2 main checkpoints targeted to date are cytotoxic Tlymphocyte associated antigen-4 (CTLA-4) and the PD-1 receptor. Examples of anti-CTLA-4 drugs include ipilimumab and tremelimumab, and examples of anti-PD-1 agents include nivolumab and pembrolizumab. These agents are given intravenously, and autoimmune toxidties are uncommon but serious, including colitis and endocrinopathies. Like MAP kinase inhibitors, the US Food and Drug Administration approved these agents alone or in combination. Unlike MAP kinase inhibitors, responses can be slower (including a potential period of "pseudoprogression" in which tumors appear to enlarge due to swelling) but more durable.

Items 22-25 Each lettered response may be selected once, more than once, or not at all. A. B. C. D. E.

Pembrolizumab Ipilimumab Dabrafenib Trametinib Imatinib

22. Anti-CTLA-4 23. Anti-PD-1 24. Anti-MEK 25. Anti-BRAF

Answer:

B, A, D, C

One due to remembering and understanding biologic agents is to understand how they are named. Monoclonal antibodies (MoAbs) end with the suffix "-mab." Chimeric humanmouse MoAbs end with "-ximab" (e.g., rituximab), humanized mouse MoAbs end with "-zumab" (e.g., bevadzumab), and human MoAbs end with "-mumab" (e.g., ipilimumab).

Newer drugs have completely changed the landscape of treating advanced melanoma. General surgeons should be aware of these newer agents for 3 reasons. First, these drugs can cause side effects that may require surgical intervention, such as skin lesions and colitis that occasionally progresses to bowel perforation. Second, these agents may significantly downstage unresectable tumors, making the tumors potentially resectable. Surgeons should work with oncologists to plan the potential resection of tumors treated with these agents in a neoadjuvant fashion. Finally, indications for these drugs, especially anti-programmed death-1 (PD-1) agents, are being extended from melanoma to a variety of cancers, including non-small cell lung, advanced urothelial (bladder and renal), advanced Merkel cell, and head and neck cancers, as well as relapsed lymphomas, just to name a few. Thus, they are rapidly affecting the overall care of multiple cancers.

Small molecule inhibitors end with the suffix "-ib." Tyrosine kinase inhibitors end with the suffix "-tinib" (e.g., imatinib), proteasome inhibitors end with "-zomib" (e.g., bortezomib), and cyclin-dependent kinase inhibitors end with "-dclib" (e.g., selidclib). MEK is part of the tyrosine kinase pathway, hence "trametinib." Also, drug names sometimes indude the name of the target (e.g., dabrafenib contains "BRAF").

26. The preoperative removal of hair from surgical sites

These new agents are of 2 general types. First, selective inhibition of the mitogen-activated protein (MAP) kinase pathway can be done by blocking mutated forms of the

A. is recommended by the Centers for Disease Control and Prevention.

15

B. C. D. E.

does not reduce the incidence of surgical site infections. has not been studied in prospective trials. should be done by wet shaving 2 days before operation. should not involve depilatory creams.

Answer:

The STOP-IT trial showed no untoward adverse events with the intervention arm. Rate of recurrent infection, surgical site infection, and length of stay were no different. In subsequent subgroup analysis, there was no difference for those with complicated intra-abdominal infection. Further, neither diabetes nor obesity led to poorer outcome in the intervention arm.

B

Antibiotics are important in the control of sepsis through the event of source control. Once source control has occurred, there is no proven benefit to antibiotic therapy beyond 4 days. Thus, risk of resistance and complications of antibiotic use can be limited. Further study is needed to determine whether antibiotic duration of less than 4 days is appropriate.

The impact on surgical site infection (SSI) rates of removing hair from operative sites remains unknown. Some—but not all—retrospective studies suggest increased rates of SSIs after hair removal. The Centers for Disease Control and Prevention recommend against it, stating that hair should be removed only when it will interfere with the operation, preferably with electric clippers (not wet shaving) and immediately before surgery. Clipping earlier than that is generally not recommended. Some advisory bodies, such as the Norwegian Center for Health Technologies Assessment, advise that earlier hair removal should be done only with depilatory creams. A recent controlled trial of more than 1500 patients randomized to hair removal or no hair removal showed no difference in SSIs.

28. A 68-year-old man presents for surgical consultation for sigmoid colon cancer. He has chronic obstructive pulmonary disorder and hypertension, and he is an American Society of Anesthesiologists class III risk. Which of the following statements is true regarding prehabilitation for this patient? A. B. C. D.

There is a small increase in cardiac events. Hospital length of stay is decreased. Intensive care unit length of stay is decreased. Inspiratory muscle training improves preoperative pulmonary function. E. Inspiratory muscle training reduces intraoperative complications.

27. A 67-year-old man presents with 3 days of worsening left lower quadrant pain. He has a known history of diverticulitis. He has insulin-dependent diabetes and a BMI of 34. On presentation, his white blood cell count is 13,500/mm3 (3600-11,200/mm3) and his temperature is 38.3°C. CT scan of the abdomen shows a 5-cm simple pericolonic abscess. A CT-guided percutaneous drain is appropriately placed. How long should broad-spectrum antibiotics be administered after source control? A. B. C. D. E.

Answer:

Interest in the use of prehabilitation to reduce postoperative complications in the elderly is increasing. Unfortunately, data on the topic are limited. Prehabilitation is more than incentive spirometry, smoking cessation, and routine ambulation. It includes inspiratory muscle training, exercises specifically focused on strengthening inspiratory muscles, and aerobic exercise. Current evidence suggests that postoperative complications are reduced with structured prehabilitation, but there is no effect on intraoperative complications. Concern has been raised that the effort required for prehabilitation might increase the risk of a cardiac event with this intervention, but this is not the case. The specific benefit to prehabilitation is unknown because there are no measurable preoperative pulmonary assessments. Intensive care unit length of stay is reduced, but not hospital length of stay.

Antibiotics are not needed. 24 hours 4 days 7 days 14 days

Answer:

C

C

The STOP-IT trial was an open-label, randomized prospective trial that compared limited duration antibiotics for intra-abdominal infection with adequate source control to antibiotic continuance until resolution of systemic inflammatory response syndrome abnormalities (SIRS). Patients were randomized to an intervention arm with infectious source control followed by 4 days of appropriate antibiotic therapy or resolution of SIRS abnormalities, defined as a temperature lower than 38°C for 1 day, white blood cell count less than 11,000/mm3, and the ability to consume at least half of a regular diet without adverse effect. Appendiceal disease was allowed as only a limited component of the study.

16

29. A 67-year-old woman presents with clinical T2, Nl, MO sigmoid colon cancer. She is scheduled for a laparoscopic sigmoid colectomy. She is placed into an enhanced recovery after surgery protocol. As part of this protocol, she is given 100 g of an oral carbohydrate load in 800 mL of liquid 2 hours before surgery. Preoperative oral carbohydrate loading will A. B. C. D. E.

transfusion in the clopidogrel group but no higher risk of perioperative complications. One study showed a higher rate of cardiac complications in the group not on clopidogrel. This finding challenges the notion of current standard practice. Continuance of clopidogrel through the perioperative period appears safe for most general surgical operations, including colectomy. Then? is no need to hold clopidogrel preoperatively, and there is no need to administer platelets intraoperatively.

reduce the risk of infectious complications. induce hyperglycemia. enhance gastric emptying. reduce insulin resistance. stimulate feelings of hunger.

Answer:

D

Enhanced recovery after surgery protocols are pre-, intra-, and postoperative bundles of care intended to improve surgical outcomes. One component of the preoperative bundle is oral carbohydrate loading. Traditional fasting before surgery leads to depletion of glycogen stores in the liver, insulin resistance, and gluconeogenesis. Multiple studies examined the potential benefit of shorter durations of nothing by mouth preoperatively and the potential benefits of carbohydrate loading. The primary benefit of carbohydrate loading is a reduction in insulin resistance caused by the surgical procedure and postoperative starvation. Perceived benefits to patients are that they are less likely to be thirsty, hungry, or anxious. There is neither a reduction nor increase in risk of infection. Gastric emptying is not affected, and patients do not become hyperglycemic.

30. A 72-year-old woman presents with a cecal cancer found on screening colonoscopy. She takes clopidogrel for a previous stroke. What is the appropriate perioperative management of her anticoagulation in anticipation of a laparoscopic right hemicolectomy? A. B. C. D.

Hold clopidogrel for 10 days preoperatively Hold clopidogrel for 5 days preoperatively Hold clopidogrel for 5 days and begin enoxaparin Continue clopidogrel and administer platelets intraoperatively E. Continue clopidogrel without interruption

Answer:

E

The manufacturer of clopidogrel recommends holding the drug for 5 days before major surgery. Given that the half-life of platelets is 7 to 8 days, this recommendation seems appropriate. However, there are increasing data in the safe use of clopidogrel through the perioperative period. Elective vascular surgery is routinely done with patients on clopidogrel and aspirin. Two recent studies addressed the use of clopidogrel in both elective and urgent general surgical procedures. In both studies, there was a slightly higher rate of

17

Perioperative Care Part II

Items 5-10 Each lettered response may be selected once, more than once, or not at all.

ITEMS 1-30 For each question, select the best possible response. Items 1-4 Each lettered response may be selected once, more than once, or not at all. A. B. C. D. E.

A. B. C. D. E.

Apixaban Dabigatran Argatroban Rivaroxaban Idarucizumab

Cefoxitin Cefazolin Cefepime No antibiotics Vancomycin

5. Prophylactic antibiotic for a fundoplication in a high-risk patient 6. Prophylactic antibiotic for a hernia repair

1. Parenteral direct thrombin inhibitor 2. Oral direct thrombin inhibitor metabolized by the kidneys

7. Prophylactic antibiotic for a laparoscopic, low-risk, elective biliary tract procedure

3. Direct Factor Xa inhibitor recommended for patients with low creatinine clearance

8. Prophylactic antibiotic for a laparoscopic or open uncomplicated appendectomy

4. Oral Factor Xa inhibitor to avoid in patients with a low creatinine clearance

9. Prophylactic antibiotic for a noncardiac thoracic procedure 10. Prophylactic antibiotic ventriculoperitoneal shunt insertion

ANSWERS:

for

an

elective

C, B, A, D

Direct thrombin inhibitors (DTIs) and Direct Factor Xa inhibitors are newer medications available to prevent and treat thromboembolic disease. Examples of parenteral direct thrombin inhibitors include bivalirudin, argatroban, and desirudin. The only available oral direct thrombin inhibitor is dabigatran. There are no parenteral Factor Xa inhibitors. Examples of oral Factor Xa inhibitors include rivaroxaban, apixaban, edoxaban, and betrixaban. Note that all of the medications in this category end in "-xaban."

ANSWER:

Antimicrobial prophylaxis may be beneficial in cleancontaminated or contaminated surgical procedures associated with a high rate of infection. Clinical practice guidelines for antimicrobial prophylaxis and surgery were developed using evidence-based methodologies to develop a standardized approach to the rational, safe, and effective use of antimicrobial agents to prevent surgical site infections (SSIs). The guidelines provide many recommendations appropriate for both adults and children. Additional recommendations include weight-based dosing in obese patients, and the need for repeat dosing when procedures exceed the half-life of the antibiotic used. The optimal time for administering a preoperative dose is within 60 minutes before the incision is made.

Dabigatran is an oral direct thrombin inhibitor that should be used with caution in patients with renal insufficiency because it is metabolized by the kidneys. Rivaroxaban is a Factor Xa inhibitor that is also metabolized by the kidney and is not recommended for patients with low glomerular filtration rates (creatinine clearance <30 mL/minute; 88-137 mL/minute). Apixaban is a Factor Xa inhibitor that can be given to patients with renal insufficiency because it has hepatic clearance. Idarudzumab is a monoclonal antibody that binds dabigatran, effectively reversing its anticoagulate activity. Direct Thrombin Inhibitor

Factor Xa Inhibitor

Parenteral Argatroban Bivalirudin Desirudin

Parenteral NONE

Oral Dabigatran

B, B, D, A, B, B

The principles of using an antimicrobial agent for surgical prophylaxis include effectiveness against expected bacterial pathogens, appropriate dose to ensure adequate serum levels during wound contamination, short duration (usually 24 hours or less), low toxicity profile, proven ability to reduce morbidity and mortality of a SSI, cost effectiveness, and a low risk of adverse effects. Agents that are approved by the US Food and Drug Administration for use in surgical antimicrobial prophylaxis include cefazolin, cefuroxime, cefoxitin, cefotetan, ertapenem, and vancomycin. Vancomycin is an alternative agent in patients with a betalactam allergy. For patients known to be colonized with methicillin-resistant Staphylococcus aureus, a single preoperative dose of vancomycin can be added to the recommended agent(s). Cefepime is not recommended for perioperative prophylaxis.

Oral Rivaroxaban Apixaban Edoxaban Betrixaban

Current as of December 2018. Table 1-4.1. Direct thrombin inhibitors (DTIs) and Direct Factor Xa inhibitors are newer medications available to prevent and treat thromboembolic disease.

19

A clean case is defined as one with no inflammation and no entry into the respiratory, alimentary, genital, or uninfected urinary tract. Fundoplication and hernia repair are both considered clean cases. The stomach is an effective barrier to bacterial colonization, partially related to its acidity. Treatment with agents that increase gastric pH are associated with an increase in the concentration of gastric organisms and the potential to increase postoperative infection rate. Prophylaxis for procedures without entry into the gastrointestinal tract is indicated for only high-risk patients, and cefazolin is an appropriate agent. Examples of high-risk patients include those with achlorhydria, morbid obesity, or cancer.

E. Hold anticoagulation for 5 days before the procedure with heparin bridge; restart anticoagulation 48 hours after the procedure; continue heparin for 5 days after the procedure.

ANSWER:

C

Direct oral anticoagulation (DOAC) use is increasing. Previously known as novel oral anticoagulation, DOAC is noted for rapid onset of action and predictable effect on the coagulation system without the need for monitoring. Many patients who use DOAC will require surgery and it is important to understand these drugs' pharmacokinetics, indications for use, and paths of elimination.

Antimicrobial prophylaxis may be beneficial in clean surgical procedures associated with a high risk of infection, such as prosthetic implants, heart valves, or ventriculoperitoneal shunt insertion, where the consequences of infection would be devastating, even if the risk of infection is extremely low. Cefazolin is appropriate prophylaxis for these cases.

DOAC have predictable pharmacokinetics. Dabigatran has a half-life of 12 to 17 hours. Rivaroxaban has a half-life of 6 to 9 hours, and apixaban has a half-life of 12 hours. Half-life is increased in patients with decreased creatinine clearance. For surgery, DOAC is held for 2 to 3 half-lives for minor surgery and 4 to 5 half-lives for major surgery.

Antimicrobial prophylaxis is warranted for most cleancontaminated procedures. The definition of a cleancontaminated wound allows the respiratory, alimentary, or genitourinary tract to be entered under controlled circumstances. Operations involving the biliary tract and appendix are included in this category. Elective, low-risk laparoscopic biliary tract procedures do not require prophylaxis, but open or elective, high-risk procedures do require prophylaxis. The biliary tract is usually sterile, and antimicrobial prophylaxis confers no benefit. All patients with a suspected biliary tract infection should receive preoperative antimicrobial prophylaxis.

CHADS2 scoring is used to decide which patients with atrial fibrillation need to have bridging of their therapy. The CHADS2 score is calculated by adding up points from the presence of congestive heart failure, hypertension, age older than 75 years, diabetes mellitus, and history of stroke or transient ischemic attack. A CHADS2 score of greater than 2 requires bridging. In the scenario presented, the patient has a CHADS2 score of 1 based on hypertension. Thus, he will not need bridging. His creatinine clearance is normal. He is having major surgery. Thus, his apixaban should be held for 4 to 5 half-lives or 2 days before the procedure.

Cefoxitin is a recommended agent for prophylaxis in uncomplicated appendicitis. If selected, cefazolin should be combined with metronidazole. The typical organisms reported from surgical site infections in patients undergoing thoracic procedures are Staphylococcus aureus and Staphylococcus epidermidis. Cefazolin is commonly used with SSI rates of 0.42 to 4% compared with up to 14% when no prophylaxis is used.

It is safe, barring any perioperative bleeding, to restart anticoagulation 24 hours after a major procedure.

12. A 57-year-old woman was diagnosed with adenocarcinoma of the right colon and is scheduled to undergo a laparoscopic right hemicolectomy. She currently takes 5 mg of prednisone each morning for rheumatoid arthritis. Appropriate perioperative management of her steroids would be

11. A 72-year-old man presents with rectal cancer and is scheduled for an open low anterior resection. He is on apixaban for atrial fibrillation. His only other medical issue is hypertension. What is appropriate management of his anticoagulation around the procedure?

A. hold prednisone the day before surgery, then restart the day of surgery. B. continue prednisone at her normal dose perioperatively. C. increase prednisone to 10 mg/day for 5 days after surgery. D. give 100 mg of hydrocortisone the day of surgery, then restart normal prednisone dose.

A. Continue anticoagulation through the procedure. B. Hold anticoagulation 24 hours before the procedure; restart anticoagulation the evening of the procedure. C. Hold anticoagulation 48 hours before the procedure; restart anticoagulation 24 hours after the procedure. D. Hold anticoagulation for 5 days before the procedure; do not bridge with heparin; restart anticoagulation 24 hours after the procedure.

20

E. give 100 mg of hydrocortisone the day of surgery, then 50 mg intravenously every 8 hours for 3 days after, then restart normal prednisone.

ANSWER:

important in the synthesis and cross-linking of collagen. Zinc is involved in the enzymatic activity of RNA and DNA synthesis and collagenase function, and deficiencies can lead to delays in epithelialization. If the vitamin and mineral stores in a patient are questionable, such as in a gastric bypass patient, the patient can be started on a multivitamin supplement.

B

Chronic glucocorticosteroid administration can affect the hypothalamic-pituitary-adrenal axis, leading to adrenal atrophy and a decreased capability to produce cortisol. This decreased production of cortisol during stress can lead to hypotension. The question is what amount of cortisol is needed during surgery to avoid this complication.

Optimizing glucose control has an impact on wound healing and wound infection. Increased glucose levels, as measured by hemoglobin A1C, lead to decreased neutrophil function. This leads to a delay in the inflammatory response and disrupts the timing of the wound-healing cycle. Impaired neutrophil function can also lead to infection. Although diabetic patients can have worse microvascular disease, no change is noted in skin blood flow with improved glucose control.

A patient with normal adrenal glands will produce approximately 10 to 15 mg of cortisol/day. Under extreme physiologic stress, the adrenals produce 300 mg/day. This number drove the initial dosing recommendation of "stress dose steroids," with 100 mg of hydrocortisone being administered 3 times a day to any patient who was on even a small dose of steroids.

Smoking causes microvascular vasoconstriction. This can affect blood flow to the wound. Patients who are active smokers have an increase in wound infection rates in addition to an increase in skin or muscle flap necrosis. If a patient quits smoking before surgery, blood flow increases. This increase in blood flow leads to a decrease in wound infection rate and skin flap necrosis.

Patients who undergo laparotomy secrete an average of 50 mg of cortisol. This amount may be lower in laparoscopy. Recent studies showed that patients undergoing surgery do not need stress dose steroids. The patient should be able to continue on her maintenance dosing schedule. Appropriate conversion to intravenous dosage may be required. In the scenario given, there is no indication to increase the amount of steroids the patient is receiving. There is also no indication to decrease the amount of steroids a patient is receiving if the dose is adequately managing the condition for which the patient is taking steroids.

Although hyperbaric oxygen is used to improve healing, there is no evidence that preoperative hyperbaric oxygen is of use in preventing wound complications. There is also no evidence that a high-protein diet affects wound healing if initiated before surgery.

Items 14-17 Each lettered response may be selected once, more than once, or not at all.

13. A 62-year-old man with a complex abdominal wall hernia is scheduled to undergo an abdominal wall reconstruction that will require skin flaps for skin closure. What will decrease his incidence of skin flap necrosis? A. B. C. D. E.

A. B. C. D. E.

Preoperative hyperbaric oxygen High-protein diet Zinc supplementation Smoking cessation Glucose control optimization

No evidence exists Evidence exists but is inconclusive Evidence exists and favors use Evidence exists and does not favor use Level I evidence

14. Cleansing of the operation site with chlorhexidinealcohol. 15. Wearing surgical scrubs. 16. Covering the wound with a dressing.

ANSWER:

D

17. Disinfecting the intravenous catheter hub.

The process of wound healing is a combination of patient factors, wound factors, and surgical technique. All point to the same aspects of wound healing: tissue blood flow, bacteria, proper tissue handling, and immunologic response.

ANSWER:

C, B, B, E

Surgical site infection is the second most common healthcareassociated infection (after respiratory infection) and results in a significant increase in healthcare costs. Guidelines were developed to reduce the incidence of surgical site infection, but many of these guidelines lack evidence or consensus.

In the preoperative period, patients can improve the chance that their wounds will heal successfully. With regard to wound healing, the patient needs adequate stores of woundrelated vitamins and minerals. Vitamins A and C are

21

Evidence-based medicine uses the best available evidence to make clinical decisions. Levels of evidence reflect the quality of the existing studies and supportive data. Level I evidence is generally based on randomized controlled trials and the highest quality of evidence with little doubt as to effectiveness. For example, the use of antimicrobial prophylaxis in dean-contaminated and contaminated surgical cases within 60 minutes of incision time is supported by good evidence and is embraced by most surgeons. Similarly, "scrub the hub" or intravenous catheter hub disinfection before injection has good evidence to support it, including randomized controlled clinical trials involving multiple operating rooms.

recommendation is to minimize the number of staff turnovers, which is an independent variable associated with increased surgical site infections. A recent review of the last decade of published literature evaluating alternatives to antibiotics in the prevention of surgical site infection highlighted perioperative practices. Tissue adhesives are easier and quicker to use than suturing a wound dosed. Although there seems to be no significant difference in rates of surgical site infection between the 2 wound closure techniques, more wound dehiscence tends to occur with tissue adhesives. A high oxygen tension seems to be important for wound healing and increases the activity of neutrophils in tissues, which may be important for resisting infection. However, randomized controlled trials and a metaanalysis examining high perioperative oxygen concentrations were contradictory for prevention of surgical site infections across all surgical procedures. A survey study through the Americas Hernia Society Quality Collaborative examining 6210 hernia repairs done by 68 surgeons and evaluating 6 different types of disposable or cloth operating room hats found no difference in the incidence of surgical site infection occurrences based on hat type. A more recent study suggested that cloth skull caps are associated with less risk of surgical site infection than disposable caps. Several papers and a meta-analysis including 17,000 patients suggested that preadmission showering with an antiseptic agent is inconclusive for the prevention of surgical site infection.

Some practices, such as wearing surgical attire in the operating room, have little or no supportive evidence through clinical trials but are guided by the concepts of Pasteur, common sense, and conventional wisdom. There are no published studies evaluating the efficacy of wearing surgical gowns. There is little evidence that wearing surgical scrubs contributes to a reduction in surgical site infection. Wearing sterile surgical gloves is accepted practice but not of proven benefit. It is doubtful that a randomized controlled trial will ever be done comparing street attire to surgical attire in the operating room. Other practices, such as the choice of agent used to cleanse the operative site before incision and wound coverage after completion of a surgical case, have been studied, and evidence favors certain practices. Results of a meta-analysis and Cochrane review both favored chlorhexidine-alcohol over aqueous povidone-iodine in general surgery cases. More recent studies showed no difference, but interpretation was hampered because variable antiseptics and concentrations were used. There is no compelling evidence that protection of a surgical wound with a dressing after case completion is of significant benefit. Although some guidelines recommend covering the surgical wound with a dressing for the first 48 hours after surgery, this recommendation is not supported by current literature. Surgical site infection rate is reported to be the same when the wound is left uncovered, is covered for 48 hours, or is covered for less than 48 hours.

19. A 73-year-old woman with a history of chronic nonvalvular atrial fibrillation treated with dabigatran is diagnosed with a 3-cm bleeding gastric adenocarcinoma. Her medical history includes noninsulin-dependent diabetes mellitus. She wishes to undergo partial gastrectomy for her cancer. Which of the following recommendations should you make? A. Convert from dabigatran to rivaroxaban preoperatively B. Convert to therapeutic heparin bridging anticoagulation before surgery C. Stop dabigatran 2 days before surgery D. Continue dabigatran E. Convert to warfarin before surgery

18. During a single surgical case, which of the following interventions decreases surgical site infection? A. B. C. D. E.

Decreased operating room personnel turnover Wound closure with tissue adhesive versus sutures Use of high FiO2 intraoperatively Disposable bouffant hats Preoperative chlorhexidine bathing

ANSWER:

ANSWER:

C

International guidelines exist for the use of long-term prophylaxis to prevent stroke in patients with atrial fibrillation. When dabigatran therapy needs to be interrupted for surgical procedures, there is no need to bridge the patient with heparin due to the rapid offset (half-life: 12-17 hours) and rapid onset (tmax 1-3 hours) of the action of dabigatran. Stopping and starting dabigatran therapy would only result in a 2- to 4-day period when patients would be subtherapeutic regarding anticoagulation.

A

Many guidelines exist for the perioperative care of surgical patients. There is variable evidence to either support or not support the interventions listed here. The best evidence exists around traffic in the operating room during a case. The

22

In a study of 1200 patients who underwent noncardiac surgery, 6.7% experienced a primary outcome of AKI. Age, diabetes mellitus, hypertension, and American Society of Anesthesiologists status were independent predictors of AKI. Atrial fibrillation, heart failure, female sex, and anemia were not risk factors.

In the RE-LY trial, more than 4000 patients who were on dabigatran or warfarin had their anticoagulation therapy interrupted around the time of an elective surgery or procedure. Patients in this study who were bridged with heparin had significantly more major bleeds than those who were not bridged (6.5% vs 1.8%; p <.001). The risk for stroke and systemic embolism was not different in patients who were bridged versus not bridged (0.5% vs 0.3%). Somewhat surprisingly, the thromboembolism risk was higher in patients who received bridge therapy with heparin (1.2% vs 0.6%), but this difference was not statistically significant. There are no demonstrable differences between dabigatran, rivaroxaban, or apixaban with respect to the risk of stroke or systemic embolism. Apixaban is associated with a lower risk of major bleeding compared with the other 2 agents, whereas rivaroxaban is associated with an increased risk of major bleeding and intracranial bleeding compared with dabigatran. Continuing these agents through major surgery or switching dabigatran to rivaroxaban is associated with increased risk of major bleeding. For the same reason, there is no justification for switching from a direct oral anticoagulant to warfarin, a Vitamin K antagonist that has a longer half-life.

Procedure-related factors were associated with developing AKI. Reoperation had a 5-fold increased risk, and emergency surgery had a 1.5-fold increased risk. Perioperative AKI was associated with a 3- to 4-fold increased risk of cardiovascular adverse events and in-hospital mortality. Patients who reached their peak serum creatinine value more than 2 days after surgery had a comparable 30-day mortality with those patients who had their peak serum creatinine within the first 2 postoperative days. There was a dose-response relationship between mortality and severity, with a 30-day mortality of 12.3%, 19.1%, and 43.2% for AKI stages 1,2, and 3 groups, respectively. Elevated C-reactive protein is also associated with AKI. Although ensuring adequate hydration and hemodynamic status perioperatively may reduce the risk of perioperative AKI, no data show that postoperative hydration will improve a patient's survival. In a meta-analysis exploring the impact of sodium bicarbonate infusion on patients with AKI, the authors were unable to demonstrate an impact on mortality risk for patients receiving bicarbonate infusion.

20. A 72-year-old woman undergoes sigmoid resection for carcinoma. Her medical history is significant for ratecontrolled atrial fibrillation. On postoperative day 1, the patient's urine output is 0.5 mL/kg/hour while her serum creatinine doubles. Which of the following statements about her 30-day mortality risk is true based on her change in renal function?

21. A 42-year-old woman presents on postoperative day 5 after an extensive lysis of adhesions. She continues to have high bilious nasogastric tube drainage, and her abdomen remains distended, mildly tender, with no bowel sounds or evidence of peritonitis. Her vital signs and urine output are normal. Laboratory data are as follows: white blood cell count = 9800/mm3 (3600-11,200/mm3), hematocrit + 42% (37-51%), sodium = 146 mEq/L (136-145 mEq/L), potassium = 3.1 mEq/L (3.5 5.0 mEq/L), bicarbonate = 24 mmol/L (2029 mmol/L), chloride = 98 mEq/L (95-105 mEq/L). The next step in the management of her ileus is

A. The severity of her acute kidney injury correlates with her mortality risk. B Acute kidney injury within the first day after surgery increases her mortality risk. B. Postoperative hydration will improve her survival. C. Sodium bicarbonate infusion will lower her mortality risk. D. Decreased C-reactive protein levels are associated with increased risk.

ANSWER:

A. B. C. D. E.

A

In patients undergoing surgery, perioperative acute renal failure is a leading cause of morbidity and mortality, with an incidence of up to 30% of patients. A collaborative network of international experts addressed the lack of a universal definition for acute kidney injury (AKI) by establishing the Acute Dialysis Quality Initiative and devised the RIFLE (risk, injury, failure, loss of function, and end-stage kidney disease) definition and staging system.

metoclopramide. potassium chloride. erythromycin. neostigmine. alvimopan.

ANSWER:

B

Postoperative ileus is a common condition after abdominal and other types of surgery. The pathogenesis of this complication is complex and involves stress-induced release of catecholamines and cytokines that inhibit colonic motility. Ileus may also be exacerbated by anticholinergic medications, opioids, and general anesthetics, as well as fluid and electrolyte abnormalities.

AKI is classified into stages based on the degree of increase in serum creatinine.

23

Hypokalemia, frequently accompanied by hypomagnesemia, is a significant contributing factor. Potassium facilitates depolarization of smooth muscle cell membranes, causing calcium channels to open such that extracellular calcium enters the cell, triggering smooth muscle contraction. In a patient with a postoperative ileus and hypokalemia, correction with potassium chloride frequently decreases the duration of the ileus. For the ileus to be effectively treated, serum magnesium must be normal.

In the postoperative period, an important concern is the optimal dosing of replacement therapy. Measurement of a platelet count is insufficient because platelet count provides no information as to platelet activity. Factor VII and fibrinogen are not directly related to Von Willebrand disease. Von Willebrand activity level is determined by ristocetin cofactor activity, which tests the ability of vWF to clump by binding to its primary receptor, platelet glycoprotein Ib; it is an indirect measure of platelet activity. Therefore, monitoring of Factor VIII activity is the superior method for monitoring a patient with Von Willebrand disease in the postoperative period.

Metoclopramide accelerates gastric emptying and stimulates gastric, pyloric, and small intestine motility but has no effect in the colon. Metoclopramide does not shorten the duration of postoperative ileus. Erythromycin acts on motilin receptors that stimulate smooth muscle contraction. Erythromycin lacks activity in the colon and does not decrease the duration of ileus. Neostigmine is an anticholinesterase that inhibits the hydrolysis of acetylcholine. This action is not selective for muscarinic receptors, which mediate gastrointestinal motility. Although neostigmine causes muscle fasciculations, it does not improve the course of postoperative ileus.

Thromboelastography (TEG) is a viscoelastic method used to measure coagulation function. Because of the lack of shear stress essential for the activation of vWF, the standard TEG assay was not thought to be of use in von Willebrand disease. A modified TEG using ristocetin activation is under investigation in the monitoring of patients with von Willebrand disease.

Alvimopan is an oral peripherally acting mu-opioid receptor antagonist that appears to hasten postoperative gastrointestinal recovery after bowel surgery and abdominal hysterectomy. Although it appears to accelerate motility and decreases the duration of ileus, it must be started preoperatively.

23. A 52-year-old man who is heparin naive is started on subcutaneous unfractionated heparin (UFH) for postoperative deep venous thromboembolism prophylaxis. The next day, his platelet count falls from 260,000/mm3 to 102,000/ mm3 (150,000-400,000/mm3). Management of his deep vein thrombosis prophylaxis should entail which of the following? A. Discontinue UFH and start low molecular weight heparin. B. Continue UFH. C. Discontinue UFH and start fondaparinux. D. Discontinue UFH. E. Discontinue heparin and start argatroban.

22. Which of the following should be monitored postoperatively in a patient with von Willebrand disease who has undergone an elective abdominal operation? A. B. C. D. E.

Platelet count Fibrinogen level Von Willebrand Factor activity Thromboelastography Factor VIII activity

ANSWER:

ANSWER:

B

Using heparin to prevent deep venous thromboembolism/pulmonary embolism is common in the postoperative period. However, heparin-induced thrombocytopenia (HIT) is a significant complication, with potential for morbidity and mortality. Pharmaceutical heparin can be unfractionated (UFH), which consists of a heterogenous mixture of sulfated polysaccharides (MW: 8,000-24,000 Da), or low-molecular weight heparin (LMWH) with molecular weight between 2,000-8,000 Da.

E

Von Willebrand disease is the most common inherited bleeding disorder. It affects up to 1% of the population. Von Willebrand Factor (vWF) plays a fundamental role in bleeding control by connecting platelets and subendothelial structures at the site of blood vessel injury, thus leading to primary hemostasis. vWF is also a carrier protein for Factor VIII, thereby assisting in clot formation after the platelet plug has been formed.

There are 2 distinct types of HIT. This patient has HIT type I. Type I or heparin-associated thrombocytopenia is a nonimmunologic response to heparin treatment, mediated by a direct interaction between heparin and circulating platelets causing platelet clumping or sequestration. HIT type I affects up to 10% of patients, usually occurs within the first 48 to 72 hours after initiation of heparin treatment, and is characterized by a mild and transient thrombocytopenia (rarely <100,000/mm’), often returning to normal. No

Patients with von Willebrand disease report a history of easy skin and mucosal bleeding, epistaxis, easy bruising, and menorrhagia. Preoperative evaluation consists of measuring Factor VIII activity, von Willebrand activity, and ristocetin cofactor activity. Desmopressin or a concentrate of vWF and Factor VIII is used for surgical prophylaxis.

24

laboratory tests are required to diagnose HIT type I, and it is not associated with an increased risk of thrombosis. Therefore, continuation of the UFH is appropriate.

Multiple immunological assays can be used to confirm the diagnosis of HIT type IIincluding platelet function tests, platelet aggregation studies, flow cytometry, antiheparin/platelet Factor 4 titers, and enzyme-linked immunosorbent assay to detect antibodies to heparin/platelet Factor 4 complex. These studies not only require a prolonged period of time for results to return but also have many false positives and negatives. As a result, the laboratory testing should be used only in conjunction with the clinical context.

Conversely, HIT type 2, is a drug-mediated, prothrombotic condition caused by IgG antibody production against platelet Factor 4. This immunologic interaction is capable of intravascular platelet activation and devastating life- or limbthreatening thrombosis by white dot or dot composed primarily of platelets. HIT type II typically occurs 4 to 5 days after starting UFH or LMWH or after a shorter period if the patient has been previously exposed to heparin.

Points (0,1, or 2 for each of 4 categories: maximum possible score = 8) 2 >50% platelet fall to nadir≥20

1 30-50% platelet count fall (or >50% directly resulting from surgery); or nadir 10-19

0 <30% platelet fail; or nadir<10

Timinga of platelet count fall, thrombosis, or other sequelae (1st day of putative immunizing exposure to heparin = day 0)

Days 5-10 onseta (typical/ delayed onset HIT); or ≤ 1 day (with recent heparin exposure) or within 30 days (rapid-onset HIT)

Platelet count fail <4 days (unless picture of rapid-onset HIT)

Thrombosis or other sequelae (e.g., skin lesions, anaphylactoid reactions)

Proven new thrombosis; or skin necrosis (at injection site); or postintervention heparin bolus anaphylactoid reaction No explanation for platelet count fail is evident

Consistent with days 5-10 fall, but not clear (e.g., missing platelet counts); or ≤ 1 day (heparin exposure within the past 31-100 days) (rapid-onset HIT); or platelet fall after day 10 Progressive or recurrent thrombosis; or erythematous skin lesions (at injection site); or suspected thrombosis (not proven); hemofilter thrombosis Possible other cause is evident

Thrombocytopenia

Other cause for thrombocytopenia

None

Definite other cause is present

Pretest probability score: 6-8 = high; 4-5 = intermediate; 0-3 = low HIT = heparin-induced thrombocytopenia. a First day of immunizing heparin exposure considered day 0; the day the platelet count begins to fall Is considered the day of onset of thrombocytopenia (It generally takes 1-3 more days until an arbitrary threshold that defines thrombocytopenia is passed). Usually, heparin administered at or near surgery is the most immunizing situation (i.e., day 0). * Table 23.1. The 4Ts scoring system. The 4 Ts Score (table 23.1) is calculated based on the degree of thrombocytopenia, timing, evidence of thrombosis, and potential alternative causes. If the score is 0 to 3, type II HIT is low probability, with a negative predictive value of 0.998. A score of 4 to 5 points indicates intermediate probability, with a positive predictive value of 0.14. A score of 6 to 8 has a high probability of type II HIT, with a positive predictive value of 0.64.

24. Which of the following statements is true regarding opioid prescriptions after general surgical operations? A. Variability in the prescriptions of opioids after discharge is limited. B. The majority of prescribed opioids are consumed after surgery. C. Educational interventions for surgeons decrease opioid prescribing. D. Most patients after general surgical procedures require refills of their narcotics. E. Preoperative opioid use is not associated with higher readmission rates.

If there is concern about type II HIT, all heparin products should be stopped, and the patient should be started on activated Factor X inhibitors such as danaparoid or fondaparinux. Alternatively, the patient can be started on direct thrombin inhibitors such as argatroban or bivalirudin.

25

ANSWER:

symptoms. Which of the following should be included in his preoperative evaluation?

C

The opioid epidemic is partly attributed to prescription opiates commonly prescribed after surgery. The prescription of opioids after surgery varies widely across all procedures. However, in a study of 642 patients, less than 28% of prescribed pills were taken and less than 2% of refills were obtained. Educational interventions to decrease opioid prescriptions reduced prescriptions by 53% compared with the number of prescriptions before the intervention. Opioid naive surgical patients become prolonged opioid users at rates 2.6 to 3.6 times higher than patients who did not undergo surgery. Patients who use opioids before abdominopelvic surgery have 9.2% higher costs, longer lengths of stay, more complications, and higher readmissions.

A. B. C. D. E.

ANSWER:

A

The preoperative assessment of a patient undergoing an elective procedure should begin with a thorough history and physical. The practice of routine laboratory, cardiac evaluation, and radiologic evaluation is not recommended without clear indication of risk for comorbidity. Current recommendations for healthy, American Society of Anesthesiologist (ASA) class I patients undergoing minor or intermediate grade surgery is to not perform resting electrocardiograms, complete blood counts, liver panels, urinalysis, coagulation tests, or basic metabolic panels. Asymptomatic men who are older than 45 years and women who are older than 50 years should undergo electrocardiogram. Routine chest x-ray is not recommended.

25. A 70-year-old man on rivaroxaban for chronic atrial fibrillation (CHADS2 <5) is scheduled to undergo an open umbilical hernia repair. What perioperative recommendation is most appropriate? A. Stop rivaroxaban 3 days before surgery, restart rivaroxaban 5 days after surgery B. Stop rivaroxaban on admission, bridge with heparin, restart rivaroxaban 5 days after surgery C. Continue rivaroxaban D. Stop rivaroxaban 1 day before surgery, restart 6 hours after the procedure E. Stop rivaroxaban 3 days before, anticoagulate with warfarin, restart rivaroxaban 3 days after surgery

ANSWER:

No laboratory studies Complete blood count Chest x-ray Electrocardiogram Basic metabolic panel

27. You are preparing to perform a right hepatectomy on a 65-year-old woman for colorectal metastasis. She has a baseline iron deficiency anemia, and after 1 month of preoperative iron supplementation, her hemoglobin is 10 g/dL (12-15.5 g/dL). The anesthesiologist asks if you want to use red cell salvage techniques during the procedure. Intraoperative red cell salvage

D

A. reduces transfusion-related immunomodulation. B. has no role outside of cardiac surgery. C. should not be used in cases of malignancy or bowel surgery. D. should be used only if intraoperative blood loss is predicted to exceed 2 L. E. requires full anticoagulation.

Rivaroxaban is a direct Factor Xa inhibitor and is a member of the class of direct oral anticoagulant used to prevent and treat thromboembolic events. The half-life of rivaroxaban is 5 to 9 hours in healthy patients and 11 to 13 hours in elderly patients. The drug is usually metabolized 66% in the liver via the CYP3A4 and CYP2J2 enzymes, and 33% is excreted unchanged. An umbilical hernia repair is considered a low bleeding risk procedure in a patient who is at low risk of thromboembolic events (CHADS2 <5). In such a situation, it is appropriate to stop the drug 24 hours before surgery and to resume it 6 hours after surgery. In extremely high-risk surgery, rivaroxaban should be stopped at least 3 days before surgery. Barring technical misadventures, it is appropriate to restart rivaroxaban at least 6 hours after the procedure.

ANSWER:

A

Red cell salvage is a process whereby shed whole blood is suctioned from the surgical field, collected in a reservoir, and then prepared for red cell reinfusion. Although this practice is most often used in cardiac surgery, other applications include orthopaedic, vascular, and trauma surgery. Systemic anticoagulation is not required, and in the process of preparing the cells for reinfusion, heparin used in the salvage process is removed. Each 200 mL unit of salvaged red cells is equivalent to the red cell mass in a unit of packed red blood cells. Because the cells are autologous, the risk of immunemediated transfusion reactions and complications, including transfusion-associated acute lung injury and transfusion-

26. A44-year-old carpenter presents to clinic with complaints of a symptomatic right inguinal hernia. His bowel and bladder functions are normal. His medical history is positive for open appendectomy and gastroesophageal reflux for which he takes pantoprazole. He denies cardiac or pulmonary

26

associated immunomodulation, are avoided. It appears that use of cell salvage reduces the risk of needing an allogenic blood transfusion by 21%, and on average, it saves patients 0.68 units of transfused blood per procedure. Cell salvage typically is reserved for procedures with an estimated blood loss of at least 500 to 1000 mL. There is no evidence for increased rates of infection or reinfusion of malignant cells with cell salvage techniques. Because clotting factors and platelets are removed from these salvaged cells, plasma and platelet transfusions may still be necessary if significant bleeding occurs.

procedures. Slightly more than one-third of patients did not interrupt the apixaban.

29. A 65-year-old man is scheduled for an elective sigmoid colectomy for diverticular disease. He has a history of coronary artery disease and underwent placement of a drugeluting stent 1 year ago. He remains on aspirin and clopidogrel. What is the optimal approach to his perioperative aspirin and clopidogrel management? A. Continue both clopidogrel and aspirin through surgery B. Hold aspirin 1 week before surgery, continue clopidogrel through surgery C. Hold both clopidogrel and aspirin 1 week before surgery D. Hold clopidogrel 1 week before surgery, continue aspirin through surgery E. Hold both clopidogrel and aspirin 1 week before surgery with a heparin bridge

28. A 65-year-old man is scheduled for an elective sigmoid colectomy for diverticular disease. He has a history of atrial fibrillation with a CHA2DS2-VASc score of 4. He takes rivaroxaban for stroke prevention. What is the optimal approach to his preoperative rivaroxaban management? A. B. C. D. E.

Continue rivaroxaban through the surgery Hold rivaroxaban 1 week before surgery Hold rivaroxaban 2 days before surgery Hold rivaroxaban 4 days before surgery Hold rivaroxaban on the morning of surgery

ANSWER:

ANSWER:

D

To mitigate the risk of stent thrombosis, dual antiplatelet agents should be continued, and any elective surgery delayed for a minimum of 1 year. Thereafter, the continued need for dual antiplatelet therapy can be assessed using the dual antiplatelet therapy score. For patients with a score of 2 or greater, dual antiplatelet therapy is typically continued for 18 additional months or longer, although elective surgery can safely proceed after 1 year.

C

The new oral anticoagulants are used routinely for stroke prevention in patients with atrial fibrillation. Rivaroxaban is an oral Factor Xa inhibitor with a half-life of between 8 and 10 hours (depending on patient age and comorbidities). It is cleared both by urinary excretion of the active drug and by metabolism to an inactive form. For major surgical procedures, such as a sigmoid colectomy, the short-term risk of a major bleeding complication is 2 to 4%, whereas a CHA2DS2-VASc score of 4 gives this patient a 4.8% chance of a thromboembolic complication over the next year (0.01%/day). Consequently, rivaroxaban should be held. Assuming this patient has normal creatinine clearance, the patient should skip 2 doses (off anticoagulation for 48 hours) before surgery. Bridge anticoagulation is not required in this patient. Assuming he has normal return of bowel function and no perioperative bleeding, he can resume rivaroxaban 2 to 3 days postoperatively.

For major surgery, such as a sigmoid colectomy, the risk of a perioperative bleeding complication is 2 to 4%. In this patient with coronary artery disease and a stent, his aspirin represents secondary prevention and should be continued through the procedure to mitigate the risk of a cardiac complication. It can also help mitigate the risk of stent thrombosis in the early postoperative period. Beyond 1 year after placement of a drug-eluting stent, dopidogrel can be safely held for a brief period around the time of surgery. It is generally held for 5 to 7 days preoperatively and resumed several days postoperatively when the bleeding risk has subsided. In patients at higher risk of stent thrombosis, a loading dose of dopidogrel can be given on resumption of the medication.

For procedures with a low bleeding risk on a patient who is also at a low risk of thromboembolic events, rivroxaban can be stopped 24 hours before surgery and resumed 6 hours after surgery or when the surgeon believes the risk of bleeding is low.

In the ARISTOTLE trial, apixaban was used for patients with a low risk of thromboembolic events and low bleeding risk

27

30. When closing a midline abdominal incision after an elective procedure, what suture technique minimizes the rate of incisional hernia development? A. Continuous sutures with 1-cm bite depth and 1-cm intersutural distance B. Continuous sutures with 5-mm bite depth and 5-mm intersutural distance C. Interrupted sutures with 1-cm bite depth and 1-cm intersutural distance D. Interrupted sutures with 5-mm bite depth and 5-mm intersutural distance E. Interrupted retention sutures with 2-cm bite depth and 2cm intersutural distance

ANSWER:

B

Fascial dehiscence with or without evisceration carries a significant morbidity and mortality. Failed surgical closures result in incisional hernias (up to 21%), dehiscence (1-4%), or evisceration, and these patients have a significantly increased mortality— as high as 35%. Approximately 100,000 incisional hernia repairs are undertaken annually; thus, addressing any modifiable risk factors for these occurrences is warranted. The principal mechanism of wound failure is sutures pulling through the fascia. Infection plays a role in approximately 30% of cases, whereas inadequate suture length and excess tension represents the primary factor in more than 60% of cases. The optimal recommended suture length to wound length ratio (SL: WL) is 4:1 (i.e., for a 15cm incision, use 60 cm of suture, not counting the discarded tails). Suture breakage and knot failure are rare. A continuous monofilament suture in a single layer of closely placed bites pulled up with minimal tension substantially reduces the risk of surgical incision failure. Small bites of 5 mm from the fascial edge and 5 mm from the last bite will result in a SL:WL ratio of at least 4:1 if minimal tension is applied when pulling up each suture (to the point where the fascial edges are just touching). Bites further from the fascial edge and spaced further apart lead to insufficient suture use; thus, excess tension is placed on each bite with normal physical activities. Interrupted sutures of any variety take more time and result in less bursting strength than continuous suture closures. In fact, meta-analyses demonstrate a significantly greater risk of incisional hernia development with interrupted suture closure of elective midline laparotomy incisions. Finally, retention sutures are placed to theoretically reduce the risk of evisceration after abdominal closure in urgent cases, although their benefit has not been rigorously evaluated. They have no known role in the primary closure of elective surgical incisions.

28

Perioperative Care Part III

hemorrhagic stroke, undergoing high-risk surgical procedures, or who have sustained severe trauma. In these patients, inferior vena cava (IVC) filters can afford protection from pulmonary embolism from lower extremity deep vein thromboses (DVTs). Absolute indications for placement of these filters are recurrent VTE despite adequate anticoagulation, contraindication to anticoagulation, complications resulting from anticoagulation, and inability to maintain or achieve appropriate anticoagulation. Relative indications include recurrent pulmonary embolism, freefloating lower extremity DVT, and iliocaval DVT. Prophylactic IVC filters are the most common indication (58%) for placement and can be used in trauma, surgical, or medical patients at high risk for DVT. IVC filter placement is most commonly performed via femoral vein approach. Overall complications are low. Early (<30 days) complications (e.g., hematoma, pseudoaneurysm, oversedation) occur in 7%. Long-term complications include recunent DVT (20%), stent migration (1.3%), and thrombosis (2.8%). Long-term complications should be avoidable because most IVC filters are removable. The current problem is that only 35 to 40% of IVC filters are removed. It is incumbent on all healthcare providers to be aware of this problem and to make sure filters are removed as soon as medically indicated.

ITEMS 1-30 For each question, select the best possible response. 1. For patients who are undergoing elective surgery and who are chronically on statin therapy, which of the following statements regarding risk is true? A. Discontinuing statins perioperatively is associated with increased stroke risk B. The benefit of continuing statins is most pronounced in those over age 75. C. Continuing statins perioperatively is associated with an increased rate of deep organ space infections. D. Continuing statins perioperatively is associated with a reduction in all-cause mortality. E. Reduction in perioperative risk is primarily due to the lipid lowering effects of statins.

ANSWER:

D

Perioperative HMG-CoA reductase inhibitors (statins) should be continued perioperatively for patients using them chronically. Statin continuation within 24 hours of elective surgery decreases all-cause mortality, especially due to cardiac events in patients undergoing noncardiac operations. This effect is not solely due to the lipid-lowering effects of statins; the anti-inflammatory properties appear to primarily provide protection. Although the benefits of continuing statins perioperatively are most marked in the reduction of cardiac events, there are also observed decreases in infectious-related complications, renal failure, and respiratory complications. There is, however, no apparent effect on central nervous system complications. Patients younger than 75, patients with heart disease or diabetes, or patients undergoing high-risk surgical procedures are at most risk, and perioperative statins should be continued if possible.

3. A 53-year-old man undergoes a laparoscopic low anterior resection with diverting loop ileostomy for a T2N0 rectal cancer. On postoperative day 6, he develops a fever and leukocytosis. Pelvic CT scan reveals a pelvic abscess, which responds to percutaneous drainage. He improves clinically and is seen in the office a week later. At that time, a drain study confirms anastomotic leak at the coloproctostomy and resolution of the abscess. His complete blood count and basic metabolic panel are normal, and he has mild abdominal tenderness at his incisions. He is tolerating a regular diet, and drain output is 50 mL/day. What is the most appropriate treatment? A. Continued observation with diet as tolerated B. Bowel rest and parenteral nutrition C. Endoscopic injection of fibrin glue D. Laparotomy with primary repair of the anastomotic defect E. Laparotomy, resection of the anastomosis, and end colostomy

2. Which of the following statements is true regarding management of inferior vena cava filter placement or retrieval? A. Early complication rates exceed 30%. B. Venous thrombosis is the most common late complication. C. Most complications occur within 30 days of placement. D. Therapeutic indications are more frequent than prophylactic indications. E. More than 50% of retrievable filters are being removed.

ANSWER: ANSWER:

B

A

Low pelvic anastomoses for rectal cancer are associated with anastomotic leak rates of 10 to 20%, with the number being higher as the anastomosis gets lower in the pelvis. These leaks can lead to severe pelvic sepsis and death. Even if the patient survives the acute episode, there is an association with worse long-term cancerspecific survival. Given these poor outcomes, most surgeons routinely protect low pelvic

Venous thromboembolism (VTE) occurs in approximately 1:1000 general-risk and 1:100 high-risk patients and can result in pulmonary embolism. Pulmonary embolism has a mortality rate of 25%. Anticoagulation remains the most common prophylactic and treatment method, yet it is contraindicated in patients with recent hemorrhage,

30

anastomoses with, a loop ileostomy. Whether temporary proximal diversion truly reduces leak rates or whether it instead simply reduces the clinical impact of leaks, including a less frequent need for surgical re-exploration, is controversial. Because of the technical skill required and the lack of distal healthy bowel, low pelvic anastomotic repair and salvage is quite difficult at the time of diagnosis. Unprotected anastomoses that leak often require takedown and permanent fecal diversion. This patient had an anastomotic leak, but it was diverted proximally. He is now clinically normal with no uncontrolled sepsis after a percutaneous drain created a controlled colocutaneous fistula. Most of these fistulas (-75%) will close spontaneously over time and not require surgical intervention, but they can take up to 6 months to heal. During this time, observation and ongoing percutaneous drainage are typically all that is necessary without a need for antibiotics or bowel rest. The anastomosis is followed with serial watersoluble contrast enemas or CT scans with rectal contrast.

as HIT antibodies). Less than 5% of these patients go on to develop the syndrome of heparin-induced thrombocytopenia (HIT). Antibody formation is lower in patients treated with low molecular weight heparin (LMWH, 8%), and the relative risk of thrombotic complications is 0.22 compared with UFH. Despite its rarity, HIT remains quite morbid, and early recognition and treatment are essential to good patient outcomes. Although significant variation exists in timing and severity, HIT typically occurs 5 to 15 days after initial heparin exposure. New-onset thrombocytopenia should alert the clinician that heparin cessation and HIT antibody testing are necessary. The "4T" score (thrombocytopenia, timing of platelet decrease, sequela of thrombosis, and other causes of thrombocytopenia) is used at many centers to differentiate patients into either low risk or intermediate-to-high risk for HIT. Patients are assigned 0 to 2 points for each T, and those at low risk (<3 points overall) have a risk of less than 2% of clinically relevant HIT. Patients with HIT can develop both arterial and venous thromboses. Prompt anticoagulation with a novel agent is warranted. Direct thrombin inhibitors such as argatroban and bivalrudin are considered first-line therapy for HIT.

Bowel rest and parenteral nutrition will not affect the output or closure rate of a diverted fistula and may lead to malnutrition and bacterial overgrowth. Endoscopic injection of fibrin glue was reported in a small series with limited success, but this would typically be reserved for patients with persistent fistulas rather than used as a primary intervention.

Desmopressin is used for patients with von Willebrand disease and has not been studied in HIT. Transfused platelets would be counterproductive in preventing thrombotic complications. Enoxaparin is associated with HIT as well and is not used due to concern for cross-reactivity. Warfarin is not used for HIT due to its slow rate of onset along with the associated transient hypercoagulability, with several reports of warfarin-induced skin necrosis being tied to HIT treatment.

Laparotomy with repair of the anastomotic defect would be aggressive and technically difficult, and such a repair, when warranted, would be better approached transanally. Laparotomy would be appropriate for uncontrolled sepsis, especially if the patient was not initially diverted. Anastomotic resection and end colostomy should be reserved for patients with severe anastomotic breakdown and associated pelvic sepsis.

5. A 55-year-old man undergoes a laparoscopic sigmoid colectomy for diverticulitis. Which of the following interventions is associated with a decreased rate of surgical site infection in this patient?

4. A 74-year-old man undergoes a Whipple procedure for pancreatic cancer and is treated with prophylactic doses of subcutaneous unfractionated heparin after surgery. On postoperative day 5, his platelet count decreases from 150,000/mm3 to 40,000/mm3 (150,000-400,000/mm3), and a heparin-induced thrombocytopenia antibody is positive. After stopping heparin, what is the most appropriate initial treatment? A. B. C. D. E.

A. B. C. D. E.

ANSWER:

Desmopressin Platelet transfusion Argatroban Enoxaparin Warfarin

ANSWER:

Intraoperative normothermia Prophylactic antibiotics for 48 hours Iodine-impregnated adhesive drapes Preoperative chlorhexidine shower Incisional gentamicin sponge

A

Surgical site infections (SSIs) are common after colorectal surgery, with several modifiable and nonmodifiable risk factors. With the increasing focus on patient outcomes as a metric for reimbursement, more attention is being paid to interventions and SSI prevention bundles to decrease infection rates, despite varying levels of validating evidence. Surgeons should be familiar with not only the SSI prevention guidelines developed by multiple organizations but also the quality of the supporting literature used to formulate these guidelines.

C

Up to 17% of patients treated with unfractionated heparin (UFH) will develop anti-heparin-PF4 antibodies (also known

31

Hypothermia is common during major abdominal surgery, and it is known to be associated with higher rates of SSI. Well-designed randomized controlled trials demonstrated decreased rates of SSI when core body temperature is maintained above 36°C.

often have a dark brown hue. Because sedated patients may have other causes of hypoxia, a high clinical index of suspicion is required. An arterial blood gas with methemoglobin level should be obtained. Methemoglobin levels in the normal patient should be close to 0%, but they may be slightly elevated in patients who smoke.

Continuing antibiotics for 48 hours after surgery does not reduce SSI rates compared with a single preoperative dose, and routine cessation of prophylactic antibiotics in the first 24 hours is appropriate. Although iodine-impregnated adhesive drapes advertise the ability to reduce the wound's exposure to skin flora, a Cochrane review demonstrated no reduction in SSI with their use, as well as some evidence that they increase infection rates. Another Cochrane review focused on preoperative chlorhexidine showers and could find no benefit to the practice compared with either placebo or a simple bar of soap. A 2010 randomized controlled trial of gentamicin sponges actually demonstrated a significantly higher rate of SSI compared with no intervention.

Methylene blue acts as an electron donor, converting methemoglobin back to hemoglobin. It should be administered at an initial dose of 50 to 100 mg intravenously. If patients do not improve after administration, there are several possibilities, inducting glucose-6-phosphate dehydrogenase deficiency and even methylene blue overdose, which causes worsening methemoglobin at very high doses because it is also an oxidizing agent. Patients with refractory methemoglobinemia can be considered for exchange transfusions or hyperbaric oxygen. Flumazenil is the primary pharmacologic treatment for benzodiazepine overdose, whereas naloxone is used for narcotic overdose. Dantrolene is administered for malignant hyperthermia. Endotracheal intubation would be appropriate if the sedated patient could not protect his airway, but it would not improve the patient's oxygenation in the presence of high levels of methemoglobin.

6. A 42-year-old man is undergoing esophagogastroduodenoscopy for new-onset dysphagia. He receives topical benzocaine along with intravenous midazolam and fentanyl before the procedure. Despite no abnormal findings during the procedure, the patient appears cyanotic and reports shortness of breath unresponsive to supplemental oxygen. Pulse oximetry shows an oxygen saturation of 88%. Arterial blood gas is obtained, showing a PaO2 of 200 mm Hg, PaCO2 of 30 mm Hg, and a methemoglobin level of 35% (0-3%). What is the most appropriate next step? A. B. C. D. E.

7. A 72-year-old woman undergoes an open sigmoid colectomy for colon cancer. Which of the following interventions reduces the incidence of postoperative ileus in this patient? A. B. C. D. E.

Flumazenil Naloxone Methylene blue Dantrolene Endotracheal intubation

ANSWER:

Methylnaltrexone Sugar-free chewing gum Wound infiltration with liposomal bupivacaine Magnesium citrate Bisacodyl suppositories

ANSWER:

C

B

Enhanced recovery after surgery (ERAS) protocols for patients undergoing colorectal surgery are derived from studies evaluating the efficacy of interventions designed to reduce the morbidity and cost of surgical procedures and therefore improve outcomes and value.

Methemoglobinemia is a well-known but uncommon complication of topical benzocaine, occurring in 1 of every 7000 exposures. It is caused by mucosal absorption of benzocaine, which can lead to the oxidation of ferrous (Fe2+) to ferric (Fe3+) iron. Iron in hemoglobin must be in its reduced (Fe2+) state for normal oxygen exchange to occur. This oxidation of hemoglobin not only leads to a functional anemia but also increases its affinity for oxygen, thus shifting the oxyhemoglobin dissociation curve to the left and leading to less oxygen delivery to tissues.

One such intervention, providing sugar-free chewing gum perioperatively, is as a method of sham feeding, with the hope that the mastication motion will promote intestinal motility through the cephalic phase of vagal enteric stimulation. The true mechanism is not entirely understood, and a randomized controlled trial of sugared chewing gum did not show a benefit, insinuating that the hexitols in sugar-free gum may have a laxative effect and contribute to bowel motility. A meta-analysis summarizing the existing literature concluded that sugar-free chewing gum is an inexpensive and effective way to reduce postoperative ileus, and it is well-tolerated without any detrimental effect on patient outcomes

Patients with methemoglobinemia will become increasingly cyanotic as methemoglobin levels increase. The cyanosis is typically unresponsive to supplemental oxygen. Pulse oximetry will generally underestimate the degree of hypoxia, with readings of 85 to 90% despite a worsening clinical picture. Of note, the PaO2 will be elevated and the blood will

32

Methylnaltrexone is currently indicated for opioid-induced constipation, but it was initially studied as an intervention to reduce postoperative ileus. Two large multicenter randomized controlled trials failed to show a reduction in postoperative ileus compared with placebo. Liposomal bupivacaine is gaining in popularity as a component of multimodal pain control in ERAS protocols. However, current data do not demonstrate superiority to conventional bupivacaine. Laxatives such as magnesium citrate and stimulants such as bisacodyl will induce diarrhea, but they will not increase small intestinal motility or reduce postoperative ileus.

Data regarding the optimal modality and timing of renal replacement therapy are conflicting. However, when patients are hemodynamically abnormal and require vasopressor support, a continuous mode of renal replacement therapy is preferred. Continuous modes require smaller volumes of blood to be removed at a time compared with intermittent hemodialysis and are better tolerated in hypotensive patients. A sodium bicarbonate infusion is used for patients with acidosis; however, starting a sodium bicarbonate infusion is generally not recommended until the serum pH is less than 7.15. Also, there are no data confirming any reduction in morbidity and mortality for its use in renal failure. Aggressive diuretic therapy used in the early stages of AKI to treat volume overload and hyperkalemia is possible. However, once a patient advances to renal failure, renal replacement therapy is the preferred modality of management. Low-dose dopamine infusions were once erroneously thought to be renal protective via a mechanism of increased renal blood flow. There are currently no data to support the use of a dopamine infusion as prevention or treatment of AKI.

8. A 65-year-old woman with well controlled type 2 diabetes mellitus presents with perforated appendicitis. She is taken to the operating room for exploration, drainage of intraabdominal abscess, and ileocecectomy. She is admitted to the surgical intensive care unit postoperatively in septic shock. Overnight, she is resuscitated with 9 L crystalloid. She is now on norepinephrine and vasopressin infusions to keep her mean arterial pressure above 65 mm Hg. On postoperative day 1, she is hypoxic with pulmonary edema on chest x-ray. Her serum potassium is 5.9 mmol/L, her serum creatinine increased from 1.5 to 5.4 mg/dL (0.4-1.3 mg/dL), her serum blood urea nitrogen is 60 mg/dL (7-20 mg/dL), her serum bicarbonate is 13 mmol/L (20-29 mmol/L), and her pH is 7.21. Her urine output is 0.3 mL/kg/hour for the last 12 hours. What is the next step in managing her renal failure? A. B. C. D. E.

9. Which of the following components of an infection prevention bundle has the most significant impact in reducing surgical site infections in elective colorectal surgery patients? A. B. C. D.

Preoperative shower with 4% chlorhexidine gluconate Mechanical bowel preparation Intraoperative abdominal irrigation with antibiotic Mechanical bowel preparation combined with oral antibiotics E. Dedicated instruments for wound closure

Continuous renal replacement therapy Intermittent hemodialysis Furosemide 80 mg intravenously Dopamine infusion Sodium bicarbonate infusion

ANSWER:

ANSWER:

A

D

Surgical site infections (SSIs) after colorectal surgery are a significant cause of morbidity and mortality and remain an important national quality indicator. Mechanical bowel preparation plus oral antibiotics was a mainstay in colorectal surgery for decades, but several publications questioned the efficacy of mechanical bowel preparation. Numerous randomized controlled trials failed to show any decrease in SSIs or anastomotic leak. A major criticism of these trials was omission of the oral antibiotic portion of the standard bowel preparation. The American College of Surgeons National Surgical Quality Improvement Program (NSQIP) retrospectively evaluated 4999 patients who underwent colectomy. Patients who had a combined mechanical bowel preparation and oral antibiotics had a lower 30-day rate of superficial SSIs, anastomotic leak, and 30-day readmission, compared with mechanical bowel preparation alone.

Acute kidney injury in the postoperative period is associated with increased morbidity and mortality. Two classification systems were proposed in the early 2000s: the RIFLE criteria and the Acute Kidney Injury Network (AKIN) staging system. These systems provide concise definitions of the extent of injury and prognosis. Both systems consider increases in serum creatinine, either an absolute number or an increase from baseline, and urine output criteria. This patient had a marked increase in serum creatinine and has oliguria. According to the RIFLE criteria, she has Failure; she is in AKIN stage III. Additionally, she is acidemic, with a serum pH of 7.21 and bicarbonate of 13 mmol/L (20-29 mmol/L), and hyperkalemic. She also shows evidence of volume overload with hypoxemia and radiographic evidence of pulmonary edema. This patient has several indications for renal replacement therapy.

Infection prevention bundles are used to reduce the rates of infection related to central line insertions and mechanical ventilation. Several interventions in the preoperative and

33

perioperative arenas are proposed to reduce the rates of SSI in colorectal patients, and institutions have grouped them together in an infection prevention bundle. One group studied the impact of each component of the infection prevention bundles: mechanical bowel preparation plus oral antibiotics, preoperative chlorhexidine shower, preoperative hair clipping, skin preparation with a standard chlorhexidine alcohol solution, intraoperative antibiotic irrigation, and a clean closure protocol with dedicated instruments. Multivariate analysis showed that the mechanical bowel preparation with oral antibiotics had the greatest effect on reducing SSIs.

The risk of stroke in patients with nonvalvular atrial fibrillation can be calculated with the CHADS2 score. The CHADS2 score is calculated by adding up points from the presence of congestive heart failure, hypertension, age older than 75 years, diabetes mellitus, and history of stroke or transient ischemic attack. The patient in our question has 2 points for hypertension and diabetes, giving him a 4.0% yearly risk of thromboembolic event with no warfarin. Dual antiplatelet therapy is not an effective treatment to prevent stroke in patient with nonvalvular atrial fibrillation.

11. Which of the following is associated with decreasing the rate of central line associated blood stream infections? 10. A 63-year-old man with a symptomatic midline ventral hernia from prior open appendectomy presents with a reducible bulge. He has a history of well-controlled hypertension, diabetes mellitus type 2, and paroxysmal atrial fibrillation. He has been on warfarin for the last 5 years. He has had worsening symptoms and pain from the hernia over the past 6 months and 2 episodes of incarceration requiring emergency department visits. Which of the following statements is true regarding this patient's perioperative anticoagulation?

A. B. C. D. E.

ANSWER:

A

Central line associated blood stream infections (CLABSI) are a significant source of morbidity and increased costs in surgical patients. The most common route of contamination and subsequent infection of a central line is migration of skin microorganisms from the insertion site to the catheter tract. To decrease the contamination of the catheter, several antiinfectious procedures were combined into a single central line insertion bundle. Several different components may be included in a central line insertion bundle; however, most include the following: hand washing, maximal sterile barrier precautions, chlorhexidine skin preparation, avoiding femoral vein placement, and removal of unnecessary catheters. The use of a central line insertion bundle reduced CLABSI by 66% in a large multi-institution trial. Additional components to a central line insertion bundle include applying a sterile dressing, replacing the dressing every 48 hours, and using a chlorhexidine-impregnated dressing,

A. His yearly risk of thromboembolic events is 8%. B. Perioperative bridging with low molecular weight heparin (LMWH) decreases his risk of thromboembolic events by 50%. C. Warfarin should be stopped 3 days preoperatively. D. Initiation of postoperative bridging with LMWH increases his risk of major postoperative bleeding. E. Dual antiplatelet therapy should be initiated to reduce the risk of perioperative stroke.

ANSWER:

Central line insertion bundle Polyvinyl chloride catheter Prophylactic intravenous antibiotics Routine guidewire exchange of the catheter Placement in femoral vein under ultrasound guidance

D

Patients with atrial fibrillation are placed on lifelong anticoagulation to reduce their risk of embolic stroke. When they are scheduled to undergo surgery, most patients have their warfarin held preoperatively, typically 5 days before surgery. Due to the concern of an embolic event during this time period, many patients undergo bridging anticoagulation with a low molecular weight heparin (LMWH). Despite very little data to support this practice, bridging anticoagulation has remained in practice guidelines. A prospective, double blind, randomized trial of 1884 low-risk patients compared bridging with LMWH with no bridging in patients with nonvalvular atrial fibrillation who had warfarin held in the perioperative period. The incidence of arterial thromboembolism and stroke was not different between the 2 groups (0.4% vs 0.3%) but the patients who underwent bridging with LMWH had more than twice as much major bleeding (1.3% vs 3.2%). Given no increase in stroke rate but higher bleeding complications in the bridging group, perioperative bridging anticoagulation is not recommended.

Several other interventions were studied but are not costeffective in reducing CLABSI rates. These are summarized in guidelines provided by the Society of Critical Care Medicine and Infectious Disease Society of America. Prophylactic intravenous antibiotics and routine guidewire catheter exchanges did not decrease the rate of CLABSI. Catheters impregnated with chlorhexidine, minocycline, rifampin, platinum, or silver can be used in patients who need a central line for more than 5 days and in settings where other components of the central line insertion bundle have not reduced CLABSI rates. No association exists between catheter material and CLABSI rates.

12. A 68-year-old man with chronic renal failure is dialysisdependent and has recurrent biliary colic. He presented for

34

elective cholecystectomy. His medical history was significant for a myocardial infarction 5 months ago, after which a bare metal stent was placed. He was taking aspirin and clopidogrel. Clopidogrel was held for 5 days before surgery, but aspirin was continued. Two days after cholecystectomy, he has chest pain and a myocardial infarction. What factor most contributed to his major adverse cardiac event after surgery?

infarction. Risk of myocardial infarction is highest if surgery is performed within 30 days of preoperative myocardial infarction and remains greater than 5% at 3 to 6 months. After 6 months, the relative risk for early postoperative myocardial infarction decreases further and remains low up to 1 year after surgery.

A. B. C. D. E.

Items 13-15 Each lettered response may be selected once, more than once, or not at all.

Withholding clopidogrel in the perioperative period Use of bare metal stent rather than drug-eluting stent Myocardial infarction within the past 6 months Dialysis dependence Male sex

ANSWER:

A. B. C. D. E.

C

Dabigatran Rivaroxaban Fondaparinux Clopidogrel Warfarin

13. Reversed by Vitamin K

Elective noncardiac surgery is associated with a small risk of postoperative major adverse cardiac events. Risk stratification before elective surgery is important, and surgeons should be aware of preoperative factors that increase this risk of major adverse cardiac events when deciding whether surgery should be performed or delayed. Demographics such as advanced age and male sex are associated with postoperative cardiac complications, as are chronic preoperative conditions such as coronary artery disease, congestive heart failure, cerebrovascular disease, and renal insufficiency. Acute conditions that carry a high risk include acute coronary or aortic disease, stroke, major trauma, or emergency surgery.

14. Selective Factor Xa inhibitor 15. Specific reversal agent is idarucizumab

ANSWERS:

E, B, A

Patients taking anticoagulants can present challenges to the surgeon. The risk of bleeding must be balanced with the need for normal or near normal coagulation. Several new oral anticoagulants are now in wide use. Dabigatran is a thrombin inhibitor that binds directly to thrombin. It lengthens the partial thromboplastin time (PIT) but in a non-dose-related fashion. Additionally, the prothrombin time (PT)/intemational normalized ratio (INR) is not useful in establishing the degree of drug activity. Although the classic thrombin time is too sensitive, the development of the dilute thrombin time is useful to estimate the quantity of the drug in circulation. As a corollary, a normal thrombin time suggests very low or no drug activity, a useful tool in an acute situation. Dabigatran is dialyzable. Idarudzumab is a monoclonal antibody preparation that is a specific reversal agent for dabigatran. It has been approved for the reversal of dabigatran in the setting of life-threatening hemorrhage and when anticipating invasive procedures or surgery that cannot be delayed for at least 8 hours.

As the population of individuals with coronary stents increases, so does the population in need of noncardiac surgery who have coronary stents. Recent cardiac stent placement is associated with postoperative major cardiac events; however, stent type is not independently associated with major cardiac event if the elective surgery is delayed more than 6 months after stent placement. At the same time, perioperative management of antiplatelet therapy can be a challenge in these patients, especially in patients with drugeluting stents. American College of Cardiology/American Heart Association guidelines recommend delaying elective surgery for a period of at least 6 months for patients with drug-eluting stents if possible, especially if dual antiplatelet therapy is discontinued perioperatively. By contrast, elective noncardiac surgery can be performed 1 month after bare metal stent placement, and the risk for postoperative myocardial infarction is highest if surgery is performed within 30 days of stent placement, regardless of whether a bare metal or drug-eluting stent were placed. Common practice is to discontinue clopidogrel and to continue aspirin in the perioperative period.

Rivaroxaban is a direct-binding Factor Xa inhibitor. It elevates both activated PTT (aPTT) and PTT but not in a consistent fashion, limiting the utility in these in determining anticoagulation activity. It is not effectively dialyzable and has no specific reversal agent approved. Andexanet alfa is a recombinant analog of factor Xa and may be available in the future as a specific inhibitor for Factor Xa inhibitors. Currently, most reversal protocols use a 4-factor prothrombin complex concentrate in life-threatening situations.

Despite recent advances in perioperative care, patients with a recent myocardial infarction remain at very high risk for postoperative cardiac complications. The risk of postoperative cardiac event decreases with greater delay in the timing of surgery after a preoperative myocardial

35

Fondaparinux binds to antithrombin III, resulting in an inhibition of Factor Xa. It elevates both the PT and aPTT but has no effect on thrombin time.

antibiotic use. The risk of infection with C. difficile is also linearly related to the degree of perioperative add suppression with both histamine receptor type 2 blockers, such as famotidine or ranitidine, and proton pump inhibitors, such as omeprazole, pantoprazole, or lansoprazole.

Clopidogrel is an antiplatelet agent that irreversibly blocks the adenosine diphosphate receptor P2Y12 on platelet membrane, inhibiting platelet activation. This is a separate action from aspirin, which inhibits thromboxane systems via an anti-Cox-1 mechanism. Both drugs affect platelets for the remainder of their lifetime.

Many drugs used in the perioperative period can predpitate or worsen the severity of delirium. Common causes of delirium indude polypharmaceutical drug effects and patient factors such as age and organic neurologic dysfunction. Diphenhydramine, often used as a sedative or "sleep aid," is a well-recognized predpitator of delirium in the elderly in the perioperative period.

Warfarin is a long-used oral anticoagulant. It inhibits the Vitamin K-dependent Factors II, VII, IX, and X. Additionally, it inhibits the anticoagulant factors proteins C and S. Warfarin interacts with KO reductase, inhibiting oxidized Vitamin K to be reduced to normal Vitamin K. It is monitored via the PT/INR assay. It is the only oral anticoagulant to be reversed by Vitamin K. It can also be reversed with fresh frozen plasma or prothrombin complex concentrates.

Serotonin syndrome is a potentially life-threatening disorder that can result from administering linezolid in a patient taking other medications that may interact with it. This is because it was originally developed for its nonselective monoamine oxidase inhibitory properties. Common drug interactions with linezolid include selective serotonin reuptake inhibitors, such as paroxetine, sertraline, or citalopram, as well as selective norepinephrine reuptake inhibitors, such as venlafaxine or mirtazapine. Symptoms include high body temperature, agitation, increased reflexes, tremor, sweating, dilated pupils, and diarrhea. Body temperature can increase to more than 41.1 °C. A host of other common medications prescribed in the perioperative period including sedatives, such as trazodone, or analgesics, such as tramadol, can precipitate the serotonin syndrome as well. Herbal products such as St John's Wort are also associated with the serotonin syndrome.

Items 16-19 Each lettered response may be selected once, more than once, or not at all. A. B. C. D. E.

Anastomotic leak Clostridium difficile infection Serotonin syndrome Delirium Wound infection

16. Ketorolac

Although not a perioperative medication, tobacco use in the perioperative period is a definite risk factor for surgical site infection and wound complications. The abstinence of tobacco products for 8 to 12 weeks before elective surgery is associated with better outcomes. Perioperative serum or urine testing of cotinine, a by-product of nicotine metabolism can be used to risk-stratify for elective surgery.

17. Pantoprazole 18. Linezolid 19. Cotinine

ANSWERS:

A, B, C, E 20. When used in the context of an enhanced recovery strategy after laparotomy with bowel resection and anastomosis, the addition of alvimopan results in

Perioperative care of the surgical patient should include a fundamental knowledge of potential negative effects of drug treatments prescribed in the postoperative period as well as ramifications of patient factors that enhance the risk of surgical procedures.

A. B. C. D. E.

Although not firmly a cause-and-effect relationship, surgeons should be aware that there is a definite association in large cohort studies between nonsteroidal antiinflammatory drugs (NSAIDS) such as ketorolac and a risk of colonic anastomotic dehiscence in both elective and urgent colon surgery. This is especially in need of clarification because narcotic-sparing regimens for enhanced recovery pathways liberally use NSAIDS as a component of care.

decreased anastomotic leaks. less nausea. increased hospital readmission within 10 days. shorter time to bowel movement decreased pain scores.

ANSWER:

D

Enhanced recovery after surgery (ERAS) programs have had several benefits, especially after colorectal surgery. They are associated with decreased length of stay without increased readmission rates and with improved patient satisfaction. Two of the key components are adequate pain control and

The perioperative risk of Clostridium difficile superinfection of the colon is not uncommon in surgical patients, especially in carriers of the bacteria or those exposed to prolonged

36

early initiation of oral intake. However, the use of opioid pain medications can contribute to postoperative gastrointestinal dysfunction and ileus. Alvimopan, a mu-opioid antagonist, can mitigate the gastrointestinal side-effect of narcotics without interfering with the analgesic effects. When used in the context of an ERAS program, alvimopan results in a shorter time to initial bowel movement after operation. Use of alvimopan is not associated with anastomotic leaks rates, postoperative nausea, increased early hospital readmission (10 days), or increased pain scores. There is added pharmacy cost with its use, but this is offset by the decreased length of stay associated with a shorter ileus period.

22. Use of a restrictive red blood cell transfusion protocol (Hgb <7 g/dL) for hemodynamically normal intensive care unit patients results in A. B. C. D. E.

ANSWER:

4 days. 7 days. 10 days. 14 days. 48 hours beyond normalization of fever and white blood cell count.

ANSWER:

D

Transfusion practices in the intensive care unit have undergone significant changes over the last 10 years. Although red blood cell transfusion can be lifesaving for someone with active hemorrhage and hemodynamic abnormality, the detrimental effects of red cell transfusions are not negligible. Based on data from large clinical trials, it has become increasingly clear that a more restrictive policy toward red cell transfusion for hemodynamically normal patients in the intensive care unit has several benefits. In this setting, mortality rates are at least equivalent (if not lower) for a restrictive policy. The percentage of patients transfused is decreased (from approximately 25% to approximately 18%), and mean direct costs of transfusion are reduced by approximately 25%. Healthcare-associated infections are decreased. Hospital length of stay and readmission rates are decreased as well with restrictive transfusions.

21. After a laparoscopic appendectomy with adequate source control for perforated appendicitis in a healthy patient, antibiotics should be prescribed for A. B. C. D. E.

increased mortality. increased length of stay. increased readmission rates. decreased percentage of patients transfused. increased fresh frozen plasma use.

A

Surprisingly little data are available on the duration of antibiotic treatment for many surgical infections. Recently, more data are emerging that support shorter durations as being equally effective.

Restrictive red cell transfusion policies are associated with similar reductions in transfusion of other blood components, including plasma and platelets.

The Study to Optimize Peritoneal Infection Therapy (STOPIT) trial enrolled adult patients with • • •

23. A 62-year-old woman is admitted with a 2-day history of right upper quadrant pain and is diagnosed with acute cholecystitis. She has no angina, and her electrocardiogram is normal, but she is on clopidogrel for a bare metal coronary stent placed 1 year ago. What is the next step in her management?

a complicated intra-abdominal infection fever, leukocytosis, or gastrointestinal tract dysfunction from peritonitis an intervention to achieve source control

Participants were randomized (1:1, open-label) to 4 full days of antimicrobial treatment after the initial source-control procedure (experimental group) or to antimicrobial therapy until 2 days after resolution of systemic inflammatory response (control group). The primary outcome of the study was a composite score of the occurrence of surgical site infections, intra-abdominal infection, or death within 30 days. A total of 518 patients were enrolled. There were no significant differences between the 2 groups in either the composite outcome or its individual components. Thus, in patients with adequate source control, a fixed 4-day course is as effective as longer courses based on fixed number of days or physiologic response.

A. B. C. D. E.

Proceed with laparoscopic cholecystectomy Delay the operation for 5 days Transfuse platelets intraoperatively Administer tranexamic add preoperatively Start a preoperative heparin drip

ANSWER:

A

For patients with uncomplicated acute cholecystitis, laparoscopic cholecystectomy generally should be done during the index hospital stay. However, in patients on antiplatelet therapy, there is always a concern for increased risk of bleeding with operation, as well as the concern for coronary stent occlusion if the medication is discontinued or reversed.

Additional studies in complicated appendicitis patients with adequate source control after laparoscopic appendectomy showed that shorter courses (in the range of 3-5 days) do not have worse outcomes.

37

For patients undergoing laparoscopic procedures, continuation of a single antiplatelet agent during the operative period is not associated with an increased incidence of operative or postoperative bleeding. Thus, many guidelines recommend continuation of a single antiplatelet agent through the operative period in patients at high risk for thrombotic complications and procedures with low-risk of postoperative bleeding (such as a laparoscopic cholecystectomy). In this case, the appropriate next step is to proceed with laparoscopic cholecystectomy. Waiting an additional 5 days will not affect the risk of bleeding and will increase the risk for conversion to open operation. Because the risk of postoperative bleeding is not increased, platelet transfusion would not benefit this patient. Likewise, tranexamic add is an antifibrinolytic agent and does not affect platelet function. There is no evidence that bridging anticoagulation with a heparin drip reduces the risk of stent thrombosis after discontinuation of an antiplatelet agent.

25. Which of the following antihypertensive medications should be continued on the day of elective surgery? A. Beta-blocker B. Calcium channel blocker C. Angiotensin converting enzyme inhibitor D. Alpha-2 agonists (clonidine) E. Furosemide

ANSWER:

Early recommendations by the American College of Cardiology (ACC) and American Heart Association (AHA) included perioperative beta-blocker therapy for patients undergoing noncardiac surgery with untreated hypertension, known coronary artery disease, or other cardiac risk factors. Since those recommendations, however, larger studies demonstrated that although initiation of beta-blockade therapy decreased the risk of perioperative nonfatal myocardial infarction, it increased the risk of stroke and death. Consequently, a more recent ACC/AHA guideline discouraged initiating new therapy within 1 day of surgery. However, for patients on chronic beta-blocker therapy, there is a strong recommendation that beta blockers be continued in the perioperative period to avoid adverse cardiac events.

24. Standardized post discharge ileostomy pathways with comprehensive patient education and close outpatient followup result in A. increased total cost of care. B. increased 30-day adverse events. C. decreased readmission rates for dehydration. D. increased hospital stays for patients who are readmitted. E. decreased patient satisfaction.

ANSWER:

A

No evidence suggests that continuation of calcium channel blockers in the perioperative period provides any benefit in terms of postoperative myocardial infarction or cardiac death. At the same time, there is also no significant detriment to continuing calcium channel blocker therapy on the day of surgery, and the decision of whether to continue can be left to the discretion of the surgical team and patient.

C

For patients with new ileostomies, readmissions can be frequent, with dehydration due to high ileostomy output being the most common cause. Several studies investigated the use of ileostomy pathways to reduce the morbidity of ileostomies and reduce the rate of readmission. Although each study varied somewhat in the details, the main components included 1) detailed preoperative education; 2) specific postoperative interventions (e.g., stoma care, dietary instructions); 3) discharge instructions (e.g., ileostomy diary, phone contact numbers); and 4) frequent contact with the patient by phone call in the first few weeks to assess progress, monitor the ileostomy, and provide new or reinforce existing instructions.

Similarly, angiotensin converting enzyme (ACE) inhibitors are commonly prescribed medications. Use of ACE inhibitors on the day of surgery is associated with intraoperative hypotension but does not cause worse cardiovascular or renal outcomes. Meanwhile, there is no convincing evidence that perioperative use of ACE inhibitors is protective. Thus, ACC/AHA recommendations suggest that it is reasonable to continue ACE inhibitor therapy in the perioperative period but holding them is acceptable as well. There is a similar concern that loop diuretics used on the day of noncardiac surgery could increase the risk of intraoperative hypotension; however, the risk of developing perioperative adverse cardiovascular events is not increased in patients on continued furosemide therapy. In these patients, however, there is no known benefit to continued therapy.

The main benefit of these pathways and frequent contact is a 50 to 100% reduction in the number of admissions due to dehydration. Overall, 30-day adverse events are decreased. Additionally, for those patients who do need readmission, their lengths of stay tend to be shorter. There is some incremental increase in the cost of the intensive out-patient follow-up, but this is more than offset by the cost saving of reduced readmissions (i.e., total cost of care goes down). Finally, patient satisfaction with these programs is very high.

Alpha-2 agonists are not recommended as cardiac prophylaxis in patients who are undergoing noncardiac surgery, because they have no role in perioperative cardiac protection and may increase postoperative nonfatal myocardial infarction. Sudden discontinuation of ongoing alpha-2 agonist therapy can result in hypertension, headache, or agitation, but continuation on the day of surgery is not proven to provide perioperative benefit.

38

26. Prophylactic placement of an inferior vena cava (IVC) filter before bariatric surgery compared with not placing an IVC filter is associated with which of the following outcomes A. B. C. D. E.

27. A 68-year-old man presents with large-volume lower gastrointestinal bleeding resulting in hypotension and tachycardia, hemorrhagic shock, and need for blood transfusions. The patient has been anticoagulated with dabigatran for treatment of atrial fibrillation, and the last dose was 4 hours ago. The optimal anticoagulant reversal agent in this patient is A. 3-factor prothrombin complex concentrate. B. fresh frozen plasma. C. activated Factor Vila. D. idarucizumab. E. andexanet alfa.

Higher postoperative deep vein thrombosis rate Lower pulmonary embolism rate Reduced use of perioperative anticoagulation Shorter length of stay Decreased all-cause 90-day mortality

ANSWER:

A

Venous thromboembolism (VTE) is a rare complication after bariatric surgery. Nonetheless, bariatric patients are considered to be at moderate risk for VTE, and it remains a leading cause of postoperative morbidity arid mortality. Factors that increase these patients' risk for postoperative VTE include prior VTE, hypercoagulable state, higher BMI, male sex, immobility, pulmonary hypertension, obesity hypoventilation syndrome, venous stasis disease, and prolonged operative time. Thus, VTE prophylaxis is standard practice in bariatric surgery. Mechanical prophylaxis is recommended for all bariatric surgical patients, along with early ambulation. In addition, most data suggest benefit for the combined use of mechanical and chemoprophylaxis in these patients.

ANSWER:

Idarucizumab is approved by the US Food and Drug Administration as a specific reversal agent for dabigatran. It is a monoclonal antibody that directly neutralizes the effects of dabigatran and has no effect on other anticoagulants. It has immediate onset and provides full reversal up to 24 hours in most patients. Prothrombin complex concentrate (PCC; 3factor), Factor Vila, fresh frozen plasma, and andexanet-alfa are not indicated for dabigatran reversal. Four-factor PCC has been used to reverse dabigatran but is not as effective as idarucizumab. Four-factor PCC is the optimal reversal agent for all Factor Xa inhibitors and warfarin in life-threatening bleeding. PCC replaces coagulation factors but does not directly neutralize the activity of the oral anticoagulants. PCC onset of action is 5 to 15 minutes and duration of effect is 12 to 24 hours.

Inferior vena cava (IVC) filters, particularly when retrievable, were of interest, especially in the high-risk bariatric population. They offer a potentially reversible, effective mode of VTE prophylaxis that obviates the risk of anticoagulation. A systematic review of IVC filter placement in bariatric surgery patients found an overall higher risk of deep vein thrombosis and pulmonary embolism in patients receiving a preoperative filter. IVC filters are associated with multiple device-related complications, including filter migration, thrombosis, embolization, occlusion, and fracture. In addition, prophylactic placement of IVC filters does not decrease the use of anticoagulation, mortality, or length of stay. Medication Dabigatran

Mechanism of Action Half-life (hours) Direct thrombin inhibitor 12-17

Rivaroxaban Factor Xa inhibitor

5-9 (11-13 in elderly)

Apixaban

Factor Xa inhibitor

12

Edoxaban

Factor Xa inhibitor

10-14

D

Plasma is sometimes indicated for warfarin reversal. Andexanet alfa is a direct reversal agent for Factor Xa inhibitors (rivaroxaban, apixaban, edoxaban; table 27.1).

Reversal

Assays

Oral charcoal (must give within 2 hours of last dose) Idarucizumab

Thrombin clotting time or Ecarin clotting time; if normal no treatment needed

3- or 4-factor PCC Andexanet alfa 3- or 4-factor PCC Andexanet alfa 3- or 4-factor PCC Andexanet alfa

Anti-Xa assay; if normal no treatment needed Anti-Xa assay; if normal no treatment needed Anti-Xa assay; if normal no treatment needed

Current as of December 2018. Table 27.1.

39

28. A 44-year-old man presents with a history of sigmoid colon stricture and colovesical fistula due to complicated diverticulitis. He underwent preoperative epidural analgesia with morphine and bupivacaine. His procedure was a laparoscopic-converted-to-open sigmoid colectomy with stapled colorectal end-to-end anastomosis in modified lithotomy position. Intraoperative findings were a large inflammatory mass densely adherent to the posterior wall of the bladder, small bowel, retroperitoneum, and sacral promontory. Immediately postoperatively, he complained of left lower extremity weakness in the femoral nerve distribution and decreased motor function. The next immediate step in management is A. B. C. D. E.

obtain a CT scan of pelvis. obtain a MRI of the thoracic and lumbar spine. remove the bupivacaine from the epidural infusion. decrease the epidural catheter infusion. discontinue the epidural catheter.

ANSWER:

Figure 28.1. Nerves of the pelvis. To minimize nerve trauma in the lithotomy position, the following should be considered: • Minimize abduction and external hip rotation. • Pad all areas at risk for tissue injury: hips, lateral fibulas, posterior thighs, and heels. • Ensure that hips and knees are moderately flexed and securely supported. • Direct the weight of the lower extremities toward the soles of the feet. • Avoid lower extremity tissue injury by ensuring that the stirrup edges do not cut into the calf (e.g., peroneal nerve), posterior thigh, or low on the Achilles tendon. • Use shorter blades and periodic relief of pressure with self-retaining retractors

B

When a patient develops lower extremity weakness immediately postoperatively with an epidural catheter in place, evaluation for possible epidural hematoma should be the first priority. The optimal diagnostic test for epidural hematoma is an urgent MRI of the thoracic and lumbar spine. In a single-institution study of 43,200 epidural catheterizations, 102 patients underwent spine MRI that confirmed 6 cases of epidural hematoma (overall incidence 1 per 10,000 epidural blocks). This patient had femoral neuropathy from deep pelvis surgery. The mechanism is compression of the femoral nerve against the pelvic sidewall as the nerve emerges from the border of the psoas muscle before exiting the pelvis at the inguinal ligament due to self-retaining retractors. Risk factors for femoral nerve compression include the following: • Thin subcutaneous fat layer (BMI <20) • Operating time greater than 4 hours • Narrow pelvis • Self-retaining retractors, with extreme lateral traction

29. A 65-year-old patient presents for colectomy for sigmoid colon cancer. He has a history of coronary artery disease and underwent heart catheterization with drug-eluting stent placement 6 months ago. He is on clopidogrel and aspirin (325 mg daily). The most appropriate plan for his antiplatelet medication is to A. delay colectomy for 6 months. B. stop clopidogrel and continue aspirin. C. stop both clopidogrel and aspirin. D. stop both and bridge with enoxaparin. E. continue both.

In a study of 2304 patients who underwent major colorectal surgery at a single institution, peripheral nerve injury occurred in 0.2% (5/2211) of open procedures and 3% (3/93) of laparoscopic procedures. There was no association between age, sex, operative time, BMI, or American Society of Anesthesiologists score and nerve injury. Nerves are prone to stretching or compression in the lithotomy position. The most common nerves affected are (1) femoral and (2) lateral femoral cutaneous (figure 28.1). In patients undergoing gynecologic pelvic surgery, a 2% incidence has been reported, with 91% complete resolution at a median time of 31.5 days.

ANSWER:

B

Dual antiplatelet therapy is indicated to prevent stent thrombosis in patients with coronary stents. When these patients need surgery, decisions regarding perioperative antiplatelet therapy should be based on the bleeding risk of the planned procedure, risk of stent thrombosis, and alternative options. This patient underwent drug-eluting stent placement 6 months ago. Modification of dual antiplatelet therapy is appropriate and safe at this time with the risk of stent

40

thrombosis being acceptably low. The risk of a bleeding complication on clopidogrel is sufficient that discontinuation in the perioperative period is indicated. Aspirin can be safely continued. Because of the underlying malignant diagnosis, delaying the operation is not appropriate. Bridging with enoxaparin will not provide additional benefit.

low (1-2 points), moderate (3-4 points), and high (>5 points) risk of VTE to determine type and duration of prophylaxis. High-risk patients should undergo chemoprophylaxis with low molecular weight heparin (LMWH). High-risk patients undergoing major abdominal surgery for malignancy have the highest risk for postoperative VTE. Two randomized trials demonstrated reduced VTE rates in these patients with 4 weeks of LMWH chemoprophylaxis compared with 1 week. The American College of Chest Physicians Chest Guidelines for prevention of VTE in nonorthopedic surgical patients state, "For patients at high risk for VTE undergoing abdominal or pelvic surgery for cancer, we recommend extended-duration, postoperative, pharmacologic prophylaxis (4 weeks) with LMWH over limited-duration prophylaxis (Grade 1B)." Shorter durations of VTE pharmacologic prophylaxis such as "only while hospitalized" or "until back to baseline ambulatory status" are inadequately short. Four weeks' duration is not associated with increased bleeding complications; however, extending duration of chemoprophylaxis beyond 4 weeks does increase the risk of bleeding complications without providing additional benefit in reducing VTE.

30. A 67-year-old man has undergone pancreaticoduodenectomy for stage II pancreatic adenocarcinoma. His Caprini score is 7. How long should he should be maintained on low molecular weight heparin? A. B. C. D. E.

Until back to his baseline level of ambulation While hospitalized 1 week 4 weeks 3 months

ANSWER:

D

Postoperative venous thromboembolism (VTE) remains a significant source of morbidity and mortality. The Caprini score (table 30.1) stratifies patients into very low (0 points), 1 Point Age 41-60 years Minor surgery BMI>25kg/m’ Swollen legs Varicose veins Pregnancy or postpartum History of unexplained or recurrent spontaneous abortion Oral contraceptives or hormone replacement Sepsis (<1 month)

2 Points Age 61-74 years Arthroscopic surgery Major open surgery (>45 minutes) Laparoscopic surgery (>45 minutes) Malignancy Confined to bed (>72 hours) Immobilizing piaster cast

3 Points Age 2:75 years History of VTE Family history of VTE

5 Points Stroke (<1 month) Elective arthroplasty Hip, pelvis, or leg fracture

Factor V Leiden

Acute spinal cord injury (<1 month)

Central venous access

Elevated serum homocysteine

Serious lung disease, including pneumonia (<1 month) Abnormal pulmonary function Acute myocardial infarction Congestive heart failure (<1 month) History of inflammatory bowel disease Medical patient at bed rest (<1 month) Abnormal pulmonary function Acute myocardial infarction Congestive heart failure (<1 month) History of inflammatory bowel disease Medical patient at bed rest

Prothrombin 20210A Lupus anticoagulant Anticardiolipin antibodies

Heparin-induced thrombocytopenia Other congenital or acquired thrombophilia

VTE = venous thromboembolism. Table 30.1.

41

Perioperative Care Part IV

necessity within 6 months of placement of a drug-eluting stent, antiplatelet therapy should be continued.

ITEMS 1-30 For each question, select the best possible response.

In the setting of pelvic surgery for cancer, continuation of aspirin as single agent antiplatelet therapy, while holding thienopyridines, offers a strategy to balance the risk of perioperative bleeding against stent thrombosis.

Items 1-3 Each lettered response may be selected once, more than once, or not at all. A. Continue current antithrombotic agents and proceed with elective procedure. B. Discontinue antithrombotic agents in time for effects to resolve before the elective procedure. C. Change antithrombotic agents and continue with elective procedure. D. Defer the elective procedure until a later date. E. Proceed with elective procedure.

4. Which of the following electric surgical devices is characterized by generating temperatures as high as 300° and having a slow heat dissipation rate? A. B. C. D. E.

1. Patient receiving warfarin for deep vein thrombosis that occurred 1 year ago and now requires wide local excision of early-stage melanoma. 2. Patient presenting with a reducible inguinal hernia 3 months after placement of a drug-eluting coronary stent followed by dual antiplatelet therapy.

ANSWER:

C

Surgeons should be familiar with how electric surgical devices work. The electricity causes molecular vibration within a cell, which generates heat and causes damage to the cell. Cell death begins to occur at 50°C. The standard monopolar energy device transfers an electrical impulse from the instrument tip through the patient to the dispersion pad. The energy provided at the tissue level causes the molecular vibration within the cell and results in cell damage. With higher energy, the damage extends further through the tissue. While on lower energy, the damage will be more limited to the surface of the tissue. The cut mode of a monopolar device uses more energy and thus is able to penetrate or "cut" through the tissue more. The cut mode can result in temperatures up to 100°C. On coagulation mode, less energy is used. Therefore, tissue damage is limited to the surface. The damage of proteins leads to coagulum formation. Coagulation mode reaches temperatures of about 60° C.

3. Patient who has completed preoperative therapy for a surgically resectable rectal cancer and has received dual antiplatelet therapy for a drug-eluting coronary stent that was placed 6 months earlier

ANSWERS:

Bipolar surgical electricity Advanced bipolar device Ultrasonic dissector Monopolar surgical electricity ND-Yag laser device

B, D, C

Antithrombotic agents include both anticoagulant agents and antiplatelet agents. When considering surgery for a patient who is currently receiving anticoagulation therapy, the surgeon must examine the nature and indication of the surgical procedure as well as the anticoagulation therapy. The surgeon must then weigh the necessity and timing of the surgical procedure against the risk of bleeding due to anticoagulation. Recommendations are available to help guide this often-difficult clinical scenario.

Bipolar devices transfer energy between the 2 tips of the instrument. Energy is transferred to the tissue that is grasped by the instrument. Again, this energy results in molecular vibration and the generation of heat, which damages the cells. Energy does not travel through any other tissue. Bipolar instruments can generate temperatures up to 90°C and are better for controlling bleeding because their effects are limited to the tissue between the jaws of the instrument. Advanced bipolar devices can generate heat up to 100°C, and they have a second aspect that allows the tissue to be cut. Heat generation with a Nd:Yag laser is less than 50°C.

In patients receiving Vitamin K antagonists like warfarin, the therapy can be interrupted before surgery. This should occur approximately 5 days before surgery. For the patient requiring a cutaneous procedure, the Vitamin K antagonists for deep vein thrombosis has been continued beyond the recommended duration of treatment. The warfarin can be discontinued. Patients with a coronary stent who are receiving dual antiplatelet therapy and require surgery should have surgical procedures deferred for at least 6 weeks after placement of a bare-metal stent and for at least 6 months after placement of a drug-eluting stent.

Ultrasonic devices convert mechanical energy, vibration of the instrument, to generate heat. The energy is transferred to the tissue between the device's jaws. The heat generated can reach up to 300°C. Therefore, ultrasonic devices can cut through tissue and control bleeding. The heat generated at the tip of the instrument, though, does not dissipate quickly. It takes approximately 20 seconds for the tips to decrease their

Inguinal hernia repair is an elective procedure, and nonoperative management is an accepted approach. In this case, deferring surgery to a later date outside the 6-month window is recommended. In patients where surgery is a

43

temperature below 100°C. Thus, they can still cause cell damage long after activation. An ultrasonic device, therefore, should not be used for dissection or to grab vital tissues (i.e., intestines or vessels) unless it has not been activated for more than 30 seconds.

SSI is to perform the repair laparoscopically. A recent multicenter VA study prospectively randomized patients with ventral hernias between open and laparoscopic repairs. The results showed that laparoscopic repair had significantly lower incidence of SSI compared with open repair (5.6% vs 23.3%). Overall, the likelihood of developing complications up to 8 weeks postoperatively was approximately 50% lower in patients whose hernias were repaired laparoscopically.

5. Versus the traditional landmark technique, the use of ultrasound guidance in the placement of internal jugular vein catheters results in A. B. C. D. E.

A recent study retrospectively analyzed more than 6000 cases and showed that the type of surgical hat worn by surgeons had no relationship to risk of SSI over a 30-day postoperative period. This study concluded that surgical hats may be chosen at the discretion of operating room personnel without fear of increased SSI, and this conclusion is supported by the American College of Surgeons.

longer cannulation times. decreased pneumothorax rate. reduced number of passes to cannulate. increased hematoma formation. no difference in overall successful catheter placement.

ANSWER:

Preoperative bowel preparation is used by approximately 12% of surgeons. Ventral hernia repair almost always involves some degree of bowel manipulation and bowel injury may occur, particularly in the subset of patients with extensive adhesions. A study evaluating the accrued data from the Americas Hernia Society Quality Collaborative examined more than 3700 ventral hernia repairs and evaluated whether bowel preparation had a protective effect. This study showed patients with bowel preparation were more likely to develop postoperative SSL

C

The use of ultrasound in guiding the localization and cannulation of the internal jugular vein reduces complication rates compared with the traditional landmark technique. Overall, greater successful catheter placement is achieved with ultrasound guidance. Inadvertent cannulation of the carotid artery and hematoma formation are reduced when using ultrasound visualization. Additionally, reduced number of passes to cannulate, greater success with initial attempt, and shorter time to successful cannulation are observed with ultrasound use. There is no statistically observed difference in the incidence of pneumothorax between the 2 techniques.

Multiple clinical trials demonstrated no difference in SSI rates between clipping and shaving when performed at the time of the operative procedure.

7. A 47-year-old woman is admitted to the intensive care unit after a complex incisional herniorrhaphy with mesh. She has a history of Crohn disease that required a right hemicolectomy and ongoing azathioprine and adalimumab support. On postoperative day 4, she develops profuse diarrhea associated with a white blood cell count of 28,000/mm3 (3600-11,200/mm3). With fluid support, she remains hemodynamically normal and is now afebrile. Investigation for Clostridium difficile identifies a strain positive for both toxins A and B (BI/NAP 1/027). Which of the following statements is true regarding her care?

6. Which of the following strategies will have the greatest impact on preventing postoperative surgical site infection for ventral hernia repair? A. Removal of abdominal hair with clippers versus shaving B. Surgeon use of bouffant hair covering versus skull cap C. Perioperative mechanical versus oral antibiotics bowel prep D. Use of biologic mesh versus artificial mesh E. Laparoscopic versus open repair

ANSWER:

A. She is at risk of multiorgan failure. B. Treatment with intravenous vancomycin and metronidazole is indicated. C. Antibiotic therapy should continue until toxin assays are negative. D. Removal of the new mesh is required to prevent necrotizing fasciitis. E. This bacterial strain is the least virulent of the known C. difficile strains.

E

Surgical site infection (SSI) is one of the most significant postoperative complications after ventral hernia repair and can increase the risk of recurrent hernia by at least 3-fold. Although the surgical community has made significant improvements in the incidence and treatment of SSIs, elimination of these events remains challenging due to their multifactorial nature. Although patient risk factors (obesity, smoking, and diabetes) for SSI remain beyond the surgeon's control, the surgeon can and should control certain aspects of the operation to minimize risk of SSL Perhaps the most important choice the surgeon can make to minimize risk of

ANSWER: 44

A

Ctostridium difficile colitis is an increasingly frequent postoperative complication Although long recognized as a consequence of specific or prolong antibotic use. asymptomatic C. difficile carrier status is now a major contributor to out-of-hospital infections, and there has been a significant spike in nursing home outbreaks Community carrier status is also a factor in the nearly 20% recurrence rate Pateints with inflammatory bowel disease (IBD; both Crohn disease and ulcerative colitis) are particularly vulnerable to C. difficile colitis and its complications,

C. Extension of antimicrobial prophylaxis to 24 hours D. Smoking cessation 4 to 6 weeks before surgery E. Optimization of preoperative HbAlC

ANSWER:

D

Surgical site infections (SSIs) are common, costly, morbid, and, in an estimated 60% of cases, preventable. An isolated SSI is associated with, on average, nearly a 10-day increased length of stay and more than $20,000 in costs during the index admission. Several, evidence-based guidelines exist to help decrease rates of SSIs. Risk factors for SSIs can be divided into modifiable (e.g., smoking status, weight/BMI, nutritional status) and nonmodifiable (e.g., age, sex, historic infections) patient factors as well as extrinsic factors (e.g., emergency procedure, blood transfusions, breach in sterile technique). Prehospital interventions include preoperative chlorhexidine bathing (unless part of a decolonization protocol), smoking cessation, blood glucose control, methicillin-resistant Staphylococcus aureus (MRSA) screening and decolonization, and bowel preparation.

Hospitalized patients with concurrent C. difficile infection and active IBD are known to have at least a 4-fold risk of mortality compared with patients with either C difficile infection or IBD alone. Prior antibiotic exposure# immunosuppression with biologic agents, and gastric acid suppression therapies amplify the risk. Further a subset of IBD patients in remission have a higher carrier rate than the general population, unrelated to antibiotic exposure or immunosuppression, Azathioprine 6-mercaptopurine, methotrexate, and infliximab are all associated with increased C. difficile in IBD patients, particularly when the patient is taking 2 or more agents, Mortality due to multiple organ failure is high# even with aggressive treatment,

Smoking cessation 4 to 6 weeks before elective surgery decreases SSIs. This is especially true when an implanted material or device is part of the procedure. A full bowel preparation, including mechanical and oral antibiotic regimen, is recommended for all colectomies but not for ventral hernia repairs. Although blood glucose optimization is recommended, reducing the HbAlC does not reduce SSIs. MRSA screening and decolonization can be of benefit; however, this requires strict adherence to bundles, otherwise they are not effective.

Several C. difficile species express A and B toxins* One especially virulent species is the BI/NAP1/027 clone that produces both toxins faster and in greater quantities compared with other clones. These clones are also capable of producing a binary toxin that leads to increased adherence of cytotoxic clostridial species to epithelium and is resistant to fluoroquinolones, levofloxacin, and moxifloxacin. As a result, this strain is harder to treat and especially virulent. In many areas of the United States, BI/NAP 1/027 accounts for upward of 50% of all recovered strains, Oral vancomycin is first-line therapy for severe and complicated C. difficile infection in IBD patients. Vancomycin enemas may be used if oral intake is not possible. Intravenous metronidazole may be added but is significantly less effective. Alternatives include fidaxomicin, a new macrocyclic antimicrobial that has 8 times the in vitro activity compared with vancomycin against clinical C. difficile isolates but is expensive. It is recommended that treatment be continued for 10 days. If active colitis is not resolved or infection is recurrent despite vancomycin or fidaxomicin, fecal microbiota transplant should be considered. Monoclonal antibodies and immunoglobulin therapies are emerging as potential adjuncts.Removal of the mesh in this case should not be necessary.

Hospital interventions shown to be of benefit include maintaining normothermia and warming if necessary, avoiding hyperglycemia, avoiding removal of hair with a razor, using an alcohol-containing preparation unless contraindicated, properly administering preoperative antibiotics, and using a wound protection device (colorectal cases). Antibiotic ointment may be of benefit in spine, joint, and cataract surgery, but it is not recommended for all cases. Proper antibiotic administration involves the appropriate agent, dosing, and timing. Antibiotic selection will depend on the procedure and the involved potential pathogens. Ideally, antibiotics should be administered within 1 hour of incision (2 hours for vancomycin or fluoroquinolones) and be redosed based on the pharmacokinetics or after and estimated blood loss of 1500 mL. No evidence supports antibiotic administration after incision closure except in the setting of implanted breast reconstruction, cardiac surgery, and joint arthroplasty.

8. A 45-year-old woman with multiple past abdominal surgeries requires an abdominal wall reconstruction for recurrent ventral/incisional hernia. The surgical plan is for a retrorectus repair with a synthetic mesh. What reduces surgical site infections? A. Preoperative mechanical bowel preparation B. Removal of hair from the surgical site

9. A 48-year-old man with diabetes presents with a malodorous ulcer on the sole of his right foot over the head

45

of the first metatarsal head. No bone is visible. He has been treated with local wound care. He is currently taking broadspectrum antibiotics. His white blood cell count is 12,000/mm3 and his HbAlC is 9%. He is febrile and tachycardic. He has a right palpable dorsalis pedis pulse. The next step in his management should be A. B. C. D. E.

to his methadone, his postoperative pain is managed with a short course of oral nonsteroidal anti-inflammatory drugs without additional opioids. He was started on efavirenz, a nonnucleoside reverse transcription inhibitor, the week before surgery. At his 2-week followup appointment, he complains of a 4-day history of nausea, vomiting, and diarrhea with associated restlessness, diaphoresis, and tachycardia. Otherwise, his physical exam, imaging, and routine labs are unremarkable. CD4 count is 500 cells/mm3 (500-1400/mm3). What is the most helpful blood test in facilitating his management?

MRI to rule out osteomyelitis. sharp debridement. negative pressure wound therapy. honey dressings. hyperbaric oxygen treatment.

ANSWER:

A. B. C. D. E.

B

Clinical practice guidelines for diabetic foot infection were published in 2016 by the Society for Vascular Surgery in collaboration with the American Podiatric Medical Association and the Society for Vascular Medicine. No debridement technique shows superiority, and initial sharp debridement is recommended based on patient tolerance, expertise, supplies, and cost-effectiveness. Urgent surgical intervention should be performed for foot infection involving abscess, gas, or necrotizing soft tissue infection.

Thyroid-stimulating hormone Gamma-glutamyl transferase Serum methadone trough level Procalcitonin level Helicobacter pylori level

ANSWER:

C

Both opioid-use disorder and HIV/AIDS are common diagnoses. They are often, as in this patient, seen together. Opioid agonist therapy, such as methadone and buprenorphine, is extremely effective treatment for opioiddependent patients. Highly active antiretroviral therapy is now widely available and reduces both morbidity and mortality in patients with HIV/AIDS. The potential for drug interactions between both categories of medication are many, and knowledge of their metabolism is crucial.

Adjunctive wound therapy options are recommended if a diabetic foot ulcer fails to decrease in size by at least 50% after a minimum of 4 weeks of standard wound therapy. Negative pressure wound therapy is one wound management approach suggested for these nonhealing or chronic diabetic foot wounds. Diabetic patients should have baseline anklebrachial index (ABI) measurements performed at the age of 50. Annual vascular examination is recommended for diabetics including foot examination, ABI, and toe pressures.

Methadone, a full opioid agonist, is dosed orally once a day. Its metabolism is complicated, with plasma concentrations following a biexponential curve. It undergoes oxidative metabolism of the cytochrome p450 system in the liver. There is significant interindividual variability in methadone's metabolism, possibly due to variability in the activity and expression of this enzyme system as well as protein-binding displacement and stereospecific binding.

The goal of an ideal wound dressing is to maintain a moist wound bed, control exudate, and avoid maceration of surrounding intact skin. Wet-to-dry dressings were a standard mechanism for debridement, but they have fallen out of favor because the debridement is nonselective and may harm viable tissue while removing necrotic tissue. Furthermore, when wet-to-dry dressings are performed correctly, removing the dry dressings can be painful. A trial showed that honey dressing was more effective in healing diabetic foot ulcers than wet-to-dry dressings and decreased the time to healing.

Nonnucleoside reverse transcriptase inhibitors (NNRTIs), such as efavirenz, are highly effective antiretrovirals. Their metabolic activity is dependent on their plasma concentrations, which remain in equilibrium with the intracellular concentrations. NNRTIs share metabolic pathways with methadone. Efavirenz both induces and inhibits several enzymes in the cytochrome p450 complex, resulting in a 50% decrease in bioavailability of methadone within several weeks of initiation of efavirenz therapy. As such, the start of efavirenz treatment has precipitated symptoms of opioid withdrawal in this patient, despite no change in his methadone dosing. A low serum methadone trough level will confirm this and allow for appropriate replacement therapy or reconsideration of the use of efavirenz.

Plain radiographs of the foot are not adequate to diagnose osteomyelitis secondary to their low sensitivity and low specificity. If additional imaging is desired, especially to evaluate for abscess, MRI is the study of choice and can help identify the presence of osteomyelitis.

10. A 26-year-old man undergoes uneventful laparoscopic preperitoneal repair of bilateral inguinal hernias with mesh. He is HIV-positive and receives maintenance methadone treatment for opioid dependence. After discussion with his primary care provider and methadone prescriber, in addition

This patient lacks any symptoms suggestive of thyroid disease, liver disease, or Helicobacter pylori infection.

46

Elevation of procalcitonin, an acute phase protein, is highly specific in differentiating infection from other inflammatory conditions in patients with autoimmune disease. Its determination in this patient would not change his care or aid in diagnosis.

cognitive dysfunction, including disorientation, hallucination, and memory and attention disturbances, all of which can complicate postoperative care and increase length of stay. A recent prospective study examined risk factors for postoperative delirium in patients 75 years and older undergoing major abdominal surgery. With an overall incidence of 24%, postoperative delirium was associated with a higher American Society of Anesthesiologists (ASA) status (ASA class III-IV), impaired mobility, and use of opioids. Even patients treated with opioid-like analgesics (e.g., tramadol) are at risk of postoperative delirium. In this study of 60 patients who received postoperative tramadol, 22 (33.6%) developed postoperative delirium compared with 6 of 58 (10.3%) of patients who did not (p = .0008). On multivariate analysis, duration of the procedure and use of propofol during the procedure did not affect the development of postoperative delirium. Use of a bladder catheter during abdominal surgery in geriatric patients is not associated with the development of delirium.

11. After a routine laparoscopic sigmoidectomy for cancer in an otherwise healthy patient when is the appropriate time to advance a patient’s diet to solid food? A. B. C. D. E.

After first bowel movement After passage of flatus When bowel sounds return Once the patient is awake and alert After the patient tolerates liquids

ANSWER:

D

Early (<24 hours) oral feeding before the return of bowel function after gastrointestinal surgery is supported by several studies and is recommended by the Society of Colon and Rectal Surgeons and the Society of American Gastrointestinal and Endoscopic Surgeons in the recent publication of clinical practice guidelines for enhanced recovery after colon and rectal surgery.

13. A 23-year-old man is admitted to the intensive care unit after a damage control laparotomy for traumatic injuries to his liver and spleen. The patient has an open abdomen and requires return to the operating room. Which of the following is associated with higher rates of primary fascial closure?

An early prospective, randomized controlled trial showed no difference in nasogastric tube insertion between patients whose diets were advanced on postoperative day 1 compared with at the time of intestinal function. In addition, hospital stays were shorter for patients in the accelerated pathway by nearly 2 days without increasing readmissions or complications. In a meta-analysis of 7 randomized clinical trials comparing early with traditional oral feeding, the early oral feeding group had a shorter length-of-stay (difference of 1.58 days; p <.009) and reduced risk of total postoperative complications (relative risk 0.70; p = .04). The early oral feeding group had no increased risk of vomiting, insertion of nasogastric tubes, anastomotic dehiscence, wound infection, or pneumonia.

A. B. C. D. E.

ANSWER:

Foley catheterization. opioid analgesia. American Society of Anesthesiologists class I-II. surgical procedure lasting more than 4 hours. use of propofol during the procedure.

ANSWER:

B

This scenario is frequently encountered. Severely injured or septic patients may not tolerate an abdominal closure at the index operation and may require resuscitation in an intensive care setting before they are able to tolerate further procedures and anesthesia. Delays in abdominal closure, however, can predispose the patient to the development of enteroatmospheric fistulas and insensible as well as sensible fluid losses from the relatively exposed peritoneal cavity.

12. In geriatric patients undergoing elective abdominal surgery, postoperative delirium is associated with A. B. C. D. E.

Delayed return to the operating room by at least 48 hours Volume resuscitation with 3% hypertonic saline Early diuresis with a furosemide drip A filling pressure of less than 10 mm Hg Continuous renal replacement therapy after initial damage control laparotomy

Delaying return to the operating room decreases the likelihood of primary fascial closure. The first return to the operating room should be as soon as possible (within 24 hours) but no later than 48 hours. The use of intravenous 3% hypertonic saline in patients with an open abdomen is associated with increased rates of primary fascial closure. The likely etiology of this success is related to the development of an osmotic gradient favoring the removal of intestinal edema into the intravascular space. Conversely, forced diuresis with a loop diuretic drip is not

B

Postoperative delirium is the most common complication of major abdominal surgery in older patients. It manifests as

47

associated with increased rates of fascial closure, despite the inherent logic of its application. Similarly, no specific evidence suggests that early institution of renal replacement therapy or keeping a patient's central venous pressures below a certain level help with the closure.

B. Nicotine inhibits the proliferation of fibroblasts and macrophages. C. Biofilm is a protective coating on healing wounds. D. Hypoglycemia delays wound healing. E. Topical antimicrobial agents cannot induce bacterial resistance.

14. Which of the following statements is true for enteral nutrition in the critically ill patient?

ANSWER:

A. Total caloric intake for the obese patient is 25 to 30 kCal/kg. B. Growth hormone supplementation increases survival in ventilated patients. C. Postpyloric feedings result in decreased mortality compared with gastric feedings. D. Total protein intake should be 2 to 2.5 g/kg of ideal body weight per day. E. The routine use of probiotics improves patient survival.

ANSWER:

B

Wound healing is affected by many environmental factors. Adequate oxygenation is critical to healing wounds. The presence of anemia does not necessarily decrease tissue oxygenation, because this is also dependent on cardiac output, capillary permeability, and vasodilation. Nicotine is a vasoconstrictor that impairs oxygen delivery, increases platelet adhesion, and inhibits proliferation of red blood cells, fibroblasts, and macrophages. The presence of diabetes mellitus has a profound effect on wound healing. Hyperglycemia can inactivate enzymes and alter the basement membrane so that transfusion of nutrients is limited. Hyperglycemia makes wounds more prone to infection and slower to heal.

D

Calculations of nutritional needs include a 25 to 30 kCal/kg estimate for most critically ill patients. This estimate is not accurate, however, in extremely thin or morbidly obese patients, and a lack of standardized recommendations persists.

Another barrier to wound healing is the presence of biofilm. This is a collection of bacteria surrounded by a self-produced polymer matrix. While commonly found in chronic, contaminated wounds, this is not a normal part of wound healing and is not protective to the wound base.

Indirect calorimetry may guide nutritional support when available. One strategy for patients where this is not available is to provide 65 to 70% of estimated total caloric needs based on ideal body weight. With sufficient protein (2 to 2.5 g/kg of ideal body weight), this strategy will promote overall weight reduction and preservation of lean body mass.

Topical antimicrobial agents target therapy against biofilms, but there is little evidence that this strategy improves wound healing rates or outcomes. Bacterial resistance also occurs with topical antimicrobial agents.

Metabolically active agents were extensively studied in intensive care unit patients. Interestingly, a large randomized trial of growth hormone in ventilated patients found no survival advantage associated with its use. Enteric tube placement is necessary to provide optimal nutritional support in ventilated patients. Postpyloric feeding tubes are associated with a decreased incidence of pneumonia in many studies, but no difference in intensive care unit length of stay or mortality has been documented. Although published trials concur that probiotic use is associated with a decreased incidence of nosocomial infections and pneumonia, overall mortality does not decrease. Published data also vary widely in terms of type of probiotic used and site of application (small bowel vs stomach). Several studies reported probioticassociated bacteremia and central venous catheter associated infections, yet others noted a decreased incidence.

16. Which of the following statements is true about perioperative opiate use? A. Low-income patients are more likely to overdose on opiates. B. Patients treated in teaching hospitals are more likely to overdose. C. Hepatic insufficiency is an Independent predictor of opioid overdose. D. More than 70% of prescribed opioid pills are not used by patients. E. Preoperative pain management with opioids decreases hospital length of stay.

ANSWER:

D

Opioid use and misuse after surgery or injury is a growing US healthcare crisis. Appreciating the risk factors for opioid addiction or perioperative overdose represents essential surgical knowledge. Preoperative opioid users may have increased hospital length of stay and infectious

15. Which of the following statements about wound management is true? A. Anemia increases wound infection rates.

48

complications. In the orthopedic population, patients report more postoperative hyperalgesia, decreased quality of life, and decreased physical function. Several reports have dispelled assumptions about patient characteristics that predict opioid dependency or overdose. Published studies of effects of preoperative opioid use on postoperative outcomes identified a prevalence of chronic opioid use to range from 9.2 to 23%. Some studies reported that chronic users are more likely to be black and less likely to be commercially insured. Multivariable analysis indicates that individuals who overdose after surgery are more likely to be female, older (>60 years old), and from a higher income quartile (>$35,000 per year). Predictors of postoperative overdose include a history of substance abuse and preexisting renal insufficiency. Hepatic insufficiency was not identified as an independent risk factor for overdose. Although there is no statistical difference in likelihood of perioperative overdose in the type of hospital (e.g., rural, urban, teaching or nonteaching), opioid overdoses are more likely in larger hospitals.

Figure 17.1. The Mallampati Classification pillars to class IV when only the hard palate is visualized. In addition to determining the Mallampati classification, the physician should recognize other risk factors that lead to difficult intubation and should prepare accordingly. These risk factors include obesity, neck immobility, interincisor distance less than 4 cm in adults, large overbite, inability to shift the lower incisors in front of the upper incisors, and a thyromental distance less than 6.5 cm. The thyromental distance is measured from the thyroid cartilage to the tip of the chin. A history of difficult intubation is one of the most important risk factors for future difficult intubations. When a difficult airway is anticipated, preparation for alternative approaches to direct rigid laryngoscopy such as fiberoptic, video laryngoscopy, blind nasal intubation, or tracheostomy should be considered.

In spite of growing understanding about the negative effect of perioperative opioid use on outcomes, analysis of prescribing habits after elective general surgery procedures suggests that clinicians continue to prescribe large numbers of opioid pills. One recent study determined that more than 70% of the prescribed opioid pills (ranging from 15-100) were not taken by patients who were discharged home; in this analysis, less than 2% of the patients requested refills, suggesting that opioids were being overprescribed. Because opioid use is a potentially modifiable behavior, controlling how much medication patients receive could mitigate some of the postoperative complications associated with opioid use.

18. A 70-year-old woman with congestive heart failure is on warfarin for a history of atrial fibrillation. She presents to the emergency department with a small bowel perforation. Which of the following is a risk factor for a stroke in this patient?

17. When evaluating an adult for possible orotracheal intubation, what is the best risk factor for predicting a difficult intubation?

A. B. C. D. E.

A. Inability to move the lower incisors in front of the upper incisors B. BMI greater than 27 C. Previous cervical spine injury with normal mobility D. Interincisor distance of 5 cm E. Thyromental distance of 7 cm

ANSWER:

Age of 70 years old Female sex Need for urgent surgery Congestive heart failure Sepsis

ANSWER:

D

The use oral anticoagulation agents are increasing nationwide for a myriad of conditions including Atrial fibrillation stent placement and thromboembolic event.

A

As such surgeons need to understand which patients require perioperative anticoagulation and have an increased risk of perioperative thrombotic events with cessation of anticoagulation. In patients with a high stroke risk, failure to resume anticoagulation in a timely fashion can be associated with perioperative thromboembolic complications. One commonly used tool to assess the risk of stroke is the CHADS2 score

A critical component of obtaining a secure airway is the evaluation of the airway. This evaluation is vitally important when considering administration of agents that induce apnea during the intubation process. The Mallampati classification (figure 17.1) is based on the structures that can be visualized with maximal oral opening and tongue protrusion while the patient is in the sitting position. Classification ranges from class I visualization of the soft palate, fauces, uvula, and

49

[table 18.1; congestive heart failure (1 point), hypertension (1 point), age older than 75 (1 point), diabetes mellitus (1point), history of stroke transient ischemic attack (2 points). The presence of clinical conditions can be used to estimate the risk of perioperative and general stroke risks. The described patient has congestive heart failure as her only risk factor and has a CHADS2 score of 1 with a relatively lower overall perioperative risk for stroke. This patient can safely undergo surgery without a heparin bridge. Risk factor

1

CHA2DS2VASC score points 1

1

1

Age >75 years

1

2

Diabetes mellitus

1

1

Stroke/transient Ischemic attack/thromboembolism

2

2

Vascular disease

-

1

Age 65-74 years

-

1

Sex category (i.e., female)

-

1

Maximum score

6

9

Congestive failure/left dysfunction Hypertension

CHADS2 score points heart ventricular

B. C. D. E.

Intravenous phosphate Intravenous antibiotics Intravenous steroids Plasmapheresis

ANSWER:

E

Patients with myasthenia gravis are at risk of hypoventilation due to diaphragmatic weakness. The etiology of this disease is autoimmune, characterized by antibodies to nicotinic acetylcholine receptors at the neuromuscular junction. Chronic treatment includes immunosuppression (e.g., with corticosteroids) and anticholinesterase medications. Several medications can contribute to worsening of myasthenia, including antibiotics and neuromuscular blockers that could have been used during surgery to treat this patient for her perforated diverticulitis. Her respiratory acidosis in the perioperative period is best managed acutely with intubation. Plasmapheresis is then used to treat the myasthenia crisis. Because she has a normal-appearing wound and normal renal function, compartment syndrome is unlikely. Hypophosphatemia, which can cause respiratory insufficiency, is a concern in chronically malnourished patients and is unlikely to be a concern in this patient who presented acutely. Intravenous antibiotics would treat worsening sepsis, which is unlikely to be the etiology of her deterioration given recent source control, normal chemistries, unchanged white blood cell count, and an unremarkable chest x-ray. Intravenous steroids are not helpful in reversing respiratory depression from myasthenia.

Table 18.1. Comparison of the CHADS2 and CHA2DS2VASc scoring systems. Note. CHADS2 or CHA2DS2-VASc score, documenting risk factors for stroke: History of congestive heart failure, hypertension history; age >75 (or age >65 years associated with one of the following: diabetes mellitus, coronary artery disease, or hypertension); diabetes mellitus; stroke or transient ischemic attack or thromboembolism history; vascular disease history; sex category.

20. A 40-year-old man is undergoing elective open distal pancreatectomy. Perioperative epidural analgesia is likely to increase

New oral anticoagulants in patients with nonvalvular atrial fibrillation:

A. B. C. D. E.

19. A 70-year-old woman with a long-standing history of myasthenia gravis presents with peritonitis and pneumoperitoneum secondary to perforated sigmoid diverticulitis. On the evening after an emergency Hartmann procedure, she is somnolent and demonstrates hypoxia, with an arterial blood gas with a pH of 7.25, PaCO2 = 62 mm Hg, PaO2 = 60 mm Hg. There is no change after administration of naloxone and neostigmine. Her blood pressure and heart rate are normal. Her abdomen is soft, and she has a colostomy. Her basic metabolic panel is normal, and she has a persistent leukocytosis but normal hemoglobin. Chest x-ray after intubation is unremarkable. After she is stabilized, what intervention should be performed to address the likely etiology of her respiratory failure?

oral opiate use. perioperative mortality. urinary tract infection. episodes of perioperative hypotension. rates of postoperative ileus.

ANSWER:

D

The rationale for use of thoracic epidural anesthesia for open cancer procedures is to optimize postoperative pain control and decrease intravenous narcotic use. However, compared with patients receiving intravenous opioids, those managed with an epidural have increased episodes of perioperative hypotension attributable to local anesthetics. This is because of the sympathetic blockade that decreases preload and afterload. Given its analgesic effects, epidural placement is likely to decrease, not increase, intravenous and oral opiate

A. Decompressive laparotomy

50

use. Although it is possible that decreased opioid use can in turn decrease complications and mortality, this finding is not consistently demonstrated. Epidural anesthesia does not directly increase rates of urinary tract infection. Epidural use is likely to decrease or not affect, rather than increase, postoperative ileus.

pacemaker. Using a bipolar instrument will also reduce this chance but will not eliminate it. An ultrasonic device does not pass electrical energy through the patient and does not require a dispersion pad for its use. Instead mechanical energy (vibration) creates heat up to 300°C, which dissects the tissue and causes coagulation. Because electrical energy does not pass through the patient with such devices, it is the best choice for this patient to avoid electrical interference with her pacemaker.

21. Six years ago, a 62-year-old woman had a quadruple coronary artery bypass graft with a cardiac pacemaker placed through her left subclavian vein. The pacemaker sits just below her clavicle and above her breast tissue. She now presents with stage II right breast cancer, and she has elected to undergo a modified radical mastectomy. You are concerned about disruption of her cardiac pacemaker with the use of surgical electricity, because she is pacemaker dependent Which of the following will most likely decrease the risk of electromagnetic interference of the pacemaker during the procedure? A. B. C. D. E.

22. For an elective clean-contaminated case, which of the following statements regarding the guidelines for prevention of a surgical site infection is true? A. Antibiotic wound irrigation lowers surgical site infection rates. B. Antibiotic powders in surgical wounds at closure reduce surgical site infection rates. C. Glucose should be controlled intraoperatively with a goal of less than ll0 mg/dl. D. Skin should be prepared with iodine-based agents unless contraindicated E. Antibiotics do not need to be continued after skin closure.

Using monopolar electricity only on blend mode Placing the dispersion pad on the right arm Placing a magnet on the pacemaker Reprogramming the pacemaker into synchronous mode Using ultrasonic shears for the dissection

ANSWER: ANSWER:

E

E

New guidelines were published in 2017 regarding preventing surgical site infections in clean and dean-contaminated cases. The new guidelines include stopping antibiotics after skin closure, meaning that prophylaxis does not need to extend a full 24 hours as previously advised. The other guidelines include skin preparation using alcohol-based solutions unless otherwise contraindicated, maintaining perioperative glucose control with a goal of less than 200 mg/dL, and maintaining normothermia. The use of antibiotic wound irrigation and antibiotic powders does not improve surgical site infection rates.

The safe use of electrical devices in the operating room includes not only the surgical energy devices like the monopolar and bipolar instruments but also the maneuvers to avoid an operating room fire or interference with other electrical devices like a cardiac pacemaker or ventricular assist device. If the patient has a pacemaker in place, it is important to know whether the patient is pacemaker dependent. If not, then the device can simply be shut off for the procedure. For most cardiac pacers, this can be done with a magnet. If she is pacemaker dependent, the device must be left on for the procedure. Pacemaker dependence means that the device will fire because the patient's own rhythm will not cause sufficient ventricular contraction. Therefore, putting the pacemaker on synchronous mode means that it will fire or "assist" only based on the patient's rhythm, which is inadequate.

Items 23-25 Each lettered response may be selected once, more than once, or not at all. A. Direct Factor Xa inhibitor B. Inhibits the activity of Vitamin K-dependent enzymes C. Nonreversible cyclo-oxygenase (COX)-l and COX-2 inhibitor D. Reversible phosphodiesterase inhibitor E. Competitive direct inhibitor of thrombin

For pacemaker-dependent patients, any electrical current via the electrical cord of the Bovie or the transfer of energy from the device to the dispersion pad should not cross paths with the pacemaker device. As such, simply putting the device on blend mode, which may reduce the overall energy flowing to the Bovie tip and into the tissue, will not avoid electrical disruption of the pacemaker.

23. Aspirin 24. Dabigatran

Placing the dispersion pad away from the pacemaker on the patient's right arm or on the patient's right thigh will reduce the amount of energy passing near the pacemaker but it will not eliminate all current that may interfere with the

25. Apixaban

51

ANSWERS:

C. water-soluble contrast study. D. upright abdominal x-rays. E. broad-spectrum antibiotics.

C, E, A

Atrial fibrillation is the most common cardiac dysrhythmia in the United States, with an anticipated incidence of 6 to 12 million older Americans by year 2050. Warfarin reduces the stroke risk by two-thirds and is commonly prescribed in atrial fibrillation. Warfarin, however, has significant limitations because of a narrow therapeutic window. Approximately onethird of patients with increased risk for stroke are not started on warfarin or discontinue it after initiation because of side effects or medication nonadherence.

ANSWER:

Percutaneous endoscopy gastrostomy (PEG) is a good solution for long-term enteral nutrition in patients who cannot swallow. In adults, PEG is commonly placed for patients with acute stroke, central nervous system trauma, and oncologic and neurodegenerative diseases. Tube dislodgment, however, accounts for up to one-third of PEG tube failures. If dislodgement occurs before the PEG tract completely matures, the stomach may separate from the abdominal wall and cause leakage of feedings and gastric contents into the peritoneal cavity. The most important step is to immediately stop feeding and verify that the tube remains in the stomach by performing a water-soluble contrast study. An exploratory laparotomy is necessary if the patient has obvious signs of peritonitis or the contrast study demonstrates peritoneal contrast spillage. Upright abdominal x-rays may show residual air instilled during the original PEG placement. Conversely, the absence of free air under the diaphragm does not rule out tube malposition. Once proper tube position is confirmed by the contrast study, providers may treat exit site infection with either antibiotics, bedside drainage, or a combination of these.

Aspirin reduces stroke risk by 20% and is used in patients with a contraindication to warfarin. Aspirin reversal with platelet transfusion is often performed in cases of lifethreatening hemorrhage. Desmopressin when given as an intravenous bolus over 30 minutes increases von Willibrand Factor and Factor VIII activity within 1 hour and increases platelet activity. The combination of platelet transfusion and desmopressin in one retrospective study did not show a decrease in intracranial hemorrhage progression or mortality. No randomized controlled trials of desmopressin in aspirin reversal have been carried out, although guidelines suggest consideration of desmopressin use in acute hemorrhage and intracranial hemorrhage. Direct oral anticoagulants are increasingly prescribed for atrial fibrillation because of their predictable anticoagulant activity, short half-life, and minimal drug-drug interactions (table 23-25.1). An example of a reversible phosphodiesterase inhibitor is cilostazol, which is commonly used to treat claudication. Dabigatran

Rivaroxaban

Apixaban

Edoxaban

Target

Thrombin

Factor Xa

Factor Xa

Factor Xa

Half-life (hours)

12-17

5-9 11-13 (in elderly)

12

10-14

Peak plasma

1

2-4

3-4

1-2

80

33

27

50

35

>90

87

55

concentration (hours) Renal clearance (%) Protein binding (%)

C

27. Four hours after pneumatic dilation of the esophagus to treat achalasia, a 72-year-old man presents with moderate chest pain and tachycardia. A Gastrografin esophagogram demonstrates an uncontained leak in the distal third of the esophagus with minimal mediastinal contamination. After resuscitation and intravenous antibiotics, the next step should be A. B. C. D.

parenteral nutrition. endoscopic closure with clips. placement of a covered stent with drainage. right thoracotomy, closure of the perforation, and pleural patch. E. esophageal exclusion, cervical esophagostomy, and gastrostomy tube.

Table 23-25.1. Properties of non-Vitamin K antagonist oral anticoagulants.

ANSWER:

26. A 78-year-old man with dysphagia underwent percutaneous endoscopic gastrostomy (PEG) 10 days ago. Overnight, the patient became confused and pulled on the PEG tube. The patient's nurse noted that marking on the tube now reads 1.5 cm. It was previously at 3 cm length. He now complains of abdominal pain, redness around the PEG site, and drainage. The next step in management is

C

Benign esophageal perforation Is a highly morbid condition requiring urgent recognition and Intervention. Compared with iatrogenic perforation, spontaneous esophageal perforation (Boerhaave syndrome) is associated with a higher mortality because of the delay in diagnosis. Regardless of etiology, the condition was historically treated with aggressive operative intervention ranging from thoracotomy with direct repair and patching to esophageal exclusion and cervical esophagostomy. however, despite Improvements in

A. exploratory laparotomy. B. bedside incision and drainage.

52

surgical technique and critical care, the reported postoperative leak rate remains between 30 and 40%. Because reported experience with esophageal perforation has Increased, less invasive options have become available, depending on the clinical circumstance.

smaller (<2 cm) tears, whereas larger defects require stent placement or sutures.

28. A 70-year-old man takes warfarin for adjunctive management of atrial fibrillation and has severe congestive heart failure at baseline. He will require urgent operative intervention for perforated sigmoid diverticulitis associated with diffuse peritonitis and a large quantity of pneumoperitoneum noted on CT imaging. His international normalized ratio is 4.0 (0.8-1.2). In addition to the administration of Vitamin K, which anticoagulation reversal regimen is best for this patient?

Nonoperative management may be appropriate in highly selected situations and seems to be moat applicable to iatrogenic perforations. Three criteria are typically used for nonoperative management of esophageal perforations: mild symptoms, perforation contained within the mediastinum and draining back into the esophagus, and minimal evidence of sepsis. If all 3 of these criteria are met, the patient can be reasonably treated with hyperalimentation, intravenous antibiotics, and cimetidine or proton pump inhibitors.

A. B. C. D.

Fresh frozen plasma Cryoprecipitate 3-factor prothrombin complex concentrate 3-factor prothrombin complex concentrate recombinant Factor Vila E. 4-factor prothrombin complex concentrate

A hybrid treatment of free esophageal perforations is now favored. Current algorithms use on esophageal stent to seal the perforation and a drainage procedure to control mediastinal contamination. The stent is placed endoscopically, avoiding the need for thoracotomy. The clinical applications of esophageal stents have expanded with increased use to include spontaneous perforations, iatrogenic perforations, anastomotic leaks, and cancer-associated perforations. The technique of stent placement involves oversizing with a 21- to 25-mm diameter self-expanding, covered stent in the longest possible length that does not cover the gastroesophageal junction or the arytenoid fold of the posterior oropharynx. The stent is placed while the patient is under general anesthesia, and the area is adequately drained at the same operation by video-assisted thoracoscopy, imageguided percutaneous drainage, or laparoscopy. A contrast esophagogram is performed 48 to 72 hours after stent placement to ensure that the leak has been sealed. The stent is usually removed within 14 days to minimize complications of stent migration, tracheoesophageal fistula, or aortoesophageal fistula.

ANSWER:

plus

E

Warfarin is a Vitamin K antagonist and common oral anticoagulant. When patients who take this medication develop life-threatening hemorrhage or require urgent operations or invasive procedures, rapid reversal of the anticoagulant effect is needed. In addition to Vitamin K, historically fresh frozen plasma and, less commonly, cryoprecipitate were the only options available for reversal. Large volumes of these blood products are required to achieve adequate reversal, which is problematic for patients who cannot tolerate rapid infusion of large volumes of colloid, such as patients with congestive heart failure. Additionally, the time required to achieve reversal can be lengthy with the use of these blood products.

Several factors preclude the hybrid approach: injury longer than 6 cm, injury that traverses the gastroesophageal junction or proximal cervical esophagus, or injury associated with a leak in the gastric conduit after esophagectomy. These injuries favor surgical repair. The most common complication is stent migration, which can be mitigated by using as long a device as possible and removing it within 2 weeks. Results of the hybrid approach have been positive in terms of leak occlusion, median hospital stay, total hospital charges, and survival. In a propensity-matched comparison of the hybrid technique with primary surgical repair, esophageal stent placement was as effective and resulted in a shorter length of stay, lower morbidity, and lower cost.

More recently, prothrombin complex concentrates (PCCs) have become available for reversal of the anticoagulant effect of warfarin. Three-factor PCC contains consistent quantities of FactorsII, IX, and X, but only trace quantities of Factor VII. Three-factor PCC may not completely reverse warfarin and can be augmented with recombinant activated factor VII (rFVIIa), which allows for complete reversal. Four-factor PCCs contain consistent quantities of Factors II, IX, X, and VII. Four-factor PCCs also contain complimentary quantities of critical anticoagulant enzymes such as protein C, protein S, and antithrombin HI, as well as heparin. Therefore, 4factor PCC achieves a more balanced replacement of the Vitamin K-dependent enzymes and may prevent untoward unopposed thrombosis during administration.

Other less invasive options are available to seal esophageal perforations, including endoscopic clips, suturing, and fibrin sealants. Experience with these options is limited, and none seems to offer any advantage over endoprostheses. Based on the limited studies, clips are currently recommended only for

When 4-factor PCC is compared with fresh frozen plasma for reversal of warfarin before urgent procedures, 4-factor PCC is effective and superior to fresh frozen plasma for rapid international normalized ratio (INR) reversal and effective hemostasis. When 4-factor PCC is compared with 3-factor

53

PCC, a more reliable correction of the INR is seen with 4factor PCC. Finally, when 4-factor PCC is compared with 3factor PCC + rFVIIa for warfarin reversal after traumatic hemorrhage, 4-factor PCC is associated with a less severe decrease in INR, a significant reduction in deep venous thrombosis rates, and a trend toward reduced mortality.

preoperative patient risk factors, it can calculate the risk of death, overall complication rate, pneumonia, heart problems, surgical site infection, urinary tract infection, blood clot, kidney failure, and estimated length of stay. NSQIP has not been validated for emergency surgery. The Eastern Cooperative Oncology Group (ECOG) Performance Status is a measurement of a patients' activities of daily living and measures patients' ability

29. What is the best independent predictor of the need to discharge to a postoperative care facility in elderly patients undergoing emergency general surgery?

30. A 72-year-old man underwent a Hartmann procedure for perforated diverticular disease and a pelvic abscess. His comorbidities included compensated chronic congestive heart failure and type 2 diabetes controlled by diet and an oral hypoglycemic agent. He was transiently hypotensive during the operation but responded promptly to fluid infusion. What is the most effective strategy to prevent perioperative acute kidney injury?

A. B. C. D.

American Society of Anesthesiologists classification Age Frailty score National Surgical Quality Improvement Program® Surgical Risk Calculator E. Eastern Cooperative Oncology Group performance status

ANSWER:

A. B. C. D. E.

C

Frailty is best conceived of as an age-related, multidimensional state of decreased physiologic reserves, which is associated with a decrease in resiliency, a loss of adaptive capacity, and an increase in vulnerability to stressors. Frailty is closely related to poor health care outcomes, including outcomes after surgery. Frailty is thought to be present in 10 to 20% of patients older than 65, and its occurrence and effects increase with age. In 2010,37% of all inpatient operations were performed on patients older than 65. Other than age, promoters of the frail state include uncompensated single end-organ dysfunction (e.g., kidney failure, heart failure), chronic infection, and malignancy. Not surprisingly, frail patients are much more likely to experience adverse outcomes after surgery.

Furosemide N-acetyl cystine Sodium bicarbonate Low-dose dopamine Maintenance of euvolemia

ANSWER:

E

Perioperative acute kidney injury (AKI) remains a common problem for surgical patients and worsens surgical outcomes, regardless of which outcome is considered. Varying degrees of AKI are seen in 50% of trauma patients and as many as 50% of patients undergoing aortic or hepatic surgeries. Even though the diagnosis of AKI may be increasing due to the increased sensitivity of the tools used to identify it, to date, there is no consistently effective strategy that is renal protective. This is because the etiology of AKI is multifactorial, and the etiology varies depending on the underlying condition that precipitated it (i.e., ischemia, sepsis, toxins). The published literature adds to the confusion due to the multiple criteria used to diagnose AKI. The American College of Surgeons Committee on Trauma defines AKI as a rise in serum creatinine to at least 3.5 mg/dL (0.41.3 mg/dL), whereas National Surgical Quality Improvement Program® uses a rise of at least 2 mg/dL over baseline. Multiple other scoring systems are used as well (table 30.1).

Baseline preoperative frailty determination can predict the outcome of surgical intervention, such as associated complications, increased length of stay, and destination at discharge (e.g., care facility), as well as hospital readmission, 30-day mortality, and long-term mortality. Frailty is superior to both American Society of Anesthesiologists class and age in predicting in-hospital complications and hospital length of stay. Many frailty scores are available to attempt to quantify this state, ranging from lengthy to brief. The Frailty index includes 50 questions related to patient history. It has gained popularity, because it does not require a physical examination, and the answers can be obtained from those who know the patient well.

Strategies used to reduce the risk have included the judicious use of intravenous fluids, avoidance of high-volume crystalloids with supraphysiologic chloride concentrations, invasive hemodynamic monitoring, and maintenance of renal perfusion with volume infusion with or without inotropes.

The American College of Surgeons National Surgical Quality Improvement Program (NSQIP®) Surgical Risk Calculator was developed in 2013 and revised in 2016, using highly detailed and accurate data from more than 2.7 million patients in 600 hospitals to estimate patient-specific postoperative complication risk for more than 1900 operations. Entering 23

Furosemide is unlikely to improve mortality or renal function because AKI is increased in patients who are or who become hypovolemic. Further, furosemide has an added toxic effect when given with other nephrotoxic drugs (e.g., aminoglycosides, nonsteroidal anti-inflammatory

54

medications). N-acetyl cystine may have some benefit in reducing contrast-induced AKI, but studies have not been consistent. It does have a very low risk of adverse side effects and is inexpensive. Likewise, sodium bicarbonate has been proposed to reduce the incidence of contrast-induced renal injury, because alkalization of urine might mitigate the toxic effect of the contrast medium. However, routine use in the intensive care unit is not recommended. In the late 1970s, low-dose (or renal-dose) dopamine was thought to be renal protective by augmenting renal blood flow and maintaining glomerular filtration. Despite overwhelming evidence to the contrary, renal-dose dopamine is still used inappropriately in many intensive care units around the world. There is evidence that the increased delivery of sodium to potentially ischemic tubules as the result of dopamine infusion might increase oxygen demand and potentiate oxygen debt.

Multiple studies have examined the optimization of fluid volume based on goal-directed strategies, which focus on cardiac output and oxygen delivery. Hypovolemia and hypervolemia are both associated with an increase in AKI in the perioperative period. Debate continues as to when goaldirected strategies should be initiated (pre- or intraoperatively), how long they should continue into the perioperative period, and which fluid is best for correction of hypovolemia. Unfortunately, the endpoints for optimization of hemodynamics do not exist for any of the techniques currently in vogue. Euvolemia is currently the best strategy, with careful monitoring of renal function in surgical patients at high risk for AKI.

System

R

1

F

L

E

RIFLE (Risk, Injury, Failure, Loss, End-Stage Renal Disease)

↑sCr x1.5 baseline or ↓eGFR >25% or UO <0.5 mL/Kg/ hour x 6 hours

↑ sCr x 2 baseline or ↓eGFR 50% or UO < 0.5 mL/Kg/ hour x 12 hours

↑sCr x 3 baseline or ↓GFR2 75% or sCr ≥ 4.0 mg/ dL and UO <0.3 mL/kg/hour x 24 hours or anuria x 12 hours

Persistent ARF x 4 weeks

Persistent ARF ≥3 months

Stage 1 ↑sCr>1.5-2.0 x baseline or UO <0.5 mL/kg/hour x 6 hours ↑sCr ≥ 0.3mg/ dL or 1.5-2.0 x baseline or UO <0.5 mL/kg/hour x 6-12 hours

Stage 2 ↑sCr 2-2.99 x baseline or UO <0.5 mL/kg/hour x 12 hours

Stage 3 ↑sCr ≥3 x baseline or ↑sCr ≥ 4.0 mg/dL with acute increase of 0.5 mg/dL or UO <0.5 mL/kg/hour x 6 hours ↑sCr ≥ 3x baseline or UO <0.3 mL/kg/hour x ≥ 24 hours or anuria ≥12 hours or need for RRT or ↓ eGFR <35 mL/ minute/1.73 m2 In patient <18 years

AKIN (Acute Kidney Injury Network)

KDIGO (Kidney Disease Improving Global Outcomes)

↑sCr 2-3 x baseline or UO <0.5 mL/kg/ hour ≥12 hours

Note. sCr = serum creatinine; eGFR - estimated glomerular filtration rate; UO = urine output; ARF = acute renal failure; RRT = renal replacement therapy. Table 30.1. Scoring criteria for acute kidney injury.

55

Problems in Related Specialties

72 hours, drainage is still incomplete, surgical drainage is warranted.

ITEMS 1-35 For each question, select the best possible response.

Category 4 parapneumonic effusions have pus within them. They have a high risk of poor outcome. Treatment is similar to Category 3 parapneumonic effusions.

1. A 34-year-old man with a right lower lobe pneumonia develops a parapneumonic effusion. What finding would confer the highest possibility of successful treatment with tube thoracostomy and antibiotics alone? A. B. C. D. E.

For this patient, a free-flowing effusion confined to 25% of the hemithorax describes a category 2 parapneumonic effusion, which would be amenable to drainage with a smallbore tube. A thickened parietal pleura and a pH less than 7.20 are criteria for a category 3 parapneumonic effusion, and purulent material within the pleural space is, by definition, a category 4 parapneumonic effusion. Although tube thoracostomy is typically the first-line therapy, it may not be sufficient. Glucose levels in the pleural effusion are not part of the criteria for categorizing parapneumonic effusions.

Pleural fluid pH less than 7.20 Thickened parietal pleura Purulent material in pleural space Free-flowing effusion confined to 25% of hemithorax Glucose of pleural fluid of 40 mg/dL

ANSWER:

D

A parapneumonic effusion is a pleural effusion arising in association with an infectious lung focus (e.g., pneumonia, lung abscess, infected bronchiectasis). Parapneumonic effusions develop in approximately 1 million patients annually in the United States. They can progress into an empyema when the effusion becomes infected. Three stages of empyema exist: (1) exudative, (2) fibropurulent, and (3) organizing. Adequate treatment of empyema includes antibiotic therapy to eradicate the infection and drainage procedure.

2. A 64-year-old man with chronic obstructive pulmonary disease presents with worsening shortness of breath, particularly when he is supine. His chest x-ray is remarkable for marked elevation of his left hemidiaphragm. The diagnosis of unilateral paralysis of the left hemidiaphragm is most reliably made by A. B. C. D. E.

The Health and Science Policy Committee of the American College of Chest Physicians developed a 4-tier categorization of parapneumonic effusions based on the pleural space anatomy, the bacteriology of the fluid, and the fluid chemistry. This schema stratifies patients with parapneumonic effusions into risk categories for poor outcomes and recommends whether drainage is necessary.

CT scan of the chest and abdomen. fluoroscopy. M-mode ultrasonography. MRI. phrenic nerve stimulation.

ANSWER:

Category 1 parapneumonic effusions are minimal, freeflowing effusions (<10 mm on lateral decubitus film) with very low risk of a poor outcome. Treatment includes antibiotics and observation.

E

Elevation of the hemidiaphragm may occur in a variety of settings. It is important deter mine whether the elevation of the hemidiaphragm is due to other anatomic factors present in the patient or to unilateral paralysis of the elevated hemidiaphragm. Unilateral diaphragmatic paralysis may be tumor related or may occur after a viral infection, surgery, or trauma. It may also be idiopathic. Paralysis resulting in chronic elevation of the of the diaphragm is more commonly unilateral than bilateral, affects the left side more often than the right side, and affects men more than women. In the absence of intrinsic lung disease, patients are often asymptomatic or have only a mild compromise of their respiratory mechanics Affected patients typically are hypoxemic and may have symptomatic orthopnea. Unilateral diaphragmatic paralysis, however, is rarely fatal.

Category 2 parapneumonic effusions are small to moderate, free-flowing effusions (>10 mm on lateral decubitus film but less than half the hemithorax) with negative Gram stain/culture of the pleural fluid and a pH greater than 7.20. They have a low risk of poor outcome. They require antibiotics and pleural fluid sampling with thoracentesis. Complete drainage of free-following fluid should be undertaken using a small-bore catheter. Category 3 parapneumonic effusions are large, free-flowing effusions involving half or more of the hemithorax on lateral decubitus film, loculated effusions, effusions with thickened parietal pleura, effusions with positive bacterial Gram stain/ culture, or effusions with a pH less than 7.20. They have a moderate risk for poor outcome. Patients should have antibiotic therapy and drainage of the parapneumonic effusion via tube thoracostomy. If, after 24 hours, drainage is incomplete, intrapleural fibrinolytics should be tried. If, after

The finding of an elevated hemidiaphragm on a chest X-ray or CT scan of the chest and abdomen is nonspecific and does not confirm the presence of a unilateral hemidiaphragm paralysis. Fluoroscopy is often cited as the method to confirm the diagnosis of unilateral hemidiaphragm paralysis. Diaphragmatic motion is recorded using fluoroscopy while

57

the patient sniffs. Brisk downward movement of the unaffected hemidiaphragm and paradoxical elevation of the affected hemidiaphragm is considered evidence of unilateral hemidiaphragm paralysis. Ultrasonography is also described as a novel method to detect paralysis of the hemidiaphragm. Both fluoroscopy and ultrasonography, however, are highly dependent on patient effort and are subject to operator error. The utility of real-time MRI for the diagnosis of unilateral paralysis of the hemidiaphragm remains to be determined. The most reliable test to quantify the mechanical function of the diaphragm is the measurement of the negative pressure generated by contraction of the diaphragm. in response to stimulation of the phrenic nerve. Electrical stimulation can be accomplished by using percutaneous needle electrodes in the neck or by single/ B bilateral magnetic cervical coils. Magnetic stimulation of the phrenic nerves is painless, reproducible, and easy to perform. Phrenic nerve stimulation is the best means to distinguish between the neuropathic conditions (phrenic nerve injury or compression) and myopathic conditions that can cause unilateral paralysis of the hemidiaphragm.

Figure 3.2.

ANSWER:

3. A 23-year-old man with a history of asthma presents with sudden onset chest and neck pain. He is afebrile, and physical exam demonstrates crepitus in his neck. His white blood cell count is 11,500/mm3 (3600-11,200/mm3). A chest CT scan is performed (figures 3.1 and 3.2). Initial management should consist of A. B. C. D. E.

A

The clinical history and CT scan findings are consistent with spontaneous pneumomediastinum (figure 3.3). This is a relatively rare condition, thought to be caused by alveolar rupture with tracking along the bronchovascular bundle to the mediastinum. Most patients present with chest pain, shortness of breath, and sometimes neck pain. Asthma is a common predisposing comorbidity. Smoking and illicit drug use are also relatively common. Careful review of the patient's history for trauma, aerodigestive instrumentation, or vomiting is important, in which case esophageal rupture would be a diagnostic consideration. White blood cell count may be mildly elevated, but fever is not common and would raise suspicion for visceral rupture. Diagnosis is suggested by physical exam and chest x-ray, showing pneumomediastinum or subcutaneous air in the neck, and subsequent chest CT scan is considered the standard of care for diagnosis. Diffuse pneumomediastinum is seen without pleural effusions (figure 3.4). If effusions or mediastinal fluid collections are present, they also raise concern for visceral rupture.

observation. esophagram. bronchoscopy. broad-spectrum antibiotics. transcervical mediastinal drain placement.

Given this patient's presenting history and clinical condition, observation without further testing is reasonable. Most patients need pain control with intravenous pain medication initially. Esophagram would be indicated when there is a history of emesis or in the presence of fever, pleural effusions, or a history of trauma or instrumentation. Bronchoscopy is not needed without a history of trauma. Antibiotics are not needed. Drain placement is not necessary either; the air typically resorbs within a 1 to 3 days.

Figure 3.1.

58

have negative cytology on the first evaluation. Second, nonexpansile lung after thoracentesis that persists despite chest tube placement in the absence of air leak defines "trapped lung," sometimes referred to as "pneumothorax ex vacuo." It is caused by a combination of tumor implants on the surface of the lung and chronic compression of the lung with scar tissue formation. Pneumothorax ex vacuo occurs in approximately half of all malignant pleural effusions. A trapped lung is a contraindication to pleurodesis of any type. The visceral and parietal surfaces are not in contact, and pleurodesis is doomed to fail. Indwelling pleural catheters are the only option for palliation in these circumstances. A second chest tube will not improve the situation with a nonexpansile lung in the absence of air leak and is an unnecessary step that will make the patient more uncomfortable with no benefit. Thoracotomy and decortication is overly aggressive in this 85-year- old man with a large effusion. Surgical morbidity and mortality are substantial for a problem that is almost certainly advanced malignancy; there is no therapeutic benefit to be had.

Figure 3.3. CT scan finding consistent with spontaneous pneumomediastinum

With expansion of targeted therapy and immunotherapy, many treatment options have minimal toxicity—obtaining tissue is critical as long as it is in line with the patient's goals of care. Thus, a single port video-assisted thoracoscopic surgery is a reasonable option to obtain pleural biopsies. This is the standard of care for evaluation of nondiagnostic cytology with a recurrent pleural effusion. The second issue is palliation of the large effusion and dyspnea. The patient's symptoms improved with thoracentesis even though the lung was trapped, which is common. Placing an indwelling pleural catheter that can be drained at home every 1 to 7 days effectively palliates dyspnea and improves quality of life, with less time in the hospital.

Figure 3.4. Diffuse pneumomediastinum is seen without pleural effusions

4. An 85-year-old old man presents with dyspnea and a chest x-ray with white out of the right chest. CT scan shows collapse of the right lung with large nonloculated right pleural effusion. Thoracentesis returns 1 L of serous fluid with improvement in his dyspnea. After thoracentesis, chest x-ray shows a pneumothorax. A chest tube is placed, which does not re-expand the lung. There is no air leak. Cytology indicates atypical cells. Malignant effusion is suspected. What is the next step in the management of this patient?

5. A patient is involved in a high-speed motor vehicle collision. He was restrained, and his airbag deployed. His primary survey is normal, and his secondary survey reveals no external injuries. He has complete imaging, and his only positive finding is a pneumomediastinum on CT scan of the chest. Which of the following statements is true regarding this finding?

A. Video-assisted thoracoscopic surgery (VATS) pleural biopsies with talc sclerotherapy B. A second chest tube and increased suction C. Right thoracotomy with decortication of the lung D. Pigtail catheter with doxycycline pleurodesis E. VATS pleural biopsies with tunneled pleural catheter

ANSWER:

A. Intubation for occult airway injury is necessary. B. Observation is appropriate treatment. C. Workup requires bronchoscopy and esophagogastroduodenoscopy. D. Mediastinal organ injury will be found in 12% of asymptomatic patients. E. Broad-spectrum antibiotics are required.

E

The case describe is a relatively common scenario in the management of malignant pleural effusions. First, pleural fluid cytology is not very sensitive, and clinical suspicion and clinical course should prompt further evaluation for malignancy. Atypical cells on cytology do not rule out malignancy, and at least 50% of the time, malignant effusions

ANSWER: 59

B

Pneumomediastinum can be spontaneous or related to trauma, instrumentation, severe coughing or vomiting, forced straining, or asthma. The most common presenting symptom is chest pain or discomfort, followed by dyspnea and neck pain. It is most often identified on CT imaging. In spontaneous pneumomediastinum, associated injury to the esophagus, trachea, or mediastinal structures is uncommon, and extensive workup, including swallow studies, bronchoscopy, or upper endoscopy, are not required. In patients who (1) have had recent instrumentation (e.g., bronchoscopy or upper endoscopy), (2) have sustained penetrating trauma, (3) had violent retching, or (4) have associated pleural effusion, the chance for mediastinal organ injury is higher and workup should be considered. Broadspectrum antibiotics are not indicated unless specific mediastinal contamination is present. Asymptomatic blunt trauma patients can be safely observed because mediastinal injury occurs in less than 2%.

Figure 6.2.

ANSWER:

Based on the history of retching and the imaging studies showing mediastinal air and gross soilage of the pleura and mediastinum, this patient almost certainly has suffered a spontaneous distal esophageal perforation secondary to emetic barotrauma, also known as Boerhaave syndrome (figures 6.3 and 6.4). Furthermore, this patient's condition has progressed to sepsis, most likely due to mediastinitis. As with all cases of septic shock, therapeutic intervention is urgent and usually begins with fluid resuscitation and broadspectrum intravenous antibiotics. In patients with Boerhaave syndrome presenting with gross mediastinal and pleural contamination, surgical intervention is the mainstay of therapy.

6. A 19-year-old college student with food poisoning has been vomiting and retching for 12 hours. He complains of chest pain, high fevers, and shortness of breath. A CT scan of the chest was obtained (figures 6.1 and 6.2). His vitals are as follows: Temp 38.9°C, heart rate = 120 beats per minute, blood pressure - 110/70 mm Hg, and white blood cell count 25,000/mm3 (3600-11,200/mm3). After administration of 2 L of normal saline and broad-spectrum antibiotics, what is the optimal next step in management? A. B. C. D. E.

B

Endoscopic placement of a covered esophageal stent Surgical repair of the esophageal tear Esophagectomy with cervical esophagostomy Endoscopic mucosal clipping Chest tube placement and 48 hours of antibiotics

Thoracotomy with primary esophageal repair is the standard of care in Boerhaave syndrome. The approach is commonly via left thoracotomy with possible phreno-laparotomy to widely expose the site of the tear, usually at the gastroesophageal junction. Copious irrigation of the mediastinum and pleural space plus debridement of all devitalized tissue is important. Once the devitalized tissue is cleared away, primary full-thickness esophageal repair is performed if possible. Reinforcement of the repair may be accomplished with a fundic wrap or pleural flap. The mediastinum and pleura are widely drained with large bore chest tubes. Esophagectomy with cervical esophagostomy would be recommended only in those patients presenting with widespread esophageal necrosis, in whom esophageal repair would be impossible. These patients usually present for treatment more than 24 hours after the inciting event.

Figure 6.1.

Some patients presenting early in the course of the condition with contained rupture and stable hemodynamics may benefit from nonoperative management. Nonoperative management should be considered only for patients with small ruptures contained in the mediastinum draining back into the esophagus, absence of pleural contamination, and no evidence of sepsis. If nonoperative management is attempted,

60

8. A spontaneous pneumothorax in a 66-year-old man with metastatic lung cancer and pleural effusion 9. A second episode of spontaneous pneumothorax in a 26year-old

ANSWERS:

A, C, B

Spontaneous pneumothorax should be managed promptly to avoid the complications of tension pneumothorax. The specific treatment is based on the patient's underlying condition. For an otherwise healthy 18-year-old patient, a small anterior chest tube directed apically is definitive treatment in more than 70% of patients presenting with a spontaneous pneumothorax. A low thoracostomy tube placed to allow drainage of an effusion is not necessary. For patients with high-risk activities such as flying in airplanes or scuba diving where another episode could be a fatal event, moredefinitive procedures such as pleurectomy, bleb resection, and pleurodesis are recommended. For patients who experience a second episode of spontaneous pneumothorax, a definitive procedure is also clearly indicated.

Figure 6.3. Esophageal rupture with air leakage into the mediastinum (white arrow) and left-sided pleural effusion.

For a patient with metastatic cancer, the issue becomes how to manage the pneumothorax and maximize his or her quality of life. A simple anterior tube will address the pneumothorax but will not be adequate to drain the effusion. A low tube thoracostomy may never be removed due to persistent exudate and may have a negative impact on a patient's quality of life. Thoracoscopy with lung resection may eliminate the source of the air leak (if it is technically possible) but is accompanied by much greater risk in this patient.

Figure 6.4. Complications of the esophageal rupture. Mediastinitis (induration of the mediastinal fat) and extensive leftsided pleural effusion with air pockets.

nothing by mouth, antibiotics, and chest tube drainage of pleural/ mediastinal fluid are necessary. Some of these patients may benefit from placement of covered esophageal stent, and some centers prefer stent placement as first-line treatment in these patients. Other centers describe the use of endoscopic clips to dose the mucosal defect associated with acute esophageal perforation. Although successful treatment of Boerhaave syndrome using endoscopic clips is possible, this method would apply only to patients early in the course of the condition and only in centers with considerable experience in advanced therapeutic endoscopy.

Elimination of the potential pleural space to allow a prompt discharge from the hospital should be the goal of therapy. Assuming the lung can be re-expanded, drainage of the chest with pleurodesis to obliterate the space can be performed at the bedside or in the operating room with sedation and pain control. Concerns do exist with the use of talc as a potential carcinogen. Animal studies have documented talc migration to sites remote from the area of application. This is not an issue, however, in this patient with metastatic disease. If the lung cannot be re-expanded, a tunneled catheter maybe appropriate because the effusion will likely recur.

Items 7-9 Each lettered response may be selected once, more than once, or not at all. A. B. C. D.

10. A 32-year-oId woman presents to the emergency department with left-sided pelvic pain. Pregnancy test is negative. Transvaginal ultrasound reveals a 3-cm tuboovarian abscess. After 2 days of intravenous antibiotics, she continues to have severe pain and fever. Repeat imaging shows the abscess has grown to 7-cm. What is the best next step in her care?

7. A spontaneous pneumothorax in an 18-year-old healthy male runner

A. Addition of antifungal therapy B. Transvaginal drainage C. Laparoscopic drainage

Anterior placement of a small pleural drainage catheter Apical pleurectomy and pleurodesis Pleurodesis with talc installation Thoracostomy tube placement mid axillary line, 8th intercostal space E. Thoracoscopy with lung resection

61

D. Left salpingo-oophorectomy E. Total abdominal hysterectomy with bilateral salpingooophorectomy

ANSWER:

ANSWER:

D

Endometriosis is a painful condition of the lining of the uterus. It affects women of reproductive age. The endometrium starts growing outside the uterus, manifesting as superficial peritoneal implants or cyst formation commonly referred to as an endometrioma. These cysts or endometriomas are formed when ectopic endometrial tissue within the ovary bleeds and results in a hematoma. They typically have fibrotic walls and surface adhesions, are filled with a thick chocolate-colored fluid, and are surrounded by ovarian tissue.

C

Tubo-ovarian abscesses can form as a result of pelvic inflammatory disease and are present in up to one-third of patients hospitalized for pelvic inflammatory disease. These abscesses form when an ascending infection travels up the fallopian tube, and they can vary significantly in size and severity. Most patients present with abdominopelvic pain and fever, but some patients also experience vaginal drainage. Physical exam will reveal lower abdominal pain and occasionally a pelvic mass. Pelvic exam may demonstrate cervical motion tenderness. Although transvaginal ultrasound is considered first-line imaging, most patients will undergo CT scanning to further clarify the size and location of the abscess.

There is debate regarding surgery for asymptomatic endometriomas. Ovarian-sparing excision for large endometriomas is appropriate. This results in improved pain control and less recurrence compared with ablation, drainage, or observation. Oral contraceptives may reduce recurrence, but they are not effective in treating an endometrioma once it is established.

The initial treatment of smaller (<4-5 cm) tubo-ovarian abscesses is intravenous antibiotics, which is successful in up to 70% of patients. Large abscesses (>7-8 cm) or patients with persistent or progressive symptoms require abscess drainage to ensure sepsis resolution. Offending pathogens are typically bacteria such as Chlamydia trachomatis and Neisseria gonorrhoeae; antifungal agents will rarely assist in abscess resolution.

12. A 71-year-old man has a 46-year history of nephrolithiasis, but no additional significant medical history. He has undergone 2 ureteral basket extractions of stones and 8 episodes in which the stones spontaneously passed. The patient recently complained of persistent left costovertebral angle and flank pain prompting CT scan, which shows large left renal calculus (figure 12.1). Workup on 2 occasions in the past demonstrated normal serum calcium and intact parathyroid hormone levels. After a successful percutaneous nephrolithotomy, stone analysis demonstrated the stones to be calcium oxalate-monohydrate and the following additional findings: sCa++ = 9.3,9.0, 9.1 (8.4-10.3 mg/dL) uVolume = 2.67 (>2.0 L/day) uCaOx = 3.25 (6-10) uCa++ = 162 (<250 mg/day) uOx = 31 (20-40 mg/day) uCitrate = 184 (>450 mg/day) upH = 5.489 (5.8-6.2) uUric acid = >0.800 (>0.800 g/day) s = serum; u = 24-hour urine

The approach to abscess drainage has evolved over time. Original recommendations included total abdominal hysterectomy with bilateral salpingo-oophorectomy; later recommendations focused on unilateral salpingooophorectomy. Such procedures are no longer used due to associated morbidity and infertility. Instead, simple abscess drainage can be done with excellent success rates. This is typically accomplished via laparoscopic drainage, with care taken to avoid spillage of the purulent material into the abdomen. Alternatively, percutaneous abscess drainage can be performed. Tubo-ovarian abscesses secondary to pelvic inflammatory disease typically occur in younger patients. The finding of tubo-ovarian abscesses in postmenopausal women should raise concern for gynecologic malignancy, which is present in almost half of older patients.

Management at this point should include 11. A 32-year-old woman with a medical history of a ruptured appendix undergoes an exploratory laparotomy for small bowel obstruction. A 6-cm endometrioma of the right ovary is discovered. What is the best treatment? A. B. C. D. E.

A. B. C. D. E.

Oral contraceptives Drainage Oophorectomy Ovarian-sparing excision Observation

62

(Tc)-99m sestamibi scintigraphy. repeat serum parathyroid hormone level. potassium citrate. bisphosphonate. furosemide.

A diagnosis of primary hyperparathyroidism is always a consideration in patient who are recurrent CaSF. The normal serum calcium levels and the history of normal intact serum parathyroid hormone (iPTH) levels excludes the diagnosis of primary hyperparathyroidism in this case. Further, the normal iPTH and the normal 24-hour uCa++ excretion make the diagnosis of normocalcemic hyperparathyroidism unlikely. As such, repeating a serum iPTH level or localizing an abnormal parathyroid gland by nuclear scanning is irrelevant. Thiazide diuretics are used, with or without alpha-blockers such as tamsulosin (which increases urinary pH and promotes passage of small stones and crystals from the collecting system and ureter) in patients with hypercalciuria, but furosemide is not appropriate. Bisphosphates, along with exercise, are used to prevent bone loss and renal stone formation in astronauts and patients subject to prolonged bed rest, but evidence for effectiveness of this strategy is largely theoretical and limited to small observational clinical data. Potassium citrate and potassium-magnesium citrate reduce the frequency of recurrent calcium-stone formation, particularly calcium oxalate stones. Citrate is thought to be an inhibitor of calcium oxalate crystal formation. This treatment would be appropriate in this patient with calcium oxalate stones and low levels of urinary citrate. Increasing the consumption of high citrate-containing fluids such as fruitB juices and lemonade is proposed as a potential strategy to reduce stone formation. Whether the beneficial effect is achieved through reducing calcium-oxalate crystal formation or by increasing urinary volume is debated.

Figure 12.1. Large left renal calculus.

ANSWER:

C

Despite decades of research and clinical experience, multiple issues regarding the physiochemical mechanisms of stone formation and treatment of the calculi in recurrent stone formers remain contested. Stone disease appears to be increasing in the United States, from 1:20 individuals to 1:11 over the past 20 years and is being associated with the increasing prevalence of obesity and metabolic syndrome. This is true across all ethnic groups and ages.

13. A 75-year-old, 82 kg, man is found on CT scan to have an enhancing left renal mass that measures 2.5 cm. Comorbidities include diabetes mellitus and compensated congestive heart failure. His serum creatinine is 1.6 mg/dL (0.40-13 mg/dL), and his estimated creatinine clearance is 46 mL/minute (97-137 mL/ minute). What is the most appropriate management approach?

Historically, men have been the prevalent stone formers, but recent epidemiologic data indicate that the gap between men and women is closing (10.6% men vs 7.1% women), as more women are being identified as calcium-stone-formers (CaSF).

A. B. C. D. E.

Most researchers agree that the initial process of stone formation begins with supersaturation of urine with solutes capable of crystal formation. Whether crystals that do form produce stones by crystal growth or crystal aggregation, the role of modulators (promoters and inhibitors of crystal formation) and the role of renal tubular epithelial injury (Randall plaques or plugs, lesions on which crystals may adhere) remains debated. Efforts to reduce supersaturation of urine by increasing fluid intake to a goal of 24-hour urine production of more than 2 L is the first step in efforts to prevent recurrent stone formation.

Cryoablation Active surveillance Radical nephrectomy Laparoscopic enucleation of the lesion Partial nephrectomy with hilar lymph node dissection

ANSWER:

B

Current literature reports an increase in the incidence of renal cell carcinoma (RCC), largely due to the increased availability and frequency of imaging studies used in patient management. The identification of incidental renal lesions in asymptomatic patients has confounded the decisions about how to proceed with workup and treatment. The greatest increase in the detection of RCC is in elderly patients, many of whom have comorbidities that complicate the decisionmaking process. As such, clinicians must determine

63

the balance between life expectancy related to the associated comorbidities and the anticipated cancer-related outcomes.

2.0 times normal reconsider their management options. Zerogrowth rate does not imply a benign nature of the lesion, but studies from Fox Chase Cancer Center and Vanderbilt University showed that there is no adverse impact on outcome that results from a delay in intervention of 6 to 12 months. The appropriate interval for repeat imaging is not clearly defined.

The goals of treatment include not only cancer-free survival but also preservation of renal function and quality of life. Radical nephrectomy, the standard of care for the management of malignant renal tumors, is associated with a 5-year survival of more than 95%, and approaching 100% in some reports, yet it comes with the cost of loss of renal function. This is acceptable in younger patients, but it increases the risk of cardiovascular events in the older patient population. Nephron-sparing surgery, including partial nephrectomy (open or laparoscopic) and ablative techniques (cryoablation and radiofrequency ablation), has equivalent short- and intermediate-term oncologic outcomes while minimizing the loss of renal function. For most early-stage RCCs, the routine addition of nodal excision, with both radical nephrectomy and nephron-sparing surgery, offers little in terms of survival benefit and is currently reserved for patients who have suspicious lymphadenopathy identified by preoperative imaging.

14. A 45-year-old man notes one episode of blood in his urine. He has no pain, urinary incontinence, urgency, or dysuria. He has no medical or surgical history and takes no medications. His vital signs are normal, and a physical examination is normal. His basic metabolic panel is normal. The only abnormality on urinalysis is 10 red blood cells per high powered field. The first step in his management should be A. B. C. D. E.

Several methods exist to determine survival based on comorbidities. The most thoroughly studied and extensively validated is the Charlson Comorbidity Index (CCI). For each unit increase in the CCI, the hazard ratio for death increases by 1.33 and increases by 1.35 for each 5 years of additional age. In the patient presented, the expected 1-year and 2-year survival is approximately 64% and 35%, respectively. When this is considered in the context that 40% of incidentally identified small renal masses (Tla, <4 cm) are benign and that another 40% will follow an indolent course, the data suggest that observation alone may be the prudent course.

urine cytology. repeated microscopic analysis in 6 months. cystoscopy. abdominal CT scan. antibiotics.

ANSWER:

C

Asymptomatic microhematuria (ASH) is defined as 3 or more red blood cells per high powered field in the absence of apparent benign causes such as trauma, infection, vigorous exercise, and viral illness. The workup focuses on detection of urologic malignancy in the upper and lower urinary tract, which occurs in 2.6 to 4% of patients with ASH. American Urological Association updated guidelines recommend a workup in patients age 35 years and older. Although intrinsic renal disease can cause hematuria, abnormal renal function or other microscopic abnormal findings in the urinalysis will typically point toward this, and a workup for neoplasm is still indicated in these patients. Multiphasic CT urogram with and without intravenous contrast to evaluate the renal parenchyma and upper urinary tracts is sensitive and specific (>90%) for neoplasms, with only a small risk of contrastrelated adverse events.

Clinical studies exist to support this approach. A Cleveland Clinic study of patients with enhancing renal lesions in an elderly cohort noted a 33% mortality at 2 years, but no mortality was related to RCC. In another study of more than 537 patients at least 75 years old, cardiovascular mortality exceeded mortality from renal malignancy and was not related to the treatment strategy used. Data from the prospective, multi-institutional Delayed Intervention and Surveillance for Small Renal Masses (DISSRM) study indicated that the 2-year survival for patients in an active surveillance arm versus a treatment arm was 96% and 98%, respectively, with a 5-year survival of 75% and 92%. Cancerspecific survival was 100% for the treatment cohort and 99% for the surveillance group. These results led the authors of this and other studies to conclude that for small renal tumors (1) active treatment is not associated with increased survival, and (2) overtreatment of these lesions is possible.

Ninety percent of bladder cancers present with hematuria, Cystoscopy is also recommended for all patients 35 years and older to evaluate for bladder neoplasm. Urine cytology is not sensitive enough, particularly for low-grade urothelial cancer, to be used in the routine workup of ASH, and negative results should not preclude a full workup. Repeat evaluation in 6 months is inadequate because substantial prior research is based on a single test result and has shown that repeated urinalyses are not necessary to improve the yield on a workup. A standard abdominal CT scan is not sufficient to detect renal neoplasms and would not be adequate. Antibiotics to treat an occult infection that may contribute to hematuria would overlook the possibility of neoplasm and are not recommended in the absence of a urinary tract infection.

Active surveillance, defined as interval imaging without active treatment, is appropriate for patients with advanced age or significant comorbidities that impose greater risks for operative mortality and complications. The mean growth rate for small renal masses is 0.28 cm/year, and as many as onethird of them have a zero-growth rate. Some authors suggest that patients whose lesions show elevated growth rates 1.5 to

64

B. C. D. E.

15. Which of the following statements is true regarding iatrogenic ureteral injury during abdominal operations? A. It occurs in 3% of radical excisions for uterine cancer. B. It is prevented by preprocedure stent placement. C. Thermal injury to the distal third of the ureter is best repaired primarily. D. Transection of the middle third of the ureter may be repaired with a Boari flap. E. Psoas hitch is indicated for ureteropelvic junction injuries.

ANSWER:

gastrointestinal hemorrhage. pneumoperitoneum. peritonitis. ileus.

ANSWER:

A

Neutropenic enterocolitis— also known as typhlitis, ileocecal syndrome, cectis, or necrotizing enterocolitis— is a severe condition that affects immunocompromised patients. Neutropenic enterocolitis was initially described in leukemic pediatric patients and is also reported in adults with hematological malignancies. The exact pathogenesis remains unknown, but the hallmarks of the disease include intestinal mucosal injury with neutropenia.

D

Ureteral injuries are a complication of extensive dissections in cases of advanced cancer, previous pelvic dissections, radiation, and infection. The incidence varies on the experience of the operator and the extent of disease. In cases of low pelvic surgery where the ureter is near the operative field, the reported incidence of injury ranges from 0.18 to 0.29% for gynecologic and 0.18 to 7.8% for colorectal procedures.

The clinical presentation includes fever, abdominal pain, neutropenia, and thickening of the bowel wall (most commonly the cecum and ascending colon). The thickness of the bowel wall correlates with mortality. Studies show a higher mortality rate in patients with ultrasound-measured colonic wall thickness of at least 10 mm compared with those with a thickness less than 10 mm. Other clinical findings of neutropenic enterocolitis include hypotension and diarrhea. The duration of neutropenia, lack of surgical intervention, and presence of severe sepsis are associated with a poorer overall survival. Gastrointestinal hemorrhage, pneumoperitoneum, peritonitis, and ileus, while associated with neutropenic enterocolitis, are not absolute diagnostic criteria.

Preprocedural ureteral stent placement should facilitate identification of the ureter in difficult cases. Risks associated with stent placement include infection, oliguria, hydronephrosis, and hematuria. Previously published series have not found a decreased incidence of injury. Surgeons hypothesized that the stent actually makes the ureter less pliable and more prone to injury during dissection. The undisputed advantage of the stent is the ability to recognize the injury promptly.

Items 17-19 Each lettered response may be selected once, more than once, or not at all.

If the injury to the ureter is not from electrocautery and is very small, primary repair over a stent can be performed. For thermal injuries or large segment transactions, the edges should be debrided to viable tissue before contemplating repair. Injuries in the distal third of the ureter may often be able to be reimplanted to the bladder, provided there is no tension. Bladder mobilization and attachment to the psoas muscle (psoas hitch) can facilitate this. When the bladder cannot be mobilized, creating a tube from the bladder (Boari flap) can be used to anastomose the bladder directly to the ureter. More proximal injuries are managed with temporary external drainage or renal autotransplantation to a position lower in the pelvis. Expertise from urologic or transplant surgeons is often necessary.

• • • • •

Pain in right lower quadrant caused by palpating the left lower quadrant Pain on extension of the thigh Pain on flexion of the thigh Pain on flexion and internal rotation of the hip Pain on flexion and external rotation of the hip

17. Obturator sign 18. Psoas sign 19. Rovsing sign

ANSWER: 16. A 7-year-old girl is receiving active treatment for acute leukemia and presents with abdominal pain. In addition to abdominal pain and neutropenia, the diagnostic criteria for neutropenic enterocolitis includes

D, C, A

For decades, the diagnosis of acute appendicitis was based on history and findings on physical examination. The typical history is anorexia, followed by epigastric or periumbilical pain that migrates to the right lower quadrant, peaking at approximately 4 hours. Physical examination findings are considered to be dictated by the physical location of the

A. bowel wall thickening on imaging.

65

appendiceal tip and the degree of inflammation (table 1719.1). Rovsing sign

Palpation of left lower quadrant resulting in pain in the right lower quadrant

Psoas sign

Patient positioned on left side and the right thigh is extended, stretching the iliopsoas muscle, eliciting pain if the tip of an inflamed retrocecal appendix is closely approximated

Obturator sign

Patient positioned supine, passively internally rotate the flexed right thigh, pain is elicited from an inflamed pelvic appendiceal tip near the obturator muscle

testis cannot be manipulated into the scrotum without causing pain due to traction on the spermatic cord.

A small proportion of retractile testes become pathologic. Indications for surgery include development of pain in the cord when manipulating the testis into the scrotum, a testis that will no longer stay in the scrotum and immediately springs back into the groin, and a decrease in testicular volume. Whenever these conditions are recognized, an orchiopexy should be performed. In one longitudinal study, approximately 16.3% of boys diagnosed with retractile testes required surgery during long-term follow-up. The mean age at diagnosis was 3.0±2.7 years and mean age at normal descent was 4.3±3.3 years. Of 64 retractile testes, 26 cases (40.6%) remained retractile until the end of adolescence. Close observation is required until the testis has descended into the scrotum or until the end of adolescence.

Table 17-19.1. Classic signs of acute appendicitis on physical examination.

These findings include right lower quadrant tenderness and frequently point tenderness at the McBurney point. Associated findings are localized peritonitis with guarding and rebound tenderness. Other classic signs on physical examination include Rovsing sign, psoas sign, and obturator sign. Flexion of the thigh or flexion and external rotation does not put the appendiceal tip in proximity to a muscle group.

21. What is the ideal way to determine whether the condition pictured (figure 21.1) is caused by an incarcerated hernia or infection? A. B. C. D. E.

20. Retractile testes A. B. C. D. E.

Ultrasound Aspiration Rectal examination Abdominal x-ray, including the scrotum Operative approach through the groin

are a variant of the undescended testis. are normal only in infants. require operation by the age of 5 years. must be in the scrotum some of the time. are normal at any age.

ANSWER:

D

A retractile testis is a variant of the normal testis. An active cremaster reflex is not uncommon in infants and young boys and may pull the testis out of the scrotum under normal circumstances, including when the child is cold or scared. This can be normal until puberty, when increased androgen secretion results in relaxation of the cremaster reflex, and the testis should be in the scrotum virtually all of the time. The 3 requirements of a retractile testis are as follows: (1) the testis can be palpated in the groin and manipulated into the scrotum; (2) after manipulation into the scrotum, the testis remains in the scrotum at least transiently, without being held there, until the cremaster reflex is stimulated again; and (3) the parent or guardian must notice that the testis is in the scrotum at least part of the time.

Figure 21.1.

ANSWER:

A

Groin pathology is common in infants and children. The differential diagnosis includes uncomplicated and complicated inguinal hernias and complicated and uncomplicated inguinal adenopathy. Medical and surgical treatments differ for each condition. The patient in the photograph may have either an incarcerated and possibly strangulated hernia or an inguinal abscess. Physical findings and diagnostic aids can assist in making the correct preoperative diagnosis. For example, a child who is eating

A retractile testis is clinically distinguishable from an undescended testis. In congenital undescended testis, the testis has never been present in the scrotum. An undescended

66

and drinking and stooling normally and looks well is unlikely to have an incarcerated hernia. A rectal exam with one hand on the groin may allow palpation of something caught in the inguinal canal and groin adenopathy or an abscess will be lateral to this.

22. Which of the following has consensus with respect to diagnosis and treatments biliary dyskinesia in adolescents? A. B. C. D. E.

Aspiration is relatively contraindicated unless there is diagnostic certainty of an abscess, because there might be a loop of bowel in the inguinal canal. An abdominal x-ray will show whether there are bowel loops below the inguinal ligament, but there could also be omentum stuck in a hernia sac and the bowel configuration may be normal despite the presence of a hernia. An ultrasound is the best diagnostic tool to examine the groin, because an abscess will have certain characteristics and be lateral to the inguinal canal. An ultrasound will also show tissue within the inguinal canal (figure 21.2).

Symptoms Diagnostic criteria Gallbladder ejection fraction Indications for surgery Lack of durable symptom relief after cholecystectomy

ANSWER:

E

Surgical treatment of children with a diagnosis of biliary dyskinesia is controversial and generally directed by symptoms as well as the results of a cholecystokininstimulated hepatobiliary scan. In a systematic review of 31 published articles, diagnostic criteria, indications for surgery, and gallbladder ejection fraction values varied. An abnormal gallbladder ejection fraction is defined institutionally and is generally less than 35%. However, there is no standard, validated method for assessing gallbladder motility in children. Some centers used a gallbladder ejection fraction up to 50% as abnormal, and 2 centers only looked at patients with hyperkinesia (gallbladder ejection fraction >70 or 80%). Diagnostic criteria were variable and included abdominal pain (e.g., "classic" biliary colic, right upper quadrant, epigastric), nausea and vomiting, fatty food intolerance, anorexia, weight loss, and a normal gallbladder ultrasound. There was no consensus on the indications for cholecystectomy. Some studies randomized patients to medical management or surgery, with nonoperative management generally including add blockers, again with variable success. Some investigators reported better success without surgery, and some reported better success with surgery.

Figure 21.2. Groin ultrasound demonstrating bowel in the hernia sac and normal testes.

If all else fails and there is still diagnostic uncertainty, the surgeon can approach this case as if it were a complicated hernia with a groin incision above the pathology (figure 21.3). If no hernia is found, this incision can be closed and the abscess drained through a counterincision directly over the area of maximal fluctuance in the groin.

Outcomes were reported in several studies after laparoscopic cholecystectomy. Four studies correlated poorer outcomes with a longer duration of symptoms. A lower gallbladder ejection fraction (<11% or <15% in separate studies) independently predicted success. Immediate relief of symptoms was more common than durable symptom relief, with many patients relapsing within several weeks of operation. Resolution was defined as complete cessation of symptoms without recurrence and ranged from 34 to 100%. Thirteen studies reported resolution of symptoms in 54 to 100% of patients for 2 weeks to 6 months after surgery. Duration of symptom resolution after surgery ranged from 6 days to 11 years, and only 66% of patients remained symptom free at long-term follow-up.

Figure 21.3. Groin incision used to exclude hernia. Groin abscess drained after initial incision closed.

67

23. The major difference between a patient-centered outcomes trial and a randomized controlled trial is A. B. C. D. E.

4 pediatric appendectomies annually appear to have similar outcomes to their pediatric surgical colleagues. Patients of general surgeons and pediatric surgeons have similar lengths of stay, readmission rates, and complication rates like wound infection as well as mortality for both complicated and uncomplicated acute appendicitis. The only statistical difference consistently identified is lower rates of negative appendectomy when the operation is done by pediatric surgeons compared with general surgeons.

choice of care. reduced cultural bias. decreased investigator bias. reduced enrollment of patients. no need for informed consent.

ANSWER:

A

In a controlled randomized trial, patient and family receive oral and written information regarding the trial and consent to the trial but are randomized to the treatment. In a patientcentered (patient and family choice) trial, patients and family receive oral and written information regarding the trial and consent to the trial, but they choose the treatment they prefer.

25. An 11-year-old girl with Crohn terminal ileitis is being treated with azathioprine and steroids, but she continues to have episodes of abdominal pain and partial bowel obstruction. Her mother is concerned that she is losing weight and becoming socially withdrawn. What is the most appropriate next step in management?

The patient and family choice design produce potential selection bias by the investigator. Bias can be minimized by using a standardized scripted consent process and welldefined inclusion and exclusion criteria. Patient and family choice design also may lead to unbalanced patient characteristics that may result in differences in treatment. For example, in a published patient and family choice trial done in a single institution comparing nonoperative to operative management of pediatric uncomplicated acute appendicitis, parents or guardians whose primary language was not English were more likely to select nonoperative management. However, those choosing nonoperative management as a personal choice were less likely to worry about recurrent appendicitis than those who were randomized to nonoperative treatment. The ability to choose the preferred treatment generally enhances enrollment in a clinical trial.

A. B. C. D. E.

ANSWER:

For children with moderate to severe Crohn disease, initiation of biologic therapy with either infliximab or adalimumab has a significant clinical benefit in 88% of patients; 50 to 60% of these patients will achieve clinical remission, with a similar number able to wean off oral steroids. Infliximab has the greatest success in this arena, with adalimumab typically reserved for second-line treatment.

shorter lengths of stay. lower surgical site infection rates. lower negative appendectomy rates. higher readmission rates. lower pelvic abscess rates.

ANSWER:

C

For children with inflammatory bowel disease, an important indicator of inadequate treatment is failure to thrive, which can include weight loss, growth delay, social withdrawal, and inactivity. The patient described is worsening despite an immunomodulator and ongoing steroids. Escalation of therapy is warranted.

24. Comparing the outcomes of general and pediatric surgeons, children with appendicitis managed by pediatric surgeons have A. B. C. D. E.

Addition of aminosalicylates Switch from azathioprine to 6-mercaptopurine Infliximab infusion Ileocolectomy Diverting loop ileostomy

Aminosalicylates have limited efficacy in Crohn disease for both children and adults and are not useful for severe disease. Although the child may need to remain on an immunomodulator, conversion from one to another (e.g., 6mercaptopurine) is not an adequate escalation of therapy. Ileocolectomy is warranted for failure of medical therapy and should be performed if biologies are ineffective. A diverting loop ileostomy may partially alleviate symptoms, but it does not remove the diseased bowel segment. This surgery may worsen the child's body image, so it should be reserved for the rare situation in which resection is not feasible.

C

Appendectomy is the most common urgent surgical procedure performed in the pediatric patient population. In the current era of subspecialization and regionalization, questions remain as to what surgical procedures general surgeons should be performing versus sending patients on to specialty surgeons or centers. The most important factor appears to be surgeon volume, not presence or absence of subspecialty training. General surgeons who perform at least

26. An otherwise healthy 2-year-old boy is referred for an asymptomatic umbilical hernia found on routine exam.

68

Examination reveals a reducible hernia with a 1-cm fascial defect. What is the most appropriate treatment? A. B. C. D. E.

Observation Abdominal binder Primary repair Repair with polypropylene mesh Repair with bioabsorbable mesh

ANSWER:

A

Small, asymptomatic umbilical hernias are very common in young children. They are present in 10 to 20% of all infants, and up to 75% of those bom prematurely with weights less than 1500 g. Complications of these hernias are rare, and incarceration or strangulation occurs in less than 1% of patients.

Figure 27.1.

Many small hernias will close spontaneously, including 80% of hernias smaller than 1 cm and 96% of hernias smaller than 0.5 cm. For this reason, the most appropriate management of small asymptomatic hernias in the first 4 years of life is observation. Abdominal binders have no proven benefit for symptom control, defect closure, or prevention of complications.

ANSWER:

C

Management of ingested and aspirated foreign bodies is a common task for surgeons caring for children, but button batteries are a special emergency. The severity of the injury of button batteries has increased for 2 reasons: (1) larger sizes can become lodged in the airway and esophagus and (2) more powerful lithium cells are being ingested (1-25% of ingestions). Button batteries produce hydroxyl radicals in the mucosa, causing a caustic injury from high pH, not from electrical, thermal, or pressure effects.

27. A 4-year-old boy presents to the emergency department with a suspicion of having swallowed a round shiny object 3 hours ago. On examination, the boy appears to be comfortable with minimal symptoms but does point to his throat when asked if he has any pain. A plain radiograph is shown (figure 27,1). Which of the following statements is true regarding management of this patient?

Button batteries have a characteristic double halo appearance on radiography that allows ready distinction from coins. Button batteries in the stomach are rarely associated with perforating gastric injury, although there is a report of an infant with a gastric perforation. One patient with a gastric button battery subsequently died from an aortoenteric fistula from a missed esophageal injury, leading to a recommendation for endoscopic evaluation of all button battery ingestions with their removal. A 2010 survey of button battery injuries included tracheoesophageal fistula (47.9%), esophageal perforation (23.3%), esophageal strictures (38.4%), vocal cord paralysis from recurrent laryngeal nerve injury (9.6%), mediastinitis, cardiac arrest, pneumothorax, and aortoenteric fistula (7 of 13 fatalities). The presence of a button battery in the esophagus is a surgical emergency requiring immediate esophagogastroduodenoscopy (EGD). In the EGD photo (figure 27.2), mucosal damage is already present.

A. Distinguishing button batteries from coins is difficult via radiography. B. Significant esophageal stricture is an unlikely complication. C. Immediate removal is indicated. D. Esophageal mucosal injury is caused by pressure effects. E. If this object were in the stomach, it could be observed.

69

Figure 28.1. Typical presentation of pediatric intussusception.

Figure 27.2. Endoscopic image of the esophagus after disk battery removal showing mucosal damage (arrows) along with metallic staining (arrowheads) from ingestion of button battery.

Confirmatory testing may be considered for an intussusception with an ultrasound, which will demonstrate a double donut in the area intussuscepted (figure 28.2).

Follow-up EGDs are indicated. For injuries dose to the aortic arch, MRI or CT scan should be done to rule out inflammation tracking from the esophagus. All battery ingestions should be reported to the National Battery Ingestion Hotline at the National Poison Control Center, to be included in its database (202.625.3333).

28. An 18-month-old girl presents with frequent crying spells after periods of normal activity. Her birth and development were normal. Immunizations are up to date. She had a lowgrade fever last week with rhinorrhea, which resolved after 72 hours. On exam, she is afebrile. Her abdominal exam is remarkable for some guarding but no peritoneal signs. Which of the following is the best next step in her management? A. B. C. D. E.

Urgent laparotomy Air contrast enema Chest x-ray Broad-spectrum antibiotics Upper gastrointestinal series/small bowel followthrough

ANSWER:

B Figure 28.2. Ultrasound demonstrating a double donut in the area

This clinical presentation is typical of pediatric intussusception (figure 28.1). Retrospective series show an average age at presentation of 20 months. This child does not have peritoneal signs, and laparotomy is clearly not indicated. Many children receive an initial plain film of the abdomen, which is often revealing for lack of bowel gas in the right lower quadrant. A chest x-ray will not examine this area and is not indicated for the resolved upper respiratory symptoms. Broadspectrum antibiotics are not indicated for a history of a viral illness that has resolved.

intussuscepted.

An upper gastrointestinal series/small bowel follow-through will identify the area of bowel obstruction but carries the risk of aspiration and does not provide a means to treat the obstruction. Air contrast enema will show the area of the intussusception (figure 28.3) and provide a noninvasive means of reduction. This may be done with ultrasound guidance or with fluoroscopic imaging to assess completeness. Recurrent intussusception rates in the first 48 hours are low, ranging from 2.7 to 6.6% in a recent meta-

70

for repair must consider this risk in addition to the risk of testicular atrophy and incarceration without repair. Randomized controlled clinical trials documented the greater success of laparoscopic repair compared with open, with fewer episodes of recurrent herniation (0.2% vs 1.0%). Laparoscopy also allows for bilateral repair without additional incisions. Controversy exists, however, regarding the contralateral inguinal region in an infant presenting with unilateral hernia. Although exceedingly low, the risk of bilateral herniorrhaphy is to both vas deferens during the dissection, which could result in later impotence. If the left side is the inguinal region with a hernia, many surgeons would argue that bilateral repair is indicated because the left processus vaginalis normally closes first. Recent series have failed to document a decreased incidence of occult contralateral patent processus vaginalis or inguinal hernia based on side of presentation (p = .099). Most authors now advocate discussing with the parents the risk of developing a future contralateral inguinal hernia (approximately 2.4% over 2 years) with the risk of another surgery and then deferring to parental judgement. Sex also further confounds the issue of whether to do a contralateral exploration in a female child. Meta-analysis has found no significant difference in the incidence of a patent processus or contralateral hernia in female children compared with male children (p = .37).

Figure 28.3. Intussusception of proximal bowel. analysis. Multiple studies document the successful management of these patients on an outpatient basis.

29. Which of the following statements is true regarding pediatric inguinal hernias?

30. A 5-year-old girl who weighs 17 kg presents to the emergency department with a penetrating injury to abdomen and evisceration of some small bowel. At exploration, she has 2 enterotomies, violation of the peritoneum in the pelvis, and what appears to be an expanding serosanguinous fluid collection in zone III. Her enterotomies are repaired with a bowel resection and primary anastomosis. She has been hemodynamically normal throughout the case. An appropriately sized bladder catheter has been placed with little return of urine; what has returned is bloody. What is the next step in management?

A. Laparoscopic repair is not recommended. B. The incidence of a patent processus vaginalis is greatest at birth. C. Inguinal hernias rarely incarcerate. D. Risk of contralateral hernia is higher for male children. E. Postoperative apnea is an expected complication in fullterm infants.

ANSWER:

B

A. B. C. D. E.

The processus vaginalis is the abdominal wall opening that allows for the descent of the testes in utero. In girls, the ovaries remain intra-abdominal and the round ligament lies within the inguinal canal attached to the uterus. In both sexes, the processus vaginalis involutes between 25 to 35 weeks of gestational age with the left side obliterating first. Persistent communication can result in fluid filling the scrotum (hydrocele) or inguinal hernia because the bowel can pass through the patent processus vaginalis. The incidence of a patent processus vaginalis is greatest at birth and gradually declines with age.

Angioembolization Suprapubic drainage Exploration of zone m No further intervention Pelvic packing

ANSWER:

C

This patient has an expanding fluid collection in zone HI after a penetrating traumatic injury. The differential diagnosis includes a vascular injury and urinary tract injury. Because there is bloody urine output after placement of a urinary catheter, a urologic injury must be suspected. She is hemodynamically normal and responds to fluid resuscitation, so it is safer to proceed even if a vascular injury is still a concern.

Inguinal hernias can incarcerate in infants, and timely repair is indicated. Incarceration rates are reported in 4.6 to 28% of patients. Special circumstances, such as with the preterm infant, need to be carefully evaluated because postoperative apnea or need for mechanical ventilation is much more common in this group (occurring in up to 9%). The ideal age

71

In an analysis of the national trauma databank, operative bladder repair was associated with improved survival in 816 pediatric patients. Because angioembolization will treat only the arterial injuries, it would not be the next step in a patient with a suspected bladder injury. Suprapubic drainage is an option to treat isolated bladder injuries but because a laparotomy is under way, the recommendation is for repair in hemodynamically normal patients. Pelvic packing might be an option if this patient were hemodynamically abnormal with active pelvic bleeding, but this maneuver would not address the source of the urine leak. It could be addressed at the time of reoperation, however. In a hemodynamically normal patient, the next best option would be to explore zone III.

occurred late in pregnancy, the intestines appear normal. However, in most cases, the intestine is thickened, edematous, and covered with exudate. Although patients with gastroschisis do not typically have associated anomalies, the protruded intestine is prone to vascular compromise, leading to a higher incidence of bowel atresia. Malrotation represents failure of normal return and rotation of the midgut during fetal development The midgut normally herniates into the umbilical cord during the sixth week of development and returns to the abdominal cavity between the 10th and 12th week. As it returns, it undergoes a 270° counterclockwise rotation along the axis of the superior mesenteric artery. The duodenal C-loop traces the path of the rotation, and the duodenum becomes fixed in the retroperitoneum. If rotation is incomplete, the cecum remains in the epigastrium, while the Ladd bands that normally fix the cecum and duodenum to the retroperitoneum continue to form. These bands cross from the cecum to the lateral abdominal wall, creating the potential for duodenal obstruction.

Items 31-33 Each lettered response may be selected once, more than once, or not at all. A. B. C. D. E.

Cardiac anomalies Defect to the right of the umbilicus Ladd bands Cystic fibrosis Currant jelly stool

Invagination of intestine into itself is known as intussusception, which is the leading cause of intestinal obstruction in children aged 6 months to 2 years. Intussusception typically begins in the terminal ileum and extends into the ascending or transverse colon, dragging the associated mesentery. The majority of cases are idiopathic and associated with a recent viral illness. Hypertrophied Peyer patches from the terminal ileum are thought to serve as lead points. Affected infants typically present with the sudden onset of severe, intermittent, and cramping abdominal pain that occurs at 15- to 20-minute intervals. Up to 70% pass stool with a mixture of blood and mucous, giving it the appearance of currant jelly.

31. Meconium ileus 32. Omphalocele 33. Gastroschisis

ANSWERS:

D, A, B

Cystic fibrosis in infants is associated with the production of viscous meconium due to pancreatic enzyme deficiencies and abnormal secretion of intestinal chloride. The thickened meconium may become impacted in the distal ileum, resulting in a high-grade intestinal obstruction. Affected patients present with failure to pass meconium and progressive abdominal distention in the neonatal period. Diagnosis is confirmed by contrast enema, demonstrating a microcolon. In uncomplicated meconium ileus, treatment with water-soluble contrast agents is often successful at relieving the obstruction.

34. A 10-year-old boy presents with 3 days of worsening diffuse abdominal pain. A CT scan shows perforated appendicitis with moderate diffuse peritoneal fluid. Which of the following statements is true regarding irrigation and suction versus suction alone of the peritoneal cavity with laparoscopic appendectomy for perforated appendicitis? A. There is no difference in the rate of intra-abdominal abscess formation. B. Irrigation reduces the number of nonpelvic intraabdominal abscesses. C. Irrigation reduces length of stay. D. Irrigation reduces duration of antibiotics. E. Irrigation increases the number of abscesses per patient.

Two abdominal wall defects in the newborn—omphalocele and gastroschisis-have different etiologies and presentations. Omphalocele is a defect of variable size in the abdominal wall through which bowel and solid viscera protrude but are covered by peritoneum and amniotic membrane. Omphalocele is thought to represent an arrest in development. Approximately 60 to 70% of affected newborns have associated anomalies, including cardiac anomalies in up to 40% of cases. By contrast, gastroschisis is characterized by an anterior wall defect to the right of the umbilicus that is usually less than 4 cm in size. Intestines freely protrude through the defect, but there is no covering sac. If the rupture

ANSWER:

A

For decades, surgical trainees have been taught "the solution to pollution is dilution." High-volume irrigation in patients such as this was thought to reduce the number of abscesses

72

and improve outcome. Two recent studies addressed this specific issue in pediatric appendicitis. In one study, children with perforated appendicitis were randomized to suction alone or peritoneal irrigation with a minimum of 500 mL of saline irrigation and suction. The rate of postoperative intraabdominal abscess formation was surprisingly high, at approximately 19%, but the same for both groups. Each group averaged approximately 2 abscesses per patient in which they formed. The location of abscess formation was also similar, with almost half occurring in the pelvis. In a similar study, the rate of intra-abdominal abscess was 4.9% and 5.0%, respectively. The length of stay and duration of antibiotics were similar for each group in both studies.

35. Which of the following statements is true regarding malrotation in the pediatric population? A. The rate of diagnosis drops off significantly after 5 years of age B. The incidence of subsequent volvulus is greater with laparoscopic Ladd procedure than with an open Ladd procedure. C. Malrotation found incidentally in a 15-year-old does not need to be fixed. D. The majority of cases are diagnosed incidentally. E. Ladd procedure includes release of Ladd bands, broadening of the mesentery, appendectomy, and placement of the small bowel to the left and the colon to the right.

ANSWER:

B

Malrotation occurs in 1/6000 live births. Approximately 90% of patients present with acute symptoms in the first year of life. Even older children may present with lifethreatening symptoms; therefore, an incidental finding of malrotation in a teenager should lead to surgical management. Urgent management of malrotation is generally performed with laparotomy, although a laparoscopic approach is possible. All of the components of the Ladd procedure can be performed laparoscopically, although it appears the incidence of recurrent volvulus is greater with a laparoscopic approach. Whether performed open or laparoscopically, the Ladd procedure includes untwisting of any volvulized bowel, release of cecal bands, broadening of the small bowel mesentery on its underside, incidental appendectomy, and placement of the small bowel to the right and the colon to the left.

73

Skin and Soft Tissue

Both hyperbaric oxygen therapy and intravenous immunoglobulins are adjuncts in the treatment of necrotizing soft tissue infection. Because there are few well-performed studies of hyperbaric oxygen therapy in necrotizing soft tissue infection, the value of this therapy is controversial. There may be a mortality benefit for patients with type 1 infections; however, hyperbaric oxygen therapy should not be initiated until all necrotic tissue is debrided. Intravenous immunoglobulins may have a mortality benefit for patients with type 2 (Group A streptococcal) infections but should not be used in type 1 or 3 infections. Immunoglobulins should not be used

ITEMS 1-24 For each question, select the best possible response. 1. A 50-year-old man with poorly controlled diabetes and a BMI of 45 has redness and pain in his scrotum and right thigh. His white blood cell count is 25,000/mm3 (360011,100/mm3), and he is febrile at 40.1°C. He undergoes debridement of his scrotum and right thigh. A necrotizing infection involving the subcutaneous tissue and fascia is confirmed; however, the underlying muscle appeared viable. He is admitted to the intensive care unit on vasopressors and continued-on broad-spectrum antibiotics. Eight hours after admission, his lactic acid increases from 5 to 8 mmol/L (2 mmol/L), his white blood cell count is 40,000/ mm3, and his vasopressor requirement has increased. What is the next step in his management? A. B. C. D. E.

2. A 58-year-old woman presents after a biopsy of a right anterior thigh Pigmented lesion. The pathology result showed superficial spreading melanoma 1.0 mm thick; Clark level II; no ulceration; mitosis rate 1/mm2. Her physical examination reveals a biopsy site on the right anterior thigh with no lymphadenopathy noted. The appropriate surgical therapy is excision of the biopsy site with

Re-exploration in the operating room Addition of micafungin CT scan of the abdomen and pelvis Hyperbaric oxygen therapy Intravenous immunoglobulin therapy

ANSWER:

A. B. C. D. E.

A

Several risk factors for the development of a necrotizing soft tissue infection include age older than 60, diabetes mellitus, obesity, intravenous drug abuse, and an immunocompromised state. There are 3 distinct types of necrotizing soft tissue infection. Type 1 infections are the most common type and are polymicrobial, consisting of Gram-positive and Gram-negative infections. Type 2 infections are caused by Group A streptococcus, and type 3 are typically caused by Clostridium species.

1-cm margin. 1-cm margin with sentinel lymph node biopsy. 2-cm margin. 2-cm margin with sentinel lymph node biopsy. 1-cm margin with groin lymphadenectomy.

ANSWER:

B

Surgical treatment of melanoma is 2-fold. First, the surgeon must decide on the adequate margin to provide "local control." In this patient, the depth of her melanoma is 1.0 mm. Appropriate margin measurements for this depth would be 1 cm (table 2.1).

The mainstay of management of necrotizing soft tissue infection is operative debridement of all involved necrotic tissue. Complete debridement of all necrotic tissue is the most important part of management, and most patients will require at least 2 operative debridements to address all of the necrotic tissue. This patient has ongoing septic shock and a worsening inflammatory state. He should be taken back to the operating room immediately for re-exploration and further debridement, if necessary. Additional imaging is not indicated unless an extension to or from the abdomen is suspected based on operative findings

Tumor Thickness (mm)

Excision Margin (cm)

In situ

0.5-1.0

0.1-1.0

1.0

1.01-2.0

1.0 or 2.0

2.01-4.0

2.0

>4.0

2.0

Table 2.1. Recommended Surgical Margins of Excision for Melanomas

In addition to surgical debridement, broad-spectrum antibiotic therapy is used until culture results are available. Coverage of Gram-negatives, Gram-positives, and anaerobes should be ensured with antibiotic choice. A cell wall inhibitor and a protein synthesis inhibitor should be initiated based on local resistance patterns. An antifungal is generally not indicated unless mucormycosis is suspected or confirmed on culture. Amphotericin B is the drug of choice because azoles and echinocandins do not have any efficacy against mucormycosis.

Next, the surgeon must assess the draining lymph node basin. This is an intermediate thickness (1-4 mm) depth melanoma. Lymph node sampling through the use of the sentinel lymph node biopsy technique is warranted. A 1-cm margin with sentinel lymph node biopsy is appropriate. Other responses combine inappropriate margins or ignore an assessment of the lymph node drainage basin. A formed groin exploration is not indicated with no clinical lymphadenopathy or biopsy-proven nodal disease.

75

3. A 37-year-old man presents with a 3-mm pigmented lesion in the nail bed of the left first toe. Biopsy confirms a subungual melanoma in situ. What is proper treatment? A. B. C. D. E.

for invasive tumor, amputation of the distal phalanx should be performed.

Shave biopsy Wide local excision Amputation at the interphalangeal joint Amputation at the metacarpal-phalangeal joint Ray amputation

ANSWER:

4. Two days after starting broad-spectrum antibiotics and debridement for a necrotizing infection of the buttock, a patient is doing well with vitals as follows: Temperature = 37.6°C Heart rate = 60 beats per minute Blood pressure = 120/56 mm Hg Respiratory rate = 10 breaths per minute White blood cell count = 6000/mm3 (3600-11,100/mm3) He has a wound that appears as shown in figure 4.1. What is the appropriate next step of management?

B

Subungual melanoma is a rare variant of malignant melanoma, making up approximately 0.7 to 3.5% of all melanomas. Because they are often unnoticed or misdiagnosed, subungual melanomas tend to present with more advanced depth. This finding led to the mistaken idea that subungual melanomas represent a more aggressive malignancy, which in turn led to the recommendation that subungual melanomas should be treated with amputation of the involved digit at the most distal interphalangeal joint. Recognition that subungual melanomas are not more aggressive than similar-depth melanomas challenged this dogma. The movement is toward a more conservative, digitsparing approach that includes nail complex excision and local flap reconstruction or application of a full-thickness skin graft. This treatment appears to be particularly suited to subungual melanoma in situ.

A. B. C. D. E.

An exhaustive literature review compared wide local excision and amputation as treatment for subungual melanoma. In the review, 82 patients from 14 studies who underwent wide local excision were compared with 801 patients from 26 studies who underwent amputation. The wide local excision patients tended to have less advanced disease, with more frequent melanoma in situ (46% vs 11%), fewer positive regional lymph nodes, and fewer distant metastases (2.4% vs 11.3%). In contrast, the amputation group had a lower local recurrence rate (2.2%) than the wide local excision group (12.2%). However, many of the wide local excision patients with local recurrence had deep melanomas that were more appropriate for amputation. Interestingly, regional and distant recurrences were similar between the 2 groups (14.6% vs 13.3%).

Place wet-to-dry dressing changes Place a vacuum-assisted closure device Place a skin graft overlying the open muscle Perform further debridement Start antifungal treatment

Figure 4.1.

ANSWER:

D

Necrotizing infection is a severe nonpurulent infection commonly caused by mixed organisms, including anaerobes, that can lead to systemic inflammatory response syndrome and multiorgan failure. The 3 types of necrotizing infections are classified on the basis on the microbes found. Type 1 includes Gram-positive cocci, Gram-negative rods, and anaerobes including Clostridium and Bacteroides. Type 2 is usually associated with beta hemolytic Streptococcus or Staphylococcus. Type 3 is associated with Vibrio vulnificus and is commonly seen in SCUBA divers (associated with infection after water exposure). After diagnosis, prompt surgical debridement and intravenous antibiotics are the appropriate first line of treatment for all of these infections. After initial debridement, it is important to reassess wounds periodically and to debride all necrotic tissues to viability before placing a skin graft or definitive closure using myocutaneous flaps. Although this patient appears to be

Amputation as the treatment for subungual melanoma does not appear to provide superior outcomes compared with wide local excision especially for in situ lesions. Randomized or prospective studies that elucidate whether one treatment is superior over the other for deeper lesions are still needed. The lack of reporting Breslow depths in many of the clinical reports make dear recommendations for treatment of deeper lesions difficult to ascertain. An approach using lesion depth similar to cutaneous melanoma is gaining support. Current data imply that melanoma in situ is treated appropriately with a digit-sparing approach. Once the conservative approach is chosen, it is important to document complete tumor clearance. If pathologic assessment reveals a positive margin

76

clinically normal, the presence of devitalized tissue should immediately prompt further debridement to healthy tissues, because rapid clinical deterioration can occur in this scenario.

follow-up of most DFSP patients but may be done for those individuals with fibrosarcomatous DFSPs. Ninety-five percent of DFSPs have a chromosome 17 to 22 translocation that results in a fusion mutation in platelet derived growth factor (PDGF). Accordingly, inhibitors of PDGF mutations, such as the tyrosine kinase inhibitor imatinib mesylate, were studied in DFSP. In a prospective phase II multicenter trial of imatinib given orally for 2 months preoperatively for large or recurrent DFSPs, tumors reduced in size in 36% of cases, which rendered some of the subsequent operations less morbid. This and other published evidence has led to studies of imatinib for locally advanced and metastatic DFSP.

Fungal infections in short-duration necrotizing infections of the soft tissue are rare, and there is no evidence to support the use of antifungals in this setting. Although wet-to-dry dressings can achieve simple debridement, they are not appropriate for necrotizing infections with large amounts of devitalized tissue. Negative pressure wound dressings (via vacuum-assisted closure devices) improve wound healing in large open wounds but require clean wound be

5. Which of the following statements regarding management of dermatofibrosarcoma protuberans is true?

6. A 37-year-old woman undergoes a femoral sentinel node biopsy for a 1.7-mm nonulcerated melanoma of the thigh. Pathology on the single node reveals a micrometastasis. She inquires about a completion groin node dissection. She should be told that this procedure

A. Wide local excision with a 5-cm margin is the standard of care. B. Overall survival is poor. C. The tumor is radioresistant. D. A sentinel lymph node biopsy should be performed. E. Imatinib can be used for locally advanced and metastatic disease.

ANSWER:

A. B. C. D. E.

is followed by postoperative radiation. improves disease-free survival. does not increase her risk of lymphedema. finds positive nodes in 50% of patients. is combined with a pelvic node dissection.

E ANSWER:

Dermatofibrosarcoma protuberans (DFSPs) are rare, usually low-grade, soft tissue tumors that typically run an indolent course. DFSPs have a slight predominance in men, individuals aged 30 to 50 years, and truncal locations, although they are also found on the extremities and head and neck. Microscopically, these tumors contain spindle cells that stain for CD34.

B

The Multicenter Sentinel Lymph Node Trial-II (MSLT-II) randomized patients with melanoma who had micrometastases in their sentinel nodes to completion node dissection or observation. More than 1900 patients entered the trial and were followed for a median of 43 months. Although the dissection group had a slightly higher diseasefree survival (attributed mostly to control in the regional nodes), melanoma-specific survival was the same between groups. That is, completion dissection can decrease nodal relapse in patients with tumors in the sentinel nodes but does not improve overall survival. This finding was not surprising, given that only 20% of completion dissections yield additional tumor in the "nonsentinel" nodes (11.5% in the MSLT-II dissection arm) and that tumor in the nodes is a predictor but not necessarily a determinant of distant metastases.

Many pathologists and surgeons consider the word sarcoma in the name DFSP to be a misnomer, because the tumor has a generally excellent survival rate and rarely metastasizes. However, like sarcomas, DFSPs have an infiltrative nature and thus often recur locally if not adequately excised. A margin of 2 cm is usually recommended. Also, like sarcomas, DFSPs are staged by size, depth, and grade. They spread to the lungs and rarely to nodes if they do metastasize. Therefore, sentinel lymph node biopsy is not part of routine care of DFSPs. These tumors are sensitive to radiotherapy, especially when given in high doses (>60 Gy). Radiotherapy is usually reserved for tumors that recur, especially if they become locally unresectable. However, in one study, radiotherapy was used postoperatively and prevented local recurrence in 12 of 14 treated patients.

Accordingly, fewer surgeons are offering completion dissection, although patients mav still request it. Patients must understand that these operations may improve their nodal control rates but will not affect overall survival and are associated with significant rates of lymphedema (24.1% in the MSLT-II trial). If a completion groin dissection is done for a positive sentinel node, it is not usually extended into the pelvis, because this technique adds to the lymphedema risk without significant benefit. Radiation therapy is not indicated

Ten to 15% of DFSPs contain a high-grade fibrosarcomatous component. These tumors appear to account for the approximately 2% of DFSPs that can metastasize, usually to the lungs. Routine chest x-rays do not have a role in the

77

for positive sentinel nodes and is reserved for large nodal tumor burden or extranodal extension.

sometimes withheld in the case of sensitive body areas such as the hands, feet, and around the eye; in the patient presented there is no such concern. Conversely, no benefit of adjuvant chemotherapy is seen, and it is usually reserved for metastatic disease. Checkpoint inhibition (specifically by the antiprogrammed cell death-1 antibody avelumab) is of value in metastatic MCC and is approved by the US Food and Drug Administration for the treatment of advanced disease. This is important, because MCC relapses in up to 48% of cases after surgery and radiation therapy.

7. A 61-year-old man presents with the arm lesion shown in figure 7.1. A biopsy is read as Merkel cell carcinoma. Physical examination is otherwise unremarkable. A metastatic workup is negative. What is the recommended treatment? A. B. C. D. E.

Mohs micrographic surgery Check-point inhibitor therapy Wide local excision only Wide local excision and postoperative radiation Wide local excision and postoperative chemotherapy

8. Keloids A. do not spontaneously regress. B. have improved cosmetic appearance after 6 weeks of topical 5-fluorouracil. C. more commonly occur in white patients. D. extend beyond the margin of the original wound. E. have excessive amounts of type III collagen.

ANSWER:

Keloids and hypertrophic scars are the result of abnormalities in the wound healing process. Both are associated with pain, erythema, pruritus, poor cosmesis, and restricted function. On histologic examination, keloids extend beyond the border of the wound. Clinically, they are noted to rarely regress and most often recur. By contrast, hypertrophic scars extend only to the wound edge, frequently regress, and are unlikely to recur. Both contain excessive amounts of type I collagen in the dermis and underlying tissue, and there is an increase in the ratio of type I to type III

Figure 7.1.

ANSWER:

D

D

Merkel cell carcinoma (MCC) is an aggressive neuroendocrine cutaneous tumor, arising from the Merkel cells of the skin. Merkel cells are mechanoreceptors in the basal layer of the dermis that lay close to nerve endings and secrete neuropeptides in response to light touch. Thus, MCCs stain with neuroendocrine stains. Although MCC is often called "rare," the incidence is the United States is rising rapidly. Series of thousands of cases have been reported. MCCs are linked to immunosuppression and carcinogenesis associated with a polyomavirus.

9. A 60-year-old obese man undergoes excision of an abnormal skin lesion on his left lower leg. His physical exam is otherwise unremarkable. He has no palpable adenopathy. Final pathology shows a 0.85-mm thick ulcerated melanoma with 6-mm clear margins. The most appropriate next step in his management is A. B. C. D.

sentinel lymph node biopsy. re-excision to a 1-cm margin. re-excision to a 1-cm margin and sentinel node biopsy. re-excision to a 2-cm margin and left groin node dissection. E. left groin node dissection.

Because they are both skin and neuroendocrine related, MCC have features in common with both melanomas and small cell lung cancers (SCLCs). Like melanomas, they are related to ultra violet radiation damage and should be staged with wide excision and sentinel node biopsy. Like SCLCs they are sensitive to radiation therapy. Retrospective studies and a randomized study showed that radiation therapy reduces the local relapse rate significantly and may improve overall survival. Thus, adjuvant radiation therapy to the primary site after resection is the standard of care. Radiation therapy is

ANSWER: 78

C

The primary treatment for melanoma is surgical excision. The status of the regional nodes is an important prognostic indicator for melanoma patients. Patients with positive sentinel nodes are known to have worse outcomes.

ANSWER:

Soft tissue masses are common, and most that are excised by nononcologic surgeons are benign. Lesions larger than 3 cm and those that are deep (i.e., below the subcutaneous fat), should be considered potentially malignant soft tissue sarcomas (STS). Ideally, these patients should be referred to centers that have multidisciplinary teams with substantial experience in dealing with STS before performing a diagnostic biopsy. Some circumstances make the "ideal" impractical, and many practitioners prefer to make a diagnosis before a referral. If a biopsy is performed, specific principles must be followed. Three-dimensional imaging, preferably contrast-enhanced MRI, should be performed on all lesions larger than 3 cm before the biopsy.

The extent of surgical excision for melanoma is guided by pathologic depth from biopsy. The goal of melanoma surgery is to achieve histologic negative margins and therefore decrease risk of recurrence. The surgical margins for invasive melanoma should be at least 1 cm for melanomas that are 1mm thick or less. For melanomas that are greater than 1 to 2 mm, adequate margins are 1 to 2 cm. For melanomas that are thicker than 2.0 mm, margins should be 2 cm. No data support a margin greater than 2 cm. Sentinel lymph node biopsy (SLNB) was developed as a procedure with less morbidity than complete node dissection. In melanoma, the sentinel lymph nodes are identified by injection or radioisotope and lymphoscintigraphy or injection of blue dye in the operating room. SLNB has lower morbidity than complete node dissection and provides information that can direct further surgery, adjuvant therapy, and overall prognosis. Patients with intermediate thickness melanoma have approximately 20% risk of having a positive sentinel node. Currently, SLNB is not recommended for patients with Tla tumors (<1 mm in depth) but should be considered in patients with thickness of 0.76 to 1.00 mm, especially if other adverse parameters are present (ulceration, increased mitotic rate, positive deep margin, young patient age). SLNB is recommended in all patients with melanoma thickness of 1 mm or greater.

Core needle biopsy, usually multiple passes (6-8 core specimens), should be obtained. Some centers recommend image-guided biopsies be performed, particularly for deep lesions. There is even a recommendation that biopsy be guided by F-FDG-PET scanning to obtain samples from the most metabolically active portion of the lesion. Fine needle aspiration biopsy, while not ideal, is acceptable in the hands of an experienced surgeon or radiologist with the back-up of experienced sarcoma pathologists. STS rarely metastasizes via lymphatics, and sentinel lymph node biopsy is rarely indicated. If incisional biopsy is performed, the incision should be placed longitudinally over the axis of the extremity to allow for en bloc excision of the scar should re-excision of the primary lesion be required. Meticulous hemostasis is mandatory. This larger lesion (>3 cm) is suspicious for an STS, and an excisional biopsy would not be appropriate.

This patient has a melanoma that is 0.85-mm thick. The margin at time of excisional biopsy is 6 mm wide. The goal margin for a 0.85-mm melanoma is 1 cm. Therefore, this patient should undergo further excision of the area to achieve wider margins. In addition, this patient meets criteria for SLNB because the tumor is ulcerated even though the melanoma is less than 1 mm thick. There is no indication for a left groin dissection until SLNB is performed, and sentinel lymph nodes are found to be positive for metastatic disease.

Traditionally, inadvertent, nononcologic resection of an STS was thought to have a negative impact on local recurrence risk and patient survival. Recent evidence suggests that local recurrence is more related to the biologic aggressiveness of the tumor and an extensive delay in performing an oncologically appropriate re-excision. Further, overall survival and disease-specific survival is not affected when the reexcision is timely and meets oncologic standards, even if residual tumor is identified.

10. A 53-year-old healthy man presents for evaluation of a lesion located on the lateral aspect of his left thigh. He identified the lesion approximately 6 months ago after a minor contusion to the area while playing tennis. He states that it has not changed in size and that it is not tender. On physical examination, the lesion appears to be deep within the vastus lateralis muscle, is approximately 5 cm in size, and is hard and nonmobile. The most appropriate next step is A. B. C. D. E.

A

11. A 52-year-old man with poorly controlled diabetes presents with right leg pain Physical examination reveals crepitus and a blue discoloration of the lateral aspect of the lower extremity. His blood pressure is normal and his heart rate is 100 beats per minute. He does have multiple blisters of different sizes. Dorsalis pedis pulse is not palpable, but triphasic flow is audible on a portable Doppler Plain radiograph of the lower extremity reveals evidence of gas tracking along the fascia. Laboratory analysis reveals hyperglycemia, elevated Hb A1C, and leukocytosis with a left shift. What is the first step in his treatment?

core needle biopsy. PET scan. fine needle aspiration. excisional biopsy. sentinel lymph node biopsy.

A. Tissue biopsy B. Limited debridement C. Hyperbaric oxygen

79

D. MRI E. Broad-spectrum antibiotics

Variable

Value

LRINEC Score Points

C-reactive protein (mg/L)

<150 >150 <15

0 4 0

15-25 >25

1 2

Hemoglobin (g/dL)

>13.5

0

Sodium (mmol/L)

11-13.5 <11 >135

1 2 0

Creatinine (mg/dL)

<135 ≤1.6

2 0

Glucose (mg/dL)

>1.6 ≤ 180

2 0

>180

1

WBCs (x 10,000 cells/mm3)

ANSWER:

E

Necrotizing soft tissue infections (NSTI) are characteristically severe and progress quickly. These infections can be accompanied by sepsis, multisystem organ failure, and death. Clinical presentation of necrotizing soft tissue infection is accompanied by swelling and erythema. Nonspecific skin changes include multiple blisters differing in size, skin bullae, and gray skin coloration. Pain out of proportion to the exam is the most consistent finding. Crepitus is described but rarely present on exam. Laboratory analysis can support this clinical diagnosis. Laboratory findings are also nonspecific and should be monitored, including serum C-reactive protein, complete blood count, and electrolytes. These laboratory parameters are commonly suggested to assess a patient for the risk of a NSTI. The LINREC score is calculated to estimate the risk. Unfortunately, a score of greater than 6 has a sensitivity of 68% and when it is greater than 8, the sensitivity is 41%. Surgical evaluation and exploration is often required to make a definitive diagnosis. Once necrotizing soft tissue infections are identified, debridement should not be limited, because prompt, wide debridement is key.

LRINEC Score Points, Sum ≤5

Risk Category

NF Probability

Low

<50%

6-7 Intermediate 50-70% >8 High >50% The LRINEC (Laboratory Risk Indicator for Necrotizing Fasciitis) score: A tool for distinguishing necrotizing fasciitis from other soft tissue infections. Table 11.1. Laboratory Risk Indicator for Necrotizing Fasciitis (LRINEC) Score System.

Imaging can facilitate diagnosis but should not delay treatment. Gas in the soft tissues can sometimes be seen on plain radiograph films. A CT scan with contrast can demonstrate the lack of enhancement of the fascia and is more specific for necrotizing soft tissue infection than air or edema. On MRI, T2-weighted images can show focal areas of abnormal signal intensity in the fascia. Another diagnostic intervention that is not helpful and can delay appropriate treatment is a tissue biopsy.

12. A 65-year-old man is diagnosed with an ulcerated 2-mm depth melanoma, He undergoes wide local excision and sentinel lymph node biopsy. What is the most powerful predictor of survival? A. B. C. D. E.

The mainstay of treatment is surgery. Delay in surgical management increases mortality and morbidity. Rapid surgical debridement can strongly impact outcome and survival. All necrotic tissue should be aggressively debrided until all remaining tissue is viable and healthy. Multiple debridements are usually required.

Sentinel lymph node status Breslow depth Ulceration Age Site of melanoma

ANSWER:

A

Sentinel lymph node biopsy (SLNB) is used in the management of melanoma and other cutaneous neoplasms. The clinical utility and prognostic significance of SLNB in melanoma patients is most clearly demonstrated in patients with intermediate thickness melanoma, defined as 1 to 4 mm in depth.

Early and aggressive antibiotic therapy is recommended. Broad-spectrum empiric antibiotics directed at the most likely organisms are necessary but should not replace surgical debridement because the antibiotics do not penetrate dead and dying tissue. Hyperbaric oxygen is proposed as an adjunctive therapy after surgical treatment. Intravenous immunoglobulins have also been evaluated as adjunctive therapy. These are not routinely recommended.

The Multicenter Selective Lymphadenectomy Trial (MSLTI) randomized patients to wide excision of the primary melanoma plus SLNB or wide excision plus postoperative nodal observation. Eligibility criteria included patients with Clark level HI and Breslow thickness of at least 1 mm or

80

Clark level TV/V with any Breslow thickness. Patients with intermediate thickness (1-4 mm) melanoma comprised the primary study group. Ten-year melanoma-specific survival rates differed by nodal status: 62.1 ± 4.8% among those with positive sentinel lymph nodes compared with 85.1 ± 1.5% among those with negative sentinel lymph nodes. On multivariate analysis, including all known confounding factors, sentinel node status was the strongest predictor of disease recurrence or death in patients with intermediate thickness melanoma. In addition, among patients found to have sentinel lymph node metastasis, the performance of further nodal dissection was associated with improved local control and melanomaspecific survival, albeit small. In a large retrospective study of patients with thick melanoma reported to the Surveillance Epidemiology and End Results program database, SLNB status provided important prognostic information; however, performance of the SLNB was not associated with a survival benefit.

Rating

Description

Clear

0 abscesses, 0 draining fistulas, 0 inflammatory nodules, and 0 noninflammatory nodules

Minimal

0 abscesses, 0 draining fistulas, 0 inflammatory nodules, and presence of noninflammatory nodules

Mild

0 abscesses, 0 draining fistulas, and 1-4 inflammatory nodules OR 1 abscess or draining fistula and 0 inflammatory nodules

Moderate

0 abscesses, 0 draining inflammatory nodules OR 1 abscess or draining inflammatory nodule OR 2-5 abscesses or draining inflammatory nodules 2-5 abscesses or draining inflammatory nodules

Severe Very severe

13. The patient pictured (figure 13.1) has hidradenitis suppurativa Which of the following statements regarding this disease process is true?

fistulas, and ≥5

fistula

and

fistulas and <10 fistulas and ≥10

>5 abscesses or draining fistulas

Table 13.2. Hidradenitis suppurativa Physician's Global Assessment Scale. Medical management should be exhausted before any surgical intervention. Medical management includes

A. This lesion represents Hurley stage II disease. B. Medical management includes tumor necrosis factor blockers. C. A colostomy should be performed at the time of excision. D. Unroofing and curettage of the sinus tracts is adequate definitive therapy. E. Topical antibiotics should be avoided.

• • • • • •

Psychological screening for issues of depression, socialization, and body image Smoking cessation Zinc gluconate 90 mg/day for 3 weeks Topical resorcinol-sulfur dressing daily ad infinitum Methylprednisolone 1-week taper Intravenous ceftriaxone (1 dose), then oral rifampin, moxifloxacin, metronidazole for 12 weeks, then rifampin and moxifloxacin for 12 more weeks

If all these fail, dapsone and or cyclosporine can be considered. Finally, consider the tissue necrosis factor blocker adalimumab. Surgical management includes unroofing and curettage of sinus tracts that are persistent. This drainage or source control helps heal the chronically inflamed tissues. Definitive surgical management includes full-thickness excision of the involved skin. Split-thickness skin grafting is used to cover excised areas. Large series of perineal excisions are reported, with only a rare need for colostomy to prevent excessive wound soilage.

Figure 13.1.

ANSWER:

Items 14-16 Each lettered response may be selected once, more than once, or not at all.

B

A. B. C. D.

In this patient with hidradenitis suppurativa, there is a wide area with chronic skin changes most consistent with Hurley stage III disease implying diffuse involvement with multiple intermediate tracts (table 13.2).

81

Antibiotics, warm compresses Antiviral therapy, warm compresses Incision and drainage Nailbed removal

≥1

E. Wide debridement of involved skin and fascia

The thumb and index finger are most commonly affected. Rapid treatment is necessary, because ischemia and necrosis can occur. S. aureus is the most common organism. Antibiotics and drainage are needed. To drain a felon, a digital block is first performed. A longitudinally oriented incision is made over the area of maximal fluctuance. Care must be taken to not cross the distal interphalangeal joint crease, because this can cause a flexion contracture. Probing the wound proximally should not be done to prevent spread of the infection into the flexor tendon sheath. The wound is then packed loosely with gauze, and a finger splint is applied. The hand is elevated and splinted.

14. Fluctuance under the nail bed 15. Erythema and pain with clear vesicles of the finger 16. Erythema, pain, and edema of the fingertip pulp

ANSWERS:

D, B, C

The most common cause of hand infections is trauma, although diabetes, HTV, and malnutrition can also increase the risk. Ninety percent of hand infections are Gram positive, and methicillin-resistant Staphylococcus aureus is becoming more prevalent. Paronychia is an infection of the nail bed or the periungual soft tissue (figure 14-16.1).

Herpetic whitlow is a viral infection of the distal finger caused by the herpes simplex virus (HSV; figure 14-16.3).

Figure 14-16.1. Paronychia Manicures, artificial nails, nail biting, and hangnails are common causes. The infection usually begins on the side of the nail bed and can spread beneath the nail if left untreated. Initial treatment is with antibiotics and warm compresses. Local drainage with the bevel of an 18-gauge needle or Freer elevator may be adequate if the abscess is localized to the side of the nail. If diagnosis is delayed, the abscess can extend below the nail, requiring partial or complete removal of the nail and exposure of the nail bed. A felon is a painful, closed-space infection of the finger pulp, most commonly caused by a puncture wound or splinter (figure 14-16.2).

Figure 14-16.3. Herpetic whitlow. Infection by genital (HSV-2) or oral (HSV-1) types are clinically similar. Herpetic whitlow is most commonly found in women with genital herpes, children with herpetic gingivostomatitis, and healthcare workers exposed to orotracheal secretions. Diagnosis is usually made by careful history and physical. It is important to distinguish whitlow from a bacterial infection because performing an incision and drainage on a herpetic whitlow can lead to a secondary bacterial infection and spread of the virus. Unlike a felon, herpetic whitlow does not appear on the pulp of the digit. A single finger is painful, erythematous, and edematous. Vesicles appear early in the process and coalesce by day 14, at which point it can be mistaken for a paronychia or felon. Herpetic whitlows usually resolve in 2 to 3 weeks, and treatment is supportive. A dry dressing should be applied and antiviral therapy should be considered. None of these localized infections requires wide debridement of skin and fascia.

Figure 14-16.2. Felon of the thumb with abscess formation. 82

17. A 56-year-old hepatitis C positive shrimper presents to the emergency department complaining of severe pain in hie left calf. On exam, he is tachycardic, tachypneic, and hypotensive; his left calf is purpuric with hemorrhagic bullae. The most likely organism responsible for this presentation is A. B. C. D. E.

Treatment of NSTIs rests on empiric broad-spectrum antibiotic coverage and source control via radical debridement of all infected tissue. Penicillin G with clindamycin or metronidazole is recommended first-line therapy. Clindamycin is preferred in type 2 NSTIs as well as clostridial infections due to its inhibition of toxin production. Antimicrobial coverage for type 3 NSTTs involving Vibrio and Aeromonas species, should consist of a third-generation cephalosporin/carbapenem and doxycycline.

Streptococcus pyogenes. Staphylococcus aureus. Clostridium perfringens. Vibrio vulnificus. Pseudomonas aeruginosa.

ANSWER:

18. A 45-year-old woman presents to the emergency department with a 4-cm fungating necrotic wound on her breast 10 days after an excisional breast biopsy. You suspect pyoderma gangrenosum. Which of the following is an associated condition with similar pathophysiology?

D

In the United States, Joseph Jones, a former Confederate surgeon during the Civil War, was one of the first individuals to describe necrotizing soft tissue infection (NSTI) in an 1871 report on 2642 cases of "hospital gangrene" he treated during the conflict. The 46% mortality rate he reported has dropped only to 25% with modem antibiotic, surgical, and critical care therapy. Such high mortality is due, in part, to the rapid, fulminant, and aggressive nature of these infections. With the onset of pain, patients can progress to widespread soft tissue necrosis, overwhelming sepsis, and death within hours. Risk factors for NSTTs include diabetes, obesity, immunosuppression, malnutrition, alcohol abuse, and intravenous drug use.

A. B. C. D. E.

Categorization of NSTIs fall into 3 types. Type 1 NSTTs are the most common and are polymicrobial in nature. They include aerobic and anaerobic species such as Streptococcus, Staphylococcus, Enterococcus, Pseudomonas, Enterobacteriaceae, Acinetobacter, Bacteroides, and Clostridium. Type 2 NSTIs are monomicrobial infections caused by Streptococcus pyogenes or Staphylococcus aureus. They can at times occur together. Finally, type 3 infections are monomicrobial infections caused by Clostridium perfringens, Vibrio vulnificus, and Aeromonas species. C. perfringens accounts for more than 70% of clostridial infections and is sometimes classified as a type 1 NSTI. Its alpha and theta toxins contribute to the myonecrosis and its resultant "gas gangrene." V. vulnificus is a marine bacterium found in raw seafood and along warm coastal areas. Aeromonas species are found in fresh or brackish water, soil, and wood. Hepatic dysfunction is a specific risk factor for V. vulnificus infection.

Scalded skin syndrome Hidradenitis suppurative Bullous pemphigoid Toxic epidermal necrolysis Necrotizing soft tissue infection

Figure 18.1.

ANSWER:

B

Pyoderma gangrenosum (figure 18.2) has long been a

Early signs of NSTIs include pain out of proportion to exam, hyperthermia, erythema, tachycardia, fever, bronzing of the skin, anesthesia of the skin, edema, and epidermolysis. Progression to later signs include hemorrhagic bullae, foul odor, "dishwater" drainage, dermal gangrene, crepitus, shock, rapid progression of erythema/pain/edema, and multisystem organ failure. Signs particular to V. vulnificus and Aeromonas species include fulminant development of hemorrhagic bullae, subcutaneous bleeding, purpura, necrosis, and gangrene.

Figure 18.2. Pyoderma gangrenosum. 83

diagnosis of exclusion has long been a diagnosis of exclusion and is often overlooked other than in patients with known inflammatory bowel disease, inflammatory arthritis, and hematological disorders. Pyoderma gangrenosum represents one of several autoinflammatory neutrophilic dermatitides that also include aseptic febrile neutrophilic dermatosis and hidradenitis suppurativa (figure 18.3).

Bullous pemphigoid (figure 18.4) and toxic epidermal necrolysis (figure 18.5) are exfoliative conditions that present

Figure 18.4. Bullous pemphigoid.

Figure 18.5. Toxic epidermal necrolysis.

Figure 18.3. Hidradenitis suppurativa.

as macular papular rashes or blistering superficial wounds similar to a second-degree bum rather than with a fungating ulcerative lesion. Scalded skin syndrome (figure 18.6) results from staphylococcal exotoxin-induced detachment of the epidermal layer.

These autoimmune conditions are potentially caused by gene mutations associated with collagen diseases, vascular conditions (including Wegener granulomatosis), myeloid dyscrasias, malignant tumors, and other hereditary disorders. These noninfectious conditions are associated histologically with dense accumulation of polymorphonuclear leukocytes with or without vasculitis, likely due to an unregulated interleukin-8 inflammatory response. Postoperative pyoderma gangrenosum is a variant that develops in surgical sites within 2 weeks postoperatively, with a mean of 7 days after surgery. Unlike typical pyoderma gangrenosum, which presents with lower extremity lesions, postoperative pyoderma gangrenosum occur most commonly in the breast, thorax, and abdomen. Postoperative pyoderma gangrenosum remains a diagnosis of exclusion but should be considered in the differential diagnosis of postoperative wound dehiscence to avoid unnecessary antibiotics or further wound debridement, especially in a patient with a personal or family history of hematologic dyscrasia, inflammatory bowel disease, or rheumatoid arthritis. Appropriate management involves reducing inflammation through anti-inflammatory and immunosuppressive therapy.

Figure 18.6. Scalded skin syndrome.

84

Clinically, it is associated with generalized erythema and superficial epidermal peeling rather than the maculopapular rash seen with toxic epidermal necrolysis and it lacks mucous membrane involvement. Necrotizing soft tissue infection (figure 18.7) represents a life-threatening, soft-tissue infection characterized by rapidly spreading inflammation and skin cellulitis and necrosis of the subcutaneous fat and fascia.

Medical therapy may be appropriate for less severe disease. Topical antibiotics such as clindamycin are associated with significant improvement in patients with minimal or mild disease. Systemic antibiotics are necessary in cases involving more severe or widely spread lesions; complete response rates of 30 to 57% occur in patients with mild disease. Other agents such as systemic corticosteroids, dapsone, and cyclosporin A were studied in small clinical studies with variable results. In patients with moderate to severe hidradenitis, the TNF alpha inhibitor adalimumab yielded a significant reduction in PGA scores by the end of the 24th week; however, relapse rates of up to 70% occurred. Surgical treatment offers the best chance at lasting cure and is currently the accepted therapy for hidradenitis, especially in more advanced cases. Unroofing individual sinus tracts or exteriorization by electrocauterization or curettage may be appropriate for minimal or mild cases at fixed locations, but both techniques are associated with a high recurrence rate in more advanced disease. The best chance at permanent cure is extensive removal of all affected skin and underlying tissue. The optimal type of closure is immediate or delayed application of a split-thickness skin graft. Primary closure is often precluded by the extensive defect after skin excision. Secondary intention closure requires painful dressing changes and prolonged healing compared with skin grafting.

Figure 18.7. Necrotizing soft tissue infection.

19. A 47-year-old man has had recurrent draining sinuses in the right axilla for many years. He now presents with 3 draining sinuses and multiple interconnected tracts across the entire axillary area compatible with severe hidradenitis suppurativa. The best option for surgical treatment should consist of

20. The use of negative pressure wound therapy after burn wound excision A. B. C. D. E.

A. unroofing sinus tracts. B. electrocauterization and curettage of the draining sinuses. C. total excision of all hair-bearing skin with primary closure. D. total excision of all hair-bearing skin with secondary intention healing. E. total excision of all hair-bearing skin with application of a split-thickness skin graft,

ANSWER:

increases acute kidney injury. increases use of nursing care. increases opioid use. decreases total treatment costs. increases rate of skin graft take.

ANSWER:

E

Excisional therapy for bum wounds is common and often requires skin grafting. Negative pressure wound therapy increases local blood flow to the wound bed and improves perfusion to wound edges and watershed areas, thus increasing the rate of skin graft take. In addition, negative pressure wound therapy helps reduce the exudate, edema, and hematoma that typify these wounds.

E

Hidradenitis suppurativa is a chronic inflammatory disease most commonly affecting the apocrine-gland-bearing skin of the axilla, inguinal, and anogenital regions. The condition typically presents after puberty and is characterized by inflammatory nodules, abscesses, sinus tracts, and local scarring. The lesions have a remitting and relapsing course with gradual progression. Several severity classifications schemes are proposed, but the 6-stage Physician Global Assessment (PGA) tool is currently the most widely used method to assess improvement in medical treatment trials (table 13.2).

Negative pressure wound therapy allows for accurate measurement and management of wound exudate permitting targeted fluid resuscitation, which can decrease pulmonary edema and acute kidney injury. An additional benefit is that negative pressure wound therapy requires less intensive daily nursing care to manage. Because application of negative pressure wound therapy results in a stable wound, the need for opioid analgesia is decreased. However, the overall cost of using this therapy is increased due to equipment costs.

85

21. Infected pressure ulcers are common among debilitated patients and can be difficult to treat. Which of the following statements is true regarding the risk for clinical recurrence of infected pressure ulcers?

alcohol and smoking status, preoperative albumin levels, obesity, and immunosuppression status. Additional risk factors for surgical site infections that are not modifiable include age, recent radiotherapy, and recent skin or soft tissue infection.

A. There is no benefit to chronic antibiotic treatment. B. Recurrence is related to the number of surgical debridement operations. C. Risk is decreased by the use of negative-pressure wound therapy. D. Admission albumin level strongly correlates with recurrence. E. Risk is decreased by the use of a myocutaneous flap.

ANSWER:

Smoking has repeatedly proven to be an important risk factor for SSIs, although the etiology of this association remains complex and not fully understood. Nicotine causes vasoconstriction, which is believed to lead to tissue hypoxia and impaired nutrient delivery. A randomized trial comparing patients who stopped smoking 6 to 8 weeks before surgery with patients who continued to smoke demonstrated a significant reduction in postoperative complication rates in the nonsmoking group. The relative risk reduction for wound complications was 83%, and the number needed to treat to prevent an SSI was 4.

A

Debilitated patients are at a continuous risk of developing pressure ulcers. Pressure ulcers can be difficult to treat. When they are infected, they have a high risk of recurrence, are associated with significant morbidity, and represent a substantial healthcare burden. The fundamental goals of preventing pressure ulcers are offloading pressure, repositioning the patient, preserving the integrity of soft tissue, and implementing risk-assessment scales.

The relationship between diabetes status and risk of SSI is also complex. Although perioperative hyperglycemia and use of diabetic medications are risk factors for SSIs, studies examining hemoglobin A1C levels among surgical patients with diabetes failed to demonstrate a correlation between tighter glycemic control and a decrease in SSIs. Shaving before surgery results in microscopic cuts and abrasions. These disrupt the skin's barrier defense against organisms and can lead to increased predisposition to infections. Guidelines recommend against the use of razors before surgery.

Treatment can involve nutritional supplementation, early and aggressive local wound care including negative pressure wound therapy, hyperbaric oxygen, surgery, and antibiotics to treat acute infection. Despite all these treatments, the rate of recurrence of infected pressure ulcers remains high. Studies suggest that recurrence of infected pressure ulcers is not related to extended duration of antibiotic therapy. In addition, recurrence rates do not correlate with the number of surgical debridements, surgical flaps, use of negative pressure wound therapy, or specific regimen of antibiotics. Markers of nutrition, such as albumin and prealbumin, are also not associated with ulcer recurrence. Neither primary closure nor use of myocutaneous flap correlate with recurrence. Chronic antibiotic therapy does not reduce recurrence.

Prospective trials failed to show a decrease in SSIs with various practice changes in the operating room, including rescrubbing; changing outer gloves; or using new instruments for fascial closure, redraping, and wound lavage. Studies evaluating the relationship between postoperative ground management and the likelihood of SSIs have had mixed results. However, in a recent review, the timing of dressing removal—early (defined as <48 hours postoperatively) versus late—was not found to be associated with the likelihood of SSI.

22. According to the American College of Surgeons and Surgical Infection Society Surgical Site Infection Guidelines 2016 updates, which of the following reduces the risk of surgical site infections? A. B. C. D. E.

23. Which of the following is an early sign of a necrotizing soft tissue infection of the lower leg? A. B. C. D. E.

Smoking cessation 4 to 6 weeks before surgery. Improving hemoglobin A1C Hair removal using razors Rescrubbing before closure in colorectal cases Removing the surgical dressing 24 hours after surgery

Crepitus Hemorrhagic bullae Confluent petechiae Hypotension Severe pain on passive motion

ANSWER: ANSWER:

A

E

Distinguishing necrotizing from nonnecrotizing infections is a diagnostic challenge of general surgery. Generally speaking, necrotizing infections require prompt tissue

Modifiable risk factors associated with the development of surgical site infections (SSIs) include glycemic control,

86

debridement and result in wounds and functional loss. By contrast, nonnecrotizing infections can often be treated with antimicrobials and without surgery. Unfortunately, the hard and definitive signs of necrotizing soft tissue infection occur late in the disease process and after the opportunity to minimize tissue loss and function. These hard signs include hypotension from shock, crepitus, hemorrhagic bullae, and confluent petechia. The latter result from tissue that is already beyond salvage. An early sign of a necrotizing infection is severe pain on clinical evaluation, such as passive motion of the extremity. The pain is disproportionate to other clinical signs. This finding should prompt further diagnostic steps to exclude a necrotizing infection.

Surgical excision to clear margins of 4 to 10 mm is the standard of treatment for the management of nonmelanoma cutaneous cancers. Standard surgical excision or Mohs micrographic surgery are both acceptable approaches. Basal cell carcinoma constitutes approximately 80% of the estimated 2 million nonmelanoma cutaneous cancers diagnosed in the United States each year, with squamous cell carcinomas making up most of the remainder. Squamous cell carcinomas are classified as "low risk" or "high risk." Highrisk lesion characteristics include lesions greater than 2 cm on an extremity, ill-defined borders, recurrent lesions, rapid growth, lesion in a prior radiation field or that develop in a chronic wound, neurologic symptoms, and immunosuppression. Pathologic features include poor differentiation, invasion depth greater than 2 mm (or Clark level > IV), perineural invasion, and lymphovascular invasion. It is unclear if high-risk lesions should have an increased resection margin, but both patient and pathologic characteristics should be considered when selecting either primary or adjuvant treatment options.

24. An 83-year-old woman who is a resident of a chronic nursing facility is referred for the lesion shown on her right arm. Punch biopsy demonstrates squamous cell carcinoma. She is chronically anticoagulated with warfarin for a prosthetic mitral valve placed more than 10 years ago. Other than severe dementia, she has no medical problems. The lesion has become a problem because of persistent bleeding requiring frequent dressing changes. What is the best management option?

A surgical approach in this patient poses multiple challenges; however, the procedure could be done with regional anesthesia and sedation. Coverage of the surgical defect with a primary split-thickness skin graft would address the immediate problem of bleeding as well as the lesion itself.

A. B. C. D.

Radiation therapy Topical 5-fluorouracil Photodynamic ablation Local excision with coverage by a split-thickness skin graft E. Discontinuation of warfarin with continued dressing changes

ANSWER:

Radiation therapy has a role in the adjuvant setting and occasionally as a primary treatment modality. Possible application of radiation therapy could include cosmetically sensitive areas of the face, positive margins after primary excision where additional tissue resection may be inappropriate, or patients who refuse primary surgical excision. For large tumors (>2 cm) treated with first-line radiation therapy, the recurrence rates are more than 30%; therefore, radiation therapy is not recommended as a first-line option in most patients. In addition, radiation therapy for the patient presented would involve many logistical problems and does not immediately address the persistent bleeding problem. Topical therapy with 5-fluorouracil or imiquimod is appropriate for management of small premalignant lesions or to address the field effect after a primary excision, but it is not appropriate for large lesions such as this. The same argument applies to photodynamic ablation. It is tempting to discontinue the anticoagulant, but this puts the mechanical valve at risk and does not obviate the need for continued dressing changes.

D

Figure 24.1. 87

Surgical Critical Care Part I

possible due to the mechanism of injury, is not supported by the radiograph and the precipitous onset of symptoms.

ITEMS 1-25 For each question, select the best possible response.

The differential diagnosis is most likely either transfusionrelated acute lung injury (TRALI) or transfusion-associated circulatory overload (TACO). Although both conditions are temporally related to the administration of blood products, clinical findings can usually distinguish between the 2 conditions. The differences are summarized in table 1.2.

1. A 19-year-old, previously healthy woman was injured after being thrown from a horse. Her only identified injury was a comminuted right femur fracture. She underwent internal fixation of her femur fracture 6 hours after admission and received an intraoperative transfusion of 2 units of packed red blood cells and 4 L of crystalloid. In the postanesthesia care unit she developed progressive respiratory distress manifested by air hunger, tachypnea (32 breaths per minute), tachycardia (142 beats per minute), a blood pressure of 90/60 mm Hg, and a pulse oximeter reading of 82% despite supplemental oxygen delivered by nasal cannula at 12 L/minute. Breath sounds are symmetrical and clear. There is no evidence of effusion by physical examination. Her chest radiograph is shown (figure 1.1). The most likely diagnosis is A. B. C. D. E.

TRALI is a type of ALI that occurs within 6-hours of the administration of a blood product and has the same oxygenation criteria as ALI due to other causes (i.e., PaO2/FiO2 ratio <300 or saturation of <90%), in the absence of any other identified etiology, hence a diagnosis of exclusion. Any blood product can produce TRALI, but the syndrome is more common with products that are plasma-rich (e.g., fresh frozen plasma, platelet packs). Although the syndrome is uncommon, it accounts for most transfusionrelated mortality in the United States and the developed world. The pulmonary injury requires the activation of neutrophils or monocytes, which are adherent to pulmonary endothelium. The activation is thought to be caused by antibodies in donor plasma as the result of exposure to allogenic, but foreign, tissue during pregnancy, prior transfusion, or tissue transplantation. When TRALI was first recognized more than 30 years ago, it was discovered that most donor products were from women and that most had been pregnant. Blood banks across North America and most of Europe instituted mitigation strategies, using only male donors, nulliparous female donors, or women who tested negative for human leukocyte antigens I, II, and III (HLA) and human neutrophil antigen (HNA) for their plasma and platelet products. That HLA- and HNA-antibodies cannot be detected in the serum of some patients with TRALI and the observation that not all patients who receive transfusion of blood products known to have antibodies against the cognate antigens develop TRALI has raised the issue of a second potential mechanism. The hypothesis of this second mechanism seems to impugn substances found in banked

pulmonary embolism. aspiration of gastric content. occult pulmonary contusion, transfusion-related acute lung injury. transfusion-associated circulatory overload.

Figure 1.1.

ANSWER:

Parameter Age Temperature Blood pressure

TRALI Any Fever, usually low grade Hypotensive, to normal

TACO Very young and elderly Normothermic Hypertensive or norma1

CVP or PAOP Response to

Normal Minimal to none

Elevated Brisk

CBC

Transient leukocytosis and thrombocytopenia

No leukocytosis

Brain natriuretic peptide

Normal

Elevated

diuretics

D

Acute respiratory distress in the immediate postoperative period has multiple potential etiologies. The clinical observations and the chest radiograph in this case suggest some type of diffuse acute lung injury (ALI). The timing of the observed respiratory distress and the diffuse, bilateral infiltrates on the x-ray make a pulmonary embolism an unlikely diagnosis. Although aspiration of gastric content is a possibility, it is unlike to cause the immediate air hunger or this degree of hypoxia. A pulmonary contusion, although

CVP = Central venous pressure; PAOP = pulmonary artery occlusion pressure. Table 1.2. Comparison of transfusion-related acute lung injury (TRALI) and transfusion-associated circulatory overload (TACO).

89

blood products, including cellular debris, biologically active lipids, and soluble CD40 ligand, all associated with the storage lesion of banked blood and blood components. These substances can induce a proinflammatory milieu, resulting in pulmonary dysfunction. In vitro studies and animal models of TRALI have added support for this concept; however, human studies have not confirmed this theory. Some researchers implied an association with the age of the blood products, but confirmatory clinical evidence is lacking as well.

cm (squared) x craniocaudal length in cm (which equates to number of slices times the thickness of the CT cuts) = d 2 * L. Periprocedural antibiotics for placement of tube thoracostomy do not reduce the incidence of empyema or pneumonia and are not routinely indicated when a chest tube is placed for traumatic hemothorax.

3. A 65-year-old woman with atrial fibrillation is admitted to the surgical intensive care unit with a subdural hematoma after a fall. She is being treated with rivaroxaban for her atrial fibrillation. What agent can be used to correct her anticoagulation?

There is no specific clinical finding or laboratory test that confirms the diagnosis and there is no specific treatment for the syndrome. As many as 70 to 90% of patients with TRALI will not respond to supplemental oxygen and will require mechanical ventilation, but mortality rates are low, at approximately 5%. The duration of respiratory dysfunction is generally less than 72 hours in most cases.

A. B. C. D. E.

2. Regarding tube thoracostomy for a traumatic hemothorax, A. the tube should be a 36 French chest tube. B. the tube should be inserted at the eighth intercostal space, just above the diaphragm. C. placement should be preceded by intravenous antibiotics. D. placement is not indicated for less than 300 mL of blood in the pleural space. E. placement is associated with a 10% complication rate.

ANSWER:

Andexanet alfa Protamine Vitamin K Fresh frozen plasma Idarucizumab

ANSWER:

A

Knowledge of which agents are used to reverse pharmacologic anticoagulation is paramount in the management of any patient, but particularly in patients who sustain intracranial hemorrhage after trauma. Andexanet alpha is used to reverse the direct oral anticoagulant (DOAC) rivaroxaban.

D

Tube thoracostomy is one of the most common interventions performed after trauma. Some simple concepts should be considered to effectively drain the pleural space while preventing complications. First, smaller tubes (28 French) are just as effective in evacuating blood from the pleural space as larger tubes (36 and 40 French). These smaller tubes may also be less painful than larger caliber tubes passing through the often-tight intercostal space. The triangle of safety for tube placement is located at the fifth intercostal space, between the pectoralis major and the latissimus dorsi, not at the eighth intercostal space. Ideally the tube is directed posteriorly and away from the fissure, but placement anywhere in the chest is documented to be effective for drainage. Recent studies suggest that the tube should be introduced at a less than 30° angle from the chest wall. Finally, the chest tube should be placed after digitally exploring the pleural space to ensure there are no adhesions. Despite all of these suggestions, tube thoracostomy is still associated with a 25% complication rate. Perhaps it is this high complication rate that reinforces the concept that chest tube placement should be avoided if less than 300 mL of blood (small hemothorax) is in the pleural space.

DOACs have increased in use in recent years. Compared with Vitamin K antagonists (warfarin) for nonvalvular atrial fibrillation and thromboembolic disease, DOACs have predictable dose-response relationships that do not require routine monitoring. Additionally, they have a short half-life compared with warfarin. Rivaroxaban has a half-life of 9 to 13 hours, which means its anticoagulant activity dissipates after 4 to 5 half-lives. Dialysis does not affect rivaroxaban's activity because it is highly bound to proteins. For reversal of warfarin anticoagulation, fresh frozen plasma, prothrombin complex concentrate (containing the 4 clotting factors that warfarin inhibits—II, VII, IX, and X), or Vitamin K are administered. For reversal of systemic heparinization, protamine can be given. For the other common DOAC, dabigatran, dialysis can be performed or reversal with idarucizumab is possible.

4. What infection control strategies can help prevent the spread of Clostridium difficile within intensive care units? A. Pairing of infected patients in the same room B. Donning gown and gloves after entering the patient’s room and discarding them after leaving the patients room C. C Terminal cleaning of rooms with bleach after patients are transferred

On CT imaging, a hemothorax is identified as layering fluid within the pleural space, typically with Hounsfield units ranging from 35 to 70. Size quantification of the hemothorax is calculated by the following equation: volume in mL = greatest depth of the hemothorax from chest wall to lung in

90

D. Using alcohol hand gel products before entering and after leaving the patients room E. E Administering of antibiotic prophylaxis with ertapenem for any operative procedures

ANSWER:

management of this patient is vasopressor support with norepinephrine. In Patients who have been volume resuscitated, norepinephrine inceases the systemic and mean arterial pressure due to vasoconstriction, with little change in heart rate and fewer arrhythmias than dopamine. Norepinephrine is more potent than dopamine, is more effective in reversing hypotension, and improves outcomes in patients with septic shock. After its initiation, norepinephrine should be titrated to achieve a mean arterial pressure of 65 mm Hg.

C

The control of the spread of Clostridium difficile in intensive care units is dependent on an effective infection control strategy. Patients with C. difficile infection should be placed on isolation precautions in a private room. Infected patients should not be paired in the same room. The use of personal protective equipment is mandatory for medical personnel caring for patients with C. difficile infection. Single use gown and gloves must be donned before entering the patient's room and discarded before leaving the patient's room. After the transfer of patients with C. difficile out of the intensive care unit, the hospital room should be terminally cleaned with bleach. The use of alcohol or gel products on the hands of caregivers is ineffective against clostridial spores and does not prevent the spread of C. difficile infections. Handwashing with soap and water is the appropriate intervention to prevent the spread of C. difficile. Antibiotic prophylaxis with ertapenem is associated with an increased incidence of postoperative C. difficile infection and is contraindicated in the management and control of C. difficile infection in the intensive care unit.

Additional crystalloid fluid in this patient is unlikely to be of benefit because his central venous pressure of 13 cm H2O indicates that he is not hypovolemic. Multiple clinical trials have failed to demonstrate a benefit to the placement of a pulmonary artery catheter in guiding the management of patients with sepsis and septic shock. A hemoglobin concentration greater than 7.0 g/dL provides adequate oxygen-carrying capacity in septic patients; therefore, the administration of additional units of red packed blood cells would not be of benefit. The administration of hetastarch can worsen outcomes and mortality in patients with sepsis and is contraindicated in the treatment of septic patients.

6. Flexible bronchoscopy in intubated patients on mechanical ventilation increases A. B. C. D. E.

5. A 24-year-old man who weighs 100 kg is admitted to the intensive care unit after undergoing a laparotomy for multiple gunshot wounds to the abdomen, during which he underwent resection of segments of his small and large bowel and repair of multiple enterotomies. Seventy-two hours later, his respiratory rate climbs to 40 breaths per minute. His blood pressure is 80/40 mm Hg, his pulse is 115 beats per minute, and his temperature is 38.7°C. His central venous pressure is 13 cm H2O. Blood, urine, and sputum specimens are sent for Gram stain and culture testing. His lab tests are remarkable for a white blood cell count of 22,000/mm3 (360011,200/mm3), a hemoglobin concentration of 8.0 g/dL (13517.5 g/dL), and a serum creatinine of 1.7 mg/dL (0.4-1.30 mg/dL). What intervention is appropriate in the management of this patient? A. B. C. D. E.

ANSWER:

D

Flexible bronchoscopy is routinely performed in intubated patients in the intensive care unit for a variety of indications. These indications include performing bronchoalveolar lavage, diagnosing the etiology of lung collapse, identifying potential sites of bleeding, or assessing the status of the tracheobronchial tree. The performance of flexible bronchoscopy in this setting, however, can have important clinical consequences for patients. Prospective studies demonstrated that flexible bronchoscopy in intubated patients on mechanical ventilation consistently increases airway resistance. In two-thirds of patients, flexible bronchoscopy decreases tidal volumes. The presence of the bronchoscope does not result in dynamic hyperinflation because of the corresponding decrease in end-expiratory lung volumes. Flexible bronchoscopy decreases PaO2 levels and increases PaCO2 levels in intubated patients. After bronchoalveolar lavage, the PaO2/FiO2 ratio may be decreased for more than an hour. Bleeding and pneumothorax are very uncommon after flexible bronchoscopy. Similarly, other potential complications, including hypertension, cardiac rhythm

2000 mL of crystalloid fluid A pulmonary artery catheter 2 units of packed red blood cells Vasopressor support with norepinephrine 500 mL of hetastarch

ANSWER:

end expiratory lung volumes. incidence of pneumothorax. PaO2. PaCO2. tidal volume.

D

This patient has developed clinical signs of sepsis. Based on the International Guidelines for the Management of Sepsis and Septic shock, the most appropriate intervention in the

91

disturbances, and hemodynamic instability, are uncommon after flexible bronchoscopy.

mechanical ventilation or a previous failed extubation. The clinical criteria should include a spontaneous breathing trial with respiratory pressure augmentation of 5 to 8 cm H2O rather than a T-piece. Pressure support during a spontaneous breathing trial of 5 to 8 cm H2O provides adequate expiratory pressure to overcome the work of breathing imposed by the artificial airway. The spontaneous breathing trial should be conducted for a period of at least 30 minutes and may be extended to 2 hours. A spontaneous breathing trial of 15 minutes is not sufficient time to assess whether a patient will successfully tolerate extubation.

7. Noninvasive ventilation is indicated as an alternative to mechanical ventilation in postoperative patients with A. B. C. D. E.

upper airway obstruction. unstable cardiac arrhythmias. hemodynamic instability. severe encephalopathy. hypoxic respiratory failure.

ANSWER:

For patients who are at high risk for extubation failure after receiving mechanical ventilation for more than 24 hours and who pass a spontaneous breathing trial, the use of noninvasive ventilation is recommended to help prevent the need for reintubation. Prophylactic flexible bronchoscopy is not recommended to remove retained secretions, because it does not improve the likelihood the patient will successfully tolerate extubation. A cuff leak test should be performed on mechanically ventilated patients who otherwise meet criteria for extubation but who are deemed to be at high risk for postextubation stridor. This particularly applies to patients who have required reintubation after self-extubation. Patients who do not have evidence of audible breathing after deflation of the cuff of the endotracheal tube should receive systemic steroids at least 4 hours before an attempt at extubation.

E

In a prospective observational study of patients with respiratory failure after abdominal surgery, noninvasive ventilation markedly reduced the incidence of reintubation, length of hospital stay, and mortality. A trial of noninvasive ventilation, rather than reintubation, is recommended as the initial intervention for surgical patients with postoperative acute respiratory failure. Noninvasive ventilation requires the presence of an intact airway. Therefore, the use of noninvasive ventilation is contraindicated in the setting of upper airway obstruction or severe encephalopathy in which patients are not able to protect their airways. Similarly, noninvasive ventilation is contraindicated in the setting of unstable cardiac arrhythmias or hemodynamic instability where the airway needs to be secured with an endotracheal tube should the rapid institution of advanced cardiac life support be required.

9. Which of the following parameters is required to characterize acute respiratory distress syndrome based on the current definition of this condition? A. B. C. D. E.

8. A 37-year-old woman develops severe acute respiratory distress syndrome after undergoing multiple laparotomies for resection of her small bowel. After 10 days of mechanical ventilation, she extubates herself and requires emergency reintubation for stridor and acute respiratory distress. She remains intubated for an additional 8 days, during which time her pulmonary function and chest x-ray improve significantly. What is most likely to optimize her chances for successful extubation?

ANSWER:

E

Acute lung injury and acute respiratory distress syndrome (ARDS) are notoriously difficult to define and diagnose. First described in the 1960s, ARDS is a form of nonhydrostatic pulmonary edema resulting from a primary pulmonary insult or a pulmonary response to extra pulmonary sources of inflammation. The result of this pulmonary insult is hypoxia, bilateral pulmonary opacities on chest radiograph, poor pulmonary compliance, and increased dead space ventilation. Initial definitions of ARDS focused on the hypoxia, the absence of elevated filling pressures, and the radiographic changes. The exclusion of other causes for hypoxemia was another aspect. Multiple terms were used in addition to ARDS to describe the pulmonary insult, including pulmonary dysfunction and acute lung injury of varying severity.

A. Spontaneous breathing trial using a T-piece B. Limiting spontaneous breathing trial to 15 minutes C. Avoiding the use of noninvasive ventilation following extubation D. Prophylactic flexible bronchoscopy to remove retained secretions E. Systemic steroids if a cuff leak is not present

ANSWER:

Pulmonary artery wedge pressure Pulmonary vascular resistance Mean airway pressure Plateau pressure Positive end expiratory pressure

E

Clinical criteria can help determine whether a patient can remain successfully extubated after an extended period of

92

Introduced in 2012, the Berlin Definition of ARDS replaced the term "acute lung injury" with a spectrum of ARDS severity based upon PaO2/FiO2 (P/F) ratio. This revision included a P/F ratio of 200 to 300 as mild with a mortality rate averaging 27%, 100 to 200 as moderate with a mortality rate of 32%, and less than 100 as severe with a mortality rate of 45%. All these calculations are done on a positive endexpiratory pressure (PEEP) of at least 5 cm H2O. This period of hypoxia also must occur within 7 days of the inciting insult.

E. Start hydrocortisone

ANSWER:

Propofol is a popular medication for maintaining sedation in the intensive care unit. Its short half-life is particularly attractive for use in traumatic brain injury. Unfortunately, the use of propofol for prolonged periods (>48 hours) or at high doses is associated with the rare but potentially fatal propofol infusion syndrome. There is no widely accepted definition of this syndrome, and its pathogenesis has yet to be fully delineated but may be caused by impaired mitochondrial function. Propofol infusion syndrome typically involves various combinations of unexplained metabolic acidosis, rhabdomyolysis, hyperkalemia, hepatomegaly, renal failure, hyperlipidemia, arrhythmia, bradycardia, and rapid progression to cardiac failure.

Only PEEP is a component of the Berlin Definition of ARDS. Initial definitions used a pulmonary wedge pressure of less than 18 mm Hg, but this was deleted in the Berlin criteria. Pulmonary vascular resistance, mean airway pressure, and plateau pressure were never included in an ARDS definition.

10. The proper duration of antibiotics for a ventilatorassociated pneumonia caused by Klebsiella pneumoniae is

The treatment is discontinuation of the propofol, hemodialysis, organ support, and sustainment of cardiopulmonary function. Propofol is also associated with the development of hypertriglyceridemia. The significance of this abnormality is controversial but may be an etiology of pancreatitis. Dexmedetomidine, a sedative agent used in the intensive care unit, can be used in place of propofol but it will not correct propofol infusion syndrome if propofol is continued. Dexmedetomidine may induce bradycardia. Hydrocortisone would be appropriate if this patient had acute adrenal insufficiency, typically manifested by hypotension refractory to fluids and vasoactive agents.

A. B. C. D.

8 days. 15 days. until the x-ray has cleared. until fever has resolved and white blood cell count has normalized. E. until the patient is liberated from the ventilator.

ANSWER:

A

Few things in medicine are supported by well-controlled randomized trials. The duration of antibiotic therapy for ventilator associated pneumonia, however, is supported with good data and strong recommendations from the Infectious Disease Society of America and the American Thoracic Society. A short course of 7 to 8 days is as safe and effective as a 14- to 15-day course. The chest x-ray in intensive care unit patients can be nonspecific, and its normalization can lag behind the resolution of the pneumonia. Fever and leukocytosis in the intensive care unit patient are also nonspecific. Patients can be dependent on mechanical ventilation for many reasons, and continuation of antibiotics for the duration of ventilation is not necessary.

Fentanyl is another commonly used medication in the intensive care unit that is known to cause chest wall rigidity at high doses. Levetiracetam is used to prevent posttraumatic seizures. Its adverse effects consist of somnolence, decreased energy, headache, dizziness, coordination difficulties, and, in rare cases, Stevens-Johnson syndrome and toxic epidermal necrolysis.

ITEMS 12-13 12. An 18-year-old man with a gunshot wound to the head is intubated in the intensive care unit. He has no response to pain, a Glasgow Coma Scale score of 3, no pupillary response, no gag reflex, and no corneal reflex. He does not have spontaneous respirations. Which of the following findings would prevent a brain death examination being performed?

11. 67-year-old woman sustained a traumatic brain injury after a motorcycle crash. She is sedated with propofol and fentanyl, receives levetiracetam as seizure prophylaxis, and has been continued on her statin and aspirin. On postinjury day 11, she has decreased urine output, hyperkalemia, and a metabolic acidosis. Her creatine phosphokinase is greater than 10,000 IU/L (60-174IU/L). What is the best first step in her management? A. B. C. D.

A

A. B. C. D. E.

Discontinue the propofol Discontinue the fentanyl Discontinue the levetiracetam Start dexmedetomidine

93

Blood pressure = 106/70 mm Hg on norepinephrine drip Hemoglobin = 8.1 g/dL (13.5-17.5 g/dL) Sodium - 161 mEq/L (136-14.5 mEq/L) Temperature = 36.5°C White blood cell count = 15,200/mm3 (360011,200/mm3)

13. You elect to perform an apnea test to confirm brain death after correcting all physiologic derangements. His pretesting vital signs and arterial blood gas on 100% FiO2 are as follows:

("doll's eyes") and the vestibulo-ocular reflex ("cold calorics"). The patient should have no corneal, gag, or cough reflexes and no spontaneous respiratory effort. Fourth, an apnea test is performed. Although the American Academy of Neurology guidelines support a single apnea test, local laws and regulations dictate whether 1 or 2 apnea tests are required. The theory behind the apnea test is that respiratory drive is based on PaCO2 levels. An abnormally high level should trigger a respiratory effort in any patient capable of doing so. Ongoing apnea in the presence of hypercarbia indicates brain death. First, the patient is preoxygenated with 100% O2 for 10 to 15 minutes. Next, the ventilator is adjusted to obtain a PaCO2 as dose as possible to 40 mm Hg. A baseline arterial blood gas is then drawn. The endotracheal tube is disconnected from the ventilator and a smaller nasal cannula tube is connected to 5 to 6 L/minute oxygen and inserted into the endotracheal tube. If spontaneous respirations, desaturation less than 85%, or hemodynamic instability occur, the apnea test is aborted. After 10 minutes, an arterial blood gas is drawn. A PaCO2 greater than 60 mm Hg (or 20 mm Hg above baseline) is considered positive, and the patient is declared brain dead.

Blood pressure = 114/70 mm Hg Heart rate = 110 beats per minute O2 saturation = 98% pH = 7.38 PaCO2 = 42mmHg PaO2 = 278 mm Hg IHCO3 = 24 mEq/L (20-29 mEq/L) During his apnea test he is administered 100% supplemental O2, and no spontaneous respirations are noted. Which of the following findings would confirm brain death after a 10minute apnea test? A. B. C. D. E.

Blood pressure = 100/40 mm Hg PaCO2 = 66mmHg HCO3 = 18 pH = 7.30 PaO2 = 200mmHg

ANSWER:

C, B

Ancillary tests are not required and should be used only if the patient is unable to complete the apnea test or any of the other exam findings are equivocal. The American Academy of Neurology recognizes 4-vessel angiography, nuclear perfusion imaging ("cerebral blood flow scan"), transcranial Doppler, and electroencephalography as acceptable ancillary tests.

In this patient, hypernatremia can contribute to coma and should be corrected before performing a brain death exam. Vasopressor use is not a contraindication if the blood pressure remains above 90 to 100 mm Hg. A hemoglobin level of 8.1 g/dL and a temperature of 36.5°C are not contraindications to performing a brain death exam. Simple leukocytosis is common in critically ill patients, and its presence alone is not a contraindication.

14. A 64-year-old man is recovering in the intensive care unit 6 hours after an exploratory laparotomy and Hartmann procedure for perforated diverticulitis with feculent peritonitis. He remains intubated since the procedure. He has no history of cardiac or pulmonary disease or symptoms. He has made 30 mL/hour of urine since the operation. His vitals since the operation have been as follows: heart rate 80 to 100 beats per minute and systolic blood pressure 130 to 150 mm Hg. He now has a heart rate of 160 beats per minute and blood pressure of 115/60 mm Hg. Electrocardiogram demonstrates atrial fibrillation without ST elevation or depression. What is the most appropriate next step in management?

Four steps are required for brain death declaration. First, the patient must have a cause of irreversible coma identified, usually by history and physical examination with imaging confirmation. A period of time from admission to performance of the brain death examination should be allowed to confirm that the patient's clinical status is not improving. This period of time is not specified in most guidelines but is generally between 6 and 24 hours after admission. Second, confounding physiologic and metabolic derangements should be identified and corrected. The patient should be normothermic and normotensive, although vasopressor support is allowed. Any intoxicants should be cleared, and medications that could obscure the clinical exam should be discontinued for at least 5 half-lives. Electrolyte abnormalities should be corrected. Glucose levels should be within normal parameters.

A. B. C. D. E.

Metoprolol Nicardipine C. Electrocardioversion D. Echocardiography Digoxin

ANSWER:

Third, a physician experienced in assessing for brain death should perform a physical examination. The Glasgow Coma Scale should be 3 and cranial nerve reflexes should be absent. The patient should have no response to painful stimuli, and the pupils should be fixed and nonreactive to light. The eyes should remain in midposition with the cervico-ocular reflex

A

Atrial fibrillation is the most common postoperative cardiac arrhythmia and may occur in up to 26% of patients undergoing noncardiac nonthoracic surgery. The incidence varies greatly with patient risk factors, such as age, preexisting cardiac disease, other co-morbidities, and the type of

94

surgery performed. It is thought to occur primarily due to adrenergic stimulation as well as systemic inflammation, fluid shifts, or electrolyte abnormalities. Typically, atrial fibrillation develops between day 1 and 4 after surgery and is self-limited. Treatment and management is the same as in the nonsurgical setting and primarily involves rate control and electrolyte correction unless the patient is hemodynamically compromised.

sedation spend more time on the ventilator than those receiving intermittent sedation as needed. Regardless of the time on the ventilator, the use of spontaneous breathing trials in conjunction with "daily sedation holidays" has the greatest effect on decreasing ventilator days. Regardless of the medication used or ventilator mode, protocols that discontinue sedation and allow for assessment for extubation lead to improved outcomes. The weaning trial should include parameters for extubation, including appropriate vital capacity, tidal volume, respiratory rate, negative inspiratory force, and rapid shallow breathing index to help predict successful extubation. Tracheostomy is not used 24 hours after an exploratory laparotomy.

Cardiac biomarkers and an electrocardiogram should be obtained to rule out an acute coronary event as the precipitating factor. Echocardiography is generally not recommended unless there is concern for underlying cardiac disease or dysfunction. The hemodynamically compromised patient should undergo immediate synchronized direct current cardioversion. Intravenous amiodarone may be administered if there is a delay in being able to perform cardioversion. Hemodynamically normal patients should have their ventricular rate controlled. When considering which agent to use, it is important to determine whether the patient has underlying chronic obstructive pulmonary disease, heart failure, or a reduced ejection fraction. Options for intravenous agents include beta-blockers, nondihydropyridine calcium channel blockers, digoxin, or amiodarone.

16. Which of the following interventions improves survival in a patient with moderate to severe acute respiratory distress syndrome? A. High-frequency oscillatory ventilation, early physical therapy, and bronchoscopy B. High tidal volume mechanical ventilation, early neuromuscular blockade, and inhaled nitric oxide C. High tidal volume mechanical ventilation, recruitment maneuvers, and bronchoscopy D. Low tidal volume mechanical ventilation, early neuromuscular blockade, and prone positioning E. Low tidal volume mechanical ventilation, high-dose steroids, and inhaled nitric oxide

When not contraindicated due to underlying disease, betablockers are first-line therapy given the underlying adrenergic stimulation that is the most likely cause of the atrial fibrillation. Nicardipine is not commonly used to treat cardiac arrhythmias; it is used to treat cerebral hypertension after cerebral aneurism rupture. Digoxin could be used, but its onset of action is too slow.

ANSWER:

The landmark ARDSNet trial demonstrated that low tidal volume ventilator settings (6 mL/kg vs 12 mL/kg ideal body weight) significantly improved survival in patients with any degree of acute respiratory distress syndrome (ARDS; 31% vs 40%). Thus, How stretch" ventilator management with low tidal volumes and escalating doses of positive end-expiratory pressure still stands as the cornerstone of ARDS management Two other therapies have since been found to improve mortality in patients with moderate to severe ARDS in single randomized-controlled trials: a short course of neuromuscular blockade (*48 hours) early in the course of ARDS and prone positioning.

15. A 60-year-old man is currently sedated and intubated 24 hours after an exploratory laparotomy for strangulated bowel due to an internal hernia. He is receiving propofol and fentanyl for sedation and analgesia. With regard to his pulmonary management, what intervention is most likely to lead to earlier liberation from the ventilator? A. B. C. D. E.

Daily sedation holiday Use of propofol Measurement of rapid shallow breathing index Early tracheostomy Assist control ventilation

ANSWER:

D

Two recent trials showed high-frequency oscillatory ventilation (HFOV) did not improve outcomes in ARDS and may even increase harm. Although physical therapy should be considered in all intensive care unit patients, even those on mechanical ventilation, there is no proven mortality benefit or improved long-term outcomes in ARDS patients. Fiberoptic bronchoscopy is clinically useful and is a very practical tool in routine intensive care unit care; however, no specific mortality benefit has been identified in patients with ARDS. High tidal volume mechanical ventilation should be explicitly avoided in ARDS patients and in routine use of

A

The decision to liberate a patient from the ventilator is challenging and should be revisited on a frequent basis to decrease ventilator time. The benefits of shortened ventilator time include shorter hospital stay and intensive care unit length of stay, decreased morbidity, and decreased mortality. A significant factor in managing intubated patients is the overuse of sedation. Patients on continuous intravenous

95

mechanical ventilation in the operating room and intensive care unit for patients without respiratory failure. Inhaled nitric oxide is a potent pulmonary vasodilator that can improve ventilation/perfusion ratio mismatch and thereby improve oxygenation. However, its use is associated with increased acute kidney injury; therefore, it should be used as a very short-term rescue therapy until more labor-intensive therapies such as prone positioning can be instituted.

approaching 25%. Necrosectomy is currently recommended only if radiologic and endoscopic drainage procedures fail. Less-invasive approaches, like percutaneous drainage done by interventional radiology and endoscopy with necrosectomy or transgastric drainage combined with percutaneous drainage, result in less requirement for open surgery, shorter hospital stays, and less morbidity. The stepup procedure combines percutaneous drains that are gradually up-sized to the point where minimally invasive retroperitoneal necrosectomy can be performed via the drain tracks if necessary to remove larger pieces of necrotic tissue.

Likewise, recruitment maneuvers can open atelectatic lung segments and may result in a transient improvement in oxygenation, but this comes at the cost of potentially inducing barotrauma or volutrauma that may have longer-term adverse effects. Finally, although some limited data suggest a potential benefit to high-dose systemic steroids in early ARDS, the risks and complications associated with this therapy are currently thought to outweigh any potential benefit. The exceptions include ARDS due to eosinophilic pneumonia and select patients with community-acquired bacterial pneumonia.

18. A 64-year-old man undergoes elective pancreaticoduodenectomy for adenocarcinoma. On the night of postoperative day 1, he becomes diaphoretic, confused, and anxious. His heart rate ranges from 130 to 165 beats per minute, with an irregular rhythm. His blood pressure is 88/60 mm Hg. He has no prior history of arrhythmias, and his preoperative electrocardiogram showed normal sinus rhythm. What is the best way to manage this patient' clinical condition?

17. Which of the following statements regarding the management of a patient with necrotizing pancreatitis is true?

A. B. C. D. E.

A. Mortality is greater than 30%. B. Infected pancreatic necrosis increases mortality to 50%. C. Open surgical necrosectomy remains the treatment of choice. D. Intervention is reserved primarily for those with symptomatic clinical deterioration. E. Percutaneous and endoscopic drainage techniques are not appropriate.

ANSWER:

Direct current cardioversion Esmolol Adenosine Diltiazem Amiodarone

ANSWER:

A

Postoperative atrial fibrillation in noncardiac surgery patients generally occurs within the first 4 days. The etiology of postoperative atrial fibrillation is likely multifactorial; however, commonly cited causes are increased circulating catecholamine levels, increased sympathetic tone, hypervolemia, and electrolyte abnormalities. Atrial fibrillation is characterized by an irregular rhythm, without distinct P waves on electrocardiogram, and a ventricular response between 90 to 170 beats per minute.

D

Acute pancreatitis is a common cause of gastrointestinal complaints that brings patients to the emergency department. Although most cases are mild or moderate in severity and are associated with a low morbidity and mortality, severe acute pancreatitis, which involves the failure of one or more organs over 48 hours has a mortality rate upward of 25%. Necrotizing pancreatitis involves necrosis of more than 30% of pancreatic parenchyma or peripancreatic tissue and accounts for only 10 to 15% of acute pancreatitis cases. It is routinely diagnosed with a CT scan with intravenous contrast.

Patients who have atrial fibrillation with rapid ventricular response can present with hypotension or shock due to inadequate cardiac output Signs of shock and hemodynamic instability in atrial fibrillation are hypotension, acute chest pain, confusion, and heart failure. Rapid treatment of atrial fibrillation is required, usually by direct current cardioversion attempting to achieve normal sinus rhythm. The overall success rate of direct current cardioversion is 90%, but the success rate decreases as the duration of atrial fibrillation increases. Some patients may need concurrent treatment with an antiarrhythmic drug such as amiodarone or sotalol before direct current conversion to prevent relapse of atrial fibrillation; however, direct current conversion should not be delayed in hemodynamically abnormal patients.

A key distinction in these patients is the presence or absence of infected necrosis, because noninfected necrotizing pancreatitis has a mortality rate of 15% versus 30% in infected necrotizing pancreatitis. Intervention for sterile necrotizing pancreatitis is optimally delayed, because twothirds will remain sterile and ultimately resolve over time. Historically, necrosectomy was a common surgical approach for dealing with infected necrotizing pancreatitis, and it was fraught with morbidity rates up to 95% and mortality rates

96

Pharmacologic therapy is preferred in patients with atrial fibrillation with rapid ventricular response who are hemodynamically normal. There is no difference in long-term outcomes comparing drugs aimed at rate control versus rhythm control. The goal of therapy in postoperative atrial fibrillation is ventricular rate of 80 to 100 beats per minute, even if the patient remains in atrial fibrillation. The first-line therapy is generally directed toward atrioventricular node blockade. Beta-blocker therapy is suggested as a first-line agent in postoperative atrial fibrillation due to increased sympathetic tone and circulation catecholamines in the postoperative period. However, calcium channel blockers remain a commonly used class of drugs.

outcomes in patients who had tight blood glucose control, between 80 and 110 mg/dL. Although tight glycemic control was rapidly adopted in critically ill patients, subsequent trials had conflicting results across diverse patient groups. A large multicenter prospective randomized trial reported increased mortality in a tightly controlled group, mainly from episodes of hypoglycemia. Based on these findings, the Society for Critical Care Medicine recommended monitoring blood glucose levels but warned against tight glycemic control. A blood glucose level of 150 mg/dL should prompt intervention with a goal of keeping the blood glucose level less than 180 mg/dL. The initial method of glucose control should be an intravenous insulin infusion. This allows for close monitoring and adjustments of the insulin infusion until a stable nutritional regimen can be initiated. Metformin is a long-acting oral antihyperglycemic agent and is inappropriate for the acute management of hyperglycemia. Insulin glargine is a longacting agent that does not allow for rapid titration of serum glucose.

Either beta-blocker or calcium channel blockers can be used as a first-line agent; however, caution should be taken when combining the 2 drugs because they both slow the atrioventricular node and can lead to heart block when given together. If the patient is still in atrial fibrillation with rapid ventricular response after an atrioventricular node agent has been given, amiodarone is another potential agent. Amiodarone acts through several mechanisms to slow the ventricular rate and may convert the patient to sinus rhythm. Given amiodarone's large volume of distribution, a loading dose followed by intravenous infusion is needed to see a clinical effect. Adenosine is not used in patients with atrial fibrillation; it can be used for diagnosis or treatment of other supraventricular tachycardias.

20. A 40-year-old man presents with choledocholithiasis and an elevated serum bilirubin. He undergoes an endoscopic retrograde cholangiopancreatography, which successfully clears the stone from his common bile duct. The following day, he develops severe epigastric pain and has a lipase of 2500 U/L (10-140 2500 U/L). A CT scan of his abdomen and pelvis shows necrotizing pancreatitis (figure 20.1). What is the best method to decrease his risk of progression to infected necrotizing pancreatitis?

19. A 40-year-old otherwise healthy man is an unrestrained driver in a motor vehicle crash. He arrives with a Glasgow Coma Scale score of 3 and blood pressure of 90/60 mm Hg. He has a positive focused assessment with sonography for trauma exam and is taken for an exploratory laparotomy and splenectomy. During his initial admission labs to the intensive care unit, his glucose level is 145 mg/dL (70—100 mg/dL). Subsequent point of care blood glucoses for 24 hours are greater than 200 mg/dL. How should this patient's blood glucose levels be managed?

A. B. C. D. E.

Early enteral nutrition Probiotics Intravenous antibiotics Intravenous corticosteroids Parenteral nutrition

A. Insulin infusion to maintain blood glucose levels between 150 and 180 mg/dL B. Subcutaneous insulin sliding scale at blood glucose of 210 mg/dL C. Insulin infusion to maintain blood glucose levels between 80 and 110 mg/dL D. Subcutaneous insulin glargine 0.3 mg/kg E. Oral metformin

ANSWER:

A

Hyperglycemia is associated with adverse outcome in critically ill patients. Observational data showed increased infections and mortality in patients with hyperglycemia, especially in patients on parenteral nutrition. A single institution randomized controlled trial showed improved

Figure 20.1. 97

ANSWER:

necrosis. Enteral nutrition is thought to enhance the integrity of the mucosal barrier and reduce the translocation of bacteria systemically. A meta-analysis of 5 randomized controlled trials of enteral versus parental nutrition in patients with severe acute pancreatitis showed a decreased incidence of pancreatic infections and mortality with early (defined as within 3 days) enteral nutrition. The IAP and APA recommend early tube feeding to reduce the risks of infection and mortality. Parenteral nutrition is associated with worse outcomes and should be used only if enteral feeding is not tolerated.

A

Infected necrotizing pancreatitis is a rare but dreaded complication of acute pancreatitis. The nomenclature surrounding acute pancreatitis was clarified in a revision of the Atlanta Classification in 2012. Severe acute pancreatitis is defined as single or multiple organ failure for more than 48 hours. Necrotizing pancreatitis is diagnosed via contrastenhanced CT scan; areas of the pancreas that do not enhance with contrast are presumed to be necrotic (figure 20.2).

21. In patients with traumatic brain injury (head abbreviated injury score >3), venous thromboembolus (VTE) prophylaxis with low molecular weight heparin compared with unfractionated heparin results in A. increased mortality. B. decreased bleeding complications. C. increased unplanned returns to operating room. D. increased incidence of heparin-induced thrombocytopenia. E. decreased VTE rate.

ANSWER:

E

Venous thromboembolism (VTE) prophylaxis in traumatic brain injury patients is particularly challenging. Severe traumatic brain injury patients are at increased risk for VTE due to their immobility. They may also have a relative contraindication to chemical prophylaxis due to the presence of intracranial hemorrhage. Thus, the risk of VTE must be balanced against the risk of hemorrhage progression. Most data now suggest that chemical VTE prophylaxis can be started within 72 hours of a stable head CT scan. Another major question is the preferable agent for chemoprophylaxis: unfractionated heparin or low molecular weight heparin.

Figure 20.2. Necrotizing pancreatitis. Areas of the pancreas that do not enhance with contrast are presumed to be necrotic.

The distinction between necrotizing pancreatitis and infected necrotizing pancreatitis can be challenging to make. Patients with progression to infection will often have a worsening clinical course and CT scan findings of extraluminal gas in the collection. A fine needle aspirate can be performed to confirm the diagnosis but is not required to begin treatment. The progression to infected pancreatic necrosis is associated with increased morbidity and mortality. To reduce the rate of infection, systemic antibiotics were used as prophylaxis. Prospective randomized trials provided mixed data regarding the efficacy of routine antibiotic use in necrotizing pancreatitis. A meta-analysis of 14 trials of patients with severe acute pancreatitis did not support prophylactic antibiotic use. The guidelines from the International Association of Pancreatology and American Pancreatic Association (IAP/APA) recommend against intravenous antibiotic prophylaxis. Probiotics have been studied prospectively; however, no reduction in infection rates have been noted. There is also no evidence that intravenous steroids reduce the rate of conversion to infected pancreatic necrosis.

For patients with traumatic brain injury, use of low molecular weight heparin for VTE prophylaxis results in lower odds ratio of VTE and lower mortality compared with unfractionated heparin. There is no difference between the 2 agents with regard to bleeding risk or unplanned return to the operating room. Heparin-induced thrombocytopenia occurs up to 10 times more often with unfractionated heparin compared with low molecular weight heparin. Low molecular weight heparin should be used with caution in patients with renal insufficiency.

22. A 58-year-old woman has been in the intensive care unit on mechanical ventilation (rate set at 12, breathing at 22) for 8 days after sustaining multiple rib fractures. She is being treated for pneumonia and develops sepsis. She is oliguric (urine output 0.3 mL/kg/hour x 2 hours) despite 2 L of fluid. She is being monitored by echocardiography. Which of the

Both animal and human studies have implicated failure of the gut mucosal barrier in the development of infected pancreatic

98

following bedside tests is the best predictor of fluid responsiveness in this patient? A. B. C. D. E.

Delirium, a state of acute decline in cognitive function, is a clinical diagnosis common in elderly hospitalized patients. However, it remains poorly understood and often undiagnosed by healthcare providers. Delirium is associated with high rates of adverse outcome and mortality; it may be caused by a single factor, but in the older patient, the cause of delirium is frequently multifactorial due to the presence of comorbid conditions, polypharmacy, and physiologic decline. The older patient is vulnerable to potential insults, wherein a single dose of medication may precipitate delirium. Similarly, surgery is a risk factor for delirium in the elderly. Among the leading risk factors for delirium in both medical and noncardiac surgery patients are age greater than 70 years, existing cognitive or functional impairment, vision impairment, and history of alcohol abuse

Physical exam Central venous pressure Pulse pressure variation with respiration Passive leg raising Respiratory variation in vena cava diameter

ANSWER:

D

Over the last 20 years, there has been a dramatic shift in the use of intravenous fluid as well as the use of invasive monitoring systems in the intensive care unit. Both over- and underuse of intravenous fluids can have detrimental effects. This situation has led to development of the concept of "fluid responsiveness." The idea is that before administering more intravenous fluids to the patient, it would be helpful to know whether the patient is likely to respond or benefit.

Delirium is associated with increased postoperative morbidity and mortality. It leads to an increased length of hospital stay, higher medical costs, and an increased likelihood of post-discharge institutionalization. In the short term, patients with delirium after major noncardiac surgery demonstrate a risk for cognitive decline that is usually recovered by 2 months. However, in the longer term, patients who suffer delirium are at a greater risk of cognitive decline after 3 years than patients without postoperative delirium. Postoperative delirium is not associated with increased risk for intensive care unit admission, return to the operating room, or wound dehiscence.

This patient is developing sepsis and oliguria and has received 2 L of intravenous fluid. If she is still hypovolemic, then additional intravenous fluids could be beneficial. However, if she is hypervolemic and oliguric due to sepsis and the onset of organ dysfunction, further fluid resuscitation could adversely affect outcomes. Thus, assessing her volume status and likelihood of responding to additional fluid resuscitation is important.

Nonpharmacologic approaches to prevention and treatment of delirium are increasingly adopted due to their recognized clinical benefit and cost-effectiveness. They consist of nursing-based protocols that are implemented daily. These interventions include reorientation, reduction of psychoactive medications, early mobilization, promotion of sleep with sleep protocols, and proper hydration and nutrition. A metaanalysis involving more than 3000 patients showed that nonpharmacologic interventions tend to reduce overall length of hospital stay. Pharmacologic approaches to prevent and treat delirium, meanwhile, have failed to show benefit.

With the decreased use of pulmonary artery catheters, noninvasive methods of assessing volume status and fluid responsiveness have become more prevalent. A large metaanalysis of 23 trials involving more than 2200 patients found that passive leg raising with monitoring of cardiac response (as done by echocardiography in this case) was the best method for assessing fluid responsiveness. Both physical exam and central venous pressure measurement are poor predictors of fluid responsiveness. Respiratory variations in both pulse pressure and inferior vena cava diameter were widely studied. However, the majority of studies for both of these techniques excluded patients with spontaneous respiratory efforts, as in this patient (set rate 12, breathing at 22). Additionally, any cardiac arrhythmias can decrease the accuracy of pulse pressure and cava diameter variation.

24. A 78-year-old nursing home resident with diabetes and congestive heart failure presented to the hospital 1 week ago with near-obstructing colon cancer and a 20-pound (10%) weight loss in the last 2 months. He is now intubated and ventilated in the intensive care unit after urgent sigmoid colectomy. Attempted enteral feeding through a nasojejunal feeding tube resulted in severe abdominal distention. The best next step to provide nutritional support is

23. A 72-year-old woman is recovering from a partial gastrectomy. On postoperative day 3, she is agitated and confused. Postoperative delirium after major abdominal surgery in older patients is associated with A. B. C. D. E.

A. B. C. D. E.

increased length of hospital stay. increased intensive care unit admission rates. a return to operating room. no change in long-term cognitive performance. wound dehiscence.

ANSWER:

A 99

trophic gastric feeds (10-20 mL/hour). parenteral nutrition. percutaneous endoscopic gastrostomy placement. peripheral parenteral nutrition. jejunostomy feeding tube placement.

ANSWER:

E. Cooling to 32°C for 48 hours

B

Timely and adequate nutritional support is an integral part of the care of the intensive care unit and postoperative patient Malnutrition is associated with morbidity and mortality in this patient population, and a catabolic, proinflammatory state can lead to oxidative stress and multiorgan failure. Thus, early feeding improves outcomes and likely plays a beneficial role in immunity. Estimates of caloric needs in the critically ill patient are weight based and are in the range of 25 to 30 Kcal/kg/day. Protein requirements in the critically ill are approximately 1.2 g/kg/day.

ANSWER:

Patients admitted to the intensive care unit after out-ofhospital cardiac arrest have a high risk of death and neurologic deficit. Targeted temperature management (TTM) is an intervention that has both short- and long-term survival and neurologic recovery advantages. Guidelines suggest TTM with urgent coronary angiography as initial steps. Percutaneous coronary intervention follows if appropriate lesions are found.

Enteral nutrition is often preferred to parenteral nutrition in the patient who cannot tolerate per os feeding, because it maintains gut integrity and modulates the stress and immune responses. Early initiation of feeds is recommended, even after gastrointestinal surgery. It is usually started as trophic feeds (at 10-20 mL/hour) and advanced incrementally to goal caloric and protein requirements within 24 to 72 hours.

The TTM trial showed that the benefits of 36°C were similar to a TTM of 33°C. Current recommendations have a target of 33 to 36°C for at least 24 hours.

Common approaches for delivering enteral nutrition to patients who are unable to maintain sufficient volitional intake are via nasogastric or nasojejunal feeding tubes, gastrostomy tubes, and jejunostomy feeding tubes. Enteral feeding, however, is limited by potential gastrointestinal intolerance or aspiration. Changing the position of the feeding tube (gastric vs postpyloric), changing the formula, or changing the rate of delivery are maneuvers that can improve gastrointestinal intolerance characterized by abdominal distention, discomfort, diarrhea, or high residual volume.

The Time-differentiated Therapeutic Hypothermia trial studied the difference in duration of hypothermia (24 vs 48 hours). This study confirmed no difference in 6-month neurologic outcomes or mortality in 355 patients randomized to TTM (33 ± 1OC) for 48 hours or 24 hours. Based on these results, current recommendations are TTM to 33°C to 36°C for 24 hours. No recommendation exists to cool to 32°C.

This patient meets Society of Critical Care Medicine and American Society for Parenteral and Enteral Nutrition diagnostic criteria for malnutrition (involuntary loss of 10% or more of usual body weight within 6 months or involuntary loss of greater than or 5% or more of usual body weight in 1 month). In intensive care unit patients at high nutrition risk, parenteral nutrition should be started as soon as possible, if adequate enteral nutrition is not tolerated. Neither peripheral parenteral nutrition nor trophic feeds will supply this patient with sufficient calories and protein.

25. A 58-year-old woman underwent an elective laparotomy for resection of pancreatic cancer with curative pancreaticoduodenectomy. She was discharged in excellent condition, but 24 hours later sustained an out-of-hospital cardiac arrest due to ventricular fibrillation. She was successfully resuscitated and admitted to the intensive care unit intubated and ventilated after successful percutaneous coronary intervention. To improve her chance of survival and neurologic recovery, what is the recommended use of targeted temperature management in this patient? A. B. C. D.

C

Cooling to 32°C for 24 hours Targeted temperature of 37°C for 24 hours Cooling to 33° to 36°C for 24 hours Targeted temperature of 37°C for 48 hours

100

Surgical Critical Care Part II

delirium. Which of the following intravenous infusions for endotracheal tube tolerance has the lowest risk of delirium?

ITEMS 1-25 For each question, select the best possible response.

A. B. C. D. E.

1. A 52-year-old patient is critically ill in the intensive care unit 5 days after urgent hepatic resection for bleeding hepatocellular cancer. He suddenly develops bright red blood in one of the surgical drains. Intravascular contrast for CT angiography is planned as a diagnostic study to consider angioembolization of the source of bleeding. What strategy is effective in reducing the risk of contrast associated acute kidney injury? A. B. C. D. E.

ANSWER:

C

Delirium is very common in the intensive care unit and is frequently related to the sedatives and pain medications used for endotracheal tube tolerance in intubated and mechanically ventilated patients. Delirium is associated with adverse outcomes in intensive care unit patients.

Isotonic saline before and after Sodium bicarbonate after Sodium bicarbonate during contrast infusion Hypertonic saline before and after Albumin after

ANSWER:

Midazolam Lorazepam Dexmedetomidine Propofol Ketamine

In 3 meta-analyses including only randomized controlled trials related to intensive care unit patients, dexmedetomidine was associated with a reduced incidence of delirium, intensive care unit length of stay, and mechanical ventilation duration, despite a significant heterogeneity among studies. Benzodiazepines, propofol, and ketamine have a higher risk of intensive care unit delirium. Unfortunately, dexmedetomidine is associated with an increase in bradycardia and hypotension.

A

Contrast-induced acute kidney injury (AKI) is reported to be the third leading cause of AKI in hospitalized patients, with associated high morbidity and mortality. Current guidelines recommend intravenous volume expansion with isotonic crystalloid solution before and after intravascular contrast use as the cornerstone treatment for the prevention of contrastinduced AKI.

3. A 65-year-old patient underwent elective resection of a large retroperitoneal] sarcoma. Postoperatively, he developed severe hypoxemia, and chest radiograph confirmed severe bibasilar atelectasis. He now requires supplemental oxygen for treatment of his severe hypoxemia but cannot tolerate noninvasive positive pressure ventilation via face mask. What is the best method to provide supplemental oxygen to reduce the risk of endotracheal intubation?

The HYDRAREA trial tested whether sodium bicarbonate was superior to isotonic saline in preventing contrastassociated AKI in critically ill patients. Two groups of critically ill patients with normal renal function (n = 307) were randomized to intravenous hydration with 0.9% saline or 1.4% sodium bicarbonate. The same administration protocol was used with each drug. During the hour before the contrast the drugs were given at 3 mL/kg and for 6 hours after the contrast the drugs were given at 1 mL/kg/hour. Approximately one-third of patients developed contrastassociated AKI in both groups (p = .81). Other similar outcomes between the 2 groups included need for renal replacement therapy (5 and 6 patients; p = .77), intensive care unit length of stay (24.7 ± 22.9 and 23 ± 23.8 days; p =.52), and mortality (25 and 24 patients; p > .99).

A. B. C. D. E.

Nonrebreather face mask Face tent High-flow nasal cannula Transtracheal oxygen Venturi face mask

ANSWER:

Infusions of sodium bicarbonate, hypertonic saline, or albumin are not recommended for prevention of contrastinduced AKI.

C

High-flow nasal cannula (HFNC) oxygen therapy is commonly used in intensive care units to provide supplemental oxygen therapy to patients with hypoxemia and respiratory insufficiency. HFNC delivers heated humidified oxygen at a flow rate of 10 to 60 L/minute and more predictable FiO2 (up to FiO21.0) through nasal prongs. The delivery of oxygen at high flow rates creates a positive pressure effect, increases end-expiratory lung volume, reduces anatomic dead space, and reduces the patients work of breathing.

2. A 92-year-old patient was admitted to the surgical intensive care unit after urgent laparotomy for perforated right colon cancer with fecal peritonitis She is intubated and mechanically ventilated. Her family reports that the last time she was in the intensive care unit, she developed severe

102

HFNC was compared with noninvasive positive pressure ventilation (NIPPV) and conventional oxygen therapy in adult patients with acute respiratory failure. Eighteen trials with a total of 3881 patients were pooled in the final analysis. Compared with conventional oxygen therapy, HFNC was associated with a lower rate of endotracheal intubation (z 2.55, p - .01) while no significant difference was found in the comparison with NIPPV (z = 1.40, p = .16). As for intensive care unit mortality and length of intensive care unit stay, HFNC did not exhibit an advantage over either conventional oxygen therapy or NIPPV. NIPPV is more difficult for patients to tolerate than HFNC or conventional oxygen therapy, related to the tight-fitting mask required for NIPPV.

The 2016 Surviving Sepsis Guidelines recommend the following: ... that stress ulcer prophylaxis be given to patients with sepsis or septic shock who have risk factors for gastrointestinal bleeding (strong recommendation, low quality of evidence). Based on the available evidence, the desirable consequences of stress ulcer prophylaxis outweigh the undesirable consequences; therefore, we made a strong recommendation in favor of using stress ulcer prophylaxis in patients with risk factors. The American Society of Health-System Pharmacists (ASHP) Guidelines made similar recommendations.

For patients with acute respiratory failure, HFNC is better tolerated as a more reliable alternative than NIPPV and conventional oxygen therapy to reduce the rate of endotracheal intubation

5. A 70-year old man underwent an Ivor Lewis esophagectomy with gastric pull-up 10 days ago His postoperative course was complicated by an anastomotic leak •that was managed with a covered stent. He is now hemodynamically normal but remains deconditioned on the ventilator. He is afebrile with a normal white btood cell count. When jejunostomy tube feeds began 4 days ago, he developed a high output of milky fluid from the right chest tube, which drained 1200 to 1500 mL per day. Triglycerides in the pleural fluid measured 400 mg/dl. (<150 mg/dL). Tube feeds were stopped, and parenteral nutrition was started. After tube feeds "halted, the chest tube output became serous rather than milky, but continued to drain more than 1 L per day. His chest x-ray shows good expansion of the right lung without effusion. The surgical team is hoping to avoid return to operating room due to patient's fragile condition. What option gives the patient best chance to avoid reoperation?

4. Which of the following factors has the highest risk for clinically important gastrointestinal bleeding from stress ulcers in critically ill patients? A. B. C. D. E.

Mechanical ventilation Renal failure Hepatic failure Shock Coagulopathy

ANSWER:

A

Stress-related mucosal disease (SRMD) and stress ulcer are common in intensive care unit patients. The underlying cause of SRMD is hypoperfusion of the mucosa in the upper gastrointestinal tract. Hypoperfusion of the mucosa predisposes an intensive care unit patient to upper gastrointestinal bleeding. Severed risk factors are associated with the development of SRMD, and national guidelines recommend stress ulcer prophylaxis for intensive care unit patients with these risk factors.

A. B. C. D. E.

Octreotide High-dose corticosteroids Thoracic duct embolization Talc pleurodesis through chest tube External beam radiation to thoracic duct

ANSWER:

Two strong independent risk factors (mechanical ventilation and coagulopathy) for clinically important gastrointestinal bleeding were identified in a landmark prospective multicenter study. Clinically important gastrointestinal bleeding is defined as overt bleeding in association with hemodynamic compromise or the need for blood transfusion. Clinically important bleeding occurred in 33 (1.5%) of 2252 patients. Respiratory failure requiring mechanical ventilation was a stronger risk factor than coagulopathy. Of 847 patients who had one or both risk factors, 3.7% of patients had clinically important bleeding (mortality rate 48.5%), compared with 0.1% in 1405 patients without these risk factors (mortality rate 9.1%).

C

Thoracic duct leak resulting in chylothorax is a well-known complication after esophagectomy. It is caused by iatrogenic injury of the thoracic duct during esophageal dissection. Chyle appears as milky white drainage from pleural space, and a triglyceride level greater than 110 mg/dL is diagnostic. Chyle is also rich in nutrients, lymphocytes, and immunoglobulins. Consequently, high-volume chyle leaks result in malnutrition and immunocompromise, with mortality rates as high as 50%. If chyle drainage is low (< 0.5 L/day), noninterventional management with a fat-free diet and somatostatin analogue usually suffices. Talc pleurodesis via a chest tube is a further option to treat patients with low-output chylothorax.

103

High-output leaks (>1000 mL/day) generally require ligation or interruption of the thoracic duct. Thoracic duct ligation after esophagectomy requires return to the operating room with either a thoracotomy or thoracoscopy and direct ligation of the thoracic duct. In deconditioned patients, like the patient in this question, re-operation is fraught with complications.

failure and severe hypoxemia. ARDS has clinical features that overlap with other causes of respiratory failure, including cardiogenic pulmonary edema, viral or diffuse bacterial pneumonia, and inhalation injury. Four criteria need to be present to make a diagnosis of ARDS: (1) acute onset of respiratory symptoms beginning within 1 week of a clinical insult, (2) bilateral patchy infiltrates on a chest x-ray or chest CT scan, (3) cardiogenic pulmonary edema or fluid overload is not the primary cause of the respiratory failure confirmed with echocardiography, and (4) a PaO2/FiO2 consistent with a severe impairment of oxygenation. The PaO2/ FiO2 is calculated by dividing the PaO2 from an arterial blood gas reading by the FiO2 delivered by the ventilator expressed as a decimal ranging from 0.21 to 1.0. Reducing positive end expiratory pressure (PEEP) would worsen his oxygenation.

Thoracic duct embolization (TDE) is a nonsurgical alternative for treatment of chylothorax. Evidence suggests that thoracic duct embolization is successful in 60 to 80% of patients and obviates the need for surgical thoracic duct ligation. After catheterization, the thoracic duct is occluded below the point of chyle leakage with embolization coils, cyanoacrylate glue, or both. The ability to visualize the lymphatic system and thoracic duct abnormality potentially increases the treatment success rate, and the minimally invasive approach eliminates the postoperative morbidity and mortality associated with reoperation. Pedal lymphangiography has been the major technical obstacle for wider adoption of TDE, and recent development of intranodal lymphangiography approached via the inguinal lymph nodes has opened the door for broader application of TDE.

The mainstay of management of ARDS is a lung protective strategy using low tidal volume ventilation (6-8 mL/kg of ideal body weight) with high levels of PEEP. Most patients with mild or moderate ARDS will recover with supportive care and lung protective ventilation. However, patients with severe ARDS may have refractory hypoxemia and elevated plateau pressures despite low tidal volume ventilation. Several adjunctive therapies have been studied for patients with severe ARDS, but none are very promising.

For patients with chylothoraces related to malignancy— usually lymphoma— the pathophysiology of the condition is decidedly different than thoracic duct injury. The mechanism by which chyle accumulates in the setting of malignancy involves infiltration of the thoracic duct and its tributaries with tumor cells causing obstruction, excess pressure, and microperforation. Thoracic duct ligation and embolization are generally ineffective in this setting. Malignant chylothoraces usually respond favorably to chemotherapy, and sometimes radiation to posterior mediastinal lymph nodes; surgical intervention is rarely necessary.

Prone positioning was initially described in case reports and retrospective series; however, a single large prospective trial demonstrated a potential benefit. Patients with severe ARDS ventilated in the prone position had an improvement in oxygenation allowing a decreased FiO2. There was also a survived advantage in patients with severe ARDS undergoing prone ventilation. Because prone ventilation is extremely labor intensive for staff and requires specialized units, these results may not be widely applicable. Also, prone ventilation would not be possible in this patient because both of his femurs are in traction.

6. A25-year-old man was an unhelmeted rider in a motorcycle versus tractor trailer crash. He was admitted to the intensive care unit with a severe traumatic brain injury with subdural hematoma, a grade III liver injury, and bilateral femur fractures in traction. On post injury day 1, his oxygen requirement increases, and he has diffuse, patchy infiltrates on chest radiograph. He is placed on 6 mL/kg tidal volume with positive end expiratory pressure (PEEP) of 15 cm H2O but still |has high plateau pressures. What intervention should be initiated next? A. B. C. D. E.

Corticosteroids may be beneficial for patients with prolonged courses of ARDS. Patients with refractory ARDS can be treated with a 30-day course of steroids starting between days 7 and 14. Corticosteroids show an improvement in oxygenation and ventilator-free days, but mortality data are mixed. There is a clear detriment to starting steroids after day 14. Inhaled nitric oxide and inhaled prostaglandins can improve oxygenation in the first 48 to 72 hours, but there is no survival benefit.

Neuromuscular blockade B. Prone position ventilation C. Reduced level of PEEP to 5 cm H2O D. Airway pressure release ventilation Inhaled nitric oxide

ANSWER:

The use of neuromuscular blockade can improve oxygenation and decrease mortality in severe ARDS. A multicenter trial of 340 patients randomized patients to neuromuscular blockade with cisatracurium or placebo within 48 hours of diagnosis of ARDS. All patients saw an improvement in oxygenation, and the patients with severe ARDS had a decrease in 90-day mortality.

A

Acute respiratory distress syndrome (ARDS) is an acute, diffuse, inflammatory lung injury resulting in respiratory

104

7. Percutaneous tracheostomy

8. An intensive care unit patient undergoes a transthoracic echocardiogram that shows a pericardial effusion. What associated feature suggests the strongest need for urgent pericardial drainage procedure?

A. has a higher complication rate than surgical tracheostomy. B. is safe without bronchoscopic guidance. C. is contraindicated in morbidly obese patients. D. is safest when performed by surgeons. E. is contraindicated in patients with recent anterior cervical spinal surgery.

ANSWER:

A. Echocardiographic evidence of inferior vena cava collapse B. 10 mm Hg decrease in systolic blood pressure during inspiration C. Effusion loculated behind the left atrium D. Patient with metastatic effusion E. Hypotension

B

Percutaneous tracheostomy is a safe and effective alternative to open tracheostomy. The percutaneous approach tends to have lower overall costs and can be carried out in an intensive care unit setting, rather than the operating room, without an increase in complication rates. The original technique for percutaneous tracheostomy included the use of a bronchoscope. Bronchoscopic guidance can visualize tracheal instrumentation and provide an additional measure of safety when performing this procedure.

ANSWER:

E

Cardiac tamponade occurs when intrapericardial fluid creates a situation where the cardiac chambers cannot fill leading to cardiogenic shock. This situation requires urgent therapeutic intervention, consisting of either percutaneous pericardial drainage or surgical pericardial window. Because the pericardium has limited compliance, even a moderate volume of pericardial fluid may impair right ventricular filling, often manifesting with a dramatic drop in cardiac output, hypotension, and shock. Therefore, hypotension in the presence of a significant pericardial effusion demands urgent pericardial drainage procedure to avoid likely hemodynamic collapse.

There are several techniques for performing bedside percutaneous tracheostomy without the aid of a bronchoscope. A modified approach, first described in 1999, included a small incision with dissection to the pretracheal space, thus allowing direct visualization of the trachea before tracheal puncture, serial dilation using the Seidinger technique, and insertion of the tracheostomy tube. Multiple large studies showed no difference in complication rates between procedures performed with and without a bronchoscope.

Other signs and symptoms of cardiac tamponade are more subtle, and, although they may help guide the decision to perform a pericardial drainage procedure, they are not as specific or urgent as frank hypotension.

Another safe and efficacious technique for tracheostomy without a bronchoscope is ultrasound-guided tracheostomy. Ultrasound can be used to visualize the trachea as well as nearby vascular structures, thus providing additional safety in lieu of bronchoscopy.

Pulsus paradoxus, defined as a 10 to 20 mm Hg decrease in systolic blood pressure during inspiration, is often cited as a sign of cardiac tamponade. Pulsus paradoxus is an exaggeration of normal diminution in blood pressure during inspiration rather than a paradoxical situation. Negative intrapleural pressure causes increased right ventricle filling, displacement of intraventricular septum to left and thus decreased left ventricle output. Consequently, stroke volume and blood pressure are diminished during inspiration. However, in normal physiologic states, this difference is quite small. When this difference exceeds 10 mm Hg in the presence of a pericardial effusion, cardiac tamponade is considered likely. Nevertheless, many conditions besides pericardial effusion can cause pulsus paradoxus, including severe chronic obstructive pulmonary disease, mitral stenosis, restrictive cardiomyopathy, obesity, and ascites.

Morbid obesity was thought to constitute a relative contraindication to percutaneous tracheostomy. One recent study evaluated percutaneous tracheostomy in patients with a BMI greater than 35 and found no increase in rates of complication compared with similar patients undergoing open tracheostomy Percutaneous tracheostomies are performed by medical intensivists, rather than surgeons, at many institutions. As long as the medical intensivists have appropriate training and credentialing, their results are equivalent to surgeons performing percutaneous tracheostomy.

Certain findings on echocardiography may indicate the presence of cardiac tamponade when a pericardial effusion is present. This list includes early diastolic collapse of the right ventricle (sensitivity -60%, specificity ~90%) and inferior vena cava plethora, defined as a diameter greater than 2.1 cm with less than 50% decrease with inspiration. However, none of these findings, by themselves, is as important as clinical

A recent study evaluated percutaneous tracheostomy in patients after anterior cervical spinal surgery and found no increase in surgical site infection. Percutaneous tracheostomy is considered safe in this group of patients.

105

hypotension in terms of indicating urgent need for pericardial drainage.

associated physical exam and lab findings. The fact that the heart is hyperdynamic with a normal ejection fractions rules out heart failure and is consistent with diminished afterload, which is the hemodynamic result of adrenal insufficiency.

Malignant pericardial effusions often develop in patients with widely metastatic carcinoma. Although malignant pericardial effusions often require pericardial drainage procedures, the presence of malignancy is not considered an indication for urgent drainage.

10. A 78-year-old man is in the surgical intensive care unit 5 days after colectomy for T2 colon adenocarcinoma. He is not intubated but is on high-flow oxygen. He has a persistent ileus with nausea and vomiting treated with a nasogastric tube and parenteral nutrition. He has increased respiratory secretions requiring frequent suctioning during which he desaturates to an oxygen saturation of 85%. His respiratory rate is 32 breaths per minute, and he is using accessory muscles to breath. The next step to manage his respiratory status is

9. A 50-year-old man presents with a traumatic brain injury and is intubated for airway protection using etomidate and succinylcholine. Twelve hours later, he is noted to have a blood pressure of 80/50 mm Hg, heart rate of 122 beats per minute, temperature of 37°C, and SpO2 of 95%. He remains hypotensive despite -fluid resuscitation. Laboratory values include the following: sodium -130 mEq/L (135-145 mEq/L), potassium - 6.1 mEq/L (3.5-5.0 mEq/L), creatinine - 2.0 mg/dL (0.4-1.30 mg/dL), and glucose = 60 mg/dL (70-100 mg/dL). His initial abdominal CT scan was normal. Echocardiogram shows a hyperdynamic heart with a normal ejection fraction. The most likely explanation for his persistent shock state is A. B. C. D. E.

A. B. C. D. E.

ANSWER:

his head injury. hyperdynamic heart failure. missed visceral perforation. acute kidney failure. adrenal insufficiency.

ANSWER:

bilevel positive airway pressure. tracheostomy. incentive spirometry. endotracheal intubation. continuous positive airway pressure.

D

This patient has impending respiratory failure and needs intubation based on his respiratory parameters and clinical situation. Avoiding intubation is always a goal of respiratory therapy, but given this clinical scenario, intubation is necessary. His respiratory rate, use of accessory muscles, secretions, and oxygen requirements all indicate the need for intubation. His clinical condition also supports intubation.

E

Primary adrenal insufficiency or Addison disease results in symptoms that occur because of title lack of mineralocorticoid and glucocorticoid made in the adrenal cortex. The most common cause is autoimmune but there are a variety of other causes, including drug suppression of glandular function. The most obvious of these is a patient whose adrenal function is suppressed by exogenous steroids taken for any number of diseases. Commonly used drugs known to suppress adrenal function include ketoconazole and etomidate.

Continuous positive airway pressure (CPAP) or bilevel noninvasive positive pressure ventilation (NPPV) can be an effective and safe intervention for the treatment of adults with acute respiratory failure after upper abdominal surgery. However, relative contraindications for these noninvasive methods of respiratory support include recurrent vomiting, large amounts of pulmonary secretions, and an inability to tolerate the devices. The patient's ileus places him at risk for vomiting and aspiration. His secretions requiring frequent suctioning would also complicate the use of a noninvasive form of ventilatory support. A mechanical impediment to the use of mask ventilation is his nasogastric tube, which will make secure mask fit difficult.

Common findings include hyponatremia and hyperkalemia. Hyponatremia is caused by both the loss of sodium in the urine and the decreased free water clearance associated with increased vasopressin and angiotensin II. Sodium is lost but free water remains in the vascular space. Other findings include unexplained hypotension, abdominal pain, vomiting, and diarrhea.

Postoperative respiratory failure is best prevented with a comprehensive program of respiratory support. This program must include preoperative and postoperative interventions aimed at improving respiratory function. Components of one program include preoperative education, incentive spirometry, mobilization, and proper pain control. Application of these interventions in isolation has limited benefit to avoid intubation. Low-quality evidence shows that CPAP in the postoperative period might reduce postoperative atelectasis, pneumonia, and reintubation. There is also insufficient evidence to show high flow nasal cannula

Although a severe head injury could damage the pituitary and cause secondary adrenal insufficiency due to disruption of the hypothalamic-pituitary-adrenal axis, this situation is rare and symptoms are unlikely to occur within 12 hours. A missed visceral injury, though possible, is minimized by the normal CT scan and unlikely to cause fluid unresponsive shock so soon. Acute kidney failure does not explain the vital signs and

106

provides a safe and efficacious respiratory support for adult intensive care unit patients and can be used to prevent reintubation routinely.

C. Chlordiazepoxide D. Clonidine E. Disulfiram

A tracheostomy is not indicated in this urgent scenario although might be needed in the future if extubation cannot be achieved.

ANSWER:

Acute alcohol withdrawal may present in patients with an alcoholic history undergoing acute surgery and trauma or in postoperative elective surgery patients where an alcoholic history is missed. Uncontrolled severe alcohol withdrawal may include seizures, alcoholic hallucinosis, delirium tremens, severe hypertension, fluid and electrolyte disorders, tachyarrhythmias, and death. Protocols for moderate to severe alcohol withdrawal largely focus on the management of psychomotor agitation with benzodiazepines. High intravenous doses of lorazepam are dangerous because the diluent includes propylene glycol, which can produce lactic acidosis, myocardial depression, and shock. When high doses of benzodiazepines are no controlling symptoms and when excess sympathetic response (hypertension, tachycardia) occurs, additional agents may be required.

11. Telemedicine in US surgical intensive care units A. B. C. D. E.

improves adherence to best-practice guidelines. is consistently associated with decreases in mortality. affects 30% of intensive care unit patients nationwide. decreases resident duty hours. is prohibited from use across state lines by federal law.

ANSWER:

D

A

Telemedicine in intensive care units is increasing in frequency but currently affects only 11% of intensive care unit patients nationally. Telemedicine appears to improve adherence to best-practice guidelines, but no consistently demonstrated decrease in intensive care unit mortality is evident. In a meta-analysis including 35 intensive care units and 41,734 patients, intensive care unit mortality and length of stay decreased, but no decrease was seen for in-hospital mortality or length of stay. A second meta-analysis (11 trials) showed lower intensive care unit and hospital mortality. Studies of telemedicine in different hospital settings are difficult to interpret due to a variety of environments and a lack of consistency in the elements of telemedicine studied.

This patient is receiving lorazepam at 10 mg/hour, which is a large dose. Barbiturates and propofol are used in cases of refractory delirium tremens to induce coma and prevent seizures. Flumazenil produces rapid benzodiazepine reversal and may induce seizures. Disulfiram is used in the outpatient management of alcohol abuse to prevent recidivism, but it is not used in acute withdrawal. Chlordiazepoxide is used as a long-acting oral agent in alcohol withdrawal but is not a preferred agent in the intensive care unit. Alpha-2 agonists, such as clonidine and dexmedetomidine are not used as primary agents and may not reduce seizure risk but may be used adjunctively to reduce benzodiazepine dose, hypertension, and tachycardia.

Telemedicine in teaching intensive care units has not decreased resident work hours, but residents have subjectively reported a positive experience with telemedicine and its perceived positive impact on patient safety. No federal laws bar the use of telemedicine across state lines.

Items 13-17 Each lettered response may be selected once, more than once, or not at all.

12. A 65-year-old man with a history of smoking and alcoholism is admitted to the intensive care unit after being found down in the street. A subdural hematoma is diagnosed and treated nonoperatively by neurosurgery. On hospital day 2, the patient has a Glasgow Coma Scale score of 14 and is hemodynamically normal. Later that day, the patient develops acute alcohol withdrawal and is started on lorazepam. Persistent symptoms require increasingly high doses of lorazepam, and the patient is intubated for airway protection. On hospital day 3, the patient is on the ventilator and is somnolent but opens his eyes with stimulation. He has no tremors but has a regular heart rate of 130 beats per minute and a blood pressure of 160/100 mm Hg on a lorazepam drip of 10 mg per hour. Which medication should be given?

A. B. C. D. E.

Phenylephrine Dopamine Norepinephrine Epinephrine Vasopressin

13. Preferred initial pressor for septic shock 14. Available in a premixed bag 15. Can cause reflex bradycardia, especially in spinal cord injury 16. Primarily used as a second-line agent 17. Acts at 5 different adrenergic receptor subtypes

A. Hydralazine B. Flumazenil

107

ANSWERS:

dysfunction. The revised adult sepsis guidelines were published in 2016. The cornerstone of sepsis therapy includes aggressive fluid resuscitation, prompt initiation of broadspectrum antibiotics, a mean arterial pressure of at least 65 mm Hg, and source control. The new recommendations specify an initial crystalloid infusion of at least 30 mL/kg within the first 3 hours. The emphasis on early and aggressive volume resuscitation is designed to improve patient perfusion until additional hemodynamic monitoring is put in place. Albumin can be used as a second-line agent, in addition to crystalloids, when patients require large volume resuscitation. The guideline document recommends against the use of hydroxyethyl starch in sepsis resuscitation, because it is associated with a higher risk of acute kidney injury and death.

C, B, A, E, D

The initial choice of vasopressor in critical care patients has evolved over the past 30 years. Selections are usually evidence based, but availability, dogma, and habit may play a role. Before 2000, dopamine was a preferred pressor in many US intensive care units. However, European trials showed the relative superiority of norepinephrine over dopamine in septic shock and discredited "renal dose" dopamine for acute kidney injury. Because of these findings, the use of norepinephrine has eclipsed dopamine. Dopamine's long history may explain its continued immediate availability in premixed bags in the United States. Norepinephrine has more alpha-adrenergic mediated vasoconstriction than dopamine and produces a greater rise in blood pressure. Dopamine is a stronger inotrope via its larger beta-adrenergic effects. It produces greater increases in cardiac output than norepinephrine.

19. Which of the following is an effective rescue strategy in a patient with severe acute respiratory distress syndrome?

Phenylephrine is a pure alpha-adrenergic vasoconstrictor. It does not produce the tachyarrhythmias and cardiac irritability of agents with beta effects but can produce bradycardia when sympathetic tone is lacking. Phenylephrine is often given via intravenous push and is the pressor most often used in peripheral intravenous lines.

A. B. C. D. E.

Vasopressin has multiple effects, including constriction of arterioles via the V1A receptor. It is typically used as a secondline agent in septic shock after catecholamine agents, when response to epinephrine is insufficient in anaphylaxis and septic shock. It is not superior to norepinephrine in septic shock.

ANSWER:

18. A 100-kg woman is admitted with cholangitis. After endoscopic retrograde cholangiopancreatography clears her common bile duct of obstructing bile duct stones, she is admitted to the intensive care unit for persistent hypotension (blood pressure 80/60 mm Hg). The recommended fluid infusion for the first 3 hours of resuscitation is

Ventilator-induced lung injury is thought to be due to the shear injury imposed by positive pressure ventilation resulting from the opening then collapse of alveolar units. Greater attention is being paid to airway pressures, with current recommendation of maintaining plateau pressures less than 30 cm H2O to avoid barotrauma and using normal (physiologic) tidal volumes, generally 6 mL/kg of predicted body weight. High-frequency oscillatory ventilation (HFOV) generated some initial enthusiasm due to the improvements seen in oxygenation and the relative constant airway pressures associated with this mode of ventilation.

1000 mL of crystalloid. 2000 mL of crystalloid. 3000 mL of crystalloid. 500 mL 5% albumin. 500 mL of 6% hydroxyethyl starch.

ANSWER:

B

Acute respiratory distress syndrome (ARDS) continues to be a management challenge in the intensive care unit. No definitive intervention has been discovered to date to reverse or prevent the development of ARDS, and treatment is generally supportive. This supports the observation that ARDS is a heterogenous process with multiple etiologies that include intrinsic respiratory injury and exogenous pulmonary factors such as injury from cytokine release due to the systemic inflammatory response syndrome. Only modest improvements in survival have occurred in the past several decades, largely due to strategies designed to protect the lung from ventilator-induced lung injury. Controversy remains over the most effective ventilation strategies: high-versus low positive end-expiratory pressure (PEEP), permissive hypercapnia, open-lung ventilation strategies, FiO2-PEEP tables, a resurgence of the "best-PEEP" concept, recruitment maneuvers, and the like.

Epinephrine acts at all 5 adrenergic receptors. It is the only listed agent that combines vasoconstriction, vasodilation, inotropy, chronotropy, bronchodilation, glycogenolysis, and lipolysis.

A. B. C. D. E.

Plat, greater than 35 cm H2O Prone positioning Positive end-expiratory pressure of 5 cm H2O or less High-frequency oscillatory ventilation Tidal volume of 8 to 10 mL/kg

C

Sepsis is a life-threatening condition that must be treated immediately to reverse hypotension and mitigate organ

108

Subsequent studies have not demonstrated any mortality benefit for HFOV, and adequate oxygenation can be achieved with current airway-protective ventilation strategies that do not require expensive ventilators and expertise.

Traditional tidal volumes of 10 to 15 mL/kg exceed normal tidal volumes in a resting adult and run the risk of overdistending open alveolar segments and exacerbating abnormal ventilation perfusion relationships (i.e., increasing physiologic shunt and producing shear stress/injury). A clinical trial comparing traditional tidal volume settings with low tidal volumes of approximately 6 mL/kg of ideal body weight demonstrated decreased mortality and increased the number of ventilator-free days in patients with acute lung injury and acute respiratory distress syndrome. An appropriate initial ventilator setting is 6 mL/kg x 70 kg = 420 mL tidal volume, with the addition of positive end-expiratory pressure (PEEP) obtained from a PEEP/FiO2 table or titrated to keep the peak plateau airway pressure less than 30 cm H2O. Minute ventilation, and consequently arterial pH, is maintained by adjusting the rate of ventilation. The goal for oxygen saturation is 85 to 95%.

It has been known for some time that placing mechanically ventilated patients in the prone position improves oxygenation. The effect is thought to be due to improvements in functional residual volume because of improvements in dorsal lung ventilation, a reduction in ventilation/perfusion mismatching (physiologic shunt), an improvement in lung recruitment, a reduction in ventilator-induced lung injury by a more homogeneous distribution airway pressure among alveolar units, and a decrease in circulating inflammatory cytokines. Prone positioning redistributes airway pressure over a larger alveolar surface area, which minimizes shear injury, particularly when PEEP is concomitantly used. Additionally, prone positioning generally allows higher levels of PEEP, which may help prevent derecruitment of alveoli. Recent clinical studies and several large metaanalyses demonstrated not only improvement in oxygenation but also better survival. Prone positioning is well tolerated but somewhat resource intensive because it requires trained staff familiar with the techniques. To be effective, patients should be kept prone for more than 12 hours per day. Prone positioning is now included in the practice guidelines for moderate to severe ARDS by the American Thoracic Society, the European Society of Intensive Care Medicine, and the Society of Critical Care Medicine.

Items 21-23 Each lettered response may be selected once, more than once, or not at all. A. B. C. D. E.

Amiodarone Calcium-channel blocker Cardioversion Intravenous magnesium Atropine

21. Monomorphic ventricular tachycardia with hypotension 22. Monomorphic ventricular ectopy

20. A 42-year-old man fell from a height of 40 feet at a construction site. His injuries include multiple left-sided rib fractures with a flail segment of his chest wall, a left-sided pulmonary contusion, and a pelvic fracture. He is appropriately resuscitated but requires intubation for respiratory distress with severe hypoxemia. His weight is 78 kg (ideal body weight - 70 kg). The most appropriate initial tidal volume setting is A. B. C. D. E.

23. Bradycardia with hypotension

ANSWERS:

In monomorphic ventricular tachycardia, the electrical conduction is prolonged when moving across the ventricles. This results in a widened QRS complex. When all of the complexes appear the same, the phenomena is called monomorphic. The pathophysiology behind this electrical activity is a result of the same ventricular focus initiating the conduction, and, hence, the contraction, within the ventricle. Immediate cardioversion is the appropriate treatment when monomorphic ventricular tachycardia occurs with hypotension.

200mL. 420mL. 600mL. 780mL. 1170mL.

ANSWER:

C, E, A

B

Monomorphic ventricular ectopy is characterized by the appearance of premature ventricular contractions (PVCs) that have the same morphology and are, therefore, called monomorphic. Single PVCs are not concerning. Couplets (2 sequential PVCs) are more concerning. Monomorphic ventricular ectopy often does not need to be treated. When required, the treatment of choice is amiodarone.

The goals of mechanical ventilation in the case presented are (1) to assist the patient in obtaining the necessary oxygenation and oxygen delivery, (2) to maintain the maximum alveolar surface area and possibly recruit atelectatic alveoli, (3) to minimize the risk of ventilator-induced lung injury, and (4) to allow the patient to provide the work of breathing appropriate under the clinical circumstances to maintain respiratory muscle tone and conditioning.

Narrow complex tachycardia is caused by an electrical impulse being initiated above the atrioventricular node and

109

the impulse traveling down the Purkinje fibers. The treatment of this arrhythmia is to block the AV node with a calcium channel blocker medication.

C. Complicated sepsis D. Severe sepsis E. Septic shock

Torsade de pointe is most commonly precipitated with a drug that prolongs the Q-T interval, such as methadone, haloperidol, and levofloxacin. Treatment is targeted at removing the offending agent and administering magnesium.

ANSWER:

Sepsis is a complex infection syndrome marked by aberrant host response and organ dysfunction. Sepsis is highly prevalent and accounts for approximately 5% of total US hospital costs. The European Society of Intensive Care Medicine and the Society

Bradycardia is treated with atropine, an acetylcholine receptor antagonist that blocks the parasympathetic activity.

24. Regarding the use of a 1:1:1 massive transfusion protocol (MTP), which of the following statements is true?

of Critical Care Medicine recently revised their consensus definitions and criteria to identify patients with sepsis. The new operational definitions emphasize organ dysfunction as a part of sepsis. The systemic inflammatory response syndrome is not specific to infection and may be adaptive in hospitalized patients. Sepsis is now defined as a sequential organ failure assessment (SOFA) score of at least 2 in the setting of suspected of infection. The quick SOFA (qSOFA) score is simpler to use and can be used at the bedside instead. qSOFA assigns 1 point each for altered mentation, respiratory rate of at least 22/min, and systolic blood pressure of 100 mm Hg or less (table 25.1). Septic shock is identified in patients with persistent hypotension requiring a pressor to maintain mean arterial pressure of at least 65 mm Hg and having a serum lactate level of at least 2 mmol/L (0.5-1 mmol/L), despite adequate volume resuscitation. With the new definitions, the terms severe and complicated sepsis are no longer needed.

A. MTP increases the overall use of blood products. B. MTP decreases 30-day mortality. C. A transfusion plan of 1:1:1 means 6 units of packed red blood cells: a 6 pack of platelets:6 units fresh frozen plasma. D. Since the implementation of MTP triggers, MTPs are used more frequently. E. A hemoglobin of 10 g/dL is one of the transfusion goals.

ANSWER:

B

C

Using early balanced resuscitation for trauma patients provides documented benefits. Early balanced resuscitation means giving equal amounts of platelets, packed red blood cells, and fresh frozen plasma to trauma patients who require transfusion. This means a ratio of 1:1:1, which means one 6 pack of platelets for every 6 units of packed red blood cells and 6 units of fresh frozen plasma. In prospective trials, multicenter trials, and meta-analyses, the ratio shows improved 24-hour survival and decreased death from exsanguination and truncal hemorrhage. Critics have countered that 30-day survival is not improved. The use of early balanced resuscitation does not increase the overall use of blood products or the implementation of a massive transfusion protocol. In other words, it does not waste more blood products. Massive

qSOFA (Quick SOFA) Criteria Points Respiratory rate ≥ 22 breaths/minute 1 Change in mental status 1 Systolic blood pressure ≤ 100 mm Hg 1 Table 25.1. Quick sequential organ failure assessment (SOFA) score.

transfusion protocol protocols do not aim for a specific hemoglobin goal. 25. A 50-year-old man presents with complicated diverticulitis. He undergoes percutaneous drainage of a pelvic abscess. His vital signs include a temperature of 38°C, blood pressure of 90/55 mm Hg, mean arterial pressure of 65 mm Hg, and respiratory rate of 24 breaths per minute. On exam, he is diaphoretic, confused, and does not follow commands. What is the most appropriate intensive care unit diagnosis? A. Systemic inflammatory response syndrome B. Sepsis

110

Trauma Part I

A grade V injury involves massive disruption of the duodenopancreatic complex or duodenal devascularization. In a retrospective review at one institution, children with grade II, III, and IV injuries were successfully managed with primary repair, and children with grade II, III and IV injuries repaired primarily did as well as or better than children with similar injuries managed by pyloric exclusion or gastrojejunostomy.

ITEMS 1-28 For each question, select the best possible response. 1. A 7-year old girl is admitted after she was a restrained passenger in a T-bone motor vehicle crash with impact on her side of the car. She is tachycardic and has an acute abdomen. At surgery, she has an isolated duodenal perforation in the second portion opposite the ampulla of Vater involving 25% of the circumference. What is the best operative plan?

Duodenal diverticulization refers to suture closure of the duodenal injury, antrectomy with end-to-side gastrojejunostomy, and tube duodenostomy. It is a complex, time-consuming procedure that is generally unnecessary. The triple tube technique of drainage is unnecessary in a grade II injury.

A. Primary repair of the duodenum B. Pyloric exclusion C. Repair the duodenum, gastrostomy, jejunostomy, and duodenostomy D. Duodenal diverticularization E. Gastrojejunostomy, nasogastric decompression, nasojejunal feeding tube

ANSWER:

2. A 22-year-old man is involved in a motorcycle crash. His injuries include a left pulmonary contusion, a comminuted open left femur fracture, and a mild closed head injury (Glasgow Coma Scale score = 12). He was placed in skeletal traction, started on antibiotics, and provided supplemental oxygen by nasal cannula while awaiting open reduction and internal fixation of his femur fracture. At 36-hours postinjury, his confusion has increased, his pulse oximeter indicates 88% saturation, and he has developed the skin lesions shown (figure 2.1). The most likely cause for these findings is

A

Blunt injuries to the duodenum are uncommon in all ages. They account for approximately 3 to 5% of all intraabdominal injuries. Blowout injuries after blunt trauma can occur due to the retroperitoneal fixed nature of the duodenum and compression against the vertebral column. Five grades of duodenal injury were described in 1990 using a standardized organ injury scale (table 1). Grade*

Type of Injury

Description

I

Hematoma

Involving single portion of the duodenum

Laceration

Partial thickness—no perforation

Hematoma

Involving more than one portion of the duodenum Disruption by <50% of circumference

II

Laceration III

Laceration

IV

Laceration

V

Laceration

Vascular

A. B. C. D. E.

a drug reaction. fat embolism syndrome. drug-induced thrombocytopenia. sequela from pulmonary contusion. progression of his traumatic brain injury.

Disruption by 50-75% of circumference of D2 Disruption by 75-100% of circumference of D1/D3/D4 Disruption by >75% of circumference Involving ampulla or distal common bile duct Massive destruction of duodenopancreatic complex

Devascularization of duodenum

*Advance one grade for multiple injuries up to grade III. D1, first portion of duodenum; D2, second portion of duodenum; D3, third portion of duodenum; D4, fourth portion of duodenum. Table 1.1 Duodenal Injury Scale (AAST-OIS).

Figure 2.1.

A grade I injury is a hematoma or laceration without perforation; a grade II laceration equates to a disruption of less than 50% of the duodenal circumference; grade III 50 to 75% circumference of D2 or 50 to 100% of D1, D3, or D4; and grade IV disruption more than 75% circumference of D2 involving the ampulla or distal common duct.

ANSWER:

B

Fat embolism syndrome (FES) was described more than 150 years ago by Zenker, but the precise etiology remains elusive. Classic features of FES include young adults (10-40 years of age), associated long-bone or pelvic fractures, male sex,

112

concomitant pulmonary injury, and delayed fracture stabilization. FES is also reported after spinal surgery, vertebroplasty, and intramedullary fixation. The reported incidence varies between 0.2% and 35%, depending on the series. The clinical features are nonspecific and generally indistinguishable from acute respiratory distress syndrome (ARDS). These include hypoxia, respiratory distress (varying from mild to severe enough to warrant mechanical ventilatory support), mental status changes, fever, tachycardia, tachypnea, retinal splinter hemorrhages, oliguria/anuria, and occasionally jaundice. One of the classic, but inconsistent, findings in FES is a petechial rash over the anterior thorax and axilla. The rash may be transient, lasting only 24-hours or less (figure 2.1). This rash differentiates FES from adverse drug reactions, which tend to be more generalized. Druginduced thrombocytopenia does not characteristically produce a skin rash (although thrombocytopenia is reported in association with FES), nor does progression of a pulmonary or traumatic brain injury

30-second loss of consciousness. He has a patent airway without difficulty breathing, and his vital signs are normal. He opens his eyes to speech and obeys commands, but he is confused and perseverating. No other injuries are noted on secondary survey. What is the most appropriate next step in his management? A. B. C. D. E.

Discharge home with instructions Observation with re-evaluation in 4 hours CT scan of the head Admission overnight Neurosurgical consultation

ANSWER:

B

Approximately 1.7 million Americans present to the emergency department annually after sustaining a traumatic brain injury (TBI). Of this number, approximately 80% are treated and released without the need for hospitalization. Among adults, falls and motor vehicle crashes are the most common causes for TBI. In the 15- to 24-year-old age group, however, sports-related injuries are second only to motor vehicle crashes as the causative etiology.

Two theories are proposed to explain the pathophysiology. The mechanical theory proposes that fat globules forced into the systemic circulation from bone marrow obstruct pulmonary capillaries, resulting in an increased shunt and hypoxia. The same mechanism is also operative in the brain and kidney, explaining the changes in mental status and renal dysfunction. Evidence to support this hypothesis comes from the identification of fat globules observed in serum, sputum, and urine and seen on duplex scans and transesophageal echocardiography. Cutaneous capillary obstruction produces the skin rash. The biochemical hypothesis suggests that fat globules are broken down to free fatty acids by lipases, which trigger activation of the inflammatory cascade. This results in pulmonary alveoli and endothelial injury, identical to that seen in ARDS. Inflammatory activation also affects other organs, explaining the central nervous system, renal, and hepatic dysfunction. Evidence to support this hypothesis is based on serum elevations of inflammatory mediators. It is likely that both hypotheses may be operative.

Closed head injuries are the most common form of TBI, and the resultant neurologic damage arises from primary and secondary processes. Primary injury occurs from the direct transmission of force to the neurons and their long axons, creating shear forces from differences in acceleration along their paths. Secondary injury results from the subsequent sequelae of the force transmission: hypoxia; hypotension; hydrocephalus; and intracranial hypertension, thrombosis, or hemorrhage. Concussions, contusions, and diffuse axonal injury are all examples of closed head injuries. For TBIs, the Glasgow Coma Scale (GCS) provides a method of stratifying severity. Determining GCS score is an integral part of the primary survey for Advanced Trauma Life Support (ATLS), assessed during the disability (i.e., D) phase. It assigns points to 3 neurologic responses: motor, verbal, and eye-opening (table 3.1). The cumulative score determines TBI severity: mild (13-15 points), moderate (9-12 points), and severe (3-8 points).

Treatment is supportive, and FES is usually self-limited. Severe cases may require ventilator support. Mortality from FES by itself is quite low due to advances in critical care and ventilatory support (<5% compared with the >30% reported several decades ago), and death is usually attributable to other associated injuries. Patients with central nervous system manifestations of FES do not appear to have a worse prognosis.

Mild severity closed head injuries are concussions that are defined by the American Academy of Neurology as a "trauma-induced alteration in mental status that may or may not result in loss of consciousness." They can be graded based on the degree of mental status change: grade 1 = confusion only; grade 2 = associated amnesia; grade 3 = associated loss of consciousness.

Early fracture stabilization is generally accepted as the most effective means of prevention. Pharmacotherapy to prevent and treat FES is disappointing, but some limited data show that prophylactic steroids may reduce FES and hypoxia, but not mortality.

Approximately 3.8 million sports-related concussions occur annually in the United States. For patients presenting to the emergency department, the workup and treatment algorithm consider the degree of initial injury as well as neurological status over time. Patients presenting with grade 1 concussions, grade 2 concussions with perievent amnesia,

3. A 17-year-old high school football player is brought to the emergency department after a head-to-head collision with a

113

and grade 3 concussions with less than 1-minute loss of consciousness may be observed in the emergency department for several hours. If patients' neurologic exam remains normal without worrisome symptoms after this period, they may be discharged home with comprehensive instructions and a reliable individual to observe them for the first day after the injury. All other patients sustaining grade 2 or 3 concussions, as well as those patients with progressive headache, vomiting, skull/facial fractures, seizure, abnormal neurologic exam, deteriorating neurologic status, alcohol/drug intoxication, coagulopathy (including anticoagulant therapy), and age older than 60 should have CT imaging to rule out intracranial injury. If the CT scan is normal and the neurologic status returns to baseline (save amnesia of the event) after observation, the patient may be discharged home under the care of a responsible individual. Otherwise, 24-hour observation in the hospital is indicated. MOTOR RESPONSE

VERBAL RESPONSE

EYE-OPENING RESPONSE

Obeys commands

6

Oriented

5

Opens spontaneously

4

Localizes to pain

5

Confused

4

Opens to speech

3

Withdraws from pain

4

Inappropriate words

3

Opens to pain

2

Flexor posturing

3

Unintelligible sounds

2

No eye opening

1

Extensor posturing

2

No sounds

1

No movement

1

"Add the 3 scores to obtain the Glasgow Coma Scale (GCS) score, which can range from 3 to 15. Add “T” after the GCS if intubated and no verbal score is possible. For these patients, the GCS can range from 3T to 10T. Table 3.1 The Glasgow Coma Scale score.

4. A 26-year-old man presents to the trauma bay after his left thigh was crushed between a wall and car. At the scene, he was bleeding copiously from a puncture wound in his thigh. He has a patent airway, is breathing normally, and is normotensive but tachycardic. His left thigh is visibly deformed with a nonbleeding puncture wound with visible bone. A pulsatile hematoma is medial to the puncture wound. He cannot dorsiflex his foot, and he has a diminished dorsalis pedis pulse. Of the findings in the trauma bay, which mandates exploration to evaluate and potentially treat a vascular injury? A. B. C. D. E.

A key component of the management of extremity trauma is determining whether a coexistent vascular injury requiring operative repair is present. In certain situations, signs on examination indicate the definite presence of a vascular injury, mandating immediate exploration of the area of injury in the operating room. Such hard signs indude active hemorrhage from the injury, absent pulse distal to the injury, the presence of a bruit or thrill near the site of injury, and an expanding or pulsatile hematoma associated with the injury. In other situations, the signs on examination are more subtle ("soft signs"), warranting further diagnostic workup and imaging, but do not mandate operative exploration. These soft signs include a large amount of reported blood loss at the scene of the injury, severe orthopedic injuries, concomitant nerve injury, subjective decrease in pulse distal to the injury, and presence of a large nonpulsatile hematoma associated with the injury. This patient has a hard sign of a vascular injury (a pulsatile hematoma) that mandates operative exploration.

Copious blood loss at scene Compound femur fracture Pulsatile hematoma Neurologic deficit Diminished pulse

ANSWER:

C

114

5. A 17-year-old is involved in a high-mechanism motor vehicle collision, sustaining a grade IV liver injury. Trace pneumomediastinum located anterior to the esophagus is noted on CT scan (figure 5.1). The presence of pneumomediastinum after blunt trauma

C. resuscitative endovascular balloon placement. D. placement of a left tube thoracostomy. E. 1 mg of epinephrine.

A. B. C. D. E.

ANSWER:

requires further evaluation with triple endoscopy. signifies an esophageal injury in 10% of cases. is identified in 20% of blunt trauma patients. is often a benign finding. correlates with the extent of an associated pneumothorax.

Witnessed traumatic arrest should prompt the surgeon to perform a resuscitative left anterolateral thoracotomy. Thoracotomy permits evaluation and treatment of intrathoracic injuries, which in this case is likely to be cardiac tamponade from a penetrating cardiac injury. The pericardium is incised from the apex toward the great vessels, anterior to the left phrenic nerve. The heart is then delivered through this anterior opening in the pericardium and the cardiac injury delineated. If the patient has intrinsic cardiac activity, digital occlusion of the injury followed by repair in the operating room is performed. If the heart is asystolic, the cardiac injury is repaired and then injection of epinephrine and defibrillation is performed. During pericardotomy, the right hemithorax may be decompressed by making an aperture through the pericardium. This action will relieve a right-sided tension pneumothorax if present in this patient with right-sided penetrating trauma.

Figure 5.1.

ANSWER:

B

Although focused assessment with sonography for trauma (FAST) exam of the pericardium may reveal hemopericardium, this is evident only if the patient's heart is beating. Without intrinsic cardiac activity, it may be difficult to identify the fluid stripe around the heart distinct from the cardiac chambers. Delaying intervention to perform a FAST exam is not warranted.

D

Pneumomediastinum is identified in only 2 to 10% of blunt trauma patients and is often a benign finding. In a recent study, only 1% of patients with identified pneumomediastinum had an esophageal injury. In most cases, patients have associated chest trauma, most commonly a pneumothorax; pneumomediastinum in these cases is caused by air dissecting along the pulmonary vasculature from injured alveoli, termed the Macklin effect. There does not appear to be a correlation between the extent on an associated pneumothorax and the volume of a pneumomediastinum. CT scan can be used as a screening tool. The identification of air located in the posterior mediastinum or throughout all mediastinal compartments (anterior, superior, and posterior) is the most concerning finding and should prompt additional contrasted imaging/endoscopy to rule out an esophageal injury.

Resuscitative endovascular balloon occlusion of the aorta (REBOA) placement is being used in some centers for patients with blunt traumatic arrest; however, REBOA will neither diagnose nor treat cardiac tamponade. In addition to performing the left thoracotomy, a concurrently placed right tube thoracostomy will decompress any associated tension pneumothorax or hemothorax. A left tube thoracostomy is not needed in a patient better suited to a thoracotomy.

7. A 26-year-old man sustains a stab wound to the abdomen. On abdominal exploration, an injury to the second portion of the duodenum with 25% loss of the lateral wall is discovered. What is the most appropriate next step? A. B. C. D. E.

6. A patient presents with a 4-cm stab wound to the chest located 2 cm to the right of the sternum. During transport, his blood pressure is 100/55 mm Hg, his heart rate is 100 beats/minute, and his respiratory rate is 28 breaths/minute. On arrival to the emergency department, he loses pulses. The most appropriate next step in this patient's management is A. focused assessment with sonography for trauma of the pericardium. B. left anterolateral thoracotomy.

115

Lateral tube duodenostomy Primary repair of the duodenum Duodenal resection and primary anastomosis Duodenojejunostomy Pyloric exclusion and gastrojejunostomy

ANSWER:

if the injury can be fully evaluated and closed. Historically, presacral drain placement and distal rectal washout was performed for all patients with extraperitoneal rectal injuries. These measures, which involve dissection of normal tissue planes and liquefication of the stool burden, increase morbidity and hence are not routinely advocated.

B

Most duodenal injuries can be primarily repaired. This is a grade II injury, with disruption of less than 50% of the circumference of the wall (table 1.1). Primary repair after debriding any devitalized tissue (commonly needed for gunshot injuries but less common for stab wounds) should be performed. Removing the uninjured wall with resection and primary anastomosis is not needed. More complex reconstructions, such as a duodenojejunostomy or resection with duodenoduodenostomy, are necessary only for grade HI injuries (disruption of 50-75% of D2 and 50-100% of D1, D3, or D4) or higher injuries. Destructive injuries to the duodenopancreatic complex often require pancreaticoduodenectomy.

9. A 26-year-old man is involved in a motorcycle collision. On imaging, a pelvic fracture and bladder injury are identified (figure 9.1). What is the most appropriate next step to manage his bladder injury? A. B. C. D. E.

Lateral tube duodenostomy may be helpful in patients who leak after a duodenal repair breaks down but should not be used at the initial surgery. Pyloric exclusion with associated gastrojejunostomy is typically used in patients requiring duodenal repairs who also have an associated pancreatic injury. In these situations, the associated pancreatic injury may cause breakdown of the duodenal repair (the sutures may dissolve from the pancreatic fluid). The diversion of the gastric contents permits adequate drainage of the area without development of a lateral duodenal fistula that is unlikely to heal.

8. A 31-year-old man presents after a gunshot wound to the right buttock. Exam under anesthesia identifies blood in the rectum at 8 to 10 cm from the dentate line concerning for an extraperitoneal rectal injury. In addition to confirming, an extraperitoneal rectal injury, what is the most appropriate next step in this patient's management? A. B. C. D. E.

Figure 9.1.

ANSWER:

D

This patient has an intraperitoneal bladder injury (figure 9.2) that should be managed with operative repair, performed via either an open or laparoscopic approach.

Direct repair Direct repair with proximal diversion Proximal diversion Proximal diversion and presacral drain placement Proximal diversion, presacral drain placement, and distal rectal washout

ANSWER:

Suprapubic tube placement Cystoscopy Foley catheter treatment for 2 weeks Operative repair Retrograde urethrogram

C

This extraperitoneal rectal injury is located quite high, hence making primary repair challenging, if not impossible. In such cases, proximal diversion with a loop colostomy is the safest management option. After proctoscopy, diversion can be accomplished with either a laparoscopic or open approach based on the patients prior surgical history and current physiology. A loop colostomy effectively diverts the fecal stream, allowing the injury to heal, while optimizing operative reversal for this temporary stoma (compared with using an end-colostomy/Hartmann, which is more difficult to reverse). If an extraperitoneal rectal injury is located much closer to the anal opening, primary repair may be considered

Figure 9.2. Extravasation of contrast in an intraperitoneal location

Laparoscopic repairs, particularly in patients without other intra-abdominal injuries, are advocated. Suprapubic tubes are typically placed for patients with urethral injuries in whom a Foley catheter cannot be passed. Cystoscopy is not indicated because the diagnosis is already confirmed on CT imaging. Extraperitoneal bladder injuries are managed with catheter drainage alone (typically Foley, rarely suprapubic tube), and

116

more than 85% will heal within 14 days. A retrograde urethrogram is used to diagnose a urethral injury; it is not indicated at this time because a Foley catheter has already been successfully placed for the CT cystogram.

A. B. C. D. E.

10. A 25-year-old man sustains a left zone II neck stab wound. He is hemodynamically normal without hard signs of an aerodigestive or vascular injury. He has dysphagia. What is the most appropriate next step in management? A. B. C. D. E.

ANSWER:

A

Patients with a traumatic brain injury and intracranial hemorrhage are at high risk of developing a venous thromboembolism (VTE; up to 25%). A traumatic brain injury is an independent risk factor for VTE. This risk is amplified by mechanical ventilation and other associated injuries. Chemical prophylaxis decreases the incidence of VTE in multiply injured patients, and guidelines recommend the use of low molecular weight heparin (LMWH) over unfractionated heparin (UFH). Concern was raised about worsening of intracranial hemorrhage with the use of chemical prophylaxis, but several retrospective studies and systematic reviews have demonstrated its safety. Early chemical prophylaxis, within 24 to 48 hours after injury, did not increase the rate of hemorrhage progression.

CT scan of the neck Esophagram Flexible esophagoscopy Angiography Neck exploration

ANSWER:

Low molecular weight heparin Aspirin 81 mg Unfractionated heparin Screening duplex imaging Retrievable inferior vena cava filter

A

The management of zone II penetrating neck injuries has evolved from mandatory neck exploration for all injuries to selective management of most injuries. CT angiogram (CTA) of the neck has emerged as the diagnostic study of choice in patients who have sustained penetrating trauma to the neck who require further workup to evaluate potential injuries. CTA is rapid to obtain, noninvasive, and an excellent screening tool for penetrating neck wounds.

Current recommendations are for sequential compression devices for the initial 24 to 48 hours after presentation, followed by chemical prophylaxis. LMWH and UFH have not been directly compared in this patient group, but both drugs have been studied. Either drug is acceptable for chemical prophylaxis.

CTA of the neck is not sensitive enough to completely rule out esophageal injury; however, it is preferred as the initial diagnostic study because it has the ability to further evaluate the potential for tracheal, esophageal, and vascular injury. When the CTA of the neck is suggestive of esophageal injury, the patient requires additional testing, especially in the setting of dysphagia or odynophagia. Further workup should include contrast esophagram or flexible esophagoscopy. The combination of both procedures yields a nearly 100% sensitivity for diagnosing esophageal injuries. Water-soluble contrast (Gastrografin) is used first with contrast esophagraphy. If this is not diagnostic, Gastrografin is followed by thin barium.

Mechanical prophylaxis with sequential compression devices (intermittent pneumatic compression) can be used in patients with contraindications for chemical prophylaxis. Compared with no prophylaxis, mechanical devices decrease the rate of VTE. However, chemical prophylaxis provides superior VTE prevention over sequential compression devices or intermittent pneumatic compression. Aspirin is used in orthopedic patients as a single agent but is currently not recommended for nonorthopedic surgery patients. Retrievable inferior vena cava filters are an attractive nonpharmacologic method to decrease VTE risk. In several nonrandomized trials, the short-term pulmonary embolus rate was decreased but the deep vein thrombosis rate was increased. Insertion complications, inferior vena cava occlusions, and filter migrations were also reported. Given these complications, inferior vena cava filter placement is not recommended over chemical prophylaxis.

Neck exploration may be indicated if an injury requires surgery; however, it is not the initial step in management of this patient. This is because the patient does not have signs of injury mandating operative intervention.

11. A 30-year-old unhelmeted man was involved in a motorcycle crash. His initial Glasgow Coma Scale (GCS) score was 13. A CT scan of his head showed a parietal skull fracture and 3-mm subdural hematoma. He was admitted to the intensive care unit for observation. Repeat head CT scan the next day shows no change in the subdural hematoma, and his GCS is now 15. In addition to sequential compression devices, what is the best way to decrease his risk of venous thromboembolism?

A surveillance protocol with venous compression ultrasonography was used in trauma patients with contraindications for chemical or mechanical prophylaxis. However, the rate of pulmonary embolism was unchanged, and false positives were reported. Routine surveillance venous compression ultrasonography is not recommended as a method of primary prophylaxis.

117

12. A 45-year-old man is the restrained driver involved in a 2-vehicle, high-impact motor vehicle collision. He presents to the trauma bay hemodynamically normal with an abdominal wall seatbelt sign. CT scan of the abdomen shows a contained retrohepatic vena cava injury as the only abnormality. The patient remains hemodynamically normal. Labs obtained return a hemoglobin of 13 g/dL (13.5-17 5 g/dL) and show the patient is not acidotic. What is the most appropriate next step in management? A. B. C. D. E.

A. B. C. D. E.

observation. angioembolization. splenorrhaphy. splenectomy. distal pancreatectomy and splenectomy.

Admission with observation Damage-control laparotomy with perihepatic packing Primary repair of the injury Endovascular stent Atriocaval shunt

ANSWER:

A

Retrohepatic vena caval injuries are some of the most feared injuries treated by even the most seasoned trauma surgeons. Because the retrohepatic vena cava is surrounded by suspensory ligaments of the liver and the diaphragm, operative mobilization of these structures can result in torrential hemorrhage and death. A surgeon may also feel compelled to surgically repair such injuries even when contained, only to realize after mobilizing the liver that bleeding will more than likely result in the death of the patient.

Figure 13.1.

ANSWER:

D

Causes of traumatic injuries in pregnancy are similar to those in the general population. Blunt trauma is the most common cause: 49% motor vehicle crashes, 25% falls, 18% assaults, 4% penetrating wounds, and 1% bums. The principle of managing a pregnant patient after injury is to focus on the patient, which yields the best outcomes for both mother and baby.

In this hemodynamically normal patient, without evidence of free rupture and ongoing bleeding, the normal anatomy may contain the bleeding such that tamponade results, and surgical intervention is not necessary. Admission with close observation in the trauma intensive care unit is the correct choice in this patient. Damage-control laparotomy with perihepatic packing, primary repair of the injury, endovascular stenting, and atriocaval shunt placement are techniques that may be used if this patient requires operative intervention for uncontrolled bleeding and hemodynamic abnormality.

Splenic injury management includes observation, angioembolization, splenorrhaphy, and splenectomy. A hemodynamically normal patient with a splenic injury is usually managed nonoperatively with a high success rate. At the other end of the spectrum is a patient who presents in hypovolemic shock that is not responsive to resuscitation. Admission hypotension is a strong predictor of the need for splenectomy, and most of these patients will require splenectomy. A distal pancreatectomy is rarely required unless a pancreatic injury is also present.

13. A 21-year-old unrestrained woman was involved in a high-speed motor vehicle collision. She is 16 weeks pregnant. She is awake and alert, complaining of left-sided chest pain and left upper quadrant abdominal pain. Her only external injuries are right lower extremity abrasions from the knee to the foot and a left knee abrasion. Her blood pressure is reported as 70 mm Hg systolic by prehospital providers, and she was given 1000 mL of lactated Ringer solution in transit. Her first emergency department vital signs include a blood pressure of 98/60 mm Hg and a pulse of 88 beats per minute. She is taken to the CT scanner, where her blood drops pressure transiently to 70 mm Hg systolic. She responds to fluid, and a unit of blood is hung, but her heart rate remains at 110 beats per minute. Her CT scan is shown (figures 13.1). The next step in her management is

Management controversy surrounds the patient who is hemodynamically abnormal and responds at least transiently to resuscitation with a known splenic injury. Angioembolization is suggested as a management option in this scenario, especially if a contrast blush is present on CT scan (figure 13.2).

118

A splenectomy is recommended because these patients are not candidates for nonoperative management with or without angioembolization. The importance of observation is stressed in reports trying to establish a role for angioembolization for splenic injuries. In this patient, who has just completed her first trimester, the goal of treatment should focus on establishing normal hemodynamics as soon as possible. She has a grade V injury but is not responding to resuscitation. Early splenectomy would quickly and reliably restore normal hemodynamics. Angioembolization exposes the patient to the risks of radiation to the fetus, complications of the intervention, and recurrent bleeding. Given the appearance of the spleen on CT scan, splenorrhaphy is not likely to be possible and would increase her rate of recurrent bleeding to a higher incidence than a splenectomy (1% vs 3%).

Figure 13.2. Grade V splenic injury with multiple contrast 14. A 20-year-old man is admitted to the trauma bay in class IV hemorrhagic shock. A tibial intraosseous (IO) device is placed for venous access. Which of the following crystalloid infusion methods is preferred?

blushes. Angioembolization improves spleen salvage rates for grade IV and V injuries compared with nonoperative management (table 13.3). Whether these spleens function normally after embolization remains unclear. However, in an outcome study, the rate of splenectomy among patients with a splenic blush on CT scan was not statistically different among those patients who had angioembolization versus those patients who did not. It is clear that proper patient selection remains the key element for appropriate use of angioembolization in patients with blunt splenic injury. Grade *

I

II

III

A. B. C. D. E.

35°C fluid by gravity 42°C fluid by pressure infusion pump 42°C fluid by gravity 24°C fluid by pressure bag 45°C fluid by manual syringe

Type-of Injury Hematoma

Description

ANSWER:

Subcapsular, <10% surface area

Laceration

Capsular tear, <1 cm parenchymal depth

Hematoma

Subcapsular, 10-50% surface area, intraparenchymal, <5 cm in diameter

Laceration

1-3 cm parenchymal depth, which does not involve a trabecular vessel

Hematoma

Subcapsular, >50% surface area or expanding, ruptured subcapsular or parenchymal hematoma >3 cm parenchymal depth involving trabecular vessels

Numerous sites can be used for intraosseous (IO) access: anterior tibia, proximal humerus, distal femur, and sternum. The anterior tibial location is preferred in children. The proximal humerus should be used only in mature adolescents and adults. Humeral access is often harder to secure, and dislodgement may be a problem if the patient is moved. Care should be taken when placing an IO device to make sure that it is the correct device for the location.

Laceration IV

Laceration

V

Laceration

Involving segmental or hilar vessels producing major devascularization (>25% of spleen) Completely shattered spleen

Vascular

Hilar vascular injury, which devascularizes spleen * Advance one grade for multiple injuries up to grade III.

Table 13.3. Splenic injury scale. The alternative is splenectomy, which can be performed in most institutions without delay with a predictable outcome. The morbidity is higher with splenectomy than successful nonoperative management, with or without angioembolization. The ideal candidate is a patient with a splenic injury who remains hypotensive despite resuscitation.

B

Crystalloid fluids (e.g., normal saline) for rapid volume expansion and viscous drugs and solutions should be administered under pressure. The pressure is needed to overcome the resistance in the emissary veins, which lead from the medullary cavity to the general circulation. The pressure device used can be an infusion pump, pressure bag, or manual injection through a syringe and stopcock. The latter obviously requires a specific individual. While using pressure infusion devices, constant evaluation for extravasation is necessary. Extravasation can be detected by swelling around the infusion site or by higher pressures being needed for continued infusion. The use of warmed crystalloid infusions is imperative for patients in shock. Hypothermia impairs coagulation, causes vasoconstriction, and promotes acidosis. The lethal triad— hypothermia, acidosis, and coagulopathy—is associated with

119

hypovolemic shock and commonly leads to aborting operative therapy in favor of a damage control approach. Warming all fluid used for resuscitation is imperative. Pressure infusers not only help when an IO is being used for access but also can warm the fluid, even at high flow rates. Most fluid warmers use a set point of 42°C, which is the same temperature for blood warming. Although the fluid can be warmed to 42°C, it will lose some heat in the tubing and usually enters the body just above normal body temperature. Thus, at very high flow rates with open body cavities, heat loss may exceed the heat transfer achieved by the fluid. Temperatures higher than 42°C carry risks of thermal injury.

Esophagoduodenoscopy is not required, although it might be used to help pass a nasoenteric feeding tube in a patient who is not totally obstructed but cannot tolerate feedings by mouth. Radiologic placement of a nasoenteric tube is another possibility. Enteral feeding is preferred over parenteral, but a feeding jejunostomy is not preferred early in management because most patients will not require any operative procedure. Exploratory laparotomy is not indicated without a trial of nonoperative management. Most duodenal hematomas will resolve in 1 to 3 weeks without surgical intervention. If duodenal obstruction persists, then surgical intervention is appropriate. Evacuation of the hematoma is usually not possible, and neither a duodenoduodenostomy nor a jejunoduodenostomy effectively treats the obstruction.

In this patient with an IO access and class IV shock, a pressure infuser should be used for all fluid administration.

16. A 24-year-old man is involved in a motor vehicle crash. He is intubated at the scene for a Glasgow Coma Scale score of 7, and 2 large bore intravenous lines are placed. On arrival to the emergency department, his pupils are equal and reactive, he has a temperature of 37.8°C, heart rate of 105 beats per minute, blood pressure of 100/60 mm Hg, and SaO2 of 86%, with an FiO2 of 0.6. On arterial blood gas, his pH is 7.45, PaCO2 is 35 mm Hg, pO2 is 57 mm Hg. Point of care glucose is 185 mg/dL (70-100 mg/dL). What is the next intervention for this patient?

15. A 23-year-old helmeted motorcyclist is injured after colliding with a stationary car. He is hemodynamically normal with a normal focused assessment with sonography for trauma examination. His chest radiograph shows a small pulmonary contusion and fractures of right ribs 9,10, and 11. He is admitted for pulmonary care. On the second day of hospitalization, the patient develops epigastric fullness and pain, which is relieved after he vomits bilious material several times. His white blood cell count and hematocrit are normal. A CT scan demonstrates a distended stomach with thickening of the second portion of the wall of the duodenum. What is the most appropriate next Step in this patient's management? A. B. C. D. E.

A. B. C. D. E.

Exploratory laparotomy Gastric decompression Feeding jejunostomy Esophagogastroduodenoscopy Parenteral nutrition

ANSWER:

Insulin infusion Hyperventilation to PaCO2 of 25 mm Hg Hypertonic saline Mannitol Increase FiO2

ANSWER:

E

Trauma is the leading cause of death in patients younger than 45 years. Traumatic brain injury (TBI) is a leading cause of death. Although many interventions can be undertaken to prevent TBI, once a TBI occurs, it is up to the surgeon caring for the patient to prevent secondary injury.

B

Blunt duodenal injury is uncommon. When it does occur, duodenal hematomas can obstruct the bowel lumen. Often seen in children, they can also occur in adults after a blow to the abdomen. Management is usually nonoperative because the hematoma typically resolves. Most patients will have signs and symptoms of a bowel obstruction that may present immediately after injury or may be delayed. CT scanning of the abdomen in patients after blunt abdominal injury will often identify duodenal abnormalities consistent with a duodenal hematoma, including duodenal wall thickening and paraduodenal edema. Perforation can be excluded when there is no extraluminal air in the pararenal spaces around the duodenum.

In 2007, the IMPACT study evaluated the risk of secondary TBI. In this study, there was a clear link between patient outcome and the absence of hypoxemia (defined as PaO2 <60 mm Hg) and hypotension (systolic blood pressure of <90 mm Hg), the presence of hypoxemia alone, the presence of hypotension alone, and the presence of both. In the scenario presented, the patient has a severe TBI with a Glasgow Coma Scale score of 7. He is normothermic and has asystolic pressure of 90 mm Hg. He is hypoxemic with a PaO2 of 57. Thus, to prevent secondary brain injury, the next step would be to increase his FiO2.

Treatment should begin with gastric decompression. After gastric decompression, a contrast study is done to identify the degree of duodenal obstruction. Parenteral nutrition is required if complete obstruction is present. Enteral nutrition can be attempted if duodenal obstruction is not complete.

Another cause for poor outcome in TBI is elevated intracranial pressure. Elevated intracranial pressure can be manifested by signs on physical exam (dilated pupils) or on direct measurement of intracranial pressure. In the absence of

120

these findings, hyperventilation, hypertonic saline infusion, or mannitol would not be indicated. There is no role for the acute infusion of insulin because his elevated glucose is a response to injury.

2. 3.

A Cochrane Database Systematic Review of antifibrinolytic drugs for acute traumatic injury included all randomized controlled trials of aprotinin and TXA. The authors concluded that the aprotinin trials did not provide sufficient data regarding mortality. TXA reduced mortality by 10%. Another Cochrane Database Systematic Review of antifibrinolytic use for minimizing perioperative blood transfusion included trials assessing TXA, aprotinin, and epsilon aminocaproic acid. The use of these agents was supported for minimizing perioperative allogenic blood transfusion in adult surgical patients. Mortality in trauma patients was not assessed in this review.

17. A 20-year-old otherwise healthy man sustains a gunshot wound to the abdomen. He is transported to the emergency department immediately after the shooting. On arrival, he is lethargic with a systolic blood pressure of 70 mm Hg, a heart rate of 140 beats per minute, and a distended abdomen. Massive transfusion protocol is initiated, and the patient is transported to the operating room for exploratory laparotomy. In addition to transfusion of blood products and surgical control of bleeding, what intervention provides a mortality benefit for this patient? A. B. C. D. E.

Recombinant Factor VIIa Desmopressin acetate Epsilon aminocaproic acid Aprotinin Tranexamic acid

ANSWER:

Although recombinant Factor Vila reduces blood product use, mortality is not affected. Desmopressin acetate does not improve mortality in acute trauma.

18. Which of the following injury patterns is the best indication for resuscitative endovascular balloon occlusion of the aorta in a patient who presents to the emergency department in extremis?

E

Primary fibrinolysis is a key component in the pathophysiology of the acute coagulopathy of trauma, and, as such, is associated with risk of mortality. In severe trauma, hyperfibrinolysis is associated with a mortality rate of 70 to 100%. Therefore, using antifibrinolytic agents to manage acute coagulopathy of trauma is plausible.

A. B. C. D. E.

Two landmark studies—the Clinical Randomization of an Antifibrinolytic in Significant Hemorrhage (CRASH)-2 and Military Application of Tranexamic Add in Trauma Emergency Resuscitation (MATTERS)—demonstrated improved mortality with the administration of the antifibrinolytic agent tranexamic add (TXA) in civilian and military trauma. Bleeding adult trauma patients who present with systolic blood pressure less than 90 mm Hg, heart rate of at least 110 beats per minute, or both, may benefit from TXA administration, with a reduction in 28-day all-cause mortality. TXA has the greatest impact reducing death in cases of severe shock (systolic blood pressure <75 mm Hg). Early TXA (<1 hour after injury) results in the greatest bleeding-related death reduction. TXA administered from 1 to 3 hours after injury also reduces bleeding-related death but to a more modest degree. TXA administered more than 3 hours after injury appears to increase risk of death due to bleeding. TXA is not associated with increased vascular occlusive events.

Gunshot wound to the neck Blunt cardiac injury Blunt thoracic aortic injury Stab wound to the chest Unstable pelvic fracture

ANSWER:

E

Hemorrhage is the leading cause of death in trauma patients. Noncompressible truncal hemorrhage is a particularly vexing problem, because progression to profound shock and death can be rapid, and the offending injuries are not amenable to direct tamponade. Proximal aortic cross-clamping via resuscitative thoracotomy allows establishment of temporary hemodynamic stability as a bridge to definitive injury repair for patients in extremis from profound hemorrhagic shock. An emerging alternative to resuscitative thoracotomy in the trauma patient is resuscitative endovascular balloon occlusion of the aorta (REBOA). This technique has been used in vascular surgery for the past 20 years, and its application is now expanding to trauma. REBOA is a minimally invasive procedure that uses a transfemoral balloon catheter, which is rapidly inserted retrograde and inflated for aortic occlusion. For control of pelvic hemorrhage, the balloon is inflated between the lowest renal artery and the aortic bifurcation (aortic zone III). For pelvic hemorrhage, balloon inflation occurs between the left subdavian artery and the celiac trunk (aortic zone I). Balloon

Current recommendations for the use of TXA in trauma based on the CRASH-2 trial are as follows: 1.

established fibrinolysis via thromboelastography (LY30 >3%). Administer only if less than 3 hours after injury. Use a dose of 1 g intravenously over 10 minutes followed by 1 g intravenously over 8 hours.

Administer to adult trauma patients with severe hemorrhagic shock (systolic blood pressure <75 mm Hg), with known predictors of fibrinolysis, or with

121

occlusion should not occur between the celiac trunk and the lowest renal artery (aortic zone II). Balloon deployment in REBOA controls inflow and stems hemorrhage, provided that the balloon is inflated proximal to the injury, allowing time to proceed to definitive injury repair in the operating room, interventional radiology, or hybrid suite. The use of REBOA in these situations is attractive, yet further assessment of outcomes is needed based on recent literature.

20. The pictured injury (figure 20.1 and figure 20.2) occurred with a circular saw. The patient is hemodynamically normal. You are in a small community hospital, 250 miles from a hospital with a replantation program. Which of the following is the most appropriate management? A. Wrap the hand in gauze moistened with saline or Ringer lactate solution, place in a sealed plastic bag, and pack on dry ice for transport. B. Wrap the hand in gauze moistened with a dextrose 5% in water solution, place in a sealed plastic bag, and pack on dry ice for transport. C. Wrap the hand in gauze moistened with saline or Ringer lactate solution, place in a sealed plastic bag, and immerse in ice water for transport. D. Place the hand in iced saline for transport. E. Place the hand in dry ice for transport.

REBOA should be confined to patients in extremis due to exsanguinating hemorrhage arising below the diaphragm. The 2 primary indications for use of REBOA in trauma are extremis due to profound hemorrhagic shock secondary to pelvic trauma or abdominal hemorrhage. REBOA should not be used in patients with blunt cardiac or aortic injury and penetrating neck or chest trauma. Patients with cardiovascular collapse due to a major intrathoracic injury should undergo resuscitative thoracotomy, because the heart and thoracic vasculature can be directly accessed for clamping, release of cardiac tamponade, and temporization of cardiac and hilar injuries.

19. A 30-year-old man is involved in a head-on motor vehicle crash. He is placed in a cervical collar. On arrival to the emergency department, he is awake and alert with normal vital signs. His secondary survey identifies no injuries. Which of the following findings indicates imaging of his cervical spine is needed? A. B. C. D. E.

Glasgow Coma Scale score of 15 Normal neurologic examination Absence of midline c-spine tenderness Inability to cooperate with the exam No evidence of intoxication

ANSWER:

Figure 20.1.

D

Although injury to the cervical spine is quite rare (occurring in 1-3% of multisystem blunt trauma patients), missing a clinically significant injury can have disastrous consequences. To address the question of which patients require screening, the National Emergency X-Radiography Utilization Study (NEXUS) decisionmaking rules were developed, applied, and studied prospectively. A patient who is awake, alert, and evaluable with no distracting injuries and neurologically normal with no midline cervical spine tenderness meets the NEXUS guidelines for removing the collar and clearing the neck clinically. Patients who cannot co-operate with the examination, whether by intoxication or other conditions, cannot be cleared clinically. Current recommendations for those who do not meet the criteria for clinical clearance is to proceed with imaging. CT scan is more sensitive than plain films.

Figure 20.2.

ANSWER:

C

Hand replantation is a viable option for many patients who have not had severe mangling or crush injuries. Replantation of a midpalmar amputation has a reported success rate of

122

86%. The surgeon or emergency medicine physician called on to manage the patient before transfer to a replantation center must be familiar with management of the amputated tissue. Cooling the tissue is essential to reduce metabolic demand before revascularization. Dry ice is too cold and runs the risk of freezing the specimen, whether in a bag or not. Placing the hand directly into the saline will lead to maceration of the tissues. The preferred method of transport is wrapped in physiologic solution moistened gauze, sealed in a plastic bag, and immersed in ice water.

monitoring and immediate access to surgery, selective management may have a role.

21. An 18-year-old man presents with a stab wound to the left chest. He is hemodynamically normal. His chest x-ray is normal. Focused assessment with sonography for trauma shows pericardial fluid. The next step in management should be

A. B. C. D. E.

A. B. C. D. E.

22. A 25-year-old man is involved in a motor vehicle crash and has the pelvic fracture seen on the x-ray (figure 22.1). His blood pressure is 80/50 mm Hg and his heart rate 120 beats per minute; he is diaphoretic. His abdomen is nontender and nondistended, and he has a negative focused assessment with sonography for trauma exam. What intervention is the best next step in management of his pelvic fracture? Exploratory laparotomy -Pelvic angioembolization Preperitoneal packing Application of an external pelvic binder Resuscitative endovascular balloon occlusion of the aorta

median sternotomy. subxiphoid pericardiotomy. cardiopulmonary bypass. transesophageal echocardiogram. left anterolateral thoracotomy.

ANSWER:

B

A subxiphoid pericardiotomy is indicated for this patient to confirm the cause of the abnormal focused assessment with sonography for trauma (FAST) exam. In recent years, support has emerged for the selective management of hemopericardium. The patient in this scenario is hemodynamically normal with a positive pericardial view on FAST exam. Neither transesophageal echocardiogram nor cardiopulmonary bypass are indicated as the next step. Mandatory exposure of the heart by either median sternotomy or left anterolateral thoracotomy leads to a high nontherapeutic operation rate. One report revealed that 38% of sternotomies for positive subxiphoid windows had no cardiac or great vessel injury requiring repair.

Figure 22.1.

ANSWER:

D

The patient described has a pelvic fracture associated with likely hemorrhagic shock (figure 22.2) and no findings that would make intraperitoneal injury the leading differential diagnosis. The first step in management for this patient is application of an external pelvic binder (figure 22.3). An external pelvic binder can be applied in the emergency department with no preparation and with a minimum of training. It immediately decreases the volume of the pelvis and tamponades pelvic fracture bleeding.

As early as 1995, in a report on echocardiography for penetrating heart trauma, 4 cases of pericardial irrigation without sternotomy were reported. In a 2014 randomized trial, hemodynamically normal patients with penetrating chest trauma had a 24-hour period of observation, then went on to subxiphoid pericardial window. Only 6 of 191 patients observed became hemodyamically abnormal and went to surgery before the end of the observation period. If blood, but not active bleeding, was found at pericardiotomy, patients were randomized to sternotomy or irrigation and drainage without sternotomy. The nonstemotomy group had a shorter intensive care unit stay and hospital stay, no life-threatening complications, and no mortality. The only death was in the sternotomy group, from a missed iatrogenic internal mammary injury. In centers with the capability for intensive

123

23. A 7-year old boy presents several hours after blunt trauma to the abdomen caused by falling onto his bicycle handlebars. He initially went home, but his mother brought him to the hospital for repetitive emesis of clear fluid. He has a bicycle handlebar imprint in his epigastrium and peritonitis on exam. CT scan reveals a transected pancreas to the left of the mesenteric vessels (figure 23.1). What is the best next step? A. Splenic-preserving distal pancreatectomy B. Diagnostic endoscopic retrograde cholangiopancreatography with placement of a pancreatic stent C. Laparoscopy and drain placement D. Bedrest, nasogastric tube decompression, and central catheter placement for parenteral nutrition E. Bed rest, nasogastric tube decompression, and nasojejunal tube placement for enteral feeds

Figure 22.2. Pelvic radiograph showing multiple fractures (arrows).

Figure 23.1.

Figure 22.3. Radiograph after placement of an external pelvic ANSWER:

binder. Note the compression of the urinary bladder, most likely caused by a pelvic hematoma.

A

The management of blunt pancreatic trauma in children is controversial, with some surgeons advocating operation and others nonoperative management for high-grade injuries. Pancreatic trauma has 5 grades: grade I is a minor contusion or superficial laceration without duct injury, grade II is a major contusion without duct injury or tissue loss, grade HI is distal transection or parenchymal injury with duct injury, grade IV is a proximal transection involving the ampulla, and grade V is massive disruption of the pancreatic head (table 23.2).

Exploratory laparotomy in this situation would be nontherapeutic and would delay treatment of the bleeding pelvic fracture. Pelvic angioembolization and preperitoneal packing are both good options for controlling pelvic fracture bleeding, but both require at least a short time for preparation and execution. Resuscitative endovascular balloon occlusion of the aorta is a promising technique that can stop arterial inflow to the pelvis. It too has a short, but real, time to prepare and execute.

124

Grade * I

II

Type-of Injury Hematoma

Description

Laceration

Superficial laceration without duct injury

Hematoma

Major contusion without duct injury or tissue loss Major laceration without duct injury or tissue loss Distal transection or parenchymal injury with duct injury

Laceration

24. A 24-year-old man presents to the emergency department with a single gunshot injury to the right frontotemporal region of the head. He has no other signs of injury. His heart rate is 40 beats per minute, and his blood pressure is 180/110 mm Hg. What is the most appropriate next step in management of his head injury?

Minor contusion without duct injury

III

Laceration

IV

Laceration

Proximal transection or parenchymal injury involving ampulla

V

Laceration

Massive disruption of pancreatic head

A. B. C. D. E.

Intravenous mannitol 1 g/kg Supine position Intravenous esmolol Hyperventilation to PaCO2 less than 25 mm Hg Intracranial pressure monitoring

ANSWER:

A

Management of severe brain trauma continues to evolve as new data regarding outcomes are published. Current Brain Trauma Foundation recommendations for the use of mannitol or hypertonic saline as hyperosmolar therapies are limited due to the lack of sufficient evidence from comparative studies demonstrating improved clinical outcomes. However, the group continues to recommend the use of mannitol before intracranial pressure (ICP) monitoring in patients with signs of transtentorial herniation or progressive neurologic deterioration not attributable to extracranial injury. Thus, waiting to place an ICP monitor is not recommended. Furthermore, randomized controlled trials have not demonstrated an improved outcome with ICP monitoring. The utility of mannitol over hypertonic saline has not been demonstrated, thus either may be used. Patients should have the head of bed elevated to decrease ICP when possible. Blood pressure should not be lowered, as an elevated blood pressure is a natural response to increasing ICP. Hyperventilation to low normal levels (30-35 mm Hg) is a short-term adjunctive therapy for patients with evidence of severe brain injury. However, level I data are lacking, and recent studies demonstrated cerebral ischemia, thus altering current expert opinion. Hyperventilation to PaCO2 of less than 25 mm Hg for prolonged periods is certainly not recommended.

* Advance one grade for multiple injuries up to grade III. Proximal pancreas is to the patients’ right of the superior mesenteric vein. Table 23.2. Pancreatic injury scale. A review article summarizing pediatric blunt pancreatic trauma management noted controversy in the management of higher-grade injuries but reported that patients with ductal disruption may benefit from operative intervention. If the ductal injury is proximal, a subset of patients may benefit from endoscopic retrograde cholangiopancreatography and stent placement in centers where this procedure can be offered to children. If endoscopic retrograde cholangiopancreatography is not available, nonoperative management with expected pseudocyst formation and later drainage is favored over operation on the pancreatic head. Although the spleen might be removed in adults with this injury, distal pancreatectomy with preservation of the spleen is favored in children with duct disruption. In a study comparing 39 children with grade HI to IV pancreatic injuries from 2 institutions that had different philosophies regarding management, those managed operatively had either distal pancreatectomy (12 patients) or only a drain placement (3 patients). Patients managed nonoperatively (24 patients) were started on parenteral nutrition and nasogastric suction. Compared with early operation, nonoperative management was associated with an 8-times greater risk of complications and 13 more days on parenteral nutrition, with a trend toward considerably longer hospitalization. Two-thirds of patients who had drain placement only also had complications, which accounted for half of the complications in the entire operative group. Pseudocyst development was the second most common complication after central venous catheter infection. None of the operative patients developed a pseudocyst, and central catheter infections were more frequent in nonoperative patients, likely related to a longer catheter duration for prolonged parenteral nutrition.

25. A 10-year-old boy presents after a firework exploded near his hands and chest. He has first- and second-degree bums to the right chest (5% total body surface area). He had no loss of consciousness, was not thrown, and has no other external, signs of trauma. This occurred outdoors, and there are no carbon deposits in his mouth or nose There are no fractures on radiographic studies. On postinjury day 1, he develops a cough and requires supplemental oxygen via nasal cannula. His chest x-ray (figure 25.1) demonstrates A. B. C. D. E.

125

pneumonia. atelectasis pulmonary contusion. inhalation injury. thermal injury to the lung parenchyma.

The primary blast injury is a blast-induced pressure wave, which extends outward from the explosion. As this intense overpressurization wave passes through the body, it results in injuries involving the lungs, gastrointestinal tract, and tympanic membranes due to disruption at air-water interfaces. The resulting lung injury is a pulmonary contusion that is not associated with rib fractures. The secondary injury is due to shrapnel resulting in penetrating wounds. The tertiary blast injury is blunt force trauma due to the body being thrown against a stationary environmental object. Quaternary blast injuries include bums, crush, and toxic inhalation. In this case, the explosion was adjacent to the chest wall, and the primary blast injury resulted in a pulmonary contusion (figure 25.3). Pulmonary contusions take time to develop and may not be evident on initial radiographic imaging.

Figure 25.1.

ANSWER:

Although a large explosion, particularly in a closed space, may force heated gas indirectly into the distal airways resulting in a thermal burn, this is not the situation in the case presented.

D

Explosions can occur in the civilian setting. The resulting physical trauma is due to any combination of 4 distinct mechanisms, which occur nearly simultaneously (table 25.2).

Category

Characteristics

Body Part Affected

Types of Injuries

Primary

Unique to high- order explosives, results from the impact of the overpressurization wave with body surfaces.

Gas-filled structures are most susceptible: lungs, gastrointestinal tract, and middle ear

• •

Results from flying debris and bomb fragments

Any body part may be affected



Secondary

• • •

Blast lung (pulmonary barotrauma) Tympanic membrane rupture and middle ear damage Abdominal hemorrhage and perforation Globe (eye) rupture Concussion (traumatic brain injury without physical signs of head injury)



Penetrating ballistic (fragmentation) or blunt injuries Eye penetration (can be occult)

Tertiary

Results from being thrown by the blast wind

Any body part may be affected

• •

Fracture and traumatic amputation Closed and open brain Injury

Quaternary

* Ail explosion- related injuries, illnesses, or diseases not due to primary, secondary, or tertiary mechanisms * Includes exacerbation or complications of existing conditions

Any body part may be affected

• • • •

Burns (flash, partial, and full thickness) Crush injuries Closed and open brain Injury Asthma, chronic obstructive pulmonary disorder, or other breathing problems from dust, smoke, or toxic fumes Angina Hyperglycemia Hypertension

• • •

Table 25.2. Mechanisms of blast injury. 126

centrally located within the spinal cord, involvement of the upper extremities is typical. The patient presents with bilateral upper extremity loss of sensation and weakness. Lower extremity function is typically preserved, and the patient usually is able to walk. Brown-Sequard syndrome results from hemitransection of the spinal cord with unilateral damage to the corticospinal tract, spinothalamic tract, and dorsal columns. Presenting symptoms include ipsilateral loss of proprioception, motor function, and light touch sensation with contralateral loss of pain and temperature sensation. Posterior cord syndrome is rare and is the result of injury to the dorsal columns. This causes loss of proprioception and vibration, although motor function is preserved. Many patients have difficulty ambulating due to the loss of proprioception.

Figure 25.3. Pulmonary contusion. Being outdoors when an explosion takes place makes inhalation injury less likely. The time course would be unusual for pneumonia to develop, and it is not consistent with the chest x-ray findings. Atelectasis can occur, particularly if there are associated fractures. However, in this case, the pulmonary contusion is isolated to where the explosion occurred adjacent to the chest wall and lacks the plate-like appearance more suggestive of atelectasis.

Conus medullaris syndrome usually results from injury to the T12 and Ll-L2 regions of the spine. Typical symptoms include loss of sensation in the saddle region with bowel and bladder dysfunction. The patient does not typically have lower extremity weakness.

Items 26-28 Each lettered response may be selected once, more than once, or not at all. A. B. C. D. E.

Anterior cord syndrome Brown-Sequard syndrome Central cord syndrome Conus medullaris syndrome Posterior cord syndrome

26. Bilateral loss of motor and sensation in the upper extremities 27. Ipsilateral paralysis and loss of proprioception with contralateral loss of pain and temperature sensation 28. Loss of sensation in the perineum, bladder dysfunction

ANSWERS:

C, B, D

Anterior cord syndrome classically occurs from a vascular injury that results in anterior spinal artery insufficiency. This leads to ischemia involving the anterior two-thirds of the cord. It can also occur after blunt trauma to the anterior spinal cord. The patient presents with loss of motor function as well as a loss of pain and temperature sensation below the level of the injury. Proprioception and the ability to sense vibration are preserved. Central cord syndrome is usually caused by hyperextension of the cervical spine in a patient with pre-existing cervical spondylosis. Because the upper extremity axons are more

127

Trauma Part II

2. A 22-year-old man presents with an isolated 2-cm stab wound to the mid left neck just anterior to the sternocleidomastoid muscle. His vital signs are normal. His voice is normal. On physical exam, there is a small hematoma (3 cm) that is stable in size with no signs of bleeding. The remainder of his exam is normal. The best next step in his management is

ITEMS 1-28 For each question, select the best possible response. 1. A 22-year-old pregnant woman presents after being ejected from a truck at highway speed. She is intubated with a Glasgow Coma Scale score of 3T, pulse of 120 beats per minute, and blood pressure 105/45 mm Hg. Her fundal height is 6 cm superior to the umbilicus. A fetal heart-rate monitor was placed. Which of the following statements is true?

A. B. C. D. E.

A. Normal heart rate of her fetus should be 85 to 100 beats per minute B. She should be placed in the right lateral decubitus position. C. Anemia of pregnancy is anticipated due to decreased red cell mass. D. CT scan of the abdomen and pelvis are contraindicated. E. The fetus has likely reached the stage of viability.

ANSWER:

esophagogastroduodenoscopy. local wound exploration. CT angiogram. catheter-based angiography. formal neck exploration.

ANSWER:

C

Historically the initial evaluation and management of penetrating wounds to the neck was driven by 3 factors: presence or absence of hard signs, location of injury, and mechanism of injury (stab or gunshot wound).

E Hard signs of vascular or aerodigestive tract injury (e.g., active bleeding, expanding or pulsatile hematoma, air bubbling in the neck) require urgent operative management by formal neck exploration. Patients with "soft" signs (e.g., small, nonexpanding hematoma, hoarseness, odynophagia) can be selectively evaluated and managed. In this group of patients (hemodynamically normal with only soft signs), multidetector CT angiogram is an excellent screening tool with sensitivity, specificity, positive predictive value, and negative predictive value all greater than 90%, especially related to vascular injuries. It is important to remember that this is a screening exam and any concerns for aerodigestive tract injury after the CT scan should be specifically evaluated. Thus, esophagogastroduodenoscopy could be indicated based on the CT scan finding but would not be the next step in this situation.

Trauma is the leading cause of nonobstetric mortality for pregnant women. Injury affects 1 in 12 pregnant women and has a major impact on maternal morbidity and fetal outcome. With a few exceptions, the diagnosis and management of the traumatically injured pregnant patient is the same as for nonpregnant patients. Due to uterine compression of the inferior vena cava, those in more advanced stages of pregnancy should be placed in the left lateral decubitus position to relieve this compression and allow for better venous return. Placement in the right lateral decubitus position has the opposite effect. Although there is concern about the teratogenic effects of ionizing radiation, this risk is highest during the period of organogenesis that occurs between 5 and 10 weeks gestation. At 20 weeks gestation, the fundal height is palpable at the umbilicus. Thereafter, it increases by 1 cm per week. In this patient with a fundal height 6 cm superior to the umbilicus, the predicted gestational age would be 26 weeks. Therefore, CT scan of the abdomen and pelvis would be safe, with a fetal exposure of only 3.5 rads.

Local wound exploration without imaging is contraindicated and risks bleeding and loss of airway if an occult major arterial injury is present. Catheter-based angiography would be used if a specific vascular injury amenable to endovascular treatment was identified. However, use as a screening tool would not identify other injuries and would risk the complications of catheter-based angiography. Formal neck exploration in patients with only soft signs would be nontherapeutic in up to 80%. Thus, for this patient with normal vital signs and a small, stable hematoma, CT angiogram is the best next step in management.

With advances in neonatal care, the age of viability continues to decline but is usually considered to be more than 23 weeks gestation or a fundal height 3 cm above the umbilicus. Red blood cell mass increases approximately 30%, but blood volume increases 40 to 50%. As a result there is a relative anemia of pregnancy. The normal fetal heart rate is 120 to 160 beats per minute. Monitoring fetal heart rate is needed to detect decelerations, tachycardia, bradycardia, or loss of variability, which are associated with adverse fetal outcomes.

Traditionally, the initial evaluation and management of penetrating neck injuries was based on the zone of injury: • • •

129

Zone I: manubrium to cricoid Zone II: cricoid to angle of the mandible Zone III: above the angle of the mandible

However, this zone-based approach had several challenges. Although it worked reasonably well with stab wounds, gunshot wounds presented problems. Specifically, gunshot wounds may traverse several zones, especially if the trajectory is transcervical. Another approach suggests initial evaluation and management should be guided by symptoms and clinical findings and not the zone of injury. Patients with hard signs of injury should undergo operative management. The only caveat is zone HI injuries, which can be very difficult to reach operatively and may be best approached by endovascular techniques. Patients with soft signs should have screening CT angiogram as their first investigation. Asymptomatic patients with normal exams can be managed expectantly.

4. For a patient requiring operative care for a traumatic pancreas injury, the strongest predictor of mortality is A. B. C. D. E.

ANSWER:

A

Pancreatic and pancreaticoduodenal injuries are challenging and require an organized and systematic approach. Mortality rates are 12 to 18% and are higher for blunt trauma compared with penetrating trauma. Shock on admission is the strongest predictor of mortality for patients with operative pancreas injuries. Injuries to the body are easier to address and have lower mortality than injuries to the head of the pancreas. The number of intra- and extra-abdominal injuries does not influence mortality, although associated vascular injuries and increasing age does increase risk of mortality.

3. A 25-year-old man sustained a gunshot wound to the left lower quadrant of the abdomen. He arrives with a normal primary survey, including normal vital. signs, but he has diffuse abdominal tenderness on secondary survey. During operative exploration, you find a through-and-through injury to the sigmoid colon with loss of more than 50% of the wall. There is local spillage of solid stool, minimal blood loss, and no other major injury. The next operative step is

5. Which of the following is most predictive of failure of nonoperative management for blunt splenic trauma?

A. primary repair of injury. B. resection of the injury, end colostomy with blind distal colon. C. resection of the injury, end colostomy and mucus fistula. D. resection of the injury with anastomosis. E. resection of the injury, planned second look.

ANSWER:

shock on admission. injury to the body of the pancreas. associated liver injury. blunt mechanism of injury. number of extra-abdominal injuries.

A. B. C. D. E.

D

Hemodynamic normality on admission Grade IV splenic injury Hemoperitoneum Age younger than 40 years Arterial contrast blush on CT scan

ANSWER:

Traumatic colon injuries can be classified as nondestructive (amenable to primary repair) or destructive (require resection). This patient has a destructive injury with loss of more than 50% of the circumference of the colon. Primary repair would likely result in narrowing or stricture. Thus, resection is required.

B

major injuries, the next step after resection should be anastomosis. In this situation, the leak rate is less than 10%. An end colostomy, with or without a mucus fistula, would require another laparotomy for this patient and is unnecessary. Likewise, damage control surgery and a second look are not required in a patient with an uncomplicated preand intraoperative course. Splenic injury can range from small to moderate lacerations or subcapsular hematomas, typically grades I to III. In hemodynamically normal patients, these injuries can often be successfully treated nonoperatively (bed rest, hemodynamic and laboratory monitoring). More significant lacerations involving segmental or hilar vessels producing devascularization of more than 25% of the spleen (grade IV) or a completely shattered spleen (grade V) are more often treated with splenectomy, but grade IV and V injuries in hemodynamically normal patients can be managed with nonoperative management with or without arterial embolization. Routine selective arterial embolization is associated with lower failure rates of nonoperative management in grade IV and V injuries (table 5.1).

After segmental resection, the question becomes how and when to reconstruct the colon. The 3 main options are (1) diversion (Hartmann procedure), (2) anastomosis, and (3) anastomosis with diversion. The main determinants of these choices are hemodynamic status and overall burden of injury. In this patient with a normal primary survey, including normal vital signs, minimal blood loss, and no other major injuries, the next step after resection should be anastomosis. In this situation, the leak rate is less than 10%. An end colostomy, with or without a mucus fistula, would require another laparotomy for this patient and is unnecessary. Likewise, damage control surgery and a second look are not required in a patient with an uncomplicated pre- and intraoperative course.

130

Grade * I

II

III

Type of Injury Hematoma

Description

Laceration

Capsular tear, <1 cm parenchymal depth

Hematoma

Subcapsular, 10-50% surface intraparenchymal, <5 cm in diameter

Laceration

1-3 cm parenchymal depth, which does not involve a trabecular vessel

Hematoma

Subcapsular, >50% surface area or expanding, ruptured subcapsular or parenchymal hematoma >3 cm parenchymal depth involving trabecular vessels

Laceration IV

Laceration

V

Laceration

Subcapsular, <10% surface area

area,

Involving segmental or hilar vessels producing major devascularization (>25% of spleen) Completely shattered spleen

Vascular

Hilar vascular injury, which devascularizes spleen * Advance one grade for multiple injuries up to grade III.

Table 5.1. Splenic injury scale. Risk factors with strong evidence for failure of nonoperative management include age older than 40, grade III or higher injury, or injury severity scores of at least 25. Moderate evidence exists for intraparenchymal contrast blush or transfusion of more than 1 unit of blood being risk factors. No evidence for failure of nonoperative management exits for hemodynamic status on admission, emergency department mean blood pressure, hemoglobin, hematocrit level, or heart rate. 6. In patients with hemorrhagic shock due to severe trauma, what is the recommended approach to plasma, platelets, and packed red blood cell (PRBC) product resuscitation during the initial phase of management?

introduced complexity into the process of resuscitation. Crystalloid, albumin, and artificial starch solutions also became popular as volume expanders during this time. Experience over the previous 30 years and several recent publications have resulted in some significant changes in the practice of resuscitation, which in many ways now resembles the historic use of whole blood. These studies demonstrated that for the acutely bleeding patient in hemorrhagic shock, empiric resuscitation with packed red blood cells (PRBCs) together with both plasma and platelets is the optimal initial management. PRBCs do not contain clotting factors or platelets; therefore, they should not be used in isolation in the resuscitation of acutely bleeding patients. Waiting for laboratory tests before initiating blood product resuscitation, even if they are point of care, is both unnecessary and potentially harmful in these circumstances. Furthermore, plasma and platelets should be given concurrently with PRBCs to avoid significant "imbalances" in the ratio of blood products at any given time. 7. A 28-year-old man presents with a single stab wound to the left chest just medial to the nipple. On examination, he is pale, diaphoretic, and lethargic. His heart rate is 120 beats per minute, his blood pressure 98/65 mm Hg, and his respiratory rate is 24 breaths per minute. A chest x-ray is taken with a paperclip on the site of the chest wound (figure 7.1). Focused assessment with sonography for trauma examination is performed and is positive for pericardial fluid. What is the best next step in the management of this patient? A. Chest CT scan B. Left tube thoracostomy C. Median sternotomy D. Subxiphoid pericardial window E. Pericardiocentesis

A. PRBCs alone until surgical hemostasis is achieved B. PRBCs for hemoglobin less than 7 g/dL, platelets when count is less than 150,000/mm3 C. Plasma, platelets, and PRBCs all given empirically D. Plasma after 10 units of PRBCs, platelets after 20 units of PRBCs E. Plasma and platelets, only if coagulation studies are abnormal

ANSWER:

C

Hemorrhage results in nearly 50,000 deaths and 2 million years of life lost annually in the United States. Up to 64% of these deaths are potentially preventable. Blood product resuscitation is essential to the survival of patients in hemorrhagic shock, and recent advances in the practice of resuscitation will hopefully reduce the number of preventable deaths. Historically, whole blood was used for resuscitation; however, the advent of component therapy in the 1960s and its increased use through the 1970s and 1980s suddenly

Figure 7.1.

ANSWER:

C

This patient has signs and symptoms consistent with obstructive shock from pericardial tamponade (diaphoresis,

131

lethargy, tachycardia, narrowed pulse pressure, tachypnea). No pneumothorax is seen on the chest x-ray; therefore, a left chest tube is not specifically required. The cardiac silhouette bears mention in this case. There is a relatively normal contour suggesting that (1) the patient does not have purely hemorrhagic shock and (2) if there is a pericardial effusion, it is likely acute because the pericardium has not had time to expand. His focused assessment with sonography for trauma (FAST) exam is grossly positive in the pericardial window, which is 97% specific for a cardiac laceration. Thus, no further diagnostic measures (e.g., pericardial window, chest CT scan) or temporizing therapeutic interventions (i.e., pericardiocentesis) are needed in this patient if surgical capability for pericardial decompression and cardiac repair is immediately available.

this situation, securing the airway as a first step in management is most important Given level HI trauma center resources, transport of this patient is necessary. Maintaining adequate oxygenation during transport will help prevent secondary brain injury. Securing the airway with endotracheal intubation is the most reliable method to maintain oxygenation and ventilation, given the GCS score, and it should be the first step in management. Further management may include a trauma CT scan if there will be a delay in transfer. A CT scan of the head can be performed, with the results being communicated to the receiving hospital. CT scan gives the most information in a short period of time about the status of the brain, and this information should be communicated expeditiously to the consulting neurosurgeon for decision making and treatment planning for interventions before transfer.

The diagnostic accuracy of pericardial FAST is degraded in patients with a hemothorax or if adequate subxiphoid or parasternal views cannot be obtained due to subcutaneous emphysema or body habitus. If adequate windows can be obtained and fluid is visualized in a patient with a clinical picture consistent with tamponade, further evaluation is not needed. In equivocal cases, gated cardiac CT scan of the chest, pericardiocentesis, or pericardial window can be used to guide further therapy. If a pericardial effusion is identified and surgical resources are not available, temporizing pericardiocentesis can buy time until the patient can be transported to a center with surgical resources.

9. A 30-year-old woman is a restrained driver in a motor vehicle crash. On initial evaluation, she has a Glasgow Coma Scale score of 15, and she is hemodynamically normal. She has mild tenderness on abdominal palpation and no seat belt sign. An intravenous contrast-enhanced CT scan of her abdomen. and pelvis reveals no injury to liver, spleen, or kidneys. The radiologist reports moderate amounts of free fluid in the pelvis with mesenteric stranding in the small bowel. The next appropriate step in her management is A. B. C. D. E.

8. A 46-year-old man is involved in a high-speed motorcycle crash; he was not helmeted. He is brought to a level III trauma center 1 hour after the event. The patient has a patent airway, bilateral breath sounds, and a systolic blood pressure of 100 mm Hg on initial survey. He is resuscitated with a good response. He is noted to have a comminuted open left femur fracture and withdraws to pain when the limb is placed in traction. His eyes are closed and open only to painful stimuli; he verbalizes that he is in pain but cannot converse normally. This man's degree of head injury can be assessed by his calculated Glasgow Coma Scale score of 10. The next step in management is A. B. C. D. E.

ANSWER:

D

Blunt hollow viscus injury (BHVI) is reported in 1 to 5% of blunt trauma patients. These injuries may be easily missed, but specific findings on CT scan can suggest the presence of BHVI. Free air or extravasation of enteric contrast are obvious signs, but many patients have more subtle findings. Often the only irregularity is bowel wall thickening, irregular contrast enhancement of the bowel wall, mesenteric abnormality, or unexplained free fluid in the abdomen. No combination of findings is pathognomonic for BHVI. The surgeon must have a high index of suspicion for these injuries and consider the mechanism of injury and physical examination findings in decisionmaking (figure 9.1). This patient has tenderness on abdominal exam and concerning findings on CT scan, both of which are predictors of BHVI. Patients in whom there is a concern for BHVI should undergo operative exploration. In selected patients, diagnostic laparoscopy may be appropriate; however, an exploratory laparotomy will allow careful inspection of the stomach, small bowel, and colon.

measure intracranial pressure. observe the patient. perform a brain CT scan. contact a level-I trauma center for transfer. intubate the patient.

ANSWER:

admission with observation. CT scan with oral and rectal contrast. focused abdominal sonogram for trauma. exploratory laparotomy. discharge.

E

This patient is a head-injured, multisystem blunt force trauma victim who is initially stable after resuscitation. His initial Glasgow Coma Scale (GCS) score, which is an easily reproducible method to evaluate degree of neurologic disability is scored as (Eyes or E = 2) + (Verbal or V = 4) + (Motor or M = 4) is 10. He has a depressed sensorium, and in

132

Rib fixation is especially useful in patients who are not able to progress to spontaneous ventilation after initially being intubated and mechanically ventilated. Maximal benefits occur when the procedure is done early after injury, usually within 3 to 5 days.

11. A 39-year-old man sustained a single stab wound to the left chest. On arrival to the emergency department, he was pulseless. Emergency department thoracotomy is indicated in this patient only if he A. did not require prehospital cardiopulmonary resuscitation. B. has signs of life in the emergency department. C. has a tension hemothorax. D. has a primary cardiac injury. E. is neurologically intact.

Figure 9.1. Blunt bowel and mesenteric injury. Further diagnostic imaging with focused abdominal sonogram for trauma or CT scan with enteric contrast is unlikely to provide clarification of a BHVI and may delay time to diagnosis. Any delay in diagnosis of a BHVI increases the morbidity and mortality associated with the injury. In patients with a high suspicion of injury, a period of observation or discharge may increase the overall complication rate.

ANSWER:

The 2015 guidelines for resuscitative thoracotomy by the Eastern Association for the Surgery of Trauma (EAST) reviewed 72 studies and 10,238 patients who underwent resuscitative thoracotomy. Based on the evidence, EAST strongly recommended that patients who present pulseless with signs of life after penetrating thoracic injury undergo resuscitative thoracotomy. Signs of life, often used interchangeably with vital signs, were defined by the American College of Surgeons Committee on Trauma in 2001: pupillary response, spontaneous ventilation, presence of carotid pulse, measurable or palpable blood pressure, extremity movement, or cardiac electrical activity.

10. A 60-year-old man is admitted with a flail right chest after a fall from a roof. He has been intubated and mechanically ventilated for 3 days with no improvement in spontaneous breathing trials. He is being considered for rib stabilization Compared with nonoperative management, the operative fixation of rib fractures in patients with a flail chest after blunt trauma reduces A. B. C. D. E.

Prehospital cardiopulmonary resuscitation, tension hemothorax, primary cardiac injury, and neurologic status are not valid indicators for the use of resuscitative thoracotomy because they do not predict survival in patients with traumatic arrest.

mortality. narcotic requirements spirometry volumes. pneumonia. costs.

ANSWER:

B

Focused assessment with sonography for trauma (FAST) exam to evaluate the presence or absence of cardiac motion may be useful in determining whether traumatic arrest patients should undergo resuscitative thoracotomy. A singleinstitution study of 187 blunt and penetrating traumatic arrest patients (77% required resuscitative thoracotomy) confirmed that overall survival was only 3.2%. Cardiac motion on FAST exam was 100% sensitive and 73.7% specific for the identification of survivors and organ donors. If both cardiac motion and pericardial fluid were absent on FAST exam, survival was zero. FAST therefore represents an effective method to identify trauma patients with higher likelihood of survival who may benefit from resuscitative thoracotomy.

D

Several randomized clinical trials compared the operative fixation of rib fractures with nonoperative management after blunt trauma. These trials have consistently demonstrated that surgical rib fracture fixation after blunt chest trauma reduces pulmonary morbidity, in particular the incidence of pneumonia and the duration of mechanical ventilation. Compared with nonoperative management, surgical rib fracture fixation increases spirometry volumes, but it does not decrease mortality rates or narcotic requirements. Hospital costs are higher with surgical rib fixation than with nonoperative treatment.

133

Items 12-15 Each lettered response may be selected once, mare than once, or not at all. A. B. C. D. E.

and distal thoracic esophagus. The lateral position has the disadvantages of not being tolerated well by hemodynamically unstable patients, providing very limited access to the abdomen, and providing practically no access to the opposite thorax. A right anterior thoracotomy is relatively rarely used in trauma. It does achieve access to the right lung parenchyma and provides limited access to the lung hilum.

Left anterior thoracotomy Left posterolateral thoracotomy Median sternotomy Right anterior thoracotomy Right posterolateral thoracotomy

The access to the heart is limited. It does have the advantage of being performed in the supine position. Right anterior thoracotomy is used in a hemodynamically compromised patient with injuries limited to the right chest.

12. Gunshot to left chest with arrest in trauma bay 13. Stab wound 1 cm left of sternum, blood pressure 96/80 mm Hg, heart rate 125 beats per minute, positive pericardial effusion on ultrasound

16. A 54-year-old man is involved in a motor vehicle collision. Portable anteroposterior chest radiograph reveals a widened mediastinum. He has no other injuries. The next step in his evaluation should be

14. Gunshot to back with injury to midesophagus by contrast study 15. Blunt injury to aorta with contained rupture 1 cm distal to the left subclavian artery

ANSWERS:

A. B. C. D. E.

A, C, E, B

The choice of incision in the treatment of thoracic trauma is vitally important in achieving adequate exposure to allow for optimal results.

chest CT scan with intravenous contrast. transthoracic echocardiography. transesophageal echocardiography. arch aortography. upright posterior-anterior and lateral chest radiograph.

ANSWER:

The left anterior thoracotomy is the incision of choice in an emergency department resuscitative thoracotomy and is performed in a supine position with the patient7 s arm abducted. Typically, it is begun at the sternal edge in the fourth or fifth intercostal space in a sharp arc to the axilla. This allows adequate access to the heart, lower thoracic aorta, and left lung. It can be performed quickly without specialized instruments. Left anterior thoracotomy does not provide good exposure to the upper chest cavity or mediastinum. It can be easily converted to a bilateral thoracotomy (clamshell) with extension across the lower sternum. The supine position also allows easy access to the abdomen, if necessary.

A

Although most patients with blunt thoracic trauma do not incur aortic injuries, a high index of suspicion must be maintained. The presence of a widened mediastinum on chest radiograph is a sensitive but nonspecific marker of traumatic aortic injury and mandates further investigation to rule out injury. Trauma patients are frequently unable to tolerate repetition of the chest radiograph in the upright posterior-anterior and lateral position, so repeat x-ray is not recommended. Also, given concern for blunt aortic injury, x-ray will not provide any new or additional information. Transthoracic echocardiography is not able to visualize the aorta at the most common site of injury (just distal to left subclavian artery). Initial enthusiasm for transesophageal echocardiography is tempered by concerns about widespread availability and accuracy. Both types of echocardiography are highly dependent on the skills of the sonographer.

A median sternotomy allows the best exposure to the heart and great vessels. It can be extended to a midline laparotomy, if required. Exposure of the lung parenchyma is limited. Median sternotomy is a poor choice for esophageal access. The need for instrumentation to divide the sternum may limit the ability to perform this incision quickly. Morbid wound complications are a risk in emergency situations.

CT scan of the chest with intravenous contrast has supplanted traditional arch aortography for 3 reasons: (1) additional information on associated injuries is available; (2) staff are readily available to perform and interpret the studies; and (3) contrast exposure is lower.

The right posterolateral thoracotomy is performed in a lateral position. It provides access to the right lung, including the hilum, diaphragm, trachea, and mid-esophagus. The lateral position may not be tolerated by hemodynamically compromised patients, due to a reduction in venous return. The access to the opposite thorax and abdomen is very limited. The left posterolateral thoracotomy is performed in a lateral position. It provides exposure to the left lung, including the hilum, the aortic arch, descending thoracic aorta, diaphragm,

134

Items 17-18 A helmeted 14-year-old sustains a hard tackle during football practice and is unconscious for approximately 1 minute. He admits to feeling dizzy after he was helped to the sidelines, but he wants to finish the practice. His Glasgow Coma Scale score is 15.

cervical spine is warranted. Although MRI is more sensitive at detecting complicated traumatic brain injury compared with CT scan, most patients presenting with concussive symptoms do not require neurological imaging studies. If level of consciousness is of concern, the athlete with the concussion should be imaged and observed in a hospital setting; frequent awakening is no longer recommended

17. Which of the following statements is true regarding his care?

Anyone who has sustained a concussive traumatic brain injury has a 2 to 5.8 times higher risk of sustaining another concussion. Recovery is not always quick. Protracted recovery is associated with numerous factors, including previous concussion, the severity of posttraumatic symptoms, female sex, a pre-existing learning disorder, and certain positions of football teams (quarterback, receivers, running backs). Although wearing a helmet decreases severe impact injuries such as skull fracture, most helmets currently in use do not reduce the incidence or severity of concussion. Catastrophic traumatic brain injury is more likely in younger athletes. One hypothesis is that the immature brain has less cognitive reserve compared with a more mature brain.

A. Returning to finish the practice is allowable. B. Immediate head CT scan should be performed. C. Sideline evaluation should include new learning and concentration assessments. D. Glasgow Coma Scale is a sensitive indicator of concussion. E. His parents should awaken him every 2 hours for the next 24 hours. 18. Regarding his prognosis, which of the following statements is true? A. Wearing a helmet eliminates the risk of chronic headache. B. His risk of sustaining another concussion is increased. C. His recovery would be more rapid than if he were younger. D. His mood will be altered for the next week. E. He can return to normal practice and play in 7 days.

ANSWERS:

Limitations on returning to school are not defined. Students with traumatic brain injury may require academic accommodations for an extended period of time. Alterations in sleep pattern, mood, and headache are common after a concussion and may last weeks or months. Return to play should be individualized, with gradual monitored reinstatement of activity. If symptoms recur or increase with activity level, activity progression should cease until symptoms resolve.

C, B

Sports-related concussions affect nearly 4 million athletes per year. Because most symptoms of neurologic dysfunction resolve within 7 to 10 days, pressure to allow young athletes to return to play early is substantial. However, symptom resolution alone is not indicative of complete recovery from a traumatic brain event. Current standards of care are based on a paradigm of rest and gradual return to play. The final decision regarding returning to play depends on findings related to cognitive and physical assessments. The recommendations make it clear that when a potential concussion is diagnosed, the athlete should not return to play on the same day.

19. A 35-year-old woman was in a head-on motor vehicle collision. She is hemodynamically normal but requires intubation due to a decrease in mental status. Before intubation, she was moving all 4 extremities. She had a cervical collar placed by prehospital personnel. The earliest her cervical collar may be removed is A. when she is able to participate in a thorough neurologic exam. B. after obtaining normal plain films (3 views) of her cervical spine. C. after obtaining a normal CT scan of her cervical spine. D. after obtaining a normal MRI of her cervical spine. E. immediately because no cervical collar is required.

Ideally, sideline evaluation after a sports-related loss of consciousness should include an assessment of vision changes with oculomotor testing, balance and postural stability, and mental status questions specific to situation, including new learning (e.g., word recall, listing months of the year backward), and concentration assessments. Standardized concussion and balance scoring systems, when combined, increase the sensitivity and specificity of a concussion diagnosis. A Glasgow Coma Scale of 15 is reassuring with respect to general verbal, motor, and pain responses but is insufficient to rule out a concussive event. Skull fractures, epidural or subdural hematomas, and cerebral edema are rare in this setting, although palpation of the

ANSWER:

C

In the awake and alert blunt trauma victim, the cervical spine can be evaluated with a thorough physical exam followed by a CT scan, if necessary. Both the National Emergency XRadiography Utilization Study (NEXUS) and Canadian Cervical Spine Rules (CCR) are acceptable methods of clearing a cervical spine clinically. Both rules require the patient to have no neurologic defects, midline cervical

135

tenderness or pain with active motion, distracting injuries, or altered mental status. If any of these are present, dedicated axial imaging is recommended. The CCR also recommends a CT scan for certain high-risk mechanisms or populations. Axial imaging has completely replaced plain x-rays because of its substantially higher sensitivity and specificity for injuries.

on parenchymal injury and ductal injury (table 20.1). Essentially, grade I and II injuries spare the duct, grade HI and IV involve the main pancreatic duct, and grade V injuries represent a massive disruption of the head of the pancreas. Type-of Grade* Injury I Hematoma Laceration II Hematoma

The obtunded population presents a different diagnostic dilemma. The risk of a missed cervical spine injury must be weighed against the potential complications of cervical spine immobilization, including increases in intracranial pressure related to decreased internal jugular venous return and occipital and submental decubiti. Until recently, axial imaging was considered insufficient to clear the cervical spine in the obtunded patient. This was due to the risk, albeit very low, of a clinically significant missed injury. Several studies quoted missed injury rates (with normal CT imaging) of approximately 3 in 1000. It was unclear what percentage of these injuries were clinically significant. A recent review by the Eastern Association for the Surgery of Trauma reversed this recommendation based on the nearly 100% negative predictive value of CT imaging and the increased risks and cost added by adding MRI to the diagnostic workup of patients with a normal CT scan. The authors noted that this recommendation may result in a "non-zero" rate of clinically significant injury.

Laceration Laceration

IV

Laceration

V

Laceration

*Advance one grade for multiple injuries up to grade III. Proximal pancreas is to the patients right of the superior mesenteric vein. Table 20.1. Pancreatic injury scale. In hemodynamically normal patients, CT imaging is the diagnostic modality of choice. Modem axial imaging can detect pancreatic injuries with a sensitivity and specificity of nearly 50% and 80%, respectively. Current, 64-slice multidetector CT scanners can detect injuries to the pancreatic duct with very high rates of sensitivity and specificity.

To be considered a diagnostically appropriate CT of the cervical spine, the study must have slices from the skull-base to the first thoracic vertebrae with sagittal and coronal reconstructions.

Grade I and II injuries diagnosed on CT scan can be safely managed nonoperatively. Several studies note very low pancreas-related morbidity from observed, low-grade injuries. In studies where grade I and II injuries were explored after axial imaging, CT scan was 91% sensitive and specific for identification of duct injury. If the pancreatic duct is not well visualized on initial imaging, additional imaging such as dedicated pancreas CT scan, magnetic resonance cholangiopancreatography, or endoscopic retrograde cholangiopancreatography (ERCP) is recommended before exploration based on initial findings alone. For grade I or II injuries found at the time of exploration, nonresectional management with or without a closed suction drain is recommended.

20. A 25-year-old man presents to the trauma bay after crashing his dirt bike. He is hemodynamically normal, and his only complaint is moderate abdominal pain. Axial imaging reveals an injury to the mid body of the pancreas. Which of the following statements is true regarding the management of blunt pancreatic injuries? A. Grade III and IV injuries encountered during exploration should undergo resection. B. Grade I and II injuries should be routinely explored. C. A draining Roux-en-Y limb should be sewn to the pancreatic margin after resection. D. Grade III and IV injuries encountered on imaging are managed with observation. E. Octreotide should be routinely administered after distal pancreatectomy.

ANSWER:

III

Description Minor contusion without duct injury Superficial laceration without duct injury Major contusion without duct injury or tissue loss Major laceration without duct injury or tissue loss Distal transection or parenchymal injury with duct injury Proximal transection or parenchymal injury involving ampulla Massive disruption of pancreatic head

Based on recommendations from the Eastern Association for the Surgery of Trauma (EAST) group, grade HI and IV injuries diagnosed on axial imaging warrant surgical exploration. The alternative to surgical exploration in the setting of a presumed duct injury is ERCP with placement of a pancreatic stent. The results from nonoperative management were not statistically worse on any single outcome studied in the EAST guidelines; however, based on a cumulative trend toward worse outcomes, the conditional recommendation of surgical resection was made. Part of the basis for this recommendation was the high rate of treatment failures in the nonoperative group leading to delays in definitive management and increased patient morbidity.

A

Pancreatic injuries as a result of abdominal trauma, while rare, can lead to significant morbidity and mortality if not managed appropriately. The American Association for the Surgery of Trauma grading scale classifies the injuries both

136



For grade III and IV injuries found at the time of surgical exploration, resection is recommended. Injuries to title left of the superior mesenteric vein should undergo a distal pancreatectomy. Stump closure with a draining roux limb does not improve leak rates. Closure techniques such as simple duct ligation with mattress suture of the pancreatic body have equivalent results. The routine use of octreotide postoperatively does not reduce fistula rates and is not recommended. Injuries to the right of the mesenteric vein are more complex, and surgical resection will be based on injuries to the surrounding structures.

A 24- to 48-hour period of monitoring is suggested. The American Burn Association formula for fluid resuscitation for a patient with more than 20% total body surface area (TBSA) recommends starting with 2 to 4 mL/kg/TBSA. This fluid resuscitation should not include albumin. Patients with a high-voltage injury may present with myoglobinuria and will require additional fluid resuscitation, but a routine increase in fluids is not indicated. Immediate systemic antibiotic therapy is not indicated. Electrical injury patients with persistent myoglobinuria or extremity compartment syndrome should be evaluated for fasciotomy and muscle debridement.

21. A 100 kg 23-year-old man was working on an outlet inside his house when a flash fire caught his shirt on fire. The patient developed 7% total body surface area second-degree burns to his hands, arms, and chest. Which of the following statements about treatment for his bums is true?

22. A 13-year-old boy was involved in a head-on collision. He was restrained in the back seat with a lap belt. He was evaluated in the emergency department, and his workup was unremarkable other than mild tenderness and a small bruise from the seat belt across his upper abdomen. The next day, he has nausea, vomiting, and mild abdominal pain. CT scan revealed complete bowel obstruction at the second and third portion of the duodenum. In addition to a nasogastric tube, what is the most appropriate next treatment for this patient?

A. A prophylactic bilateral escharotomy of the hands and arms should be performed. B. An electrocardiogram is not necessary. C. His fluid resuscitation amount should be increased by 25%. D. Immediate fluid resuscitation should start with albumin. E. Systemic antibiotics are not recommended.

ANSWER:

A. B. C. D. E.

E

It is important to classify electrical injuries based on the voltage. Low-voltage injuries are less than 1000 V; highvoltage injuries are 1000 V and higher. Low-voltage bums are generally localized. In the United States, domestic wiring operates on an alternating current at 120 V. Therefore, this bum is classified as a low-voltage burn. Low-voltage burns are usually localized to the point of contact. If contact with the low-voltage source is prolonged, deep tissue damage occurs with little lateral extension.

Urgent laparotomy Trickle gastric feeds Laparoscopy Parenteral nutrition Surgical jejunostomy

ANSWER:

D

Duodenal injuries are uncommon, partly because of the duodenum's protected location in the retroperitoneum. A hemodynamically normal patient with a CT scan showing a duodenal injury should be further evaluated. Laparoscopy is not recommended because a full Kocher maneuver must be performed, or a small laceration may be missed.

High-voltage bums are associated with deep extension and underlying tissue damage similar to a crush injury. Because of this, these patients are at risk for the development of compartment syndrome during the first 48 hours after an injury. High-voltage bums are more common in industrial settings and not in homes. With high-voltage injuries, escharotomy is not sufficient. A fasciotomy is required for compartment syndrome treatment.

Grade I and grade II hematomas are diagnosed by CT scan, and initial management should be expectant with nasogastric decompression and oral intake withheld. A hematoma may progress to obstruction. The period for development of this obstruction varies from hours to days. Nonoperative management is appropriate for up to 14 days; therefore, urgent laparotomy is not indicated in this hemodynamically normal patient. After 14 days, operative management should be performed, which includes drainage of the hematoma and simple repair.

Cardiac abnormalities occur after both low- and high-voltage injuries Electrocardiogram should be part of the initial evaluation in all patients. Prolonged cardiac monitoring is not usually necessary but suggested when the following are present: • • •

Cardiopulmonary resuscitation in the field

Early enteral feeding has benefits, and a jejunostomy is preferred. Trickle gastric feeds are not appropriate due to the obstruction. In the absence of a jejunostomy, parenteral nutrition should be started. Parenteral nutrition should be continued until the obstruction resolves, thus allowing for

Loss of consciousness Electrocardiogram abnormality or evidence of ischemia Documented dysrhythmia before or after admission to the emergency department

137

enteral feeding by mouth. If the obstruction persists beyond 14 days, a surgical jejunostomy is appropriate.

Lateral compression (LC): anterior injury-rami fractures (a) LC I: sacral fracture on side of impact (b) LC II: crescent fracture on side of Impact (c) LC III: type I or II injury on side of impact with contralateral open-book injury

23. A 74-year-old man is involved in a high-speed motor vehicle collision. After initial resuscitation, he is hemodynamically normal and the only abnormality identified on evaluation is a pelvic fracture (figure 23.1). What is the next step? A. B. C. D. E.

Anterior-posterior compression (APC): anterior injury = symphysis diastasis/rami fractures (a) APC I: minor opening of symphysis and sacroiliac (SI) joint anteriorly (b) APC II: opening of anterior SI, intact posterior SI ligaments (c) APC III: complete disruption of SI joint

External pelvic fixation CT scan with intravenous contrast Angiographic embolization Exploratory laparotomy Placement of pneumatic antishock garments

Vertical shear (VS type) Vertical displacement of hemipelvis with symphysis diastasis or rami fractures anteriorly, iliac wing, sacral facture or SI dislocation posteriorly Combination (CM type): any combination of above injuries

Table 23.1. Young-Burgess classification of pelvic fractures. After initial evaluation, in hemodynamically normal patients, a CT scan with intravenous contrast of the abdomen and pelvis should be obtained. If there is no evidence of active bleeding, patients who continue to be hemodynamically normal should be treated with pelvic stabilization. As long as they remain hemodynamically normal, these patients do not require angiographic embolization, exploratory laparotomy, external pelvic fixation, or pneumatic antishock garments.

Figure 23.1.

ANSWER:

24. A 19-year-old man was intubated in the field. He arrives at the emergency department with a single gunshot wound to the chest just medial to the left nipple. He is unresponsive and has bilateral breath sounds, a heart rate of 140 beats per minute, and no detectable blod pressure. A focused assessment with sonography for trauma exam demonstrates a large hemopericardium. What is the best next step in the management of this patient?

B

Pelvis injuries from high-energy trauma are frequently associated with concomitant internal injuries, including hemorrhage, intra-abdominal injuries, bladder/urethral injuries, and nerve deficits secondary to disruption of the pubic ring.

A. B. C. D. E.

After consideration to airway, breathing, and circulation, initial inspection for concern of a pelvic injury should include a search for external bleeding, blood at the penile meatus/vagina, and the position of the lower extremities and iliac crests. A common classification system for pelvic fractures is the Young-Burgees classification (table 23.1).

Pericardiocentesis Resuscitative left anterolateral thoracotomy Subxiphoid pericardial window Median sternotomy Resuscitative endovascular balloon occlusion of the aorta

ANSWER:

A

This patient presents in extremis after a gunshot wound in the "cardiac box." A focused assessment with sonography for trauma exam demonstrates a hemopericardium leading to a diagnosis of pericardial tamponade, which is consistent with the clinical picture. This patient has a penetrating cardiac injury that needs to be addressed immediately to prevent complete cardiac arrest. A left anterolateral thoracotomy will allow for rapid and adequate exposure of the heart and

138

pericardium to allow decompression and repair of any injury. A median sternotomy provides excellent exposure for this kind of injury but requires equipment that may not be immediately available in an emergency department and takes longer to perform than a thoracotomy.

The ankle-brachial index (ABI) is an important part of the physical examination in patients with knee dislocations. Abnormal ABI values (<0.9) help identify patients with vascular injury and can obviate routine angiography in all patients with a history of knee dislocations and ligamentous injury. An angiogram is the gold-standard to help identify peripheral vascular injury, but it is invasive and involves the use of nephrotoxic contrast agents. In patients with palpable pulses and no hard or softs signs of vascular compromise, obtaining an ABI before committing to angiographic imaging is prudent.

A pericardiocentesis in this scenario will result in an excessive amount of time lost. Moreover, it is diagnostic only and still requires an urgent operation to fix the cardiac injury. Similarly, a pericardial window is a reasonable diagnostic procedure if the patient already had a laparotomy ongoing and there is suspicion for pericardial tamponade. In this scenario, the diagnosis is clear and diagnostic procedures will delay definitive management and potentially lead to patient mortality.

Routine surgical exploration of patients with knee dislocations is not indicated when there is no evidence of impending vascular compromise.

Resuscitative endovascular balloon occlusion of the aorta (REBOA) involves access through a patient's femoral artery and placement of an occlusive balloon in the aorta to stem bleeding distal to the occlusion. Chest trauma that could result in aortic or cardiac injury is a contraindication for the use of REBOA because it may exacerbate the injury.

26. A 25-year-old woman with a gunshot wound to her abdomen and a 4-hour transport time arrives with a temperature of 35°C. She has a blood pressure of 80/40 mm Hg and a heart rate of 140 beats per minute. As you prepare for surgery, which of the following statements is true regarding treatment of her hypothermia?

25. A 23-year-old man arrives awake and alert after a singlevehicle motorcycle crash. He complains of severe right knee pain associated with edema around the knee joint. X-rays demonstrate no evidence of a fracture. Pedal pulses are palpable and symmetric. The next step in management is A. B. C. D. E.

A. Warm insulating blankets and a warming gown device prevent further heat loss. B. Blood products should not be heated to more than 30°C. C. Peritoneal lavage with warm saline will normalize her core temperature D. Correcting the blood loss does little to treat the hypothermia. E. Airway rewarming effectively raises core temperature.

continuous leg compartment pressure measurement. arterial vascular duplex the next day. ankle-brachial index. right lower extremity angiography. surgical exploration of the popliteal artery.

ANSWER: ANSWER:

C

A

Hypothermia (<35°C) is an independent predictor of mortality. Hypothermia after traumatic injury results from blood loss and is compounded by a cold external environment at the time of injury and during the phases of treatment. Warming the local environment—the trauma bay or the operating room—may help to prevent further heat loss but will not correct the hypothermia. Similarly, warm insulating blankets and wanning gown devices will help prevent further heat loss. Correcting the blood loss with transfusion and stopping the bleeding will correct the hypothermia. Peritoneal lavage with warm saline and airway rewarming will not increase the core temperature of a patient sufficiently to treat hypothermia.

Severe knee pain, edema, and the lack of a fracture raise the possibility of significant ligamentous instability and a history of knee dislocation. Knee dislocations are notorious for causing popliteal artery injuries, which need to be ruled out early in the evaluation to prevent the potential for irreversible vascular compromise. Palpable pulses indicate that distal perfusion is maintained and no acute ischemia is present; however, they do exclude the presence of a clinically significant popliteal injury. The measurement of compartment pressures is relevant when there is high suspicion for the development of leg compartment syndrome, such as with high-grade tibial fractures. When elevated compartment pressures are found, early fasciotomies are prudent. Continuous measurement of compartment pressures will not help in the diagnosis of a popliteal vascular injury.

Transfused blood products should be warmed to a normal body temperature before transfusion, because transfusing cold products will decrease the core temperature. Packed red blood cells are usually refrigerated at 20 to 25°C and fresh frozen plasma is stored at 0 to 5°C. Rapid transfusers can heat to 37°C and transfuse a whole unit within a few minutes. Rewarming is imperative in traumatically injured patients.

A vascular duplex study can identify flow-limiting lesions. Obtaining the study 24 hours after admission is too late and may result in significant ischemia if an injury is missed.

139

27. A 24-year-old man presents to the emergency department after a motor vehicle. collision. There is a spider-web pattern on his side of the windshield, but he denies loss of consciousness. He follows commands, his eyes spontaneously open, and he speaks in clear but nonsense phrases. His primary and secondary surveys are otherwise negative. A CT scan of his head is normal. After a few hours, his Glasgow Coma Scale score (GCS) is 15. With regard to management this patient, which of the following statements is true? A. B. C. D. E.

He is at no risk for future neurologic symptoms. He is still at significant risk for sudden death. Seizure prophylaxis is indicated for 3 months. Long-term effects can include cognitive changes. Symptoms rarely occur if GCS is 15.

ANSWER:

Figure 28.1.

D

This patient has a mild traumatic brain injury or a concussion. A concussion does not require the patient to have lost consciousness. The fact that his Glasgow Coma Scale score returned to normal quickly and his head CT scan was normal should not reassure the clinician that no traumatic brain injury exists. Although he is not at risk for sudden death due an intracranial hemorrhage, he is still at significant risk for morbidity and future symptoms. This does not include seizures, however, so seizure prophylaxis is not indicated. His symptoms are more likely to present as headaches, photophobia, difficulty concentrating, amnesia, tinnitus, and gait instability. These symptoms are not always self-limited. All patients who experienced a traumatic brain injury should be referred to someone who treats these injuries to ensure there are no further symptoms. Symptomatic patients should undergo cognitive evaluation and probable neuropsychological rehabilitation. Left untreated, these subtle symptoms may later manifest as cognitive impairments and mood disorders that may result in job loss, divorce, and social isolation. Additionally, these patients should refrain from contact activities until cleared by healthcare providers.

Figure 28.2.

28. A 21 -year-old woman presents after a motor vehicle crash at high rate of speed. She was found unresponsive at the scene with agonal breathing and was intubated. She has significant facial and head trauma and abdominal road rash. Her chest x-ray was normal. She is hemodynamically normal. Focused assessment with sonography for trauma (FAST) exam reveals no hemoperitoneum. Diagnostic CT scan is obtained (figures.28.1and28.2). The next step in management is A. repeat FAST. B. diagnostic peritoneal lavage. C. angiographic embolization. D. diagnostic laparoscopy. E. laparotomy.

140

ANSWER:

related to the suspected bowel injury. Urgent laparotomy is indicated from these findings.

E

Gastrointestinal hollow viscus injury after blunt chest and abdominal trauma is not common (0.6-1.2% of all trauma cases). Early recognition of these injuries significantly reduces morbidity and mortality.

Repeat focused assessment with sonography for trauma examination will not be helpful, because it is effective in identifying hemoperitoneum only. Diagnostic peritoneal lavage is not indicated, because the CT scan findings confirm hollow viscous injury. Angiographic embolization is indicated only for hemorrhage related to solid organ injury. Diagnostic laparoscopy would be helpful if the CT scan findings were equivocal, but in this case, the CT scan findings have confirmed bowel injury.

Physical exam is not accurate in detecting these injuries, and contrast-enhanced CT scan is the mainstay for diagnosis. CT scan findings in this patient include (1) several foci of extraluminal gas within the right hemiabdomen and pelvis, which is relatively confluent anterior to the liver (figure 28.3); (2) small to moderate volume free abdominal fluid, some of which is high attenuation (figure 28.4); and (3) no evidence of solid organ injury.

Figure 28.3. Pneumoperitoneum (arrows).

Figure 28.4. Free fluid. These findings are concerning for bowel injury, with suspicion directed toward the right colon. Small-volume high-attenuation free fluid surrounds the liver dome. No discrete liver laceration is seen, and the free fluid may be

141

Vascular

ITEMS 1-24 For each question, select the best possible response.

ANSWER:

1. When placing a port for outpatient chemotherapy, what decreases the risk of inadvertent arterial placement?

This patient is presenting with chronic mesenteric ischemia. Chronic mesenteric ischemia is a disorder in which blood flow to the intestines is inadequate. The main etiology of chronic mesenteric ischemia is atherosclerosis of the celiac, superior mesenteric, or inferior mesenteric arteries. Other rare causes of chronic mesenteric ischemia include median arcuate ligament syndrome, vasculitis, or fibromuscular dysplasia. Nonocclusive mesenteric ischemia is a form of chronic mesenteric ischemia that occurs in patients with a low flow state. There is a female preponderance for chronic mesenteric ischemia.

A. B. C. D. E.

Internal jugular access Ultrasound guidance Fine-gauge needle for initial access Steep Trendelenburg positioning Neutral neck position

ANSWER:

B

The most important finding in chronic mesenteric ischemia is that of postprandial pain, usually occurring after a meal and lasting anywhere from 90 minutes to 2 hours. Fear of eating is a result of this pain and leads to patients' eating smaller meals or missing meals. This leads to weight loss. Patients may also present with diarrhea. Once a patient reaches endstage chronic mesenteric ischemia, pain patterns may change, with abdominal pain being more constant. This is due to blood flow not being able to meet even the most minimal metabolic demand. Lastly, chronic mesenteric ischemia can present with ulcers in the stomach. Elderly patients with ulcers where Helicobacter pylori is negative and there is no history of NSAID use should undergo screening for chronic mesenteric ischemia.

Arterial puncture is more likely to occur with an internal jugular rather than a subclavian approach (3% vs 0.5%). Subclavian access is more likely to be complicated by pneumothorax or hemothorax. Real-time ultrasound prevents arterial injury in the internal jugular position.It reduces, but does not completely eliminate, the risk of complications (from 8.4% to 1.4%) Experience also reduces the risk of complications. Catheter insertion by a physician with more than 50 prior insertions compared with a surgeon who has performed fewer than 50 is half as likely to result in a mechanical complication. More than 3 insertion attempts during one procedure is 6 times more likely to lead to a complication than when venous cannulation is achieved on the first attempt. After 3 attempts, the surgeon should either abort the procedure or ask for assistance.

The diagnosis of chronic mesenteric ischemia is first and foremost clinical. Surgeons need to be attuned to the diagnosis. Duplex ultrasound is a good screening test, because it does not involve radiation. Unfortunately, it is operator dependent and thus its use is limited in regions where experience with ultrasound is not high. CT angiography is the diagnostic test of choice for chronic mesenteric ischemia. It not only provides 3-dimensional imaging of the vasculature of the abdomen but also can rule out other causes of abdominal pain.

Pressure monitoring, with either column manometry or pressure transducer before placement of dilator, potentially prevents arterial injury. Neck positioning is dependent on the chosen access site. Generally, it is best to place a roll behind the shoulders. Fine gauge needle use and steep Trendelenburg positioning are also good practices and facilitate the procedure, but neither of these practices definitively reduces the risk of inadvertent arterial catheterization.

Hepatobiliary scanning is typically used for biliary disease. The patient in the scenario does not present with typical biliary symptoms. Upper endoscopy and colonoscopy could be considered. However, in this patient with postprandial pain, diarrhea, and weight loss, findings on these tests would only lead to confusion as to the diagnosis and would delay treatment of the primary etiology. There is no indication for diagnostic laparoscopy in this patient.

2. A 77-year-old woman presents with 6 months of progressive abdominal pain and weight loss, despite a normal appetite. She reports epigastric and periumbilical pain approximately 1 hour after eating, with frequent associated diarrhea. Her physical exam and abdominal ultrasound were unremarkable. Which of the following would be the next step in her evaluation? A. B. C. D. E.

C

3. A 25-year-old man is shot in the right flank. At the time of laparotomy, a right medial visceral rotation is performed. A nonpulsatile, but large and expanding, retroperitoneal hematoma is visualized. The hematoma is opened, and the site of bleeding is identified as a laceration of the infrarenal inferior vena cava involving 75% of its circumference. The patient's blood pressure is 80/40 mm Hg, his pulse is 115

Hepatobiliary scan Upper endoscopy CT angiogram of the abdomen Colonoscopy Diagnostic laparoscopy

143

beats per minute, and his hematocrit is 21% (45-52%). The best next step in the management of this patient is A. B. C. D. E.

when simple ligation of the IVC would save the patient's life. Similarly, although a spiraled saphenous vein graft is a potential surgical option for repair of the IVC, it is appropriate only in the elective setting, owing to the time required to create and place a spiraled saphenous vein interposition graft.

packing of the retroperitoneal hematoma. ligation of the inferior vena cava. suture repair of the inferior vena cava. a spiraled saphenous vein interposition graft an endovascular stent graft.

ANSWER:

4. A 70-year-old man has a 4.0-cm abdominal aortic aneurysm on a surveillance ultrasound. He is asymptomatic. He is advised to undergo yearly surveillance ultrasound imaging. Referral for repair of his abdominal aortic aneurysm should occur when the diameter of his abdominaI aortic aneurysm is

B

The acute management of penetrating injuries to the inferior vena cava (IVC) should be guided by both the clinical status of the patient and the extent of the injury to the IVC. In the clinical scenario presented, the patient is hypotensive, tachycardic, and anemic, indicating significant volume depletion as the result of acute blood loss. Once a patient is in shock from a penetrating injury to the IVC, progressive acidosis, hypothermia, and coagulopathy will occur unless the hemorrhage can be rapidly controlled. Undertaking complex surgical repairs when a patient is in shock due to ongoing hemorrhage is not advisable. Given the finding of a laceration involving 75% of its circumference, the best surgical intervention for this patient is ligation of the IVC.

A. B. C. D. E.

4.5 cm. 4.8 cm. 5.1 cm. 5.4 cm. 5.7 cm.

ANSWER:

E

The size at which an abdominal aortic aneurysm should be repaired was studied in multiple randomized clinical trials. These trials all used surveillance imaging with abdominal ultrasound to detect the presence of an abdominal aortic aneurysm in patients and then followed these patients with regular scheduled ultrasound scanning over time. These trials, known as the UK Small Aneurysm Trial (UKSAT), the Aneurysm Detection and Management trial (ADAM), the Comparison of Surveillance versus Aortic Endografting for Small Aneurysm Repair trial (CAESAR), and the Positive Impact of Endovascular Options for Treating Aneurysms early trial (PIVOTAL), all concluded that close observation with periodic ultrasound screening is as safe as either open repair or endovascular aortic reconstruction, as long as the abdominal aortic aneurysm is less than 5.5 cm in diameter.

Ligation of the significantly injured infrarenal IVC is an accepted practice in the setting of damage control laparotomy, with a reported survival rate of 40%. The most common morbidity of ligation of the infrarenal IVC is swelling of the lower extremities. In some patients, this swelling can be severe enough to cause an acute compartment syndrome. In most patients, however, ligation of the IVC is tolerated well. On long-term follow-up, lower extremity edema or dysfunction is uncommon in patients who have undergone emergency ligation of the IVC. Concerns regarding the potential long-term consequences of ligation of the IVC should not override the need to perform the procedure when it is necessary to save a patients life. In this setting, with the finding of a laceration of the infrarenal IVC involving 75% of its circumference, packing of the retroperitoneal hematoma will not reliably control the patients hemorrhage.

The results of these 4 trials demonstrated no advantage to early repair either via open surgery or endovascular surgery for smaller abdominal aortic aneurysms, defined as those between 4.0 and 5.5 cm in size. All of the available evidence suggests that the optimal care for these patients is routine monitoring with ultrasound surveillance. More recent clinical trials focused on the efficacy of endovascular aneurysm repair for smaller aortic aneurysms have not shown a benefit. Neither open nor endovascular repair of an abdominal aortic aneurysm should be undertaken until the aneurysm is more than 5.5 cm in diameter.

Although an endovascular stent graft may be an attractive option in some patients, in the acute setting where the hematoma has been opened and the patient is in shock, there is not sufficient time to place an endovascular stent graft. An endovascular repair is more appropriate when an injury to the IVC is identified preoperatively and the patient has been adequately resuscitated. Direct suture repair of the IVC should be reserved for patients with lacerations that are less than 50% of the circumference of the IVC, who are not actively in shock, and in whom an expeditious suture repair of the IVC can be undertaken.

5. A 64-year-old man has a medical history of hypertension, diabetes, coronary artery disease, and aortic stenosis. He has undergone coronary artery bypass grafting and aortic valve replacement. He presents with swelling that involves just his

Complex repairs of the IVC, including the placement of venous patches or interposition grafts, should be avoided

144

third toe with associated plantar ulcer (figure 5.1). In addition to antibiotics, what is the next step in his management? A. B. C. D. E.

presents with exposed metatarsal and thus carries a diagnosis of osteomyelitis. For this reason, incision and drainage, debridement, and hyperbaric therapy would not be indicated. This patient requires amputation for source control of his infection. Because only one toe is infected, a transmetatarsal amputation would be too extensive, whereas a ray amputation would allow source control and preserve tissue.

Incision and drainage with alginate dressing changes Debridement with negative pressure wound therapy Hyperbaric therapy Transmetatarsal amputation Ray amputation

Items 6-7 A 65-year-old man undergoes laparoscopic right adrenalectomy for an enlarging mass. He is managed with appropriate venous thromboembolism prophylaxis. On postoperative day 2, he develops new-onset right calf swelling. Pathology of the adrenal mass is an adenoma. Duplex studies demonstrate acute obstruction in the right soleal vein. He is otherwise asymptomatic. 6. How should this patient's clinical condition be managed? A. B. C. D. E.

Vitamin K antagonist therapy for 30 days Observation with repeat duplex study in 2 weeks Catheter-directed thrombolysis of the lower extremity Placement of an inferior vena cava filter Low molecular weight heparin for 30 days

7. On the patient's fourth postoperative day, he develops hypoxia while remaining normotensive. A CT scan (figure 7.1) is obtained. Which of the following should the patient's management include? A. B. C. D. E.

Figure 5.1.

ANSWER:

E

Placement of an inferior vena cava filter Anticoagulation therapy for 3 months Systemic thrombolytic therapy Pulmonary thromboendarterectomy Aspirin therapy initiation

This picture is consistent with a diabetic foot infection (figure 5.1). Key point in the evaluation of the patient include extent of the infection, underlying factors that may have led to the infection, and microbes that are causing the current infection. Management of diabetic foot infections involves wound care, antibiotics, and surgery. Wound care consists of debridement of necrotic tissue, dressing changes, and elimination of pressure. Most diabetic foot infections seen by surgeons are polymicrobial. Thus, when deciding on antibiotics, broadspectrum coverage should be initiated. On examination, the surgeon needs to assess the extent of infection and whether osteomyelitis is present. Evidence of the diagnosis of osteomyelitis includes grossly visible bone, the ability to probe bone, ulcer duration longer than 1 to 2 weeks, and ulcer size greater than 2 cm2. If osteomyelitis is suspected, the diagnosis can be confirmed with imaging, either plain film or MRI.

Figure 7.1.

In the scenario presented, the patient is higher risk for endocarditis due to his aortic valve replacement. Thus, source control needs to occur quickly. In addition, the patient

ANSWERS: 145

B, B

The diagnosis of venous thromboembolic events and their management are re-evaluated on a recurring basis, and the American College of Chest Physicians provides clinical practice guideline updates based on these reviews. One of the challenges involves the role of whole-leg ultrasound examinations (including distal veins) in patients with suspected deep vein thrombosis (DVT). The current guidelines discourage routine whole-leg ultrasound examinations in patients with suspected DVT, which has reduced the frequency of diagnosing isolated distal DVT.

8. Which of the following statements regarding the use of temporary arterial shunts in trauma is true?

The rationale is that other assessments may already indicate that isolated distal DVT are either unlikely to be present or unlikely to cause complications. Approximately 15% of untreated isolated distal DVT are expected to subsequently extend into the popliteal vein and may cause pulmonary embolism. Therefore, it is unacceptable to neither anticoagulate nor perform surveillance to detect thrombus extension. Efficacy of anticoagulant therapy exists for the treatment of proximal DVT and pulmonary embolism. Whether the benefits of anticoagulation outweigh its risks in patients with isolated distal DVT is uncertain because of their lower risk of progressive or recurrent venous thromboembolism. In patients with acute proximal DVT of the leg, the recommendation is for anticoagulant therapy alone over catheter-directed thrombolysis and the recommendation is against the use of an inferior vena cava filter. These recommendations extend to patients presenting with distal DVTs. In patients requiring anticoagulation for treatment of DVT of the leg or pulmonary embolism and no history of cancer, 3 months anticoagulant therapy is recommended with dabigatran, rivaroxaban, apixaban, or edoxaban over Vitamin K antagonist therapy (figure 7.2).

ANSWER:

A. B. C. D. E.

Systemic anticoagulation is required. The most common site is in the thorax. Optimal shunt time is less than 6 hours. Blunt traumatic injuries are the most common indication. Shunt-related complications occur more frequently in torso injuries than extremity injuries.

C

Vascular injury during trauma can be treated by ligation, embolization, stenting, direct repair, or vascular reconstruction using autologous or prosthetic materials. Occasionally, temporary intravascular shunts will be required in the damage control setting or during staged orthopedic procedures to temporarily re-establish blood flow (figure 8.1).

Figure 8.1. Vascular shunt. Temporary arterial shunts allow for rapid control of hemorrhage and re-establishment of distal circulation much more quickly than a complex arterial reconstruction. This can allow the surgeon to address other associated injuries and get the patient to the intensive care unit quickly to correct hypothermia, coagulopathy, and acidosis. Later, when the patient has been resuscitated, definitive arterial reconstruction can be performed. In the setting of concomitant vascular injury and complex orthopedic injuries, temporary intravascular shunts can allow for rapid reestablishment of distal perfusion, then orthopedic injuries can be definitively addressed without concern for disrupting a newly fashioned arterial repair (figure 8.2). Afterward, the vascular injuries can be definitively repaired.

Figure 7.2. Deep vein thrombosis of the leg. There is no role for aspirin as an alternative to anticoagulant therapy in the acute treatment of venous thromboembolism or pulmonary embolism. With respect to the role of catheterdirected thrombolytic therapy or pulmonary thromboendarterectomy, the recent recommendation is that few patients with acute pulmonary embolism without hypotension should be treated with thrombolytic therapy.

146

Aortoenteric fistulas are missed on more than 85% of endoscopic evaluations, presumably due to their distal nature, but CT scan has a 93% sensitivity of identifying them. Once identified, swift action should be taken to avoid exsanguination. Historically, outcomes for surgery were poor, but a newer study demonstrated 21% overall mortality at 30 days, with 0% mortality for those with normal blood pressure at the time of surgery compared with 60% of those in hemodynamic shock at presentation.

Figure 8.2. Vascular shunt (plastic tubing) that temporarily connects 2 severed ends of artery. This allows for rapid reperfusion of the distal segment and deferring of complicated repair in an austere forward surgical facility.

Optimal shunt dwell times are not clearly established. No shunt-related complications are reported with dwell times less than 6 hours. However, after 6 hours, shunt thrombosis or dislodgement and distal ischemia are seen more frequently. These shunt-related complications appear to be related to size of the vessel being shunted, with smaller vessels having more complications. Therefore, shunts used in the torso in large vessels tend to have fewer complications than extremity injuries. Systemic anticoagulation does not decrease shunt thrombosis and is not recommended, especially given the high percentage of associated injuries in these patients. The role of local and systemic heparin administration during temporary intravascular shunts placement is controversial. Placement of temporary intravascular shunts is most common in the setting of peripheral arterial injuries with a penetrating mechanism.

Antibiotics and bowel rest are not adequate to prevent hemorrhage and death because they have not removed the fistula or the septic focus. Endoscopic clipping is inadequate to address the infected graft. Although patch angioplasty is a tempting alternative, most experts agree that the entire graft must be explanted to eliminate the infection and prevent farther pseudoaneurysms. After fistula repair and graft excision, the aorta is oversewn, and the distal perfusion is restored with an extra-anatomic bypass to avoid graft contamination. Endoscopic injection of a fibrin sealant, in combination with an endovascular stent, is described for thoracic aortoenteric fistulas. Although such a maneuver is certainly promising, given its less invasive nature, there is not sufficient evidence at this point to support its utility in abdominal aortoenteric fistula.

9. A 78-year-old man presents with a single episode of hematemesis. He has a history of an open abdominal aortic aneurysm repair 4 years ago. Esophagogastroduodenoscopy reveals a dot in the third portion of the duodenum. CT angiography reveals a fistula between the aorta and duodenum. What is the most appropriate treatment?

Endovascular stenting of aortoenteric fistulas has emerged as a reasonable alternative to graft excision and extra-anatomic bypass. Stenting is associated with lower in-hospital mortality, but it does carry a nearly 50% risk of reinfection. Currently, stenting is seen as a bridge to definitive open surgery and is typically used for patients who are hemodynamically abnormal or otherwise unfit for the larger operation.

A. Intravenous antibiotics and bowel rest B. Esophagogastroduodenoscopy with endoscopic dipping of the fistula C. Endoscopic injection of fibrin glue D. Fistula takedown with patch angioplasty E. Fistula takedown with graft removal and extra-anatomic bypass

ANSWER:

10. Among high-risk patients on chemoprophylaxis for venous thromboembolism prevention, the greatest risk factor for failure is

E

Upper gastrointestinal hemorrhage is common in elderly patients. Although these patients often have other potential causes, including peptic ulcer disease and nonsteroidal antiinflammatory drug use, a history of an abdominal aortic aneurysm repair should always raise suspicion for an aortoenteric fistula.

A. B. C. D. E.

Aortoenteric fistulas are more common in patients with an open abdominal aortic aneurysm repair compared with those with an endovascular repair, but they have been described in both groups. They typically occur 1 to 5 years after abdominal aortic aneurysm repair, with the most common etiology being graft infection with an anastomotic pseudoaneurysm that erodes into the adjacent bowel. Fistulas usually involve the proximal aortic anastomosis and the distal duodenum.

cumulative missed doses. trauma. epidural catheter use. male sex. BMI greater than 30.

ANSWER:

A

Venous thromboembolism (VTE) events remain a major cause of potentially preventable morbidity and mortality in high-risk trauma and general surgery patients. However, these patients also pose significant challenges in providing appropriate chemoprophylaxis. Patient characteristics put them at high risk, and they may have relative contraindications to chemoprophylaxis. Interruptions in prophylaxis due to procedures or operations are common.

More than 70% initially present with a "herald bleed" that is followed shortly by high-volume hemorrhage and death.

147

A review of more than 200 high-risk trauma and general surgery patients found that only age older than 50 and cumulative missed doses correlated with an increased risk of VTE. Trauma (compared with general surgery), male sex, and obesity were not independent risk factors in this study. Also, epidural catheter use was not associated with increased risk of VTE.

weeks) either while awaiting return of renal function or until permanent access is obtained.

Items 12-13 11. A 63-year-old man presents with severe abdominal pain and bloody diarrhea 2 months after a myocardial infarction treated with coronary stent placement. His blood pressure is 98/60 mm Hg, and his heart rate is 115 beats per minute. An abdominal CT scan with contrast is obtained (figure 12.1).

In an analysis of traumatic brain injury patients, interrupted chemoprophylaxis had the highest odds ratio of predicting VTE compared with other risk factors. Conversely, male sex and BMI were not significant predictors.

11. A 42-year-old woman with stage HI chronic kidney disease (glomerular filtration rate of 32 mL/min/1.73 m2; 90120 mL/min/1.73 m2) and a BMI of 32 was admitted to the intensive care unit with acute oliguric renal failure 3 days after a hysterectomy. She needs vascular access for continuous renal replacement therapy. The most appropriate option is

A. B. C. D. E.

temporary dialysis catheter, right internal jugular vein. temporary dialysis catheter, left subclavian vein. temporary dialysis catheter, right femoral vein. tunneled right subclavian catheter. tunneled left internal jugular catheter.

ANSWER:

A

Most postoperative or post-trauma patients who develop acute kidney injury requiring renal replacement therapy will recover kidney function and not need long-term dialysis. However, this patient has pre-existing stage III chronic kidney disease (glomerular filtration rate of 32 mL/min/1.73 m2; 90-120 mL/min/1.73 m2) and thus would be at very high risk for eventually needing chronic long-term dialysis. Thus, when planning her temporary, acute venous access, it is important to be aware of her long-term needs and the potential complications of short-term venous access.

Figure 12.1. 12. After fluid resuscitaion, the most Appropriate next step in management is A. B. C. D. E.

In general, the preferred vein for short-term dialysis access is the right jugular vein because it gives a straight trajectory. The left internal jugular can also be used, but due to its circuitous route can have more issues with flow. The subclavian veins should be avoided if possible due to concerns of stenosis, especially in patients who are likely to need permanent dialysis access in the arm (either fistula or graft). Femoral access is a reasonable second choice if the jugular is difficult or not available, but this access is associated with higher infection rates (bacteremia, especially in patients with high BMI) and the theoretical risk of iliac vein stenosis complicating renal transplant if that should happen. Tunneled catheters are usually not placed in the acute, intensive care unit setting but are reserved for the subacute phase when a patient needs long-term access (>3

diagnostic laparoscopy. systemic anticoagulation. flexible saigmoidoecopy. upper endoscopy. catheter-based papaverine infusion.

13. After initial management is commenced, the most appropriate next step m management is A. B. C. D. E.

observation. repeat CT scan in 12 hours. flexible sigmoidoscopy. operative embolectomy. mesenteric bypass.

ANSWERS:

148

B, D

This patient has a superior mesenteric artery (SMA) embolus, most likely from a cardiac source (figure 12.2).

Items 14-17 Each lettered response may be selected once, more than once, or not at all. A. B. C. D. E.

Lifelong warfarin therapy 3 months apixaban therapy 4 to 6 weeks prophylactic dose fondaparinux 4 to 6 weeks rivaroxaban therapy Observation

14. Popliteal vein thrombosis in a 52-year-old woman after uncomplicated laparoscopic cholecystectomy 15. Isolated calf vein thrombosis in a healthy 36-year-old man after ankle sprain. 16. A second episode of superficial femoral vein thrombosis in a 62-year-old female breast cancer patient on tamoxifen 17. Superficial thrombophlebitis at the knee in a 56-year-old perimenopausal woman

ANSWER:

B, E, A, C

Historical recommendations for the treatment of venous thromboembolism (VTE) include treatment with parenteral anticoagulation for at least 5 days and transition to maintenance anticoagulation with warfarin. Current guidelines recommend a minimum of 3 months of therapy. A longer duration of treatment (e.g., 6 months) may be associated with a decreased incidence of postthrombotic syndrome. Although Factor Xa direct oral anticoagulants (DOACs) eliminate the need for transitioning with parenteral anticoagulation, the recommendations for duration of therapy remain unchanged. Randomized controlled trials demonstrate equivalent efficacy, and meta-analysis of phase III trials shows DO AC use to be associated with a decreased incidence of recurrent VTE and VTE mortality. For a patient with a postoperative cholecystectomy, 3 months of apixaban is adequate therapy.

Figure 12.2. Superior mesenteric artery embolus. The CT scan demonstrates a large filling defect in the SMA distal to the proximal branches. The initial treatment includes fluid resuscitation and systemic anticoagulation to minimize any future clot propagation. After these treatments have been initiated, expeditious operative embolectomy with restoration of the blood flow to the bowel is indicated. An operative embolectomy is the most appropriate approach to remove the embolus. Bypass of the SMA obstruction is required in SMA thrombosis, but it is not indicated with embolic disease. Catheter-based vasodilation with papaverine may be appropriate in low-flow intestinal ischemia but has no role in occlusive disease.

Isolated calf thrombosis associated with an injury in an ambulatory patient should not receive anticoagulants. In a randomized controlled study of 107 patients, the rate of progression to proximal VTE was low at 3.7% and not significantly different between patients treated with anticoagulation and compression versus compression alone. There were no VTE-associated deaths and no increased incidence of venous recanalization with heparin use. For a 36year-old man with an ankle sprain, NSAID therapy for pain control is sufficient treatment.

The time to restoration of perfusion is paramount. Although assessment of the bowel viability is necessary after reestablishment of blood flow, neither endoscopy or laparoscopy is indicated when there is ongoing ischemia. Determination of the need for resection of bowel is typically done at the completion of the embolectomy and may involve a "second look" laparotomy at 12 to 24 hours after the initial operation.

Recurrent VTE in patients with cancer can be difficult to manage. For the patient with a history of breast cancer on tamoxifen, there is an additional risk of VTE due to tamoxifen in addition to the thrombotic risk accompanying her cancer diagnosis. Current recommendations for treatment are for

149

lifelong anticoagulation unless there are contraindications to pharmacotherapy such as active bleeding. Inferior vena cava filter placement may be considered for these patients.

venous hypertension with development of venous collaterals and eventual endothelial damage that predisposes patients to acute or chronic venous thrombosis. Anatomic compression may occur at numerous sites, and the attendant symptoms are often expressed as venous hypertension distal to the obstructed area. If untreated, venous compression can lead to chronic, symptoms that limit the ability to work and quality of life. Approximately 60% of cases of superior vena cava syndrome are caused by malignancy, whereas intravascular catheters and implanted cardiac devices account for the majority of benign cases. Face and neck swelling, headache, conjunctival edema, dyspnea, and cough are the most common symptoms (figure 18-22.1).

Recurrent VTE in patients with cancer can be difficult to manage. For the patient with a history of breast cancer on tamoxifen, there is an additional risk of VTE due to tamoxifen in addition to the thrombotic risk accompanying her cancer diagnosis. Current recommendations for treatment are for lifelong anticoagulation unless there are contraindications to pharmacotherapy such as active bleeding. Inferior vena cava filter placement may be considered for these patients. A patient with a superficial venous thrombosis does not have a deep venous thrombosis. There are, however, increasing amounts of data documenting that many of these patients will have a concomitant VTE and an increased risk of future VTE. American College of Chest Physicians guidelines from 2012 recommended treatment with fondaparinux for 45 days based on the results of randomized clinical trials comparing anticoagulation with nonsteroidal anti-inflammatory drugs and observation. Reduction in the rate of recurrent superficial venous thrombosis was observed in addition to a reduction in future VTE. Another venous problem is catheter-associated thrombus which is not an uncommon finding in patients with lower extremity central venous access (up to 30%). Current recommendations are for prompt removal of the catheter and treatment similar to other proximal VTE events with 3 months of anticoagulation. Other options for therapy include the use of local thrombolytic therapy for more proximal thrombosis when central venous access will continue to be required. Items 18-22 Each lettered response may be selected once, more than once, or not at all. A. Varicocele B. Varicose veins below the knee C. Unilateral arm pain and swelling D. Iliofemoral deep vein thrombosis E. Headache and conjunctival edema

Figure 18-22.1. Superior vena cava syndrome. Photographs of patient showing reduction in swelling of the face, neck, and upper extremities. (A) At initial presentation and (B) after treatment (hospital day 8).

Treatment options depend on the etiology. Patients with malignant lesions receive radiation with or without chemotherapy. In benign cases, the indwelling device is removed, and the patient is anticoagulated. A recent series of patients with superior vena cava syndrome due to either malignant or benign causes who underwent venoplasty and stenting showed promising outcomes. Chronic compression of the axillary-subclavian vein at the level of the thoracic outlet is referred to as Paget-Schroetter syndrome, also known as effort thrombosis or venous thoracic outlet syndrome (figure 18-22.2).

18. Superior vena cava syndrome 19. Paget-Schroetter syndrome 20. Nutcracker syndrome 21. May-Thurner syndrome 22. Popliteal vein entrapment

ANSWERS:

E, C, A, D, B

Extrinsic compression of veins by adjacent arteries, ligaments, muscles, or bones is most commonly seen in young, healthy individuals. Extrinsic compression also can be caused by adjacent neoplasms and metastatic lesions. Regardless of etiology, venous compression can lead to -

Figure 18-22.2. Paget-Schroetter syndrome, also known as effort thrombosis or venous thoracic outlet syndrome. Opened subclavian vein demonstrating a first rib compression.

150

Compression is most commonly seen in the dominant arm of active patients, and acute thrombosis is often provoked by excessive overhead exercise. The site of compression is at the costoclavicular triangle bordered by the first rib inferiorly, the medial aspect of the clavicle superiorly, the subclavius muscle and costoclavicular ligament medially, and the insertion of the anterior scalene muscle laterally. Muscular or ligamentous hypertrophy, bony abnormalities, or callus formation from prior fractures may lead to narrowing of the costoclavicular space. The most common symptoms are unilateral arm pain and swelling, often accompanied by the development of chest wall venous collaterals (figures 18-22.3 and 18-22.4).

The acute onset of symptoms usually heralds development of axillosubclavian vein thrombus. Treatment involves lysis of the thrombus, followed by surgical decompression of the vein. Compression of the left renal vein between the superior mesenteric artery and the aorta is known as nutcracker syndrome (figure 18-22.5).

Figure 18-22.5. Nutcracker syndrome The syndrome is associated with left flank and abdominal pain, often accompanied by gross or microscopic hematuria. Because the left gonadal vein drains into the left renal vein, men may develop left testicular pain and varicocele formation. Women may present with pelvic pain, dysmenorrhea, dysuria, and dyspareunia. Many treatment options are available, including left renal vein transposition and venous stenting. Patients with minimal symptoms may be treated conservatively, as spontaneous regression has been reported in some cases. Compression of the left common iliac vein between the right common iliac artery and sacral promontory or fifth lumbar vertebra is known as May-Thumer syndrome (figures 18-22.6 and 18-22.7).

Figure 18-22.3. Patient with Paget-Schroetter syndrome showing swelling and discoloration of the right arm and hand at rest.

Figure 18-22.4. Prominent superficial veins over the right upper

Figure.18-22.6. Illustration demonstrating the anatomic

arm and shoulder reflective of Urschel's sign.

compression seen in May-Thurner syndrome.

151

Left iliac vein compression is a common variant, and 50% compression is reported in up to 25% of asymptomatic adults. The syndrome becomes clinically significant when compression results in hemodynamic changes, such as flow reversal or development of varicosities. Chronic compression leads to endothelial damage that predisposes patients to thrombosis. The most common presentation is acute ileofemoral deep vein thrombosis manifest as sudden onset left leg swelling (figure 18-22.8).

Figure 18-22.7. A

Figure 18-22.8. May-Thurner syndrome and iliac arteriovenous fistula in an elderly woman.

In the presence of thrombus, standard therapy is anticoagulation and compression stockings. Currently, many experts advocate catheter-directed thrombolysis of the thrombus followed by placement of a self-expanding stent to relieve the venous compression.

Figure 18-22.7. B. Diagnosis and treatment of May-Thurner syndrome. (A) Infused CT scan shows compression of the left common iliac vein by the right common iliac artery. The right common iliac vain appear enlarged. (B) Left pelvic venogram shows nearly complete obstruction of the left common iliac vein with venous drainage of the left leg occurring through pelvic and paraspinal collateral

Popliteal vein compression may occur in up to 25% of normal adults and is considered to have no pathologic consequence in asymptomatic individuals. The cause of compression is usually an aberrant course of the medial head of the gastrocnemius muscle; other causes include an adjacent popliteal artery aneurysm, popliteal cyst, or popliteus muscle sling (figures 18-22.9a and 18-22.9b).

152

Many patients have associated popliteal artery compression. When the venous compression is hemodynamically significant, affected patients may develop venous thrombosis or varices below the knee (figure 18-22.10). Chronic compression can lead to symptoms of chronic venous insufficiency. Treatment depends on the degree of symptoms: compression stockings for control of edema, anticoagulation for deep vein thrombosis, and surgical decompression if both artery and vein are involved.

23. A43-year-old woman underwent endovenous laser therapy for treatment of symptomatic varicose veins in the right leg. On routine postoperative ultrasound examination 1 week later, she has extension of a saphenous vein thrombus to the saphenofemoral junction (SFJ). Treatment should consist of A. B. C. D. E.

Figure 18-22.9a. Classification of gastrocnemius medial head anomaly. Type 1: an aberrant medial arterial course around normal medial head of gastrocnemius muscle. Type 2: abnormal head of the gastrocnemius muscle which is laterally inserted on the distal femur with medial displacement of popliteal artery. Type 3: an aberrant accessory slip from the medial head of the gastrocnemius muscle wraps around the normally positioned popliteal artery and entraps it.

aspirin. dopidogrel. enoxaparin. warfarin. high ligation of the SFJ.

ANSWER:

A

Minimally invasive techniques using endovenous radiofrequency or endovenous laser ablation have supplanted high ligation and stripping as the preferred method of treating symptomatic varicose veins due to superficial venous reflux. Both techniques involve percutaneous introduction of a radiofrequency or laser catheter into the great or small saphenous vein. Radiofrequency or thermal energy is then delivered throughout the vein treatment length, resulting in endothelial destruction and thrombus formation. The ensuing inflammatory reaction ultimately leads to fibrosis with vein ablation. Venous ultrasound examination of the ipsilateral leg is indicated in the early postoperative period to ensure that the vein is ablated and that the thrombus has not extended into the deep venous system. As experience with these techniques increases, extension of the thrombus from the great saphenous vein into the femoral vein or from the small saphenous vein into the popliteal vein is recognized more frequently. This so-called endothermal heat-induced thrombosis (EHIT) occurs in 1.4 to 5% of patients. Several EHIT classification schemes guide therapy. The 4-level classification system first proposed by Kabnick and colleagues provides a useful framework, as shown in table 23.1

Figure 18-22.9b. Classification of gastrocnemius medial head anomaly. Type 4: the popliteal artery located deep in the popliteus muscle or beneath fibrous bands in the popliteal fossa. Type 5: any form of entrapment that involves the popliteal artery and vein. Type 6: functional type normally positioned popliteal artery which is entrapped by normally positioned gastrocnemius with hypertrophy.

153

Class

Thrombus Location

Treatment

1

Saphenofemoral or saphenopopliteal junction

Aspirin

2

Extension into common femoral or popliteal vein with cross-sectional area <50%

Aspirin

3

Extension into common femoral or popliteal vein with cross-sectional area >50%

Systemic anticoagulation

4

Total occlusion of the common femoral or popliteal vein

Systemic anticoagulation

C. argatroban. D. bivalirudin. E. a retrievable vena cava filter.

ANSWER:

C

Heparin-induced thrombocytopenia (HIT) should be suspected in this patient with a history of heparin exposure and a decreased platelet count This immune-mediated process is a dangerous and potentially lethal complication of unfractionated heparin (UH) and, less frequently, low molecular weight heparin (LMWH). Heparin-induced thrombocytopenia is caused by the formation of antibodies against the heparin-platelet factor 4 (PF4) complex. The antibody binds simultaneously to the heparin PF4 complex and to the Fc platelet receptor, activating platelets and promoting thrombin formation. Thrombosis is the most severe consequence of heparin-induced thrombocytopenia; venous thrombosis is more common than arterial thrombosis.

Table 23.1. Kabnick classification. . Current treatment recommendations are based on EHIT class. Close monitoring with serial ultrasound examinations should be used in all patients diagnosed with EHIT. EHIT classes 2,3, and 4 are deep vein thrombosis (DVT). Two published studies used the following treatment protocol: oral aspirin 81 mg or 325 mg daily for EHIT class 1 or 2 and systemic anticoagulation for EHIT class 3 or 4. Systemic anticoagulation was achieved until the thrombus retracted caudal to the saphenofemoral or saphenopopliteal junction. Using this protocol, 70 patients with EHIT were studied at a single center. No participant with EHIT class 3 or 4 progressed, and 2 of 34 patients (6%) with grade 2 had progression to grade 3 and required anticoagulation. In a study of 201 limbs with EHIT in 194 patients, EHIT resolution occurred in 78% of limbs within 4 weeks and remained as class 1 in 13.5%. Although 4.5% showed progression on serial ultrasound examinations, all subsequently resolved within 4 weeks.

The essential sign of heparin-induced thrombocytopenia can be an absolute drop in platelet count to less than 150,000/mm3 (150,000-400,000/m3) or a relative drop of 30% to 50% from baseline count. The thrombocytopenia typically occurs 5 to 14 days after institution of heparin but can occur sooner if the patient has had a previous heparin exposure. Although thrombocytopenia due to other causes may be associated with bleeding, patients with HIT are at high risk for thrombotic complications. The diagnosis of HIT requires laboratory confirmation, either with functional platelet activation assays or immunoassays such as the enzyme-linked immunosorbent assay. However, if HIT is clinically suspected, the clinician should not wait for positive test results to immediately stop heparin from any source (UH, LMWH, heparin-bonded catheters, and heparin flushes). Because the 30-day risk of subsequent thrombosis is 20 to 50%, alternative anticoagulation should be instituted. Vena cava filters are not indicated because there are several anticoagulants available. Published clinical guidelines from the American College of Chest Physicians recommend treating HIT with lepirudin, argatroban, or danaparoid. Of these, argatroban is currently the only FDA-approved agent. Lepirudin is no longer manufactured. Although bivalirudin is not yet approved for use in HIT, it is effective in preventing thrombotic complications. However, because bivalirudin is renally excreted, it is relatively contraindicated in patients with renal insufficiency. Warfarin should not be used in the acute setting, because it causes a decrease in proteins C and S, which can worsen the thrombotic risk. Warfarin can be started at low doses (5 mg/day) when platelet counts increase greater than 150,000/mm3, overlapping with a nonheparin anticoagulant for at least 5 days. The use of direct oral anticoagulants such as rivaroxaban, dabigatran, and apixaban is under investigation, but these agents are not currently recommended to treat HIT.

The patient presented here has class 1 EHIT. Treatment should consist of daily aspirin and monitoring with serial ultrasound examinations every 1 to 2 weeks. Clopidogrel is not necessary unless the patient is allergic to aspirin. The aspirin can be stopped when the thrombus retracts caudally. In the rare case that the thrombus progresses to class 3, anticoagulation should be instituted and continued until the thrombus retracts caudally or for the full duration of DVT treatment. Anticoagulation is not indicated now, nor is high ligation of the saphenofemoral junction.

24. Two weeks after an aortofemoral bypass, a 67-year-old man with chronic renal insufficiency presents with acute swelling in his right leg. Duplex ultrasonography confirms acute occlusive thrombus in the right femoral and popliteal veins. He is started on unfractionated heparin. Twenty-four hours later, his laboratory values are remarkable for a platelet count of 57,000/mm3, which has dropped from a preoperative value of 175,000/mm3 (150,000-400,000/mm3). The next best step in treatment should be A. continued unfractionated heparin. B. change to low molecular weight heparin.

154

Related Documents

Sesap 17th Vol 2
February 2021 0
Sesap 17th Vol 1
February 2021 0
Morometii Referat Vol 2
January 2021 1
10280 Vol 2 Spa
February 2021 0

More Documents from "LordNieve"

Sesap 17th Vol 1
February 2021 0
Sesap 17th Vol 2
February 2021 0
Tourstournage_02.pdf
February 2021 2
Le Nectar Cachete
January 2021 0
Bremmer-humansacrifice
January 2021 2